Você está na página 1de 277

Clinical

Cases and
OSCEs in
Surgery
Commissioning Editor: Laurence Hunter
Senior Development Editor: Ailsa Laing
Project Manager: Annie Victor
Designer: Kirsteen Wright
Clinical
Cases and


OSCEs in
Surgery
SECOND EDITION

Manoj Ramachandran
BSc(Hons) MBBS(Hons) MRCS(Eng) FRCS(Tr&Orth)
Consultant Paediatric and Young Adult Orthopaedic Surgeon, The Royal London and
St. Bartholomews Hospitals, Barts and The London NHS Trust, London; Honorary Senior
Lecturer, William Harvey Research Institute, Barts and The London School of Medicine
and Dentistry, University of London, UK

Marc A Gladman
MBBS DRCOG DFFP PhD MRCOG MRCS(Eng) FRCS(Gen Surg)
Professor of Surgery, Blacktown/Mount Druitt Hospitals and Chair of Surgery, School of
Medicine, University of Western Sydney, New South Wales, Australia

Adam Poole (First Edition)


BSc(Hons) MBBS(Hons) MRCS(Eng)
Director, Arcus, London, UK

Photographer: Pramod Achan Contribution from:


MBBS FRCS(Eng) FRCS(Tr&Orth) Thomas P J Crompton
Consultant Orthopaedic Surgeon, The Royal London MBBS BSc MRSC
and St Bartholomews Hospitals, Barts and The Specialist Surgical Registrar (SE Thames Rotation),
London NHS Trust, London, UK London, UK

Edinburgh London New York Oxford Philadelphia


St Louis Sydney Toronto 2011
First Edition Elsevier Science Limited 2003
Second Edition 2011 Elsevier Limited. All rights reserved.

No part of this publication may be reproduced or transmitted in any form or by any means, electronic
or mechanical, including photocopying, recording, or any information storage and retrieval system,
without permission in writing from the publisher. Details on how to seek permission, further
information about the Publishers permissions policies and our arrangements with organizations such
as the Copyright Clearance Center and the Copyright Licensing Agency, can be found at our website:
www.elsevier.com/permissions.

This book and the individual contributions contained in it are protected under copyright by the
Publisher (other than as may be noted herein).

First Edition by Manoj Ramachandran and Adam Poole.

ISBN 978-0-7020-2994-3

British Library Cataloguing in Publication Data


A catalogue record for this book is available from the British Library

Library of Congress Cataloging in Publication Data


A catalog record for this book is available from the Library of Congress

Notices
Knowledge and best practice in this field are constantly changing. As new research and experience
broaden our understanding, changes in research methods, professional practices, or medical
treatment may become necessary.

Practitioners and researchers must always rely on their own experience and knowledge in evaluating
and using any information, methods, compounds, or experiments described herein. In using such
information or methods they should be mindful of their own safety and the safety of others, including
parties for whom they have a professional responsibility.

With respect to any drug or pharmaceutical products identified, readers are advised to check the
most current information provided (i) on procedures featured or (ii) by the manufacturer of each
product to be administered, to verify the recommended dose or formula, the method and duration of
administration, and contraindications. It is the responsibility of practitioners, relying on their own
experience and knowledge of their patients, to make diagnoses, to determine dosages and the best
treatment for each individual patient, and to take all appropriate safety precautions.

To the fullest extent of the law, neither the Publisher nor the authors, contributors, or editors, assume
any liability for any injury and/or damage to persons or property as a matter of products liability,
negligence or otherwise, or from any use or operation of any methods, products, instructions, or
ideas contained in the material herein.
The Publisher

Working together to grow The


publishers
libraries in developing countries policy is to use
paper manufactured
www.elsevier.com | www.bookaid.org | www.sabre.org from sustainable forests

Printed in China
PREFACE
Preface to the Second edition
Increasingly, the art of surgical examination is postgraduate or exit examination level. We have
being lost from the repertoire of clinical skills preserved the hierarchy of difficulty of surgical
of undergraduate and postgraduate students cases first introduced in our original edition and
of surgery. Unlike many other branches of those candidates that would like to challenge
medicine in the twenty-first century, many their surgical brains are invited to proceed on
surgical diagnoses are made on the basis to the more difficult cases. We have also
of clinical assessment alone and thus the introduced photographs of key manoeuvres
maintenance and fine-tuning of clinical skills during specific surgical examination routines in
remain imperative to good practice in surgery. order to make the leap from text to practice that
The primary aim of this book is to demystify and much easier. In addition, we have updated the
simplify the clinical assessment of surgical text and references and added a section on
cases. We have included lessons learned from communication skills.
our own personal experience gained from We hope that the changes we have made will
surgical examinations and have provided Top help you prepare with confidence for any
Tips, acronyms and up-to-date summaries of surgical examination you may have to face in
current practice wherever possible. the near future.
This second edition is aimed squarely at Manoj Ramachandran
candidates preparing for any surgical
Marc A Gladman
examination, be it at undergraduate,
London 2010

Preface to the First edition


This book is designed for candidates sitting star rating given to each case, three stars being
both the MRCS clinical section as well as the most frequently encountered.
undergraduate clinical examinations in surgery, Instruction: This is the same for short cases
with the objectives of explaining how the and OSCEs and defines the flow of the case
examinations work, and of smoothing the which follows.
process of passing. The 133 cases in the book
Top tips: These are included to emphasize
are carefully framed to allow interpretation for
specific areas (often favourites of examiners),
both short cases and OSCEs. In each case, an
which cause confusion or are described
example of the opening instruction is given,
differently by different surgeons and teachers.
followed by a discussion of the steps required
to complete the examination and to pass. Finish your examination here: This instruction
is added to demonstrate where the marking
From the start, we set out to create this book
sheets for an OSCE, or the expectations of a
from within the context of the examination. The
short-case examiner, are likely to conclude.
following is a list of the ways in which the
Going beyond this is unlikely to score any
ordering, selection of cases, and format of each
further marks and you are more likely to impress
case are designed to help you to pass.
by answering some supplemental questions
Structured bays: Cases are listed in clusters accurately.
that reflect the historical organization of
Questions and advanced questions: These are
examination cases into areas that examiners
designed to fit in with both short-case and
have the experience to cover. It is unlikely that
OSCE formats, and also cover supplemental
a vascular surgeon would feel confident
questions asked following history scenarios.
examining an orthopaedic station (particularly at
MRCS level). Procedures and props: We have included
examples of common procedures and props that
Common to rare: In OSCEs particularly,
come up in the skill-based examination format,
commoner cases appear much more frequently
such as reduction of fractures and description of
than rare cases in the actual clinical exam.
intramedullary nails and external fixators.
This book lists cases in decreasing order of
frequency of appearance in the examination Thanks for buying and reading this book, and
within each sub-section; this is denoted by the good luck with your exams.
ACKNOWLEDGEMENTS
Acknowledgements for the second edition
The authors would like to thank Mr Pramod and Trauma, South East Thames rotation,
Achan, Consultant Orthopaedic Surgeon, The London for his significant contribution to the
Royal London and St Bartholomews Hospitals, Communication Skills cases.
Barts and The London NHS Trust, London for The authors would also like to thank the
his role as photographer of the high-quality following current medical students (and one
images that have beautifully complemented the postgraduate surgeon) at Barts and The London
updated text of the new edition of this book. School of Medicine and Dentistry, University of
We would also like to thank Dr Elizabeth Owen London for volunteering to act as photographic
for her excellent body-art painting on our models: Katie Chan, Harry Craven, Marc
student models and Ms Noemi Montes for her Gladman, Archchana Radhakrishnan, Catherine
vital help with the photographic editing. Rees, Emily Shepherd and Natalie Soobadoo.
The authors would also like to thank Tom
Crompton, Specialist Registrar in Orthopaedics

Acknowledgements for the first edition


The authors are grateful to the following Emma Jackson, Will Jackson, Tim McCormick,
individuals for their advice: Ian McDermott, Navin Ramachandran, Sally
Rachel Bell, Joanna Broomfield, Timothy Richardson, Marc Swan, Hazel Warburton and
Cheadle, Paul Dilworth, Barry Ferris, Richard Dan Weaver.
Harrison, Charlie Knowles, Gordon Kooiman,
INTRODUCTION
This introduction describes and discusses the case. In fact, there is no fundamental difference
different types of assessment of candidates in between the examination style required it is
these examinations and describes the different only the assessment and marking schemes that
scenarios that might be presented. are different. Examining an inguinal hernia, or a
Before going any further a note of caution. thyroid lump, or taking a history from a patient
It is often said that the OSCE is completely with abdominal pain, is the same in each.
different from the short case and therefore the However, because the OSCE is an objective
methods used for preparing for the clinical examination, the marking schedules are much
exam in surgery should be shredded and the more clearly defined, and deviation (on the part
process begun from scratch. This is not the of the examiner) from this is not allowed.

SHORT CASES
Format The pros of short cases are that they:
Allow good candidates to progress rapidly to
At the beginning of the examination, candidates harder cases or more complex supplemental
wait in a specific central area to be collected by questions
the examiners, who work in pairs. One asks the Give flexibility for examiners to choose
questions and the other listens and often makes different patients who are waiting in the bay,
notes. The examiners lead you round the which is less boring for both examiner and
patients, who are organized in clusters (or patients
bays), and choose which patients you meet Allow rapid assessment of clinical skills
and in which order. across areas, e.g. in superficial lesions,
It is possible to include the description of a cases vary from skin lesions to lumps and
prop, or an X-ray or another data-interpretation bumps to thyroid nodules, etc.
style question, but these are usually Incorporate data interpretation questions,
supplemental to the major theme, which is the such as chest X-rays, as appropriate
physical examination of a particular part of a Test clinical skills across a broad spectrum.
patient. The vast majority of the time will be The cons of short cases include that:
spent examining the patient and answering
questions on the background problem or They allow little control of choice of patients
treatment options. an individual examiner picks (except the
presence of the co-examiner)
The examiners choose how many patients you
They can emphasize favourite clinical signs,
see per bay, which can vary between just one
which may not reflect clinical relevance
patient to six or seven. The only time limitation
is on the whole bay, which may be 10 or 15 It is difficult to control the marking scheme
minutes. Within that time it is up to the to ensure transparency and fairness
examiner how many patients the candidates see They are almost entirely subjective
and how deep (and difficult) the supplemental It is difficult for the candidate to feel
questions become. In Final MB short cases confident about doing well (or badly) as the
there is usually only one bay, where all the questions tend to get increasingly difficult.
cases are examined, which might be part of a
ward or a day surgery unit.

OBJECTIVE STRUCTURED CLINICAL EXAMINATION (OSCE)


of the syllabus. The time spent at each station
Format is fixed (often 710 minutes) and is the same for
every candidate, irrespective of how well, or
The OSCE examination takes the form of a fair, badly, the candidate is performing at the
where candidates approach different examiners station. Often a bell rings between stations to
at different stations (or in different rooms let the examiners know to move on to the next
altogether), who test them on specific aspects candidate. Each OSCE would contain between
viii Introduction

10 and 20 stations. The whole examination to history taking and in particular, examination
therefore lasts at least 2 hours and can be of communication skills. Dummies and
much longer. mannequins (such as for trauma, breast
In general therefore, an OSCE takes much examination or scrotal examination cases) are
longer to complete and the time spent on each also being used much more commonly for the
case (or scenario) is often longer than in the clinical parts of examinations.
short case format. The marking sheet the The pros of using simulated patients are that
examiner has in front of him is pre-set and only they:
allows them to score on specific criteria that are Allow accurate portrayal of typical patients,
standard for every other examiner as well. e.g. response to grief, being given a
The pros of OSCEs are that: diagnosis or information on the treatment
The marking scheme is explicit and therefore of a relative
seen as being fairer Are the most effective way of testing
They reduce inter-examiner variability, and communication skills
usually mean assessment by a larger number Contribute to discussion of each candidates
of examiners in total because each scenario performance and even the mark awarded
is examined by a different clinician The cons of using simulated patients are that:
They allow the possibility of assessment by They reduce the number of clinical scenarios,
other doctors (e.g. specialist registrars, and tend to increase history taking and
medical educators) or other healthcare communications stations
professionals In the same way as practicing basic
There tends to be much greater emphasis on resuscitation on a dummy, it is different in a
patient-centred examining, including real life situation
communication skills and rapport, i.e. tests It can be difficult to believe if the same actor
greater range of skills (not just clinical is used for more than one scenario with the
examination) same candidate.
They allow for much more extensive use of
simulated patients see below.
The cons of OSCEs include that they: Range of testing
Are repetitive for examiners and patients
One conclusion about OSCEs is that they dont
seen as being boring and may lead to
just test clinical examination technique. In fact
error
the areas they test are classified into five
Provide little or no scope for examiners to different headings:
push very strong candidates
1. Clinical examinations
Make it easier to score an average mark, and
more difficult to pull out a clear fail or an 2. History taking
exceptional candidate 3. Data analysis
May present patients as having a certain set 4. Communication skills technique
of characteristic symptoms or signs, which 5. Practical skills
may not mirror their personal clinical So how do you know which of these is being
situation tested in a given station?
Usually under-represent unusual cases as
they focus on common scenarios.
Clinical examinations
Simulated patients Who will be at the station (other than
examiners)?
Simulated patients are actors. There is a A patient with an identifiable pathology
growing industry of simulated patients across (inguinal hernia, thyroid lump, etc.)
medical education. Actors were originally used
Occasionally a mannequin
in teaching and assessment in general practice,
and the success of this has led to a huge What will be available to you?
expansion into other specialties over the last 5 Anything required to adequately complete
years. Actors can, of course, be trained and will the examination, e.g. in a thyroid scenario, a
play a clinical scenario very effectively. Clearly glass of water is provided; in a vascular bay
there are drawbacks and their use is confined a hand-held Doppler probe is provided
Introduction ix

How will the scenario begin? How will the scenario begin?
Normally examine , or have a look at , With an explicit instruction to comment on a
and you will be directed to the side of the prop or a set of data
patients examination couch, or to the area What kind of questions will be used?
where they are sitting
Often very specific (and quite closed)
What kind of questions will be used? questioning will be used to ensure you
These will often close in on the pathological understand the clinical significance of any
problem, especially if the candidate is getting abnormality you pick up
sidetracked with something which is not on What kind of supplemental questions should you
the marking sheet for the scenario expect?
What kind of supplemental questions should you Usually these will relate to the clinical
expect? situation which has been diagnosed, and are
Supplemental questions might be asked (as unlikely to relate specifically to history or
included in the chapters of this book) to examination technique.
ascertain background knowledge and
understanding of potential treatments.
Communication skills
History taking
Who will be at the station (other than examiners)?
Who will be at the station (other than Simulated patient
examiners)? What will be available to you?
A simulated patient or a real patient Probably a sheet detailing the
What will be available to you? communications exercise (which is usually
Possibly paper on which to make notes as given to you in advance to allow you to
you take the history prepare)
How will the scenario begin? What kind of questions will be used?
You may be asked to gain some information None, the scenario is a test of your rapport
about the symptoms a patient is describing and communication with the patient, not with
and to formulate a differential diagnosis the examiners
Be aware of the time; you are not going to What kind of supplemental questions should you
be able to complete a whole history but expect?
should focus on answering the exact None, for the same reason.
question posed, without going into a whole
stream of closed questioning
Practical skills
What kinds of questions will be used?
During the scenario none, but if you are Who will be at the station (other than the
interrupted you should take from this that examiners)?
you may be getting side-tracked
Nobody
What kind of supplemental questions should you
What will be available to you?
expect?
A prop or mannequin
Again supplemental questions may relate to
further parts of the assessment of the How will the scenario begin?
patients symptoms. With an instruction to demonstrate a specific
technique, such as advanced trauma
Data analysis life-support, or suturing, or reduction of a
Colles fracture on the examiners arm
Who will be at the station (other than examiners)? What kind of questions will be used?
Nobody Usually you talk through as you are
What will be available to you? proceeding with the case; the only role the
examiners have is to ensure that you can
Here a prop will be used which might be
adequately perform the specific skill
arterial blood gases, blood laboratory results,
joint aspiration results, histopathology results What kind of supplemental questions should you
or possibly an X-ray, CT scan or barium expect?
series Possibly none.
x Introduction

SCORING SYSTEMS
We set ourselves one objective in writing this understand under what basis you will be
book to help you to pass any surgical assessed and how you will score marks.
examination and the first stage is to

SCORING IN SHORT CASE ASSESSMENTS


As mentioned above, this is largely subjective, Coming up with possible further
but marks here are awarded for: examinations or tests that could be done
Introducing yourself to the patient and Thinking of a list of differential diagnoses,
establishing rapport or a definite diagnosis, and a list of
Taking care to appropriately expose the investigations that would tip you towards a
patient (as described in each individual particular cause
chapter) Following the train of thought of the
Examining the relevant parts of the body examiner, picking up on suggestions and
including starting with the hands letting yourself be taught technique at the
bedside.
Accurately identifying the pathological
problems (if there are any)

SCORING IN OSCE ASSESSMENTS


This is an objective test, and there is a specific 9. Thanking patient and washing hands
marking sheet, which might look like this: It is possible to come up with a marking
Bay 1 Superficial lesions scheme for each case in this book by picking
Case 8 Thyroid examination out the detail of the examination and making a
list of the things you would need to do in order
Done well = 2, Done adequately = 1, Not done = 0
to demonstrate competence. In the same way
Elements being assessed:
as in the short cases, there comes a point
1. Introdution to patients where you should finish your examination and
2. Adequate exposure tell the examiner how you would proceed. This
3. Observing neck from front is clearly listed under each case in the book.
4. Observing swallow test and protrusion of The examiner indicates if you should continue,
tongue and this would imply there are more marks yet
to be awarded.
5. Palpating neck from behind
6. Checking for cervical lymphadenopathy At the end of each case your marks are allotted,
then totalled at the end of the entire
7. Percussion and auscultation from the front
examination to come up with a score which
8. Mentioning the need to check clinical thyroid
translates into a pass/fail.
status

FAILING THE CLINICAL EXAMINATION


Failing a clinical exam is most likely if you are and establish rapport, making them like you. It
not seen to show due concern for the patient, isnt as simple as this, and at all levels you are
such as not introducing yourself, not exposing also expected to ask questions or examine
adequately, and not asking permission before intelligently and come up with the right answers
examining. The examiners may be trying very to most of the questions. You dont fail the
hard to give a hint that you are heading in whole examination for failing one OSCE though,
completely the wrong direction. Ignoring these and one of the most important things to do is
hints, and not listening carefully enough to the brush yourself down after each station and
question, may also lead to a failed case. Gross get on with trying to pass the next. We all
lack of knowledge or understanding is the third naturally emphasize in our minds the things
possibility. that havent gone so well, and this will tend to
A common mistake in OSCEs is to assume that psychologically knock you down during OSCEs.
you pass if you show concern for the patient Work on ways of concentrating on what you
Introduction xi

have done well at each station and move on to with the one he examines immediately before
the next, keeping your mind as fresh and alert or afterwards
as possible. Inter-examiner variability where different
In the OSCE, reducing as many variables as examiners have wildly different expectations
possible from the assessment reduces the of the appropriate amount of knowledge
chance that a candidate who should have required to pass
passed will actually fail (i.e. the false-negative Testing one single modality where,
rate). Variables that are reduced (or eliminated) instead of just being tested on clinical
in this format include the following: examination, a range of skills (as above) is
Intra-examiner variability where an examined.
examiner (by chance) chooses a harder set A pass mark for the OSCE may therefore be
of cases for a given candidate compared more fairly ascertained than in short cases.
This page intentionally left blank
CONTENTS
SECTION 1 SUPERFICIAL LESIONS

1 Lumps and ulcers history 3 22 Keratoacanthoma 43


2 Lumps and ulcers examination 3 23 Neurofibroma 44
3 Lipoma 5 24 Papilloma 45
4 Sebaceous cyst 6 25 Pyogenic granuloma 45
5 Ganglion 7 26 Seborrhoeic keratosis 46
6 Neck examination general 8 27 Solar keratosis 47
7 Cervical lymphadenopathy 12 28 Digital clubbing 48
8 Thyroid examination 14 29 Branchial cyst 49
9 Solitary thyroid nodule 19 30 Dermoid cyst 50
10 Multinodular goitre 22 31 Thyroglossal cyst 51
11 Diffuse thyroid enlargement 24 32 Radiotherapy marks 53
12 Thyroid history 25 33 Dermatofibroma 54
13 Hypertrophic and keloid scars 26 34 Hidradenitis suppurativa 55
14 Squamous cell carcinoma 28 35 Kaposis sarcoma 55
15 Malignant melanoma 29 36 Pharyngeal pouch 56
16 Basal cell carcinoma 32 37 Cystic hygroma 57
17 Pressure sores 34 38 Chemodectoma 58
18 Grafts and flaps 35 39 Furuncles 59
19 Ptosis 36 40 Pyoderma gangrenosum 60
20 Facial nerve palsy 38 41 Vascular malformations 61
21 Salivary gland swellings 40

SECTION 2 ABDOMEN AND TRUNK

42 Inguinal hernia 64 51 Examination of the scrotum


43 Abdominal examination general approach 86
general approach 68 52 Hydrocoele 87
44 Surgical jaundice 71 53 Epididymal cyst 88
45 Stoma 73 54 Varicocele 89
46 Hepatomegaly 75 55 Right iliac fossa mass 90
47 Incisional hernia 78 56 Transplanted kidney 92
48 Umbilical/paraumbilical hernia 79 57 Ascites 93
49 Splenomegaly 81 58 Epigastric mass 95
50 Inflammatory bowel disease 83 59 Pleural effusion 96
xiv Contents

60 Dysphagia 98 68 Chest post-lobectomy/


61 Enlarged kidney 100 pneumonectomy 110
62 Common surgical scars 102 69 Median sternotomy 111
63 Breast examination general 70 Testicular tumour 112
approach 103 71 Enterocutaneous fistula 114
64 Breast lump 105 72 Mouth signs in abdominal
65 Post-mastectomy breast 106 disease 116
66 Breast reconstruction 107 73 Epigastric hernia 117
67 Gynaecomastia 109 74 Femoral hernia 118

SECTION 3 MUSCULOSKELETAL AND NEUROLOGY

75 Orthopaedic history taking 92 Examination of the shoulder 159


general approach 122 93 Gait 165
76 Osteoarthritis of the hip 123 94 Osteochondroma 166
77 Osteoarthritis of the knee 130 95 Popliteal fossa swellings 167
78 Dupuytrens contracture 137 96 Hallux rigidus 169
79 Carpal tunnel syndrome 139 97 Casts 170
80 Rheumatoid hands 142 98 Simulated reduction of
81 Osteoarthritis in the hands 144 fractures 171
82 Ulnar nerve lesions 145 99 Lumbar disc herniation 172
83 Hallux valgus 147 100 Brachial plexus lesions 175
84 Hammer toes 149 101 Ivory osteoma 178
85 Mallet toes 150 102 Chondroma 179
86 Claw toes 151 103 Charcots joints 180
87 Mallet finger 152 104 Winging of the scapula 181
88 Trigger finger 153 105 External fixators 182
89 Ingrowing toenail 154 106 Intramedullary nails 183
90 Ligamentous and cartilaginous 107 Pagets disease of bone 183
knee lesions 155 108 Achondroplasia 185
91 Radial nerve lesions 157
Contents xv

SECTION 4 CIRCULATION AND LYMPHATIC SYSTEMS

109 Varicose veins 188 122 Neuropathic ulcer 214


110 Venous ulcer 193 123 Lymphoedema 215
111 Peripheral arterial system 124 Hyperhidrosis 217
examination 194 125 False aneurysm 218
112 Vascular effects of the 126 Thoracic outlet obstruction 219
diabetic foot 199
127 Thromboangiitis obliterans
113 Amputations 200 (Buergers disease) 221
114 Peripheral arterial system 128 Superior vena cava
history 202 obstruction 222
115 Abdominal aortic aneurysm 203 129 Carotid artery aneursym
116 Carotid artery disease 205 and dilated common carotid
117 Popliteal aneurysm 207 artery 223
118 Ischaemic ulcer 208 130 Lymphangioma 224
119 Post-phlebitic limb 210 131 Arteriovenous fistulae 224
120 Gangrene 211 132 Coarctation of the aorta 226
121 Raynauds phenomenon 212 133 Atrial fibrillation 227

SECTION 5 COMMUNICATION SKILLS (WITH THOMAS CROMPTON)

134 Introduction to communication 139 Information giving


skills 230 breast carcinoma 234
135 Information gathering 140 Information giving
back pain 231 testicular tumour 235
136 Information gathering 141 Information giving
trauma call 232 loss of function 236
137 Information gathering 142 Information giving
vascular referral 232 the angry patient 237
138 Information giving obtaining
informed consent 233 Index 239
This page intentionally left blank
1
SECTION

SUPERFICIAL LESIONS
1 Lumps and ulcers history *** 3
2 Lumps and ulcers examination *** 3
3 Lipoma *** 5
4 Sebaceous cyst *** 6
5 Ganglion *** 7
6 Neck examination general *** 8
7 Cervical lymphadenopathy *** 12
8 Thyroid examination *** 14
9 Solitary thyroid nodule *** 19
10 Multinodular goitre *** 22
11 Diffuse thyroid enlargement *** 24
12 Thyroid history *** 25
13 Hypertrophic and keloid scars *** 26
14 Squamous cell carcinoma ** 28
15 Malignant melanoma ** 29
16 Basal cell carcinoma ** 32
17 Pressure sores ** 34
18 Grafts and flaps ** 35
19 Ptosis ** 36
20 Facial nerve palsy ** 38
21 Salivary gland swellings ** 40
22 Keratoacanthoma ** 43
23 Neurofibroma ** 44
24 Papilloma ** 45
25 Pyogenic granuloma ** 45
26 Seborrhoeic keratosis ** 46
27 Solar keratosis ** 47
28 Digital clubbing * 48
29 Branchial cyst * 49
30 Dermoid cyst * 50
31 Thyroglossal cyst * 51
32 Radiotherapy marks * 53
33 Dermatofibroma * 54
34 Hidradenitis suppurativa * 55
35 Kaposis sarcoma * 55
36 Pharyngeal pouch * 56
37 Cystic hygroma * 57
38 Chemodectoma * 58
39 Furuncles * 59
40 Pyoderma gangrenosum * 60
41 Vascular malformations * 61
Case 2 Superficial lesions 3

CASE 1 LUMPS AND ULCERS HISTORY ***

INSTRUCTION Onset
Ask this gentleman a few questions about his
When did you first notice it?
lump/ulcer.
What made you notice it?
Were there any predisposing events
APPROACH (e.g. trauma, insect bite)?
It is common in cases and OSCEs at finals, and
the MRCS, to be asked to take a focused Continued symptoms
history from a patient presenting with relatively
common problems, such as a lump or ulcer. How does it bother you, i.e. what symptoms
Listen carefully to the instruction. After does it cause? (Ask particularly about pain)
introducing yourself and establishing the Has it changed since you first noticed it?
patients name and age, go straight to (colour, shape and size changes are
questions about the lump or ulcer. You may important in malignant melanoma)
continue on to further relevant surgical Have you noticed any other lumps?
questions such as fitness for anaesthesia. The
Has it ever disappeared or healed?
examiner will usually stop you once you have
extracted the necessary information. You may
not always be asked to continue to examine the Treatments and cause
patient.
What treatments have you had in the past
for this?
TOP TIP What do you think is the cause of the lump/
If the examiner tells you the patients name, then do ulcer?
not embarrass yourself by asking his name again this You will usually find that as you extract the
only shows that you have not been listening to the relevant information, the examiner will move you
examiner! onto the examination relatively quickly.

TOP TIP
When asked to take a history, keep eye contact with
VITAL POINTS the patient throughout your questioning. Dont stare at the
Ask the following questions about the lump/ lump!
ulcer:

CASE 2 LUMPS AND ULCERS EXAMINATION ***

give a diagnosis and to describe the specific


INSTRUCTION
features which have led you to this conclusion.
Examine this lump.

VITAL POINTS
APPROACH
Inspect
Most clinical examinations in surgery include the
description of a lump. The examiners may even
Site most accurately measured with
expect an on-the-spot diagnosis. The description
respect to a fixed landmark, such as a bony
given here of the examination technique is
prominence
complete and exhaustive, but be prepared to
4 Superficial lesions Case 2

Size measure the dimension in centimetres of the lump and then flick one side of it,
(if the lump is large enough, be seen to use a feeling the other side for a percussion
measuring tape/ruler, but do not use a tape wave (most commonly performed in ascites,
on a small lump as it can appear awkward) Case 57)
Shape Fixation decide which plane the lump is in
Skin changes by determining which structures it is
Symmetry attached to, e.g.:
Scars Skin see if you can move the skin over
the lump
Colour
Muscle move the lump in two planes
Ask the patient if the lump is tender before
perpendicular to each other, ask the
proceeding with palpation.
patient to then tense the relevant muscle
and reassess the motion in the two
Palpate planes.

Surface smooth/irregular Percuss


Edge well/poorly defined
Consistency soft/firm/hard Dull/resonant (the latter indicating an air-filled
Temperature using the dorsal surface of mass).
the examining fingers or hand
Tenderness Auscultate
Transilluminability using a pen torch on one
side of the lump and looking through an Bruits or bowel sounds may be heard.
opaque tube, such as an empty Smarties
tube (this is difficult and cumbersome to
perform in a well-lit room and we therefore Finish your examination here
recommend not taking an empty Smarties
tube into the exam, especially if the lump is
a hydrocele!) Completion
Pulsatility place a finger on opposite sides
of the lump Say that you would like to:
expansile pulsation = fingers pushed
Examine the draining lymph nodes
apart
Assess the neurovascular status of the
transmitted pulsation = fingers pushed in
area/limb
the same direction (usually upwards)
Look for similar lumps elsewhere
Compressibility/reducibility press firmly on
Perform a general examination (as necessary).
the lump and release
compressible = lump disappears on
pressure but reappears on release, TOP TIP
e.g. arteriovenous malformations
When assessing consistency, imagine:
reducible = lump disappears on pressure
but reappears only when another Soft, comparable with the consistency of the flesh of
opposite force is applied, such as your nostrils (i.e. the ala)
coughing in hernia examination Firm, comparable with your nasal septum
Fluctuation (for small lumps) rest two
Hard, comparable with the bridge of your nose.
fingers of one hand on opposite sides of the
lump and press the middle of the lump with
the index finger of your other hand if the
fingers are moved apart, the lump is
fluctuant. (Repeat the test at right angles to Mnemonic
the first in order to confirm your findings.)
This is also known as Pagets sign (see We use the following mnemonic to remind us
Case 107) what to do with a lump. It is very useful as an
Fluid thrill for large lumps ask the patient aide-memoire for completeness, but note that it
to place the edge of his hand on the centre does not provide you with the correct order for
examination:
Case 3 Superficial lesions 5

Should The Children Ever Find Lumps Readily E Edge/Expansility and pulsatility
SS  ize/Site/Shape/Surface/Skin changes/ F Fluctuation/Fluid thrill/Fixation
Symmetry/Scars L Lymph nodes/Lumps elsewhere
T Temperature/Tenderness/Transilluminability R Resonance/Relations to surrounding
C Colour/Consistency/Compressibility structures and their state, e.g.
neurovascular status

A note on ulcers
Ulcers should be examined in a similar way to a lump, Undermined = pressure necrosis or tuberculosis
but important additional points to look for on Rolled = basal cell carcinoma
examination can be remembered in the form of the
Everted = squamous cell carcinoma
mnemonic BEDD:
Describe which structure is visualized at the base of
Base. Look for the presence of granulation tissue,
the ulcer, e.g. is the ulcer down to fascia, muscle
slough (i.e. dead tissue) or evidence of malignant
or bone?
change
Discharge. Is the discharge serous (clear),
Edge. Five types of edges to be aware of are:
sanguineous (blood-stained), serosanguineous
Sloping = a healing ulcer (usually venous or (mixed) or purulent (infected)?
traumatic)
Individual ulcers, e.g. arterial, venous, neuropathic,
Punched-out = ischaemic or neuropathic (rarely
are considered in the appropriate sections.
syphilis)

CASE 3 LIPOMA ***

INSTRUCTION Palpate
No specific instruction.
Lobulated surface
May be soft or firm depending on the nature
APPROACH of the fat within the lipoma and the
temperature at which it liquefies
Examine as for any lump (see Case 1).
If soft and large in size, may show fluctuation
Slip sign describes the manner in which
VITAL POINTS a lipoma tends to slip away from the
examining finger on gentle pressure
Lipomas can occur anywhere in the body where
Skin freely mobile over the lipoma (compared
there are fat cells, although they most
with a sebaceous cyst)
commonly occur in the subcutaneous layer of
Try and elicit which layer the lipoma is in,
the skin, particularly in the neck and trunk.
e.g. whether subcutaneous or intramuscular
(in the latter case, the lipoma disappears on
Inspect contraction of the relevant muscle).

Discoid or hemispherical swelling Completion


May appear lobulated
Look carefully for scars (may be a recurrent Say that you would like to ask the patient:
lipoma).
How the lipoma affects their lives, e.g.
cosmetic symptoms, pain
Whether they have noticed similar lumps
elsewhere.
6 Superficial lesions Case 4

Hibernomas, which consist of brown fat cells


QUESTIONS similar to those seen in hibernating animals
(a) What is a lipoma? BannayanZonana syndrome rare
autosomal dominant hamartomatous
A lipoma is a benign tumour consisting of disorder, characterized by multiple lipomas,
mature fat cells. Multiple, painful lipomas are macrocephaly and haemangiomas.
known as adiposis dolorosa or Dercums
disease, and are associated with peripheral (b) How are liposarcomas classified?
neuropathy. Liposarcomas can be classified
pathologically into three main groups:
(b) Do lipomas undergo malignant change?
1. Well-differentiated
It is thought that malignant change in a 2. Myxoid and round cell (poorly-
lipoma does not occur differentiated myxoid) liposarcoma
Liposarcomas arise de novo and usually 3. Pleomorphic liposarcoma.
occur in an older age-group in deeper
tissues of the lower limbs.

(c) How would you treat a lipoma? Francis X. Dercum (18561931). North American
neurologist, born in Philadelphia.
Non-surgical: reassure and watch and wait
Surgical: if the patient wants it removed, e.g.
pain, cosmesis. Some surgeons remove
lipomas using suction lipolysis via a small, FURTHER READING
remote incision. Usually this is performed Dalal KM, Antonescu CR, Singer S: Diagnosis
under local anaesthetic. However, nuchal and management of lipomatous tumors. J Surg
lipomas have extremely fibrous septae and Oncol 97(4):298313, 2008.
are difficult to excise, and any lipoma close Dei Tos AP: Liposarcoma: new entities and
to a joint may communicate with the joint evolving concepts. Ann Diagn Pathol 4(4):252
and it may not be possible to excise it under 266, 2000.
local anaesthetic.
www.cancerhelp.org.uk/help/default.
asp?page=18503 information for patients on
ADVANCED QUESTIONS lipoma removal.

(a) Do you know of any variants of lipomas or


syndromes associated with lipomas?
Angiolipomas, which have a prominent
vascular component

CASE 4 SEBACEOUS CYST ***

INSTRUCTION Usually solitary


Found most commonly on the face, trunk,
No specific instruction. neck and scalp
Punctum present at apex of cyst in 50%.
APPROACH
Examine as for any lump (see Case 1). Palpate

Smooth surface
VITAL POINTS Firm to soft on palpation
Inspect Punctum may exhibit plastic deformation on
palpation
Smooth hemispherical swelling
Case 5 Superficial lesions 7

All sebaceous cysts are attached to the skin, Epidermal cyst (EC) thought to arise from
therefore the cyst does not move the infundibular portions of hair follicles
independently from the skin. Trichilemmal cysts (TC) thought to arise
from hair follicle epithelium and so are most
Completion common on the scalp, and are frequently
multiple; these cysts have an autosomal
dominant mode of inheritance.
Say that you would like to ask the patient:
How the cyst affects their lives, e.g. (b) What is a Cocks peculiar tumour?
cosmetic symptoms Proliferating trichilemmal cysts are usually
Whether they have noticed similar lumps solitary, occur on the scalp in 90% of cases,
elsewhere. and can grow to a large size and ulcerate.
Clinically and histologically, they may resemble
a squamous cell carcinoma, in which case it is
QUESTIONS known as a Cocks peculiar tumour. Very rarely,
(a) What are the complications of a malignant transformation can occur.
sebaceous cyst?
(c) What is Gardners syndrome?
Infection frequent complication, there may Multiple epidermal cysts may be part of
be an associated discharge Gardners syndrome, which is also associated
Ulceration with:
Calcification (trichilemmal cysts, see Adenomatous polyposis of the large bowel
below) this may cause the cyst to feel Multiple osteomata of the skull
hard on palpation
Desmoid tumours.
Sebaceous horn formation (hardening of
Note that Gardners syndrome is now part of
a slow discharge of sebum from a wide
the spectrum of familial polyposis coli
punctum)
syndromes, which includes familial
Malignant change. adenomatous polyposis.
(b) How would you treat a sebaceous cyst?
Non-surgical: may be left alone if small and Edward Cock (18051892). English surgeon at Guys
asymptomatic Hospital, who was the nephew of Sir Astley Cooper
and performed the first pharyngectomy in England.
Surgical: to prevent recurrence, complete
excision of cyst and its contents is required Eldon J. Gardner (19091989). American geneticist
which requires removal of an elliptical portion and Professor of Zoology, Utah State University.
of skin containing the punctum.
FURTHER READING
ADVANCED QUESTIONS Dastgeer GM: Sebaceous cyst excision with
minimal surgery. Am Fam Physician 43(6):1956
(a) What are the different histological 1960, 1991.
subtypes of sebaceous cysts?
www.intelihealth.com/IH/ihtIH/WSIHW000/
Two types of cysts are recognized according to 9339/9779.html information for patients on
their histological features: sebaceous cysts.

CASE 5 GANGLION ***

INSTRUCTION APPROACH
Examine this gentlemans hand. Expose to elbows and ask the patient to
place his hands palm upwards on a pillow
(if available).
8 Superficial lesions Case 6

sheath. The origin of ganglia is controversial


VITAL POINTS
they are seen as a pocket of synovium
Ganglia can occur anywhere in the body, communicating with the joint or tendon sheath,
although they are commonly found around the or as a myxomatous degeneration of fibrous
wrist, on the dorsum of the hand and on the tissue.
dorsum of the ankle. In fact, the most common
soft-tissue mass found in the hand is a (b) What is the differential diagnosis?
ganglion. Bursae
Cystic protrusions from the synovial cavity of
Inspect arthritic joints
Benign giant cell tumours of the flexor
Usually single sheath (indistinguishable from flexor sheath
Hemispherical swelling ganglia)
Look carefully for scars (may be recurrent). Rarely, malignant swellings, e.g. synovial
sarcoma.

Palpate (c) How would you treat a ganglion?


Non-surgical: watch and wait, or aspiration
Smooth surface followed by 3 weeks of immobilization
May be multiloculated (successful in 3050% of patients). (The old
May be soft and fluctuant (especially if large) method of striking the ganglion with the
or firm (if small with tense, viscous contents) family Bible is now out of favour!)
Associated with a synovial lined structure Surgical: complete excision to include the
such as a tendon or joint neck of the ganglion at its site of origin.
Weakly transilluminable due to its viscous
fluid contents.
(d) What complications are associated with
surgical treatment of a ganglion?
Wound complications, e.g. scar, haematoma,
Completion
infection
Recurrence can be as high as 50% but
Say that you would like to ask the patient:
can be lower if care is taken to completely
How the ganglion affects their lives, excise the neck
e.g. cosmetic symptoms Damage to adjacent neurovascular
Whether they have noticed similar lumps structures.
elsewhere
Which hand is dominant (considering
treatment options) FURTHER READING
Their occupation (also to consider treatment Thornburg LE: Ganglions of the hand and wrist.
options). J Am Acad Orthop Surg 7(4):231238, 1999.
www.med.und.nodak.edu/users/jwhiting/ganglia.
QUESTIONS html information for patients.

(a) What is a ganglion?


A ganglion is a cystic swelling related to a
synovial lined cavity, either a joint or a tendon

CASE 6 NECK EXAMINATION GENERAL ***

INSTRUCTION APPROACH TO THE NECK


Examine this gentlemans neck. Note that the patient is usually sitting in a chair
and may have a glass of water next to him
Case 6 Superficial lesions 9

If there is a glass of water, be prepared to examination of a thyroglossal cyst


examine the thyroid gland in full (see Case 31)
Expose the whole neck down to both A thyroid lump does not move on protrusion
clavicles this may necessitate undoing the of the tongue.
top buttons of a shirt or even taking off a
polo neck jumper
Swallowing
Ask the patient to remove any jewellery
present.
Place the glass of water in the patients
hands
TOP TIP Ask him to take a sip of water, hold it in his
mouth and swallow when you ask him to
The examiners may try to catch you out by placing
the patient on a chair with its back against the wall. Your As he swallows, inspect the lump if it
first move is to ask the patient to stand up and move the moves on swallowing, it is likely to originate
chair away from the wall, allowing you to access and from the thyroid gland.
examine the patients neck from behind.
Palpate (from the back)

The neck is best (and first) palpated from


behind the patient
VITAL POINTS Be as gentle as possible, as you are unable
to watch the patients face for pain
Inspect (from the front)
Use the fingertips of both hands to elicit the
Site of the lump, e.g. midline, supraclavicular physical signs
fossa Begin by showing the examiner that you
Other features on inspection of the lump, know the borders of the two main triangles
e.g. size, skin changes, scars (see Case 1). of the neck and tell him which triangle the
lump is in (Fig. 1)
The anterior triangle of the neck is
Protrusion of the tongue bordered by the anterior border of
sternocleidomastoid, the midline and the
Ask the patient to open his mouth and stick ramus of the mandible
his tongue out as far as possible The posterior triangle of the neck is
If the lump moves on protrusion of the bordered by the anterior border of
tongue, it is likely to be a thyroglossal cyst trapezius, the clavicle and the posterior
(this is because the cyst is usually related to border of sternocleidomastoid
the base of the tongue by a patent or fibrous Next, determine whether the lump is solid or
track, which runs through the central portion cystic. You should now be ready to consider
of the hyoid bone) proceed with the differential diagnosis (Table 1 and Fig. 2).

Mandible

Posterior border of Midline


Borders of Borders of
sternocleidomastoid
posterior anterior
triangle Anterior Anterior border of triangle
of the border of sternocleidomastoid of the
neck trapezius neck

Clavicle
Figure 1 Posterior and anterior triangles of the neck.
10 Superficial lesions Case 6

Table 1 Differential diagnosis of neck lumps


Position Solid Cystic
Midline Thyroid swelling (Case 8) Thyroglossal cyst (Case 31)
Anterior triangle Lymphadenopathy (Case 7) Branchial cyst (Case 29)
Chemodectoma (Case 38) Cold abscess (secondary to tuberculosis)
Posterior triangle Lymphadenopathy Pharyngeal pouch (Case 36)
Cystic hygroma (Case 37)
Within sternocleidomastoid Sternocleidomastoid tumour

Submental gland, dermoid Submandibular


or thyroglossal cyst gland

Parotid gland
Carotid
chemodectoma
Branchial cyst

Thyroid nodule Thyroglossal


cyst

Cystic hygroma

Tip of Thyroid
cervical rib nodule
Figure 2 Locations of the most common swellings in the neck.

TOP TIP Examination of cervical lymph nodes


Multiple lumps palpable within the neck are invariably
The cervical lymph nodes (Fig. 4) are best
lymph nodes.
examined using the up-and-down technique:
Use gentle rotating movements of the
fingertips this allows you to palpate even
the smallest nodes
Continuing the examination
If the patient tries to help you by raising their
chin, ask him to drop his chin this makes
If at this stage you think that the lump is thyroid
the examination easier by relaxing the
in origin you should proceed to examine the
anterior neck muscles
thyroid gland in full (Fig. 3 and see Case 8).
Begin by moving from the chin backwards,
If you have attempted a differential diagnosis palpating the submental, submandibular and
you should be prepared to offer additional parotid glands and pre-auricular nodes
evidence for your suggestions see individual
Move your fingers behind the ears and feel
cases.
the mastoid (post-auricular) nodes
If you have not found a lump at this stage you Go down the anterior border of the
should examine the neck thoroughly using the sternocleidomastoids, feeling the anterior
up-and-down technique as in Table 2.
Case 6 Superficial lesions 11

General approach

Inspect
Protrusion of tongue
Swallowing

Palpate from the back

Define triangle of neck

If you know the


differential diagnosis If lump is
state this and move NECK midline continue
on to discuss each DECISION with thyroid
individual diagnosis CIRCLE examination
(see individual
cases)

If you haven't found a


lump proceed with up-and
-down technique

Figure 3 Neck decision circle approach to examination of the neck.

Table 2 The up-and-down technique Move up the posterior border of the


Stage Procedure sternocleidomastoids, feeling the posterior
triangular nodes
1 Palpate from the chin backwards to below
Finish by palpating the occipital nodes at the
the ears
back of the neck.
2 Move your hands behind the ears and
palpate DOWN the anterior border of
sternocleidomastoid to the clavicle Palpate (from the front)
3 Move laterally along the clavide and then Confirm your findings if necessary by feeling
UP the posterior border of the lump from the front, watching the
sternocleidomastoid patients face carefully for signs of
4 Finish by palpating the back of the scalp discomfort.
for occipital nodes
Percussion and auscultation
triangular nodes, including the jugulodigastric
(tonsillar) node See individual cases.
Move laterally along the clavicular region,
feeling for both supraclavicular and
infraclavicular nodes
12 Superficial lesions Case 7

Parotid Preauricular

Postauricular

Occipital
Submental
Jugulo-
Submandibular digastric
Anterior cervical
chain Posterior
triangular
nodes

Infraclavicular Supraclavicular
Figure 4 Typical grouping of lymph nodes.

Finish your examination here (following a complicated or breech birth) and


normally disappearing over the first 46 months
of life. Babies may present with a torticollis.
With early diagnosis, non-surgical treatment
Note with active stimulation and passive stretching
and occasionally using Botulinum toxin
Sternomastoid tumour is an ischaemic injections; with late diagnosis, it may require
contracture of a segment of the muscle seen to surgery.
appear in the first 12 weeks after birth

CASE 7 CERVICAL LYMPHADENOPATHY ***

INSTRUCTION Additional points on inspection


Examine this gentlemans neck.
Site of the lump, e.g. midline, supraclavicular
fossa
APPROACH Other features on inspection of any lump,
e.g. size, skin changes, scars (see Case 1).
Approach as you would a neck examination
(see Case 6). Note that cervical lymph nodes
are the commonest neck lumps found in the Additional points on palpation
clinical cases. (from the back)

Use the up-and-down routine as detailed in


Inspection, protrusion of the tongue, Case 6 to examine thoroughly for cervical
swallowing, palpation lymphadenopathy
Remember also that the lymph nodes should
(See Case 6). be examined as for any other lump (see
Case 1) and particularly note:
Consistency tends to be firm but may
be rubbery
Case 7 Superficial lesions 13

Number solitary, multiple or matted to Lymphoma and leukaemia


each other Infection (see below)
Fixation skin tethering in tuberculous Sarcoidosis
nodes or malignancy. Tumours (primary/secondary).
Infectious causes can, as always, be further
Finish your examination here subclassified:
Bacterial
Tonsillitis, dental abscess (b-haemolytic
Completion streptococcus)
Tuberculosis
Say that you would like to: Viral
Examine the face and scalp carefully for a Cytomegalovirus
primary site of infection or neoplasia Infectious mononucleosis (Epstein-Barr
Perform a full examination of the ear, nose virus)
and throat (say that you would request a Human immunodeficiency virus
formal full ENT examination), including the
Protozoal
salivary glands and the thyroid gland
Toxoplasmosis
Examine the rest of the lymphoreticular
system, including palpation of the abdomen (c) How would you investigate
for hepatomegaly and splenomegaly this gentleman?
Look for a primary site of infection or
Blood tests:
neoplasia above the umbilicus, e.g. chest
examination Haematological: full blood count,
erythrocyte sedimentation rate
In a female patient a breast examination
would also be indicated as breast Biochemical: thyroid function tests,
malignancy can metastasize to the neck. angiotensin converting enzyme levels
which may be raised in sarcoidosis
Serological: monospot or PaulBunnell
QUESTIONS test looking for atypical mononuclear cells
in infectious mononucleosis
(a) What questions would you like to ask Radiological:
this gentleman? Ultrasound
Directed to the possible causes (see (b) below): CT scan
Symptoms from the lump itself, e.g. the MRI scan
duration, pain (e.g. in lymphomas pain is Histological
experienced on alcohol ingestion, although Fine-needle aspiration cytology (FNAC):
this is not specific to lymphomas), other False-positive rate 03%, false-
lumps elsewhere negative rate 110%
General symptoms, e.g. night sweats, loss of Errors reduced by experience of
appetite, loss of weight clinician and cytologist
Local symptoms, e.g. intraoral diseases such Excision biopsy (see below).
as tooth decay
Systemic disease, e.g. serious medical
illnesses, previous surgical operations ADVANCED QUESTIONS
(thinking of neoplasia)
(a) What results might you expect from the
Social history ethnic origin (patients from
high-risk areas for TB including the Indian FNAC and how would you proceed?
subcontinent), foreign travel, contact with If malignant:
animals (cat scratch fever), risk-factors for Is it squamous cell carcinoma? Do not
HIV infection. perform open lymph node excision biopsy
(spoils the field for subsequent block
(b) What causes of cervical lymphadenopathy
dissection of the neck and may reduce
do you know of? survival), refer to ENT surgeon for full
Think of the acronym LIST when considering assessment to include panendoscopy to
this answer: find a primary tumour. Random biopsies
14 Superficial lesions Case 8

from multiple sites may be needed, along Limited dissection is now in favour
with sputum cytology and chest X-ray (supra-omohyoid only in oral and
Is it adenocarcinoma? Continue to open oropharyngeal carcinoma and lateral only
lymph node excision biopsy and look for in hypopharyngeal and pharyngeal
primary from breast or intra-abdominal tumours) in conjunction with radiotherapy
viscera such as pancreas or stomach Radical neck dissection:
Is it lymphoma? Continue to open lymph Clear all lymphatic tissue from mandible
node excision biopsy as a whole node is above to clavicle below, and from the
required for detailed histology and marker midline to the anterior border of the
studies trapezius laterally
If inflammatory: Incisions used include the wineglass, the
Is it tuberculosis? Do not perform open standard y and the McFee incision
lymph node excision biopsy (may result in Details of the dissection itself are beyond
chronic sinus formation) treat as for postgraduate level.
tuberculosis
Is it another infectious or inflammatory
disorder? Continue to open lymph node J. R. Paul (18931971). North American physician and
excision biopsy and treat according to pathologist.
underlying cause.
W. W. Bunnell (19021966). North American
(b) What surgical options are available in the physician.
management of cervical lymphadenopathy? Thomas Hodgkin (17981866). English physician,
St Thomass Hospital and Curator of the Pathology
Open lymph node excision biopsy:
Museum at Guys Hospital.
Best performed under general anaesthesia
M. A. Epstein (born 1921). English physician and
Beware biopsy in the posterior triangle
Professor of Pathology, Bristol.
due to risk of damaging the spinal
accessory nerve which is quite Yvonne Barr (born 1932). English physician.
superficial damage leads to shoulder
and arm pain, paralysis of trapezius
and winging of the scapula
In addition patients should be warned of FURTHER READING
damage to the facial nerve if the surgical Peters TR, Edwards KM: Cervical
approach includes dissection around the lymphadenopathy and adenitis. Pediatr Rev
parotid gland 21(12):399405, 2000.
Block dissection of the neck: Tracy TF Jr, Muratore CS: Management of
Classic operation involves removing the common head and neck masses. Semin Pediatr
sternomastoid, jugular vein and accessory Surg 16:313, 2007.
nerve

CASE 8 THYROID EXAMINATION ***

A hoarse voice (recurrent laryngeal nerve


INSTRUCTION
palsy)
Examine this ladys thyroid gland. Warm and sweaty hands
(hyperthyroidism).
The objectives of thyroid examination are to:
APPROACH
1. Confirm that the abnormality lies within the
See general approach to examination of the thyroid gland (as opposed to other neck
neck (Case 6) structures)
As you start the examination, you should be 2. Determine whether there is diffuse
looking for clues of thyroid dysfunction such enlargement of the thyroid (smooth or
as: nodular) or a solitary nodule
Case 8 Superficial lesions 15

3. Examine structures around the thyroid


4. Assess the thyroid status of the patient.

TOP TIP
Examination of the thyroid should be directed at
achieving these objectives and is easiest performed in
three parts:
Part 1: The thyroid gland itself
Begin the examination in front of the patient, then
move to the back before finally returning to the
front ready for Part 2
Part 2: Structures around the thyroid
Assess trachea and oesophagus; recurrent
laryngeal nerve
Part 3: The thyroid status.

PART 1: THE THYROID


GLAND ITSELF Figure 5 Examination position for palpation of the
Examine (from the front) thyroid gland from behind.

Inspection, protrusion of the tongue,


swallowing
(See Case 6).

Additional points on inspection


Obvious midline lump (see Notes (below) for
definition of goitre)
Scars horizontal skin crease incision is
most common following previous thyroid
surgery
Raised jugular venous pulse due to neck
vein obstruction from mass effect.

Examine (from behind) Figure 6 Palpation of each lobe of the thyroid gland.

Palpation
Describe the features of the lump (see Case
(See Figure 5). 1) gently push on one edge of the lump so
that you can palpate the other edge with ease
Additional points on palpation (be gentle!) (see Figure 6) feel particularly
Ask the patient to protrude the tongue for:
again checking for a thyroglossal cyst Size
while gently palpating the thyroid gland Tenderness
from behind
Mobility
Repeat the swallow test, asking the patient
Consistency
to take another sip of water, hold it in the
Most importantly, try to work out whether
mouth and swallow when you indicate. Feel
there is:
the thyroid gland rise, proving the mass
arises from the thyroid Diffuse enlargement of the thyroid
smooth or nodular or a solitary nodule
16 Superficial lesions Case 8

Figure 7 Palpation of the cervical lymph nodes using


the up-and-down technique.
Figure 8 Assessing for tracheal deviation during thyroid
examination.

Move on to examining the cervical lymph Neoplasms: benign (follicular adenoma); malignant
nodes performing the up-and-down primary (papillary, follicular, medullary, anaplastic
technique (Fig. 7 and see Case 6). squamous cell carcinoma and malignant lymphoma)
or secondary (metastatic from breast/kidney)
Examine (from the front) Dominant nodule of a multinodular goitre
masquerading as an apparently solitary nodule.
Percussion and auscultation
Listen over the thyroid for a systolic bruit
this is caused by a hypervascular thyroid
which is almost pathognomonic of Graves PART 2: STRUCTURES AROUND
disease THE THYROID
Percuss over the sternum from the notch Gently palpate the trachea for deviation by
downwards listening for a change in placing one finger over the trachea. It should
percussion note if there is retrosternal lie equidistant between the heads of the
extension. clavicles (Fig. 8)
Ask the patient if she has had any problems
TOP TIP swallowing or has noticed any change in her
voice this completes the examination of
A classification scheme for thyroid enlargement the structures around the thyroid gland
Diffuse enlargement smooth or nodular You could ask the patient to repeat a
sentence that you read out in order to listen
Multinodular goitre (see Case 10) for the hoarse voice characteristic of a
Toxic (i.e. hyperthyroid) = Graves disease previously damaged or infiltrated recurrent
(see Case 11) laryngeal nerve.
Simple colloid goitre (see Case 11)
Thyroiditis, e.g. subacute (granulomatous) de PART 3: THYROID STATUS
Quervains; autoimmune (Hashimotos); or Reidels This includes examination of the hands and
(invasive fibrous). In these cases, the thyroid may be eyes, and occasionally knowing other areas
tender to examine for further evidence of thyroid
Neoplastic goitre, benign/malignant. dysfunction. You will not usually be asked to
continue to perform this part of the examination
Solitary nodule (see Case 9)
if the patient has normal thyroid status (i.e. is
Degenerative cysts euthyroid).
Case 8 Superficial lesions 17

Move on to the hands examining finger rapidly down, while


watching the patients eyes. The globe will
There are seven signs to look for in the hands: follow the finger but the lid will lag behind
when the sign is positive
1. Increased sweating (due to hyperthyroidism)
4. Ophthalmoplegia. Fully assess eye
2. Palmar erythema (due to hyperthyroidism) movements, enquiring for the presence of
3. Thyroid acropachy (a feature of Graves diplopia (Fig. 11). The superior recti and
disease, see Case 11) also known as inferior oblique muscles are most commonly
pseudoclubbing affected, leading to diplopia when looking
4. Onycholysis (Plummers nails, see Case 10) up and out. In more advanced cases, it
5. Areas of vitiligo (white patches of skin
hyperpigmented borders, seen in association
with autoimmune disorders such as Graves
disease)
6. Pulse tachycardia or atrial fibrillation in
hyperthyroidism, bradycardia in
hypothyroidism
7. Fine tremor best demonstrated by placing
a sheet of paper on the outstretched hands
with palms facing downwards.

Proceed to the eyes

There are also seven signs to look for in the


eyes, the latter six being associated with
Graves disease (see Case 11):
1. Loss of hair on outer-third of eyebrows
(hypothyroidism)
2. Lid retraction raised upper eyelid but the
whiteness of the sclera is not visible around
the iris also known as Dalrymples sign
(Fig. 9) Figure 10 Assessing for lid lag during thyroid
examination.
3. Lid lag. Ask the patient to look up at an
examining finger and to follow it when you
move your finger (Fig. 10). Move the

Normal

Lid retraction
Elevation of the upper
eyelid

Exophthalmos
Sclera visible all round
the iris
Figure 11 Assessment of ocular movements for the
Figure 9 Eye signs in Graves disease. presence of ophthalmoplegia.
18 Superficial lesions Case 8

becomes impossible for the patient to Proceed to complete a full


complete this movement systemic examination
5. Exophthalmos both eyelids move away
from the centre of the iris so that the It may only be necessary to say that you
whiteness of the sclera is visible below or all would like to complete a full systemic
round the iris examination, although you should be
6. Chemosis the venous and lymphatic prepared to do it should the examiner so
drainage is disturbed by the protrusion of the desire
eye and the appearance is oedematous and The relevant systems to be examined include
wrinkled the cardiorespiratory system and the lower
7. Proptosis the eye has protruded so far limbs:
forward that it is visible beyond the level Look for signs of heart failure
of the supraorbital ridge when looking over
Inspect the shins for pretibial myxoedema
the head of the patient from behind finger
(seen in Graves disease, see Case 11)
(Fig. 12).
Test for proximal myopathy by assessing
In the normal eye, the upper eyelid is halfway
the strength of the muscles of the upper
between the pupil and the superior limbus,
arm (seen in Graves disease)
while the lower eyelid is at the level of the
inferior limbus. Test the reflexes supinator jerks are
inverted and ankle jerks are slow-relaxing
in hypothyroidism.

Finish your examination here

Completion

Say that you would like to:


Ask the patient how the thyroid mass is
affecting their life
Continue to assess the patients thyroid
status by asking her a few questions (see
Case 12).

J. Dalrymple (18041852). English ophthalmologist.


F. de Quervain (18681940). Swiss surgeon who
described subacute thyroiditis with self-limiting
inflammation of the gland, pathologically characterized
by giant cells and granuloma, which is probably as a
result of viral infection. Some 50% of patients may
experience mild hyperthyroidism.
Hakura Hashimoto (18811934). Japanese surgeon
who described an autoimmune thyroiditis often
associated with mild hypothyroidism. The pathology
is thought to be due to apoptosis induced by
lymphocytes bearing Fas ligands combining with
thyrocytes bearing Fas.

Notes

1. The term goitre is non-specific and


describes any swelling of the thyroid gland.
Figure 12 Examining for evidence of proptosis. It does not imply any pathology. It is derived
Case 9 Superficial lesions 19

from the Latin for throat (guttur). Goitres This is Pembertons sign do not elicit in the
become visible when they are three times examination, as the patient may faint.
the normal size, weighing over 50g. Goitres 3. There are some other physical signs of the
can be graded according to the World Health eye which are of historic interest that are
Organizations (WHO) grading scheme: included here for sake of completeness:
Grade 0: No palpable or visible goitre Stellwags sign: C. Stellwag von Carion
Grade 1: Palpable goitre (18231904), Austrian ophthalmologist
Grade 1A: Goitre detectable only by infrequent blinking in hyperthyroidism
palpation Joffroys sign: A. Joffroy (18441908),
Grade 1B: Goitre palpable and visible with French neuropsychiatrist absence of
neck extended wrinkling of the forehead when the patient
Grade 2: Goitre visible with neck in normal bends her head and looks up
position Mobius sign: P. J. Mobius (18531907),
Grade 3: Large goitre visible from a distance. German neurologist difficulty in
convergence elicited in a patient with
2. Patients with large retrosternal goitres
ophthalmoplegia.
develop signs of compression on raising
their arms above their heads, leading to 4. The term vitiligo is derived from the Latin
suffusion of the face, giddiness or syncope. vitellus for spotted calf.

CASE 9 SOLITARY THYROID NODULE ***

INSTRUCTION (c) What do you know about solitary thyroid


nodules?
See Cases 6 and 8 for the general examination
More common in females (F : M ratio = 4 : 1)
of the neck and thyroid gland.
Occur most commonly in the fourth and fifth
decade
SPECIFIC POINTS ON 10% in middle-aged are malignant but 50%
EXAMINATION OF THE NECK are malignant in the young and the elderly
Fine-needle aspiration cytology (FNAC) is the
Palpable nodule which moves on swallowing
most important investigation if benign,
but not on protrusion of tongue
leave alone and if malignant, surgery is
Determine the characteristics of the nodule required.
lump (see Case 1)
Palpate for associated cervical (d) How would you investigate and treat a
lymphadenopathy. solitary thyroid nodule?
All patients should undergo triple
QUESTIONS assessment:
Clinical examination
(a) What is the arterial supply to the Radiological assessment, usually
thyroid gland? ultrasonographic
Pathological, most commonly cytological
See Figure 13.
following FNAC.
(b) What are the causes of a solitary Note
thyroid nodule?
Contrary to what is still contained in some older
Prominent nodule in a multinodular goitre textbooks, technetium or iodine radio-isotope
Cyst (e.g. from haemorrhage into a nodule) scanning is ONLY of value in current clinical
Adenoma practice when assessing solitary nodules in
Carcinoma/lymphoma thyrotoxic patients. In this setting, it is used to
Thyroiditis (see Case 8). determine whether the nodule is functional and
the cause of thyrotoxicosis, as this clearly has
implications on further treatment. It has NO
20 Superficial lesions Case 9

Superior thyroid artery supplies upper


pole and anastamoses with inferior
thyroid artery behind the thyroid gland

Common
carotid
artery Thyroid
cartilage

Thyroid
gland

Inferior
thyroid artery

Thyrocervical
Trachea trunk

Thyroidea ima (in 3% of people)


originates from brachiocephalic Subclavian
trunk or arch of the aorta artery
Figure 13 Anatomy of the arterial supply to the thyroid gland.

value in differentiation of benign and malignant Anaplastic = 5%


nodules due to poor sensitivity and specificity. Lymphoma = 2%
The treatment is then dependent on the Papillary and follicular varieties are
findings (Fig. 14). collectively known as differentiated thyroid
cancer
(e) What do you know about Papillary
thyroid adenomas? Begins with P for paediatric = commonest
Almost all are follicular adenomas in children and young adults
Usually 24cm and encapsulated at Ends with y for yellow = lymph =
presentation lymphatic spread to lymph nodes
Indistinguishable from carcinomas on FNAC, Multicentric
as the presence of a capsule cannot be 90% of children have nodal metastases at
demonstrated surgery
Surgical excision is needed to confirm Follicular
diagnosis. Begins with F for fifty = mean age is 50
years at presentation
ADVANCED QUESTIONS Ends with r for red = blood = spread via
bloodstream
(a) What do you know about FNA cannot distinguish cancer from
thyroid malignancy? follicular adenoma
The incidence is low: approx. 4 per 100000 Note that 80% of follicular lesions on FNA
per year are adenomas
Histological varieties are: Treatment of differentiated thyroid cancer:
Papillary = 70% Thyroidectomy is the treatment of choice,
although the precise extent of the
Follicular = 15%
procedure depends on the type and size
Medullary = 8% of the cancer and remains controversial
Case 9 Superficial lesions 21

Solitary thyroid
nodule

Hyperthyroid Euthyroid

USS / FNAC USS / FNAC

123 Cystic
I scan Benign Malignant
Suspicious
Inadequate

Hot nodule Hot nodule Asymptomatic Pressure Surgery <4 cm >4 cm


<3 cm >3 cm symptoms OR
tracheal blood stained
deviation aspirate
increasing OR
size recurrence
after aspiration
Repeat Review after
FNAC and 6/12 and
discharge discharge
Therapeutic 131 I Surgery in 6/12 Surgery if no change Surgery

Figure 14 Management of solitary thyroid nodules. FNAC, fine-needle aspiration cytology; USS, ultrasound scan.

Total thyroidectomy is advocated due to influences the behaviour of any metastatic


its ability to treat multifocal tumour, tumour
decrease local and distant recurrence, Medullary
facilitates treatment with 131I and Arises from the parafollicular C cells
allows postoperative monitoring with (derived from ultimobranchial bodies)
thyroglobulin concentration measurement. which produce calcitonin, a polypeptide
A unilateral total lobectomy and which decreases blood calcium
isthmusectomy is adequate for minimal 90% are sporadic cases
(<1cm) lesions
10% are familial and may be associated
Some cancer will be diagnosed following with multiple endocrine neoplasia type 2a
lobectomy for a solitary nodule, especially and 2b (see below)
follicular tumours that cannot be identified
Familial cases are associated with
on FNAC. In such cases where frankly
mutations of the ret proto-oncogene if
invasive cancer has been demonstrated,
mutation is present, 100% risk of
completion total thyroidectomy is required
developing medullary carcinoma and
There is no evidence to support radical therefore prophylactic thyroidectomy is
and mutilating block dissection of the indicated in childhood
neck except in the case of extrathyroidal
Treatment is radical surgery with
papillary carcinomas
follow-up using sequential calcitonin
Postoperative treatment includes: (1) 131I assays
scan to exclude residual thyroid tissue/
Anaplastic
metastases. Total uptake should be <1%.
Begins with A = aged = occur in the
If it is greater, then therapeutic doses of
131 elderly
I are administered until all residual
uptake is ablated. (2) Lifelong thyroxine Resection of the thyroid is rarely possible
for replacement and to suppress TSH Treatment with radiotherapy and
levels <0.1IU, as this favourably doxorubicin gives best survival of 1 year
22 Superficial lesions Case 10

Lymphoma Pituitary adenoma


Tru-Cut biopsy is often needed for Primary hyperparathyroidism
diagnosis Multiple endocrine neoplasia type 2a (Sipple
Treated with radiotherapy and syndrome):
chemotherapy. Autosomal dominant
(b) What do you know about thyroid cysts? Phaeochromocytoma
Medullary carcinoma of the thyroid
True cysts with a completely smooth wall are
very rare Primary hyperparathyroidism
Most are composite lesions with colloid Multiple endocrine neoplasia type 2b:
degeneration, necrosis or haemorrhage in Same as IIa but no parathyroid involvement.
benign or malignant tumours
Only benign if completely abolished by
John H. Sipple (born 1930). North American Professor
aspiration
of Medicine, New York.
Note that cytology can be false-negative in a
P. Wermer (1898-1975). Contemporary North
third of malignant cysts.
American Physician, Columbia University, Presbyterian
Hospital, New York
Notes

Multiple endocrine neoplasia type 1 (Wermer FURTHER READING


syndrome):
Sadler GP: The thyroid gland. In: Lennard TWJ,
Autosomal dominant editor: Endocrine surgery, London, 2006,
Pancreatic islet cell tumour Elsevier Saunders, pp 4377.

CASE 10 MULTINODULAR GOITRE ***

INSTRUCTION OTHER POINTS ON SYSTEMIC


See Cases 6 and 8 for the general examination EXAMINATION
of the neck and thyroid gland. Feel the pulse atrial fibrillation is seen in
40% of patients with multinodular goitre.
SPECIFIC POINTS ON
EXAMINATION OF THE NECK QUESTIONS
With a large multinodular goitre it should be (a) What are the features of a multinodular
relatively easy to palpate the thyroid as the goitre (MNG)?
patient swallows some water:
Describe the features of the lump: Progression of simple diffuse goitre to
nodular enlargement
Multinodular
Middle-aged women
May be large in size
Positive family history. Malignant change
There may be one nodule which is more
occurs in 5% of untreated MNGs
prominent than the others
Overactivity in parts of an MNG may lead to
Check the position of the trachea, which
mild hyperthyroidism (Plummers syndrome)
may be deviated by a large multinodular
goitre, and percuss for retrosternal No ophthalmic features are seen (these are
extension. characteristic of Graves disease, see
Case 11).
Case 10 Superficial lesions 23

(b) How would you manage a multinodular greater risk of damage to recurrent
goitre? laryngeal nerves and parathyroids.
After taking a history and performing a (d) What are the indications for surgery?
clinical examination, most patients do not
need any intervention The five Ms
The patient usually presents because of: Mechanical obstructive symptoms
Cosmetic reasons, or they have noticed a Malignancy
lump in their neck Marred beauty cosmetic reasons
Discomfort Medical treatment failure thyrotoxicosis
Tracheal compression causing Mediastinal (retrosternal) extension unable
shortness of breath to perform FNAC or monitor change
Oesophageal compression causing clinically.
dysphagia
Worries about malignancy ADVANCED QUESTIONS
Onset of hyperthyroidism
Investigate if: (a) How can you tell the difference between
Prominent nodule toxic multinodular goitre and Graves disease?
Features suspicious of malignancy, such (See Table 3).
as cervical lymphadenopathy or recurrent
laryngeal nerve palsy
Investigate using:
Thyroid function tests hyperthyroid?
H. S. Plummer (18741936). North American
Ultrasound dimensions of goitre and physician. Also described:
nodules, look for dominant nodules/cysts
for FNAC Plummer nails: concave or ragged edge to the
nail-bed seen in early onycholysis occurring in
Chest X-ray a retrosternal goitre may
thyrotoxicosis (most prominent in fourth and fifth
compress the trachea.
fingers)
(c) How would you treat a multinodular Plummer sign: inability of patient to sit in a chair as
goitre? a result of thyrotoxic myopathy
Non-surgical: Plummer treatment: the use of iodine to treat
Remove goitrogens, e.g. remove cabbage thyrotoxicosis
from diet PlummerVinson syndrome: iron deficiency
Thyroxine 0.10.3mg/day causes anaemia associated with dysphagia and post-
regression in 5070% of patients cricoid oesophageal webs in middle-aged women.
probably because multinodular goitres (Note that this is the North American variation
increase in size as a result of raised this syndrome is known as the PatersonBrown
thyroid-stimulating hormone levels Kelly syndrome in the UK and the
If thyrotoxicosis, treat as in Graves WaldenstromKjellberg syndrome in Scandinavia!)
disease (see Case 11)
Aspiration of cysts with cytology to
exclude malignancy (see Fig. 14)
Radioiodine for elderly patients, Table 3
particularly those unfit for surgery
Surgical: Toxic multinodular Graves disease
Bilateral subtotal thyroidectomy without goitre
need for postoperative replacement of Older age-group Younger age-group
thyroxine Nodular enlargement Diffuse enlargement
More recently, total thyroidectomy is the Eye signs not present Eye signs present
preferred procedure due to the risk of
pathological change in the thyroid Atrial fibrillation present in Atrial fibrillation
remnant (malignancy/increasing in size, 40% of patients uncommon
further nodularity) necessitating further No associated Autoimmune diseases
re-do surgery, which carries a much autoimmune diseases commonly associated
24 Superficial lesions Case 11

Huysmans D, Hermus A, Edelbroek M, et al:


FURTHER READING
Radioiodine for nontoxic multinodular goitre,
Hisham AN, Azlina AF, Aina EN, et al: Total Thyroid 7(2):235239, 1997.
thyroidectomy: the procedure of choice for
multinodular goitre, Eur J Surg 6:403405, 2001.

CASE 11 DIFFUSE THYROID ENLARGEMENT ***

INSTRUCTION Defects of thyroid hormone production


(rare).
See Cases 6 and 8 for the general examination
of the neck and thyroid gland. (c) What are the features of Graves disease?
Commoner in females (9:1)
SPECIFIC POINTS ON Results from polyclonal immunoglobulins
against thyroid-stimulating hormone receptor
EXAMINATION OF THE NECK which bind and stimulate the receptor
Describe the features of the lump: these antibodies are found in 90% of
Diffuse enlargement (not nodular) patients
May be large in size Hyperthyroidism with goitre
Non-tender Thyroid eye disease (see Case 8)
Gently palpate the trachea for deviation (from Thyroid acropachy
a large goitre), and ask the patient if she has Pretibial myxoedema
had any problems swallowing (from a large Normochromic normocytic anaemia, raised
goitre) or noticed any change in her voice erythrocyte sedimentation rate and
Remember to percuss over the sternum for a hypercalcaemia can also occur
retrosternal extension of a large goitre. Associated with other autoimmune
conditions such as Type 1 diabetes and
pernicious anaemia.
QUESTIONS
(d) How do you treat Graves disease?
(a) What are the causes of a diffusely
Medical:
enlarged thyroid gland?
Antithyroid drugs, e.g. carbimazole,
Simple colloid goitre methimazole, propylthiouracil to inhibit
Graves disease thyroid peroxidase
Thyroiditis (Hashimotos, de Quervains or Beta-blockers, e.g. propanolol to reduce
Riedels, see Case 8). the effects of excess circulating thyroxine
on the cardiac system
(b) What do you know about simple Radioiodine:
colloid goitres? Treatment of choice (only absolute
Commonest form of thyroid abnormality contraindications are pregnancy and
Secondary to hyperplasia of the gland to lactation)
meet physiological demand for thyroxine Single oral dose of 131I causes direct
Secondary to defective production of thyroid radiation damage to the replication
hormone mechanisms of thyroid follicular cells
Causes are as follows: Risks include early hyperthyroidism, late
Iodine deficiency (commonest cause hypothyroidism and late
worldwide) hyperparathyroidism
Increased physiological demand Surgery:
puberty, pregnancy and lactation Particularly useful for: patients who refuse
(commonest cause in the UK) radiation therapy or relapse after an
Goitrogens (less common) uncooked adequate course, pregnant patients or
cabbage, lithium and anti-thyroid drugs those wishing to become pregnant within
Case 12 Superficial lesions 25

4 years, patients under the age of 40 years Table 4 NO SPECS classification of thyroid eye
and those with nodular or large goitres disease
Bilateral subtotal thyroidectomy leaving Class 0 N No signs or symptoms
behind approx. 4g of thyroid tissue on
each side of the trachea
Class 1 O Only signs of upper lid retraction
and stare, with or without lid lag
Increasingly, total thyroidectomy is
preferred, as this removes the possibility and exophthalmos
of recurrent disease, appears to improve Class 2 S Soft-tissue involvement
the outcome for patients with significant Class 3 P Proptosis
eye disease and eliminates the need for
Class 4 E Exophthalmos
annual TFT monitoring to assess remnant
function. It does, of course, demand Class 5 C Corneal involvement
thyroxine replacement therapy. Class 6 S Sight loss due to optic nerve
involvement
(e) What are the complications
of thyroidectomy?
Complications of thyroidectomy can be divided
into those that are general to any operation (e.g. ADVANCED QUESTIONS
risks of anaesthesia) and those that are specific
(a) What is the pathology of thyroid eye
to thyroidectomy alone. They can also be
disease?
divided into immediate (within 24h), early (within
30 days) and late (after 30 days) they (mostly) Exophthalmos is secondary to retroorbital
begin with the letter H: inflammation and lymphocytic infiltration,
Immediate: leading to oedema and an increase in
Haemorrhage, leading to airway retrobulbar orbital contents
obstruction from secondary laryngeal Lid lag is secondary to sympathetic
oedema; patients who have recently had overstimulation and restrictive myopathy of
a thyroid operation should have a pair of levator palpebrae superioris.
suture cutters by their bed if this
complication occurs the sutures should (b) How do you classify the severity of thyroid
immediately be removed and an eye disease?
anaesthetist called Use Werners mnemonic NO SPECS (see
Hoarseness from damage to the recurrent Table 4).
laryngeal nerve
Hyperthyroidism severe and is known
as thyroid storm R. J. Graves (17971853). Irish physician, Dublin.
Early:
(H)infection a rather weak H!
Hypoparathyroidism, leading to
Hypocalcaemia
FURTHER READING
Late: Weetman AP: Graves disease. N Engl J Med
Hyperthyroidism recurrent 343(17):236248, 2000.
Hypothyroidism
Hypertrophic scarring.

CASE 12 THYROID HISTORY ***

INSTRUCTION APPROACH
This lady is complaining of a swelling in her It is important to ascertain the symptoms arising
neck. Ask her a few questions about her thyroid from the swelling, the thyroid status, other
gland. associated symptoms and any relevant medical
history.
26 Superficial lesions Case 13

Table 5 Hyper- and hypothyroidism


Hyperthyroidism Hypothyroidism
General Increased appetite but loss of weight Decreased appetite and gain in weight, lethargy
Thermoregulatory Preference for cold Preference for hot weather
Dermatological Increased sweating Dry skin, peaches and cream complexion, loss
of hair especially outer third of eyebrows
Musculoskeletal Proximal myopathy (autoimmune) Muscle fatigue
with wasting and weakness
Gastrointestinal Change in bowel habit, particularly Constipation
diarrhoea and frequent defaecation
Cardiovascular Tachycardia, atrial fibrillation Bradycardia
Gynaecological Oligomenorrhoea, amenorrhoea Menorrhagia
Psychiatric Nervousness, easy irritability, Slow thought, speech and action, depression,
emotional lability, insomnia, psychosis dementia
Neurological Fine tremor Symptoms of carpal tunnel syndrome (see
Case 79)

Symptoms arising from the swelling Other associated symptoms

Duration and change in size note Ask about eye symptoms, e.g. protruding or
particularly if the swelling has suddenly staring eyes, difficulty closing eyelids, double
increased in size (can occur if there is vision (secondary to ophthalmoplegia) and
haemorrhage into a necrotic nodule, pain in the eye (secondary to corneal
subacute thyroiditis or a rapidly growing ulceration).
carcinoma)
Cosmetic symptoms Relevant medical history
Discomfort during swallowing/dysphagia
oesophageal compression Previous operations on the thyroid gland
Dyspnoea (tracheal compression) Previous or current medication, e.g.
Hoarseness due to recurrent laryngeal antithyroid drugs, thyroxine, iodine-
nerve paralysis secondary to malignant containing medications
infiltration Radioiodine therapy for previous Graves
Pain not common but can occur in disease (eye signs may persist)
thyroiditis or anaplastic carcinoma. Move on to assessing fitness for surgery if
relevant and time permits.
Thyroid status

Table 5 shows the symptoms of hyper- and


hypothyroidism.

CASE 13 HYPERTROPHIC AND KELOID SCARS ***

INSTRUCTION APPROACH
No specific instruction. Your description is likely to be based solely on
inspection.
Case 13 Superficial lesions 27

Table 6 Table 7

Features Hypertrophic Keloid scars Features Hypertrophic scars Keloid scars


scars Age Any age (commonly Puberty to 30
Appearance Scar confined to Scar extends 820 years) years
wound margins beyond wound Gender M=F F>M
margins Race All races Black and
Site Across flexor Earlobes, chin, Hispanic races
surfaces and neck, shoulder,
skin creases chest

VITAL POINTS (d) How do you treat these scars?


Recurrence can be as high as 55% with surgical
The scar can be on any part of the body
revision alone, and therefore a combination of
where there has been an incision in the skin
the treatments outlined below is often
Describe the scar point out that the scar employed:
area is more prominent than the surrounding
skin and add details as in Table 6. Non-surgical: mechanical pressure therapy
(day and night for up to 1 year), topical
silicone gel sheets
Finish your examination here Surgical: revision of scar with closure by
direct suturing, local Z-plasty or skin grafting
to avoid excessive tension
Completion Intralesional steroid and local anaesthetic
injections: using triamcinolone in
Say that you would like to ask the patient: combination with lignocaine.
How the scar affects their lives, e.g.
cosmetic symptoms. ADVANCED QUESTIONS
(a) What other associations have been
QUESTIONS described with keloid and hypertrophic scars?
(a) What do you know about the epidemiology (See Table 8).
of hypertrophic and keloid scars?
(See Table 7).
Jean Louis Albert coined the name keloid in 1806 in
(b) What types of wounds are prone to order to describe this overhealing phenomenon,
hypertrophic and keloid scar formation? although they had been originally described in the
Wounds associated with: Smith Surgical Papyrus (2500 BC).
Infection
Trauma
Burns FURTHER READING
Tension, especially over the sternum, such
Wolfram D, Tzankov A, Plzl P, et al:
as after coronary artery bypass grafting
Hypertrophic scars and keloids a review of
Wounds on certain areas of the body, see
their pathophysiology, risk factors, and
Table 6.
therapeutic management. Dermatol Surg
(c) Is there a difference in the clinical course 35(2):171181, 2009.
of hypertrophic and keloid scars?
Hypertrophic scars tend to appear soon after
injury and usually regress spontaneously, while
keloid scars appear months after injury and
continue to grow.
28 Superficial lesions Case 14

Table 8

Associations Hypertrophic scars Keloid scars


Biochemical Normal rate of collagen synthesis but increased Increased rate of collagen synthesis
breakdown of collagen by collagenase activity (increased proline hydroxylase activity)
and increased collagenase activity
Genetic Not proven Significant predisposition in Black and
Hispanic races
Oxygen levels Relative hypoxia due to wound tension? No link
Immunology May be important, but no specific associations Increased IgG, IgM and C3 levels, and
known antinuclear antibodies to keloid fibroblasts

CASE 14 SQUAMOUS CELL CARCINOMA **

INSTRUCTION Completion
Examine this gentlemans face.
Say that you would like to ask the patient about:
Predisposing factors (see below)
APPROACH How the lesion affects his life, e.g. cosmetic
symptoms.
Sit or kneel in front of the patient in order to be
at the same level as his face, and examine as
for any lump. QUESTIONS
(a) What is your differential diagnosis?
VITAL POINTS
Benign skin lesions:
Inspect Keratoacanthoma
Infected seborrhoeic wart
May occur on any part of the face (usually in
areas of sun-exposed skin where skin looks Solar keratosis
weathered) Pyogenic granuloma
Appears vascular (redbrown) Malignant skin lesions:
Raised and everted edge Basal cell carcinoma
May be of considerable size (>1cm) Malignant melanoma (amelanotic).
There may be erosion of the facial
architecture if the tumour is advanced
(b) What are the predisposing factors for
squamous cell carcinomas (SCC)?
May have central ulceration.
Congenital:

Palpate Xeroderma pigmentosum (see Case 16)


Acquired:
Regional cervical lymphadenopathy (may be Environmental agents, e.g. sunlight, ionizing
due to metastases or secondary infection radiation, industrial carcinogens such as
only 5% have metastasized by the time of arsenic
presentation) (see Case 7). Pre-existing skin lesions, e.g. solar keratosis
(see Case 27), Bowens disease (see below)
Finish your examination here Infections, e.g. viral warts (human papilloma
virus 5 and 8)
Immunosuppression, e.g. in anti-rejection
treatment post-transplant and in HIV
infection may develop multiple SCCs
Case 15 Superficial lesions 29

Chronic cutaneous ulceration, e.g. chronic are seen to extend in all directions into the deep
burns, chronic venous ulcers (Marjolins dermis and subcutaneous fat. The tumour itself
ulcer). may be well-differentiated (with production of
keratin), moderately-differentiated or poorly-
(c) What treatment options are available differentiated.
for SCC?
Primary lesion:
Professor J. T. Bowen (18571941). American
Excision with 1cm margin
dermatologist. Bowens disease is an intraepidermal
Mohs staged chemosurgery with
carcinoma presenting as a single brown-red irregular
histological assessment of margins and
plaque usually on the trunk that increases in size and
electrodesiccation for lesions of the
may progress to invasive SCC. The condition is also
eyelids, ears and nasolabial folds
associated with subsequent development of visceral
Radiotherapy for unresectable lesions malignancies, usually 57 years later, particularly if
Nodal spread: the affected area of skin has never been exposed to
Surgical block dissection if palpable nodes the sun. Excision with at least a 0.5cm margin is
or in cases of Marjolins ulcers but the recommended. Newer treatment options include
benefit of prophylactic block lymph node topical diclofenac, topical imiquimod and photody-
dissection with Marjolins ulcers is not namic therapy. When seen on the penis, vulva or oral
proven cavity, it is known as Erythroplasia of Queyrat (French
Radiotherapy. dermatologist c.1900).
R. Marjolin (18121895). French surgeon.

ADVANCED QUESTIONS
(a) What do you know about the pathology FURTHER READING
of SCC?
Garcia-Zuazaga J, Olbricht SM: Cutaneous
The tumour arises from epidermal cells that squamous cell carcinoma. Adv Dermatol
normally migrate to the skin surface to form 24:3357, 2008.
the superficial keratinizing squamous layer. www.britishskinfoundation.org.uk/standard.
Full-thickness epidermal atypia is seen (vs basal aspx?id=90 general information about SCCs.
atypia only in solar keratosis) and tumour cells

CASE 15 MALIGNANT MELANOMA **

Presence of naevi is the most common


INSTRUCTION
predictor of risk of malignant melanoma
Examine the lesion on this ladys right leg. Commonest cancer of young adults aged
between 20 and 39 years
Commoner in women than men.
APPROACH
The four commonest types are:
The patient should already be adequately
exposed. Examine as for any lump (see Case 1). Superficial spreading melanoma
Most common type (70%)
Occurs most often on the legs of women
VITAL POINTS and the backs of men
Inspect Red, white and blue in colour
Irregular edge
Found most commonly on the legs of young Usually palpable but thin.
women and the trunk of middle-aged men,
but location and characteristics depend Nodular melanoma
on type Second most common type (1530%)
Occurs most often on the trunk
30 Superficial lesions Case 15

Polypoid in shape and is raised bleeding, change in colour, shape or


Smooth surface thickness
Irregular edge Ask the patient about predisposing factors
Frequently ulcerated. (see below).

Lentigo maligna melanoma


Arises in a lentigo maligna (Hutchinsons QUESTIONS
melanotic freckle see below)
(a) What is your differential diagnosis?
Occurs most often on the face or dorsum of
the hands and forearms Benign skin lesions:
Underlying lesion is flat and brown-to-black Moles: increased numbers of melanocytes
in colour with an irregular outline producing too much melanin (also called
Malignant area in the lesion is usually thicker, pigmented naevus)
and darker in colour. Freckles: normal numbers of melanocytes
but each producing too much melanin
Acral lentiginous melanoma
Lentigo: increased numbers of melanocytes
Least common producing normal amounts of melanin
Occurs on hairless skin (such as subungual Pigmented seborrhoeic keratoses
area, and palms of hands and soles of feet),
Dermatofibromas (see Case 33)
and is more common in Oriental and Black
races Thrombosed haemangiomas
Irregular area of brown or black Malignant skin lesions:
pigmentation. Pigmented basal cell carcinomas
There are other rarer types of melanoma (e.g. (see Case 16).
amelanotic melanoma, with no pigmentation
and a poorer prognosis) but these are less likely
(b) What are the predisposing factors for
to be encountered in the clinical cases. It is also malignant melanomas?
possible to have intracranial melanoma as there Congenital:
is melanin in the substantia nigra, and also in Xeroderma pigmentosum (see Case 16)
the retina. Dysplastic naevus syndrome (also known as
B-K mole or FAMM syndrome) risk of
Features of a pigmented skin lesion suspicious developing malignant melanoma is 100% if
of malignancy two family members are affected
Loss of normal surface markings around the lesion Large congenital naevi
(e.g. skin creases) Family history in first-degree relatives
(increased risk by one and a half times)
Presence of ulceration
Acquired:
Evidence of bleeding from the lesion
Sunlight (particularly ultraviolet light):
Marked variation of colour within the lesion especially in fair-skinned people with red hair
Presence of a halo of brown pigment in the skin Pre-existing skin lesions, e.g. lentigo
around the lesion maligna, more than 20 benign pigmented
Presence of satellite nodules of tumour around the naevi (the latter increases the risk three
lesion. times)
Previous melanoma (increases the risk three
and a half times).
Finish your examination here
(c) How do you stage malignant melanomas?
There are two pathological staging systems in
Completion use, both of which have prognostic value. Both
staging systems are based on the depth of
Say that you would like to: invasion of the tumour from the epidermis.

Examine the draining lymph nodes The first is Clarks levels of invasion (Fig. 15),
described in 1969 (Table 9).
Ask the patient about symptoms from the
lesion that may indicate malignancy, e.g. The second is Breslows thickness (Fig. 16),
rapid increase in the size of a mole, itching, described in 1970 (Table 10).
Case 15 Superficial lesions 31

V Subcutaneous fat
IV Reticular dermis
III Papillary/reticular interface
II Papillary dermis
I Intraepidermal

Epidermis
Papillary Epidermis
dermis

Dermis
Reticular
dermis

Subcutaneous fat
Subcutaneous
fat Figure 16 Breslows thickness of malignant melanoma,
Figure 15 Clarks level of melanoma invasion. which relates to thickness of the tumour itself.

Table 9 Table 10

Clarks Extent of tumour 5-year Breslows thickness 10-year survival (%)


level survival (%) <0.76mm 92
I Epidermis only 98 <3mm 50
II Invades papillary dermis 96 <4mm 30
III Fills papillary dermis 94 Lymph node involvement <40 (8-year survival)
IV Invades reticular dermis 78
V Subcutaneous tissue 44
invasion

Breslows thickness is a better prognostic Table 11


indicator because the reticular dermis is not
uniformly thick in different parts of the body. Clinical stage Histopathological stage
A four-stage clinically based system is also in IA <0.75mm or Clark level II
use, which is more accurate for prognosis IB 0.751.5mm or Clark level III
(Beahrs and Myers 1983) (Table 11). IIA 1.54.0mm or Clark level IV
(d) What treatment options are available for IIB >4.0mm or Clark level V
malignant melanoma? III Lymph node metastasis in one
regional drainage area or >5mm
Surgical excision
in transit metastasis
Main lesion:
IV Advanced regional metastasis or
Veronesi has been instrumental in leading distant metastasis
opinion as to the most appropriate surgical
management of melanoma:
32 Superficial lesions Case 16

Lesions <0.76mm excise with a 1cm above. In addition, the following are also known
margin of grossly normal tissue to be indicators of poor prognosis:
Lesions 0.761.0mm excise with a 2cm Increasing age of the patient
margin Male patients
Lesions >1.0mm excise with a 3cm Melanomas on the trunk (especially the
margin back), scalp, hand and foot
Excision should be down to deep fascia Ulceration of the tumour
Nodal spread: Depigmentation and amelanotic melanomas
If clinical suspicion of nodal metastasis, Aneuploidy and high mitotic index.
lymph node biopsy or fine-needle aspiration
cytology (FNAC)
Sir John Hunter (17281793). First described
If palpable lymph nodes, therapeutic block
malignant melanoma in 1787. (See Case 117.)
dissection.
Sir Jonathan Hutchinson (18281913). English
Palliation/adjuvant therapies surgeon, London Hospital and Professor of Surgery,
Immunotherapy, e.g. vaccines to raise an Royal College of Surgeons. He described a flat
anti-melanoma antibody response, pigmented, brown-to-black melanocytic naevus with
monoclonal antibody therapy, cytokine malignant potential that occurs on sun-damaged skin
interferon alfa therapy. on the face, and on the dorsum of the hands and
forearm. The freckle itself represents an increased
Prevention: most important to mention this in
number of melanocytes at the dermoepidermal
your answer
junction. It occurs in the fifth to seventh decades, and
Avoidance of causative factors, e.g. public after a period of time (1030 years), it transforms into
education campaigns to reduce sun a malignant melanoma, heralded clinically by the
exposure. development of a black or tan nodule. Also described
Hutchinson triad (eighth nerve deafness, notched teeth
and interstitial keratitis in congenital syphilis).
ADVANCED QUESTIONS
(a) What do you know about the pathology of
malignant melanoma? FURTHER READING
Bataille V, de Vries E: Melanoma Part 1:
On microscopy, malignant melanomas consist
epidemiology, risk factors, and prevention. BMJ
of loose nests of melanocytes in the basal cell
337:a2249, 2008.
layer which invade the epidermis (leading to
destruction and ulceration) and penetrate Beahrs OH, Myers MH: Manual for staging of
deeper into the dermis and subcutaneous fat. cancer. American Joint Committee Cancer.
Philadelphia, 1983, Lippincott, p 117.
(b) Do you know of any prognostic indicators Thirlwell C, Nathan P: Melanoma Part 2:
for malignant melanoma? management. BMJ 337:a2488, 2008.
Clarks levels, Breslows thickness and the www.skincancerfacts.org.uk information on all
four-stage clinical system have been described types of skin cancers for patients.

CASE 16 BASAL CELL CARCINOMA **

INSTRUCTION APPROACH
Examine this gentlemans face. Sit or kneel in front of the patient in order to be
at the same level as his face, and examine as
for any lump.
Case 16 Superficial lesions 33

VITAL POINTS Finish your examination here

Completion
TOP TIP Say that you would like to:
If you need to see the side of the patients face, Examine for regional lymphadenopathy (but
e.g. ear, stay still while sitting or kneeling and ask the note that metastases are extremely rare,
patient to turn his head to the appropriate side it looks BCCs are locally aggressive)
unprofessional to move back and forth around the patient!
Ask the patient about predisposing factors
(see below).

Inspect QUESTIONS
(a) What is your differential diagnosis?
Occurs on hair-bearing sun-exposed skin of
elderly people, especially around the eye The two main differential diagnoses to consider
Single or multiple are:
Features of basal cell carcinomata (BCCs) Benign keratoacanthoma especially if it is
depend on the clinical type and can be sloughing at its centre (see Case 22)
divided into: Malignant squamous cell carcinoma
Raised above the skin: particularly the nodulo-ulcerative type with a
Nodular/nodulo-ulcerative rolled edge (see Case 14).
Most common type
(b) What are the predisposing factors for
Well-defined rolled, pearly edge basal cell carcinomas (BCC)?
Central ulceration
Congenital (rare):
Cystic
Xeroderma pigmentosum (familial
Large cystic nodule condition associated with failure of DNA
Not raised above the skin: transcription, leading to defective DNA
Pigmented repair) also known as Kaposis disease
Contains melanin (see Cases 16 and 35)
Can be confused with malignant Gorlins syndrome (see below)
melanoma (see Case 15) Acquired (very common):
Sclerosing (also known as morphoeic) Sunlight (particularly ultraviolet light in the
Flat or depressed tumour UVB range)
Ill-defined edge Carcinogens, e.g. cigarette smoke,
May be ulcerated (occurs late) arsenic
Cicatricial (also known as field-fire or Previous radiotherapy
bush-fire) Malignant transformation in pre-existing
Multiple superficial erythematous lesions skin lesions, e.g. naevus sebaceous.
interspersed with pale atrophic areas
(c) What treatment options are available
Superficial
for BCC?
Erythematous scaly patches
Treatment options available are:
Can be confused with Bowens disease
(see Case 14). Tumours raised above the skin: excision with
0.5cm margin (maximum)
Tumours not raised above the skin: wider
Palpate margin of excision, particularly if at inner
canthus of eye, nasolabial fold, nasal floor
Fixation of the BCC deep to the skin is a and ear frozen section may be necessary
sign of deep local invasion. to ensure adequate excision
Other approaches: radiotherapy and Mohs
surgery (see Case 14).
34 Superficial lesions Case 17

ADVANCED QUESTIONS R. J. Gorlin (19232006). American Professor of Oral


Pathology, University of Minnesota. Gorlins syndrome
(a) What do you know about the histology (naevoid basal cell epithelioma syndrome) is an
of BCC? autosomal dominant condition presenting in early
adult life with multiple basal cell carcinomas,
On microscopy, BCCs have many patterns but
keratocysts of the jaw, palmar and plantar pits,
the most common features are islands and
mesenteric cysts and scoliosis.
nests of basaloid cells in the dermis (like those
seen in the basal cell layer of the epidermis).
The cells exhibit high mitotic rates and
peripheral palisading (cell islands arranged
radially with long axes in approximately parallel FURTHER READING
alignment). Often, there is ulceration of the
epidermis. Epstein EH: Basal cell carcinomas: attack of the
hedgehog. Nat Rev Cancer 8(10):743754, 2008.
(b) What do you know of the pathology Gorlin RJ: Nevoid basal cell carcinoma
of BCC? syndrome. Dermatol Clin 13(1):113125, 1995.
The hedgehog signalling pathway is important Roewert-Huber J, Lange-Asschenfeldt B,
in embryological development and is highly Stockfleth E, et al: Epidemiology and aetiology
conserved through evolution. Recently patched, of basal cell carcinoma. Br J Dermatol
a member of the pathway, was found to be 157(Suppl 2):4751, 2007.
important in Gorlins syndrome. Inherited www.skincancerfacts.org.uk information for
patched gene mutations underlie the syndrome, patients on all skin cancers, including BCCs.
in which a key feature is multiple basal cell
carcinomas (BCCs). The gene is also mutated in
sporadic BCCs.

CASE 17 PRESSURE SORES **

INSTRUCTION TOP TIP


Have a look at this ladys heel. A classification of pressure sores
Stage 1: Abnormal area of skin with erythema that will
APPROACH not blanch indicates extravasated blood from
cutaneous capillary beds
Examine as for ulcers (see Case 110). It is
Stage 2: Partial thickness skin loss a shallow abrasion
important to bear in mind when examining the
wound
peripheral vascular system of patients in the
circulatory bay that ulcers may be due to Stage 3: Full thickness skin loss with fat at the base of
pressure necrosis as well as peripheral vascular the wound
disease. Stage 4: Extensive soft-tissue loss through deep fascia,
often with underlying muscle necrosis.

VITAL POINTS ON INSPECTION


You should be able to pick up all of the points
simply from inspection. The American National
Completion
Pressure Ulcer Advisory Panel Classification
should be kept in mind when describing your
findings. Say that you would like to take a history,
looking for predisposing factors (see below).
Case 18 Superficial lesions 35

Carcinomatosis
QUESTIONS
Infection.
(a) Where are pressure sores most
commonly found? ADVANCED QUESTIONS
Pressure sores can occur over any bony
prominence, the commonest areas being: (a) How do you treat this condition?
Sacrum Prophylaxis: regular skin inspection, frequent
Greater trochanter turning of immobile patients (24-hourly),
Heel massage, toileting, the use of special
mattresses and cushions which redistribute
Lateral malleolus
the pressure on at-risk areas
Ischial tuberosity
Non-surgical: optimize tissue perfusion and
Occiput. oxygenation, treat infection as it arises, use
various topical dressings as required and
(b) What conditions increase the risk of
provide nutritional support. Specifically,
developing pressure sores?
vitamin C, zinc and multivitamins should be
Immobility and prolonged bed-rest are the most prescribed. Several other techniques such
important factors, particularly secondary to as hyperbaric oxygen, hydrotherapy and
conditions such as: ultrasound are in use depending on local
Cardiopulmonary disease policy
Trauma Surgical: debridement of dead tissue (which
Neurological disease, e.g. paraplegia often does not require anaesthesia and can
Bone and joint disease be performed by the tissue viability nurse)
and reconstruction using a variety of fascial
Prolonged operative procedures, particularly
and muscle-containing composite flaps, e.g.
if there are intraoperative episodes of
buttock rotation flap for sacral sores.
hypotension
Conditions that slow wound healing can (b) What do you know about the
increase the severity and risk of pressure pathophysiology of pressure necrosis?
necrosis:
Prolonged weight-bearing and mechanical shear
Metabolic disorders: forces act on areas of soft-tissue overlying bony
Diabetes mellitus prominences, leading to both occlusion and
Deficiencies of vitamins and trace metals, tearing of small blood vessels, reduced tissue
e.g. vitamin C, zinc perfusion and ischaemic necrosis.
Drugs:
Steroids
FURTHER READING
Post-chemotherapy (also radiotherapy)
Underlying disease: Reddy M, Gill SS, Kalkar SR, et al: Treatment of
pressure ulcers: a systematic review. JAMA
Tissue hypoxia such as in peripheral
300(22):26472662, 2008.
vascular disease
Renal failure Reddy M, Gill SS, Rochon PA: Preventing
pressure ulcers: a systematic review. JAMA
Jaundice
296(8):974984, 2006.

CASE 18 GRAFTS AND FLAPS **

INSTRUCTION QUESTIONS
You may be shown a patient who has had an
(a) What is a skin graft?
operation involving a skin graft or a flap. It is
important to be aware of the principles involved A skin graft involves the transfer of skin from
and the various types of grafts and flaps that a donor site to a recipient site independent of a
may be encountered. blood supply. The graft takes by acquiring a
36 Superficial lesions Case 19

blood supply from a healthy donor bed. Skin Random or axial: the latter is based on a
grafts may either be full thickness or partial named artery or vein.
thickness, but contain the entire epidermis, with
a portion of the underlying dermis. The dermis (f) What are the indications for flap
does not regenerate, but the epidermis reconstruction?
regenerates from the adnexal elements of Situations where skin grafts will not take (see
skin hair follicles, sebaceous glands and above)
sweat glands within the dermis. When the aim is to reconstruct with tissue
that is like-for-like (bone, joint, tendon,
(b) What tissues do skin grafts not take on? nerve, epithelial lining, etc.) to promote
Unhealthy, necrotic and infected tissue optimal structure, function and cosmesis
Irradiated tissue When blood supply has to be imported to
Exposed cortical bone without periosteum areas of doubtful viability, e.g. pressure
Tendon without peritendon sores, complex trauma.
Cartilage without perichondrium.
(g) What is the reconstruction ladder?
(c) How do you harvest a skin graft? This is the array of plastic surgical
Use hand-held skin graft knives (e.g. Watson reconstruction techniques of increasing
and Braithwaite modifications of the Humby complexity that is available to the surgeon and
knife) or electric- or gas-powered which is used according to their suitability for
dermatomes, the latter producing a graft of individual patients:
even thickness from almost any site, with Healing by secondary intention (i.e.
little expertise needed for operation granulation) and then by primary intention
Donor site is usually one that can be easily (excision and closure) prior to reconstruction
concealed, e.g. inner thigh, buttock or Skin graft
inner arm. Local flap
Distant flap
(d) What is a skin flap?
Composite flap
A skin flap consists of tissue, or tissues,
Island flaps vs pedicled flaps
transferred from one site of the body to another,
Free tissue transfer
while maintaining a continuous blood supply
through a vascular pedicle. Composite neurovascular free tissue transfer.

(e) How do you classify skin flaps?


FURTHER READING
Site: local or distant (also known as a free
flap) Andreassi A, Bilenchi R, Biagioli M, et al:
Classification and pathophysiology of skin
Contents: can contain any tissue capable of
grafts. Clin Dermatol 23(4):332337, 2005.
transfer, including omentum and bowel

CASE 19 PTOSIS **

INSTRUCTION VITAL POINTS


Examine this gentlemans face and tell me your Ptosis is best observed with the patient sitting
diagnosis. up and the head being held by the candidate.

APPROACH TOP TIP


This case is likely to be a spot diagnosis.
Remember that the definition of ptosis is The upper eyelid is raised by the action of levator
drooping of the upper eyelid associated with the palpebrae superioris. This muscle is of dual origin and
inability to elevate the eyelid completely. innervation (a favourite topic for surgical examiners):
Case 19 Superficial lesions 37

Mainly skeletal muscle innervated by the third Finish your examination here
cranial nerve (oculomotor)
A thin sheet of smooth muscle (Mllers muscle)
that is supplied by postganglionic sympathetic Completion
nerve fibres arising from cell bodies in the
superior cervical ganglion
Say that you would like to:
Complete ptosis follows third nerve palsy the eyelid
Take a history from the patient to try to find
droops in all positions
the cause of their ptosis.
Partial ptosis follows an ipsilateral sympathetic nerve
lesion this is Horners syndrome (ptosis, meiosis,
anhydrosis and enophthalmos), which can be QUESTIONS
overcome on asking the patient to look up.
In surgical exams, ptosis is most likely to
be due to Horners syndrome, possibly
secondary to:
Lower brachial plexus injury (Dejerine
Inspect Klumpke paralysis, see Case 100)
Pancoasts tumour of the lung (an apical
Is it unilateral or bilateral? lung carcinoma that invades the cervical
Note whether ptosis is partial or complete by sympathetic plexus, associated with shoulder
asking the patient to look upwards and arm pain due to brachial plexus invasion
Look at the size of the pupil of C8T2, and a hoarse voice or bovine
Small pupil in Horners syndrome (look for cough due to unilateral recurrent laryngeal
other signs of Horners, see above) nerve palsy and vocal cord paralysis).
Large pupil in third cranial nerve palsy
(look at the position of the eye (Fig. 17) (a) What causes of ptosis are you aware of?
down and out in third nerve palsy) and Unilateral:
test the reaction of the pupil to light and Third cranial nerve palsy complete ptosis
accommodation pupil does not react in Horners syndrome partial ptosis
third nerve palsy.
Syphilis
Bilateral:
Congenital ptosis
III III
Myopathies myasthenia gravis, dystrophia
myotonica
Syphilis.

(b) What surgical treatments are available?


VI III A blepharoplasty can be performed: excess
eyelid skin and fat are removed.

IV III
Figure 17 The actions of the IIIrd, IVth and VIth nerves
Henry Pancoast (18751939). Professor of Radiology,
on the eye movements of the right eye. III = oculomotor,
Pennsylvania, USA.
IV = trochlear, VI = abducent.
38 Superficial lesions Case 20

CASE 20 FACIAL NERVE PALSY **

Take a history to determine the duration and


INSTRUCTION
effects of the condition on the patient
Have a look at this ladys face. Examine for taste with salt/sweet solutions
(involvement of the chorda tympani, an
afferent branch of the facial nerve)
APPROACH
Test the patients hearing (hyperacusis can
In a surgical case, think of surgical causes, e.g. result from involvement of the nerve to
parotid gland tumours, old skull fracture these stapedius muscle, an efferent branch of the
result in lower motor neurone palsy of the facial nerve).
seventh cranial nerve (facial nerve).

QUESTIONS
VITAL POINTS
(a) What are the causes of facial nerve palsy?
Inspect systematically
Intracranial:
General: loss of facial expression Vascular cerebrovascular accident
Eyelids: on blinking, the affected side closes Tumour acoustic neuroma
after the normal eyelid (Bells sign the Infection meningitis (rarely)
eyeball moves vertically upwards on the Intratemporal:
abnormal side when the eye is closed) Infection acute and chronic otitis media,
Eyes: widened palpebral fissure herpes zoster (Ramsay Hunt syndrome)
Nasolabial fold: flatter on affected side Idiopathic Bells palsy (see below)
Mouth: the affected side droops and moves Trauma surgical, accidental, e.g. basal
less when talking. skull fracture
Tumour paraganglioma, squamous cell
Test the muscles carcinoma of external or middle ear,
metastases, e.g. breast
involved systematically
Extratemporal:
Occipitofrontalis: raise your eyebrows Tumour parotid gland malignancy
spared in upper motor neurone facial nerve Trauma surgical, accidental, e.g. facial
palsy as the forehead has bilateral cortical lacerations.
representation
Orbicularis oculi: close your eyes as tightly ADVANCED QUESTIONS
as you can
Orbicularis oris: show me your teeth (a) What are the branches of the facial nerve?
Buccinator: puff out your cheeks.
(See Figure 18).
Motor:
Look for an obvious cause Nerve to stapedius
Nerve to posterior belly of digastric
Look for a scar over the parotid gland
Five divisions within the parotid gland
indicating iatrogenic facial nerve damage
temporal, zygomatic, buccal,
Look for parotid gland enlargement mandibular and cervical to supply the
Look in the external auditory meatus for muscles of facial expression (such as
herpes zoster (Ramsay Hunt syndrome, see orbicularis oculi, buccinator and
below). orbicularis oris)
Secretomotor via greater superficial
Completion petrosal nerve to lacrimal, nasal and palatine
glands
Say that you would like to: Taste via chorda tympani to anterior
two-thirds of the tongue
Case 20 Superficial lesions 39

Facial nerve
nucleus

Pons Superficial
Lacrimal nasal salivary
and palatine glands Petrous nucleus
bone
Nucleus
solitarius

Greater superficial Internal acoustic


petrosal nerve meatus

Tongue
(taste to
anterior 2/3) Chorda
tympani
nerve
Stylomastoid
foramen
Temporal
Motor Zygomatic
branches to
Buccal
facial muscles
and platysma Masseter Parotid gland
Cervical

Figure 18 Branches of the facial nerve.

Sensory uncommon sensory component of


FURTHER READING
facial nerve carrying cutaneous impulses
from the anterior wall of the external auditory Sweeney CJ, Gilden DH: Ramsay Hunt
meatus known as nervus intermedius or pars syndrome. J Neurol Neurosurg Psychiatry
intermedia of Wrisberg. 71(2):149154, 2001.
Tiemstra JD, Khatkhate N: Bells palsy:
diagnosis and management. Am Fam Physician
76(7):9971002, 2007.

Sir Charles Bell (17741842). Scottish physiologist Protection of the eye during sleep, wearing dark
and Professor of Surgery, Edinburgh, who founded the glasses during the day and use of artificial tears
Middlesex Hospital and Medical School, London. He High-dose prednisolone to reduce nerve oedema
described rapid, unilateral facial weakness associated which prevents weakness becoming paralysis
with or preceded by an ache below the ear which used if presentation is within a few days of onset.
worsens for 12 days and then resolves spontane-
J. Ramsay Hunt (1874 1937). Professor of Neurology,
ously within a few days in the majority (85%) of
Columbia University, New York. He described
cases. Treatment involves the following:
involvement of the facial geniculate ganglion with
Physiotherapy (massage, electrical stimulation, herpes zoster resulting in lower motor neurone facial
splint to prevent drooping of the lower part of the nerve palsy, severe ear pain and visible vesicles in the
face) external auditory meatus, on the eardrum, on the soft
palate or in the tonsillar fossa.
40 Superficial lesions Case 21

CASE 21 SALIVARY GLAND SWELLINGS **

INSTRUCTION Palpate from behind


This gentleman is complaining of a swelling on
Walk behind the patient and enquire about
his right cheek. Examine it and tell me what you
tenderness before palpating the swelling
think.
Is the swelling unilateral or bilateral?
Is it fixed to the skin or underlying muscle?
APPROACH Ask the patient to clench his teeth, which
tenses the masseter and makes the anterior
Sit or kneel in front of the patient in order to be
border of the parotid gland more prominent
at the same level as his face, and examine as
Examine for other features as for any lump
for any lump (see Case 1).
(see Case 2) see below for features of
malignancy
VITAL POINTS Continue on to look for cervical
lymphadenopathy using the up-and-down
Inspect routine (see Case 6).

Swelling in the region of the parotid gland


(which lies wedged between the
Other tests
sternocleidomastoid muscle and the
mandible) (Fig. 19) and the submandibular These tests may be described in an examina-
gland (at the angle of the jaw, wedged tion you are unlikely to be asked to perform all
between the mandible and mylohyoid) of them:
Look for scars and the opening of a fistula Look inside the mouth with a pen torch at
(the latter can occur following parotidectomy the opening of the parotid duct (Stensens
or long-standing parotid traumatic injury) duct), which can be found opposite the
Stand back and look for facial asymmetry second upper molar, and at the opening of
which occurs if the seventh nerve is involved the submandibular duct (Whartons duct) on
with a parotid lesion (see Case 20). the floor of the mouth adjacent to the
frenulum linguae look for inflammation and
pus, or the presence of a stone

External auditory
meatus
Zygoma

Parotid duct
Mastoid process

2nd upper molar


tooth
Branches of the
facial nerve Stensen's duct
(opening of parotid duct)

Mandible
Parotid gland

Sternocleidomastoid Masseter
muscle muscle
Figure 19 Anatomy of the right parotid gland.
Case 21 Superficial lesions 41

Palpate the parotid duct and submandibular


duct openings wearing a pair of gloves, e.g.
QUESTIONS
presence of stone (a) What is the differential diagnosis of a
Palpate the submandibular gland bimanually unilateral swelling of the parotid gland?
with a finger in the mouth and another finger
below the angle of the jaw. (See Table 12).

(b) How would you diagnose a benign parotid


Finish your examination here tumour?
These are adenomas and there are several
varieties, the two most important of which are
Completion pleomorphic adenoma (commonest) and
Warthins tumour (second commonest) (Table 13)
Say that you would like to: Investigations:
Test the facial nerve (see Case 20) which Fine-needle aspiration cytology for diagnosis
may be involved in malignant parotid
MRI to exclude deep lobe involvement.
tumours
Surgical treatment is superficial parotidectomy
Perform a full ear, nose and throat
(if superficial lobe of gland only involved) or total
examination.
parotidectomy with preservation of the facial
nerve (if deep lobe of gland or both lobes
involved).

Table 12

Arising inside the parotid gland Arising outside the parotid gland
Neoplasia Soft-tissues
Benign, e.g. pleomorphic adenoma Lipoma, sebaceous cyst
Malignant tumours of the parotid gland Dental origin
Lymphoma and leukaemia* Infection
Stones Muscular origin
Sialolithiasis Hypertrophy of masseter muscle
Infection/inflammation Bony origin
Mumps* Winged mandible
Acute sialadenitis Transverse process of atlas/axis
Chronic recurrent sialadenitis Neoplasia
Human immunodeficiency virus salivary gland disease Infratemporal fossa and
Autoimmune parapharyngeal tumours
Sjgrens syndrome*
Infiltration
Sarcoidosis*
Lymph node origin
Parotid lymph node enlargement
Neural origin
Facial nerve neuroma
Vascular origin
Temporal artery aneurysm
Systemic diseases
Alcoholic liver cirrhosis
Diabetes mellitus
Pancreatitis
Acromegaly
Malnutrition
*Can present as bilateral swellings.
42 Superficial lesions Case 21

Table 13 Other investigations include Schirmers test


for xerophthalmia (strip of filter paper
Pleomorphic adenoma Warthins tumour inserted into each fornix and hyposecretion
<50 years old >50 years old confirmed by wetting of less than 5mm
Smoking important in 5min normal is 15mm), slit-lamp
risk factor examination of the cornea and lip biopsy for
histological examination of the minor salivary
Tail of parotid, superficial Tail of parotid, glands
to upper part of superficial to upper Treatment involves the use of artificial tears
sternomastoid part of sternomastoid and saliva, use of systemic steroids and
Facial nerve rarely involved Facial nerve rarely careful follow-up due to increased risk of
involved lymphoma development.

(c) What clinical features would make you ADVANCED QUESTIONS


suspect that a parotid swelling is malignant
in nature? (a) What are the complications
Rapid growth and pain (on history) of parotidectomy?
Hyperaemic hot skin Specific complications include:
Hard consistency Immediate
Fixed to skin and underlying muscle Facial nerve transection (intraoperative)
Irregular surface or ill-defined edge Reactionary haemorrhage
Facial nerve involvement. Early
Wound infection
(d) What is Sjgrens syndrome?
Temporary facial weakness (neurapraxia)
Autoimmune condition 90% occur in
Salivary fistula
women at an average age of 50 years
Division of the greater auricular nerve
Intermittent or constant swelling of one or all
loss of sensation to the pinna
of the salivary glands
Late
Clinical diagnosis if at least two of the
following triad is present: Wound dimple
Keratoconjuctivitis sicca (dry eyes) Freys syndrome (auriculotemporal
syndrome) increased sweating of
Xerostomia (dry mouth)
the facial skin when eating, due to
Associated connective tissue disorders
reinnervation of divided sympathetic
such as rheumatoid arthritis (50% of
nerves to the facial skin by fibres of the
cases), scleroderma, systemic lupus
secretomotor branch of the
erythematosus, polymyositis or
auriculotemporal nerve.
polyarteritis nodosa
If no associated connective tissue disorders
are present, this is known as primary
Sjgrens disease (note that Mikulicz L. Frey (18891944). Polish physician, Warsaw. She
syndrome is enlargement of the salivary and was killed by the Nazis.
lacrimal glands secondary to sarcoidosis,
J. von Mickulicz-Radecki (18501905). Professor of
lymphoma or tuberculosis, associated with
Surgery, Breslau, Germany.
dry mouth and dry eyes, but no arthritis)
O. W. A. Schirmer (18641917). German
Pathology is lymphocyte-mediated
ophthalmologist.
destruction of the exocrine glands secondary
to B-cell hyper-reactivity and associated loss H. S. C. Sjgren (18991986). Professor of
of suppressor T-cell activity Ophthalmology, Gothenburg, Sweden.
Patients are at 40 increased risk of N. Stensen (16381686). Professor of Anatomy,
developing lymphoma, usually B-cell Copenhagen, Denmark.
non-Hodgkins type A. S. Warthin (18661931). Professor of Pathology,
Several antibodies present, e.g. anti-salivary Ann Arbor, Michigan, USA.
antibodies, rheumatoid factor, but two
T. Wharton (16141673). English physician, St
specific antibodies present anti-SSA-Ro
Thomass Hospital, London, UK.
and anti-SSB-La
Case 22 Superficial lesions 43

The only known risk-factor for salivary gland


FURTHER READING
tumours is exposure to radiation
Chandana SR, Conley BA: Salivary gland The most common malignant salivary gland
cancers: current treatments, molecular tumour is the mucoepidermoid tumour,
characteristics and new therapies. Expert Rev which is most common in the parotid gland
Anticancer Ther 8(4):645652, 2008.
The most common malignant salivary gland
tumour occurring in the submandibular
Notes on salivary gland tumours gland, sublingual gland and the minor
salivary glands is adenoid-cystic carcinoma
80% of salivary gland tumours occur in the The treatment of malignant salivary gland
parotid gland, 80% of these parotid tumours tumours involves total excision of the
being benign, with 80% of these benign involved gland with preservation if possible
tumours being pleomorphic adenomas of associated nerves (facial nerve in the case
In contrast, only 10% of salivary gland of the parotid gland and the lingual or
tumours occur in the submandibular gland, hypoglossal nerves in the case of the
with only 60% of these submandibular submandibular gland) unless there is direct
tumours being benign (i.e. submandibular infiltration of the nerve by tumour. Adjuvant
gland tumours are twice as likely to be radiotherapy may also be used.
malignant)

CASE 22 KERATOACANTHOMA **

INSTRUCTION How the lump affects their lives, e.g.


cosmetic symptoms.
Examine this gentlemans face.

QUESTIONS
APPROACH
(a) What is a keratoacanthoma?
Examine as for any lump (see Case 1).
A keratoacanthoma is a benign overgrowth of
hair follicle cells that produces a central plug of
VITAL POINTS keratin. It is rapidly growing, forming within 6
Found on sun-exposed parts of the body weeks and regressing after 6 weeks, leaving a
Commoner in males. depressed scar. Clinically and cytologically, they
may look similar to well-differentiated squamous
cell carcinomas. Occasionally, rapidly growing
Inspect malignant melanomas may appear similar.

Dome-shaped with central crater (containing (b) How would you treat this condition?
keratin) Non-surgical: leave alone if asymptomatic
Normal skin colour (except for the central (particularly in young patients)
core which is brown or black due to keratin). Surgical: complete excision of lesion with
histology (particularly in elderly patients
Palpate where there should be a high index of
suspicion for squamous cell carcinoma).
Firm consistency (except for the central core
which is hard) FURTHER READING
Fully mobile over deep tissues (as they occur
in the skin). Schwartz RA: Keratoacanthoma: a clinico-
pathologic enigma. Dermatol Surg 30(2 Pt 2):
326333, 2004.
Completion

Say that you would like to ask the patient:


44 Superficial lesions Case 23

CASE 23 NEUROFIBROMA **

INSTRUCTION (b) What is neurofibromatosis?


This is the presence of multiple neurofibromas
No specific instructions neurofibromata can
in a patient, in combination with other
occur on any part of the body.
dermatological manifestations (six caf-au-lait
spots). It is an autosomal dominant condition.
APPROACH There are thought to be two types of
neurofibromatosis:
Examine as for any lump (Case 2).
Type 1 (von Recklinghausens disease)
defective gene on chromosome 17
VITAL POINTS Type 2 (known as MISME syndrome
Multiple inherited schwannomas,
Inspect meningiomas and ependymomas no relation
to NF1) defective gene on chromosome 22
May be solitary or multiple (the latter being with variable penetrance. Cutaneous signs
known as neurofibromatosis see below) are less often seen in this type.
Pedunculated nodules
(c) What complications can neurofibromata
If arising from deeper nerves, can result in
severe deformity due to diffuse enlargement give rise to?
of the peripheral nerve with involvement of Pressure effects, e.g. spinal cord and nerve
the skin (plexiform neurofibroma) root compression
Look for associated caf-au-lait spots in Deafness with involvement of the VIIIth
neurofibromatosis (light brown macules cranial nerve
which are greater than 1.5cm in diameter Sarcomatous transformation: occurs only in
six or more suggest a diagnosis of von Recklinghausens disease in 513% of
neurofibromatosis) if you see one or two, cases
ask the patient to point out any others he or Intra-abdominal effects: obstruction, chronic
she may have anywhere else. gastrointestinal bleeds
Skeletal changes: can cause kyphoscoliosis,
Palpate cystic changes and pseudoarthrosis.

(d) How would you treat a patient with a


Soft (fleshy) in consistency.
single neurofibroma?
Non-surgical: leave alone if asymptomatic
Completion and if patient does not want intervention
Surgical: indicated only if malignant growth
Say that you would like to ask the patient:
suspected; post-excision, local regrowth is
How the lump(s) affects their lives, e.g. common as neurofibromata cannot be
cosmetic symptoms surgically detached from the underlying nerve.
If multiple neurofibromata, say that you
would like to test the cranial nerves
(particularly the eighth) and measure the ADVANCED QUESTIONS
blood pressure (associated with
phaeochromocytoma).
(a) What is the histological appearance of
a neurofibroma?
Consist of Schwann cells which appear as
QUESTIONS bundles of elongated wavy spindle cells
(a) What is a neurofibroma? Associated with collagen fibrils and myxoid
material
A neurofibroma is a benign tumour derived from
Often not encapsulated (unlike
peripheral nerve elements.
neurilemomas the other common benign
tumour of peripheral nerves which are
always encapsulated).
Case 25 Superficial lesions 45

Friedrich Daniel von Recklinghausen (18331910). FURTHER READING


Professor of Pathology, Strasbourg, Germany. Also Williams VC, Lucas J, Babcock MA, et al:
described von Recklinghausens Disease of Bone Neurofibromatosis type 1 revisited. Pediatrics
(osteitis fibrosa cystica seen in hyperparathyroidism) 123(1):124133, 2009.
and haemochromatosis.

CASE 24 PAPILLOMA **

INSTRUCTION Similar lumps elsewhere


How the lump affects their lives, e.g.
No specific instruction. cosmetic symptoms
Associated conditions there is a link with
APPROACH pregnancy, diabetes and intestinal polyposis.

Examine as for any lump (see Case 1).


QUESTIONS
VITAL POINTS (a) What is a papilloma?
Also known as skin tags or fibroepithelial A papilloma is an over-growth of all layers of
polyps the skin with a central vascular core. They are
Can occur anywhere on the skin, particularly increasingly common with age.
on the neck, trunk, face or anus
Pedunculated swelling (may be sessile) (b) How would you treat a papilloma?
Flesh-coloured The simplest surgical technique is to excise
Soft to palpation. the papilloma with a sharp pair of scissors,
controlling bleeding from the central vascular
component with a single suture. Alternatively,
Completion diathermy can be used to control the bleeding
at the same time as the excision.
Say that you would like to ask the patient
about:

CASE 25 PYOGENIC GRANULOMA **

INSTRUCTION VITAL POINTS


Examine this ladys face. (Pyogenic Inspect
granulomata are found most commonly on the
hands and face in children and young adults, Bright-red or blood encrusted hemispherical
and on the gums and lips in pregnant women.) nodule
May be sessile or pedunculated
APPROACH May be associated with a serous or purulent
discharge
Examine as for any lump (see Case 1).
Can be skin coloured if long standing
(epithelialization).
46 Superficial lesions Case 26

Palpate QUESTIONS
Soft in consistency (fleshy) (a) What is a pyogenic granuloma?
Slightly compressible (due to vascular origin)
A pyogenic granuloma is a rapidly growing
May bleed easily (so palpate only if the capillary haemangioma which usually measures
examiner asks you to). less than 1cm in diameter. It is neither
pyogenic nor a granuloma.
Completion
(b) How would you treat this condition?
Say that you would like to ask the patient: Non-surgical: regression is uncommon,
Whether they can remember a previous except those arising in pregnancy, and so
injury in this area (this association with they are best treated surgically, though
trauma is now thought to be less strong, but occasionally a silver nitrite stick can be
show the examiner that you are aware that attempted
there is thought to be a link between the Surgical: curettage with diathermy of
two) the base or complete excision biopsy
How long the lump took to appear (rapid (if recurrent, consider malignancy,
growth in a few days) e.g. amelanotic melanoma).
How the lump affects their lives, e.g. pain,
cosmetic symptoms, bleeding. FURTHER READING
Giblin AV, Clover AJ, Athanassopoulos A, et al:
Pyogenic granuloma the quest for optimum
treatment: audit of treatment of 408 cases.
J Plast Reconstr Aesthet Surg 60(9):10301035,
2007.

CASE 26 SEBORRHOEIC KERATOSIS **

that bleed slightly (do not attempt to do this


INSTRUCTION
in the exam!).
Have a look at this ladys face.
Completion
APPROACH
Say that you would like to ask the patient:
Examine as for any lump (see Case 1).
About similar lesions elsewhere (note that
sudden onset of multiple seborrhoeic
keratoses is associated with visceral
VITAL POINTS
malignancy this is known as the Leser
Commonly found on the trunk and face but Trlat sign)
can occur anywhere How the lesion affects her life, e.g. cosmetic
Single or multiple symptoms, catches on clothes.
Round or oval in shape
Stuck-on appearance QUESTIONS
Varying degree of pigmentation light brown
to black (in black people, seborrhoeic (a) What is a seborrhoeic keratosis?
keratoses on the face are known as
dermatosis papulosa nigra) A seborrhoeic keratosis is a benign overgrowth
of the basal cell layer of the epidermis.
Surface appears velvety or warty
Histologically, it is characterized by:
Can be picked off the skin, leaving behind
pink skin and one or two surface capillaries Hyperkeratosis (thickening of the keratin
layer)
Case 27 Superficial lesions 47

Acanthosis (thickening of the prickle cell (b) How would you treat this condition?
layer) Non-surgical: can be left alone on patients
Hyperplasia of variably pigmented basaloid wishes as it is a benign lesion
cells Surgical: as the keratosis lies above the level
This condition can be confused clinically with of the surrounding normal epidermis, it can
acanthosis nigricans. be treated by superficial shaving or cautery.

E. Leser (18281916). German surgeon.


W. Trlat (18281890). French surgeon.

CASE 27 SOLAR KERATOSIS **

INSTRUCTION QUESTIONS
Have a look at this gentlemans face. (Solar
(a) What is a solar keratosis?
keratoses are commonly found on sun-exposed
parts of the face and dorsum of the hands of Solar keratoses are squamous cell carcinomata
elderly people.) in situ. Histological appearances include:
Hyperkeratosis (thickening of the keratin
layer)
APPROACH
Focal parakeratosis
Examine as for any lump (see Case 1). Irregular acanthosis (thickening of the prickle
cell layer)
Basal layer atypia only (versus atypia in all
VITAL POINTS
layers of the epidermis in squamous cell
Usually multiple carcinoma see Case 14).
Yellow-grey or brown in colour
Begin with thickening of skin which can (b) What is the risk of progression to invasive
become unsightly and catch on clothing squamous cell carcinoma?
Scaly surface If untreated, 25% progress to invasive
Can occur as a solar horn on the pinna of squamous cell carcinoma.
the ear these are also benign.
(c) How would you treat this condition?
Non-surgical: cryotherapy, topical application
Completion
of 5-fluorouracil (cytotoxic agent), retinoic
acid (to reverse the damaging effects of
Say that you would like to ask the patient sunlight)
about:
Surgical: shaving of affected skin.
Similar lesions elsewhere
How the lesion affects his life, e.g. cosmetic
symptoms. FURTHER READING
Dinehart SM: The treatment of actinic keratoses.
J Am Acad Dermatol 42(1/2):2528, 2000.
48 Superficial lesions Case 28

CASE 28 DIGITAL CLUBBING *

Palpate the wrist joints for tenderness in


INSTRUCTION
hypertrophic pulmonary osteoarthropathy
Examine this patients hands. (HPOA) rapid painful digital clubbing is
nearly always due to bronchial carcinoma
Examine the toes to look for digital clubbing
APPROACH
Take a history and examine the patient to
This case is a spot diagnosis. As for any case elicit the duration of digital clubbing, e.g. is it
involving the hands, expose to above the from birth, and to look for underlying causes
elbows and ask the patient to place his hands (see below).
palm upwards on a pillow (if available). If the
instruction is to inspect the nails, then just look
at them without going through this routine. TOP TIP
Note that the correct term is digital clubbing the
word clubbing on its own may have several connotations,
VITAL POINTS e.g. something done on a Friday night (or more
Inspect worryingly, to a seal )!

Exaggerated anteroposterior and longitudinal


curvature to fingernails (Fig. 20)
Loss of angle between nail and nail bed (this
QUESTIONS
can be more clearly seen by approximating
the dorsal aspects of the terminal phalanges (a) What are the causes of digital clubbing?
of the fingers of both hands after flexing at
the interphalangeal joints, and is known as The most common cause of clubbing is
Lovibonds sign or the diamond sign) idiopathic. The other causes can be divided
Drumstick or parrot-beak appearance into:
of the nail (also known as doigts Gastrointestinal:
Hippocratique). Liver cirrhosis (especially primary biliary
cirrhosis)
Palpate Inflammatory bowel disease (especially
Crohns disease)
Increased bogginess/fluctuation of nail bed Malabsorption (coeliac disease, tropical
elicit this by supporting the patients finger sprue)
with your two thumbs and use your index Gastrointestinal lymphoma
fingers to demonstrate fluctuance. Respiratory:
Bronchial carcinoma (most commonly
Completion squamous cell)
Chronic suppurative lung disease
Say that you would like to: (abscess, bronchiectasis, cystic fibrosis,
empyema)
Fibrosing alveolitis
Mesothelioma
Increased Cardiac:
longitudinal
Cyanotic congenital heart diseases (e.g.
curvature
Spongy Fallots tetralogy, transposition of the
Nail/nail-fold angle
nail bed great arteries)
greater than 180
Infective endocarditis
Bulbous Atrial myxoma (rare)
terminal Rare causes:
phalanx Familial, e.g. hazel nails (usually seen
Figure 20 Digital clubbing. before puberty), pachydermoperiostitis
Case 29 Superficial lesions 49

(idiopathic familial HPOA with post- (b) How do you grade digital clubbing?
pubertal digital clubbing, bone changes, Grade I: increased glossiness and cyanosis
increased sweating of palms and soles of the skin at the root of the nail associated
and marked thickening of the skin, with increased fluctuation at the base of the
forehead and scalp) nail bed
Graves disease (pseudo-clubbing also Grade II: loss of angle between nail and nail
known as thyroid acropachy) bed (see above)
Unilaterally seen in axillary artery Grade III: drumstick appearance of nail (see
aneurysm and brachial arteriovenous above)
malformation.
Grade IV: bony changes involving the wrists
and ankles, sometimes the elbow and knees
ADVANCED QUESTIONS (HPOA).

(a) What do you know about the


pathophysiology of digital clubbing? Hippocrates (460379 BC). Greek physician, born in
Kos, commonly thought of as the founder of medicine.
Several theories have been put forward to try and
explain the mechanisms behind digital clubbing: E. L. A. Fallot (18501911). Professor of Hygiene and
Legal Medicine, Marseilles, France. He described
The current favoured explanation is failure of congenital cyanotic heart disease due to a
platelet precursors to become fragmented combination of (1) ventricular septal defect, (2) right
into platelets within the pulmonary ventricular outflow tract obstruction, (3) right
circulation they are easily trapped in the ventricular hypertrophy and (4) overriding aorta.
peripheral vasculature and release platelet-
derived growth factor and vascular
endothelial growth factor leading to
promotion of vascularity and ultimately FURTHER READING
clubbing
Myers KA, Farquhar DR: Does this patient have
Vasodilatation of nail-bed vessels secondary clubbing? JAMA 286(3):341347, 2001.
to an unidentified mediator (candidates
include ferritin, bradykinin, prostaglandin and Spicknall KE, Zirwas MJ, English JC 3rd:
5-hydroxytryptamine), which is normally Clubbing: an update on diagnosis, differential
inactivated in the lung but may persist in diagnosis, pathophysiology, and clinical
those with digital clubbing where inactivation relevance. J Am Acad Dermatol 52(6):1020
is defective or there 1028, 2005.
Increased growth hormone
Organs supplied by the vagus are affected Note
by digital clubbing vagotomy can reverse
digital clubbing in bronchial carcinoma Hippocrates first described digital clubbing over
Tumour necrosis factor. 2400 years ago!

CASE 29 BRANCHIAL CYST *

INSTRUCTION VITAL POINTS


Examine this gentlemans neck. Usually presents in a young adult in the third
decade
Equally common in males and females
APPROACH Found in anterior triangle of neck in front
Approach as for neck examination (see Case 6). of the upper or middle third of the
sternocleidomastoid
Smooth, firm swelling that is ovoid in shape
with its long axis running forwards and
downwards
50 Superficial lesions Case 30

Fluctuant on palpation
QUESTIONS
Usually opaque on transillumination (due to
desquamated epithelial cell contents) (a) What is a branchial cyst?
May be hard and fixed to surrounding
structures in the presence of established or A branchial cyst is thought to develop because
recurrent infection of a failure of fusion of the embryonic second
and third branchial arches. An alternative, and
Look carefully for the opening of a fistula in
currently popular, hypothesis is that it is an
this area (a branchial fistula runs between
acquired condition due to cystic degeneration in
the tonsillar fossa and the anterior border of
cervical lymphatic tissue. The cysts are lined by
the sternocleidomastoid).
squamous epithelium.

TOP TIP (b) How would you diagnose a branchial cyst?


Clinical examination
There are three surgical definitions that you should be
able to roll off your tongue: Fine-needle aspiration opalescent fluid
containing cholesterol crystals or pus.
Cyst: an abnormal sac containing gas, fluid or
semisolid material, with an epithelial lining (c) How would you treat a branchial cyst?
Sinus: a blind-ending track, typically lined by epithelial The cyst may be surgically excised whole if
or granulation tissue, which opens onto an epithelial possible, although this may be difficult if
surface there has been previous infection
Fistula: an abnormal communication between two Bonneys blue dye can be injected into the
epithelial surfaces (or endothelial surfaces, e.g. fistula/sinus allowing accurate surgical
arteriovenous fistula). excision and therefore reduces recurrence
rates
Complicating infections may be treated with
antibiotics
Completion Complications include recurrence of the cyst
and development of a chronic, discharging
Say that you would like to ask the patient sinus.
about:
Associated symptoms, e.g. pain, symptoms FURTHER READING
of infection
Tracy TF Jr, Muratore CS: Management of
The effect of the lump on his life. common head and neck masses. Semin Pediatr
Surg 16(1):313, 2007.
www.pedisurg.com/PtEduc/Branchial_Cleft_
Cyst.htm information for parents on their
childrens neck lumps.

CASE 30 DERMOID CYST *

INSTRUCTION VITAL POINTS


No specific instruction can occur in various Inspect
sites.
Smooth spherical swelling
APPROACH Soft and may fluctuate
Non-tender
Examine as for any lump (see Case 1).
Look for associated scar from previous injury
(especially in adults).
Case 31 Superficial lesions 51

TOP TIP The midline of the nose (nasal dermoid


cysts)
Dermoid cysts lie deep to the skin in the The midline of the neck and trunk
subcutaneous tissues. They differ from lipomas as they
Suspect if you see a child or young adult in the
are not within the fat layers, and from sebaceous cysts as
exam:
they are not attached to skin.
Acquired due to forced implantation of skin
into subcutaneous tissues following an injury.
Normally found in areas of the body prone to
injury such as fingers. Suspect if you see an
Completion adult in exam.

Say that you would like to ask the patient: (b) How would you treat this condition?
How the cyst affects their lives, e.g. Congenital surgical treatment involves
cosmetic symptoms complete excision but the full extent of the
Whether they have suffered an injury cyst should be established with suitable
previously (if you suspect that it is an radiographic views (X-ray or CT scan)
acquired cyst). This is especially important in midline
cysts which may communicate with the
cerebrospinal fluid so exclusion of a bony
QUESTIONS defect is vital before surgery
(a) What is a dermoid cyst? Acquired surgical treatment involves
complete excision of the cyst.
A dermoid cyst is a skin-lined cyst deep to
the skin. They may be congenital or acquired
Congenital due to developmental inclusion FURTHER READING
of epidermis along lines of fusion of skin Turkyilmaz Z, Karabulut R, Bayazit YA, et al:
dermatomes and are therefore found Congenital neck masses in children and their
commonly at: embryologic and clinical features. B-ENT
The medial and lateral ends of the 4(1):718, 2008.
eyebrows (internal and external angular
dermoid cysts)

CASE 31 THYROGLOSSAL CYST *

opening of a thyroglossal sinus with


INSTRUCTIONS seropurulent discharge, which follows
Examine this gentlemans neck. rupture or incision of a thyroglossal cyst),
scars (see Case 8).

APPROACH
Protrusion of the tongue
Approach as you would a neck examination
(see Case 6). Ask the patient to open his mouth and stick
his tongue out as far as possible
Inspect (from the front) If the lump moves on protrusion of the
tongue, it is likely to be a thyroglossal cyst
Site of the lump note that 75% are in the (this is because the cyst is usually related to
midline, 25% are either a little to the right or the base of the tongue by a patent or fibrous
the left track which runs through the central portion
Smooth and rounded of the hyoid bone) a lump from the thyroid
gland does not move on protrusion of the
Other features on inspection of the lump,
tongue.
e.g. size, skin changes (you may see the
52 Superficial lesions Case 31

Swallowing Rarely present at birth, 40% present in the


first decade and can even present late in the
Place a glass of water in the patients hands, ninth decade.
ask him to take a sip of water, hold it in his
(c) How do you treat a thyroglossal cyst?
mouth and swallow when you ask him to
As he swallows, inspect the lump if it Treatment is essentially surgical
moves on swallowing, it is likely to originate Operation of choice is Sistrunks operation
from the thyroid gland Inject patent track with dye at the start of the
Note that thyroglossal cysts also move on operation
swallowing, so ask the patient to stick his Excise cyst and the patent or fibrous track
tongue out before proceeding with the which runs through the central portion of the
thyroid gland examination (see Case 8). hyoid bone (which is also excised)
May have to dissect up to the origin at the
foramen caecum (see below)
Palpate (from the back)
If central portion of hyoid bone not excised,
high incidence of recurrence.
Repeat the two above tests, this time palpating
the cyst gently from behind the patient to
ensure that the diagnosis is correct. ADVANCED QUESTIONS
(a) What is the embryological origin of a
Finish your examination here
thyroglossal cyst?
Results from persistence of part of the
Completion thyroglossal tract, which marks the
developmental descent of the thyroid gland
Say that you would like to: At the 4th week of development, the thyroid
appears as a midline diverticulum and
Take a history from the patient, particularly
descends ventrally to the pharynx between
concentrating on how the lump affects his
the developing second arch as a duct (which
life, e.g. cosmetic symptoms.
normally involutes)
The origin of the tract can persist as a
QUESTIONS midline dimple the foramen caecum at
the junction of the vallate and filiform
(a) What is your differential diagnosis? papillae of the tongue.
The differential diagnosis includes other midline (b) What are the pathological features
neck lumps: of thyroglossal cysts?
Thyroid nodules and masses (including Lined by stratified squamous or ciliated
pyramidal lobe) pseudostratified columnar epithelium
Enlarged lymph nodes May also contain thyroid or lymphoid tissue,
Other cysts dermoid and epidermoid which can undergo malignant change
Subhyoid bursae. If malignancy occurs, usually of thyroid
papillary type.
(b) What do you know about the epidemiology
of thyroglossal cysts?
Rare FURTHER READING
Worldwide distribution Foley DS, Fallat ME: Thyroglossal duct and
Equally common in males and females other congenital midline cervical anomalies.
Semin Pediatr Surg 15(2):7075, 2006.
Case 32 Superficial lesions 53

CASE 32 RADIOTHERAPY MARKS *

preventing mitosis. Normal tissues have a


INSTRUCTION
greater ability to repopulate in response
Have a look at this ladys chest. to radiation-induced cell depletion than
tumours.

APPROACH (b) Which normal tissues are particularly


As this is a spot diagnosis, the patient should affected by radiotherapy?
be adequately exposed to their waist if they Tissues with rapid turnover:
are covered up, explain what you are going to
Epidermal layers of the skin
do and obtain sufficient exposure. Approach as
Small intestine
you would a breast examination (see Case 63).
Bone marrow stem cells
Tissues with a limited ability to repopulate:
VITAL POINTS Spinal cord
Inspect Gonads oocytes and spermatocytes.

(c) What are the side-effects of radiotherapy?


For any evidence of chest wall or breast
disease the patient may have undergone a Early:
unilateral mastectomy. General: malaise, fatigue, loss of appetite,
nausea and vomiting
Current radiotherapy Skin: as above
Bone marrow suppression: particularly if
India-ink marks irradiation to the pelvis and long bones
Erythema Gastrointestinal: diarrhoea
Desquamation Late:
Skin markings to delineate area of treatment. Skin: as above
Lungs: pneumonitis, pulmonary fibrosis
Previous radiotherapy Heart: ischaemic heart disease
Arteries: radiation arteritis, especially the
Telangiectasia. carotids post head and neck radiotherapy;
this leads to subsequent stenosis and distal
ischaemia
Completion Spinal cord: myelopathy
Visceral damage: constricted fibrotic bladder,
Say that you would like to: bowel obstruction secondary to strictures
Take a history to determine the duration and and adhesions, renal impairment due to
side-effects of radiotherapy on this lady. depletion of renal tubular cells
Gonadal damage: infertility
ADVANCED QUESTIONS Thyroid: hypothyroidism due to depletion of
thyroid follicular cells
(a) How does radiotherapy work? Eyes: cataracts
Secondary malignancies: increased risk of
High-energy X-rays interact with tissues to
solid tumours and also of leukaemias (the
release electrons of high kinetic energy, which
risk of the latter being 12% at 15 years,
cause secondary damage to adjacent DNA via
with an even higher risk if chemotherapy with
an oxygen-dependent mechanism. The damage
alkylating agents are used in conjunction).
is either repairable or non-repairable, the latter
manifesting itself as chromosomal abnormalities
54 Superficial lesions Case 33

(d) How are the side-effects of Prior chemotherapy to increase sensitivity


radiotherapy minimized? of tumour to radiotherapy
Lead shields to protect the eyes and gonads Regional hypothermia useful in superficial
tumours and bulky non-vascular tumours
Dose fractionation to allow recovery of
normal host tissues Radiolabelled antibodies delivering high
levels of radiation locally to the tumour.

CASE 33 DERMATOFIBROMA *

INSTRUCTION QUESTIONS
Examine this ladys legs.
(a) What is a dermatofibroma?
A dermatofibroma (also known as a fibrous
APPROACH histiocytoma) is a benign neoplasm of
Examine as for any lump (see Case 1). dermal fibroblasts. Previous theories that
dermatofibromas are a reaction to a previous
injury or insect bite have now fallen out of
VITAL POINTS favour. Recent thinking favours the concept that
it is a result of an abortive immunoreactive
Inspect process, featuring dermal dendritic cells as
initiators of the disease.
Can occur anywhere but are more common
on the lower limbs of young to middle-aged (b) What is the differential diagnosis?
women It is important to exclude malignant tumours
Small pink or brown pigmented such as:
hemispherical nodules
Malignant melanoma (see Case 15)
Smooth in appearance.
Basal cell carcinoma (see Case 16).

Palpate (c) How would you treat this condition?


Non-surgical: leave alone if asymptomatic
Firm woody feel (characteristic) and if patient does not want intervention
They are part of the skin and are therefore Surgical: simple excision followed by
fully mobile over deep tissues. histology.

Completion FURTHER READING


Nestle FO, Nickoloff BJ, Burg G:
Say that you would like to ask the patient:
Dermatofibroma: an abortive immunoreactive
What symptoms they are experiencing from process mediated by dermal dendritic cells?
the lump, e.g. cosmetic. Dermatology 190(4):265268, 1995.
www.skinsite.com/info_dermatofibromas.htm
information for patients.
Case 35 Superficial lesions 55

CASE 34 HIDRADENITIS SUPPURATIVA *

INSTRUCTION QUESTIONS
Examine this ladys left axilla.
(a) What is hidradenitis suppurativa?
Hidradenitis suppurativa, also known as acne
APPROACH inversa, is now considered a disease of follicular
Examine as for any lump (see Case 1). occlusion rather than an inflammatory or
infectious process of the apocrine glands.
Abscesses form recurrently and this causes
VITAL POINTS the characteristic permanent disfiguring of the
skin. It usually affects young women, with a
The skin is thickened and may be ulcerated;
prevalence of 0.30.4% in industrialized
watering-can sinuses may be seen
countries.
Look for signs of any active current infection
(tenderness/increased temperature/ (b) How would you treat hidradenitis
erythema). suppurativa?
Hidradenitis can be extremely uncomfortable,
COMPLETION cosmetically unpleasant and distressing for the
patient; they are also problematic to treat
Say that you would like to ask the patient
satisfactorily.
about:
Well-localized abscess: incision and drainage
Symptoms arising from this condition and
under antibiotic cover
how they affect the patient
Larger lesions: radical excision and full-
Any other affected areas, e.g. perineum,
thickness skin grafting usually harvested
groins
from the groins or abdomen
Predisposing factors, e.g. diabetes mellitus.
New treatments like tumour necrosis factor-alfa
inhibitors have given clinicians more options
against this difficult disease.

FURTHER READING
Alikhan A, Lynch PJ, Eisen DB: Hidradenitis
suppurativa: a comprehensive review. J Am
Acad Dermatol 60(4):539561, 2009.

CASE 35 KAPOSIS SARCOMA *

INSTRUCTION VITAL POINTS


Have a look at this skin lesion. Inspect

Purple papules or plaques


APPROACH
Solitary or multiple
Examine as for any lump (see Case 1). Can be found anywhere on skin or on
mucosa of any organ but usually found on
the limbs, mouth, tip of the nose or palate.
56 Superficial lesions Case 36

Completion ADVANCED QUESTIONS


Say that you would like to: (a) What varieties do you know of Kaposis
Take a history, e.g. ethnic origin, previous sarcoma?
transplant Classic Kaposis sarcoma:
Ask the patient about underlying Initially described in Ashkenazi Jews
immunocompromise.
Found on the legs of elderly men
Confined to skin
QUESTIONS Not fatal
AIDS-associated Kaposis sarcoma
(a) What do you know about Kaposis
Found in one-third of patients with AIDS
sarcoma?
(diagnostic of AIDS)
Derived from capillary endothelial cells or More common in homosexual patients
from fibrous tissue One-third develop a second malignancy,
Linked to human herpes virus 8 (HHV-8) e.g. leukaemia, lymphoma
also known as Kaposis sarcoma herpes Endemic (central African) variety
virus (KSHV) Aggressive invasive tumour
(b) How would you treat this condition? Ultimately fatal
Good response to chemotherapy
Leave alone if asymptomatic and if patient
does not want intervention Transplantation-associated Kaposis sarcoma
Intervene only when extensive or for Following high-dose immunosuppressive
cosmetic reasons: therapy
Local radiotherapy Often regress when treatment stopped.
Chemotherapy interferon-,
doxorubicin, intralesional vinblastine Moricz Kaposi (18371902). Hungarian dermatologist.
Mention that Kaposis is usually present in Also described the rash in systemic lupus erythemato-
advanced HIV infection and so adequate sus as a butterfly rash and xeroderma pigmentosum
anti-retroviral therapy is required. (Kaposis disease).

FURTHER READING
Schwartz RA, Micali G, Nasca MR, et al: Kaposi
sarcoma: a continuing conundrum. J Am Acad
Dermatol 59(2):179206, 2008.

CASE 36 PHARYNGEAL POUCH *

May be very little to find except for a cystic


INSTRUCTION
swelling low down in the anterior triangle of
Examine this gentlemans neck. the neck (Fig. 21)
Deep palpation produces a squelching sound
due to free fluid in the pouch
APPROACH
Halitosis is a frequent feature, as food is
Approach as for neck examination (see regurgitated into the neck.
Case 14).
Completion
VITAL POINTS
Say that you would like to ask the patient
Most commonly seen in the elderly
about:
Case 37 Superficial lesions 57

(b) What are the complications of a


pharyngeal pouch?
Chest infection due to pulmonary
aspiration
Thyropharyngeus
Diverticular neoplasia in less than 1% of
Killian's cases.
dehiscence
Cricopharyngeus (c) What investigations would you perform to
help you in your diagnosis?
Pharyngeal Barium swallow usually diagnostic
diverticulum Inferior Rigid endoscopy if neoplasia suspected.
constrictor
of pharynx (d) How would you treat a pharyngeal pouch?
Figure 21 Anatomy of the pharyngeal pouch. Non-surgical: Leave alone if small and
asymptomatic
Associated symptoms, e.g. regurgitation Surgical:
leading to coughing, dysphagia Minimally invasive surgery: Dohlmans
Complications (see below) procedure endoscopic diathermy
The effect of the lump on his life. resection of the posterior pharyngeal wall
Ask to listen to the patients chest. or endoscopic stapling less risk of
fistula formation and consequent
mediastinitis
QUESTIONS Surgical excision: simple inversion and
oversewing (diverticulopexy) as pouch is
(a) What is a pharyngeal pouch? left in situ, risk of missing a possible
diverticular carcinoma, or
A pharyngeal pouch is formed by the herniation
diverticulectomy.
of pharyngeal mucosa (known as a pulsion
diverticulum) through its muscular coat at its
weakest point (Killians dehiscence) between the FURTHER READING
thyropharyngeal and cricopharyngeal muscles
that make up the inferior constrictor. Patients Aly A, Devitt PG, Jamieson GG: Evolution of
are usually symptom-free for a long period of surgical treatment for pharyngeal pouch. Br J
time followed by dysphagia and hoarseness, Surg 91(6):657664, 2004.
associated with regurgitation of undigested
foods, and associated weight-loss.

CASE 37 CYSTIC HYGROMA *

INSTRUCTION VITAL POINTS


Examine this gentlemans neck. 5065% are present at birth, but
occasionally may present in late childhood
or adulthood
APPROACH Located in the posterior triangle of the neck
Approach as for neck examination (see Case 6) Lobulated cystic swelling
note these patients are more frequently found Soft and fluctuant
in paediatric cases. Compressible (usually into another part of
the cyst)
Brilliantly transilluminable.
58 Superficial lesions Case 38

Completion Before delivery:


May obstruct delivery
Say that you would like to: After delivery:
Look in the oropharynx (a large cyst Respiratory obstruction
may extend deeply beneath the Obstruction of swallowing.
sternocleidomastoid muscle into the
retropharyngeal space) (c) What investigations would you perform to
Ask the patient how the lump affects his life. help you in your diagnosis?
Investigations are mainly radiological:
Chest X-ray to map the caudal extent of
QUESTIONS
the cystic hygroma
(a) What is a cystic hygroma? CT/MRI scanning especially if complex.

A cystic hygroma is a congenital cystic (d) How would you treat a cystic hygroma?
lymphatic malformation found in the posterior
Non-surgical:
triangle of the neck. It is probably a
Aspiration injection of sclerosant
developmental anomaly formed during the
coalescence of primitive lymph elements. It Surgical:
consists of thin-walled, single or multiple Excision may be partial (to relieve
interconnecting or separate cysts which symptoms) or complete (as a one-stage
insinuate themselves widely into the tissues at procedure).
the root of the neck.

(b) What are the complications of a FURTHER READING


cystic hygroma? Bloom DC, Perkins JA, Manning SC:
Complications include cosmetic symptoms but Management of lymphatic malformations. Curr
important problems are encountered in the Opin Otolaryngol Head Neck Surg 12(6):500
perinatal period: 504, 2004.

CASE 38 CHEMODECTOMA *

INSTRUCTION An overlying palpable external carotid


artery
Examine this gentlemans neck. True expansile pulsation from a soft or
very vascular tumour
APPROACH Due to intimate relationship with the carotid
arteries, the lump can be moved from side to
Approach as for neck examination (see Case 6). side but not up and down
May be bilateral.
VITAL POINTS
Located in the anterior triangle of the neck, QUESTIONS
at the angle of the jaw
(a) What is a chemodectoma?
Be gentle when examining in this area as
pressure on the carotid bifurcation can A chemodectoma is a tumour of the
induce a vasovagal attack paraganglion cells of the carotid body located
Usually the lump is solid and firm at the bifurcation of the common carotid artery.
Pulsatile but not expansile in nature (be They are usually benign (but locally invasive),
extra gentle as soon as you realize it is but occasionally, they are malignant with
pulsatile!) May be due to: potential to metastasize to local lymph
Transmitted pulsation from the adjacent nodes.
carotid arteries
Case 39 Superficial lesions 59

(b) What investigations would you perform to Ultrasonic surgical dissection may also be
help you in your diagnosis? used
Duplex ultrasound Radiotherapy:
Angiography shows a hypervascular mass For patients unfit for surgery
displacing the bifurcation of the carotid For large tumours.
arteries
CT/MRI to delineate the extent of the FURTHER READING
tumour.
Sajid MS, Hamilton G, Baker DM; Joint Vascular
(c) How would you treat a chemodectoma? Research Group: A multicenter review of carotid
Surgical: body tumour management. Eur J Vasc
Endovasc Surg 34(2):127130, 2007.
Surgical excision (with preoperative
embolization if the tumour is large)

CASE 39 FURUNCLES *

INSTRUCTION QUESTIONS
Examine this gentlemans right axilla.
(a) What is a furuncle?
A furuncle results from infection of hair follicles
APPROACH with Staphylococcus aureus.
Examine as for any lump (see Case 1). These
are common in A&E but are rare in (b) How would you treat this condition?
examinations. Non-surgical: risk-factor modification, e.g.
establishment of good diabetic control and,
for recurrent infections, eradication of
VITAL POINTS nasal carriage of Staphylococcus aureus
Can affect any hair-bearing area of the skin, with antiseptics and/or antibiotics, e.g.
particularly the face, neck, buttocks, groins chlorhexidine and mupirocin
and axillae Surgical: incision and drainage for large and
Small pus-containing swelling (when the painful boils.
contents become solid, it is known as a boil)
(c) What is a carbuncle?
Tender on palpation
A carbuncle is an extensive infection of hair
May be multiple.
follicles by the same organism, with involvement
of adjacent follicles and development of draining
Completion sinuses. It is associated with diabetes and is
treated with a combination of systemic
Say that you would like to ask the patient antibiotics and surgical incision.
about:
Other affected areas
FURTHER READING
Predisposing factors such as diabetes
mellitus, steroid treatment and other Bernard P: Management of common bacterial
immunodeficiencies. infections of the skin. Curr Opin Infect Dis
21(2):122128, 2008.
Ask to test the urine or blood for sugar.
60 Superficial lesions Case 40

CASE 40 PYODERMA GANGRENOSUM *

Myeloproliferative disorders, e.g.


INSTRUCTION
polycythaemia rubra vera, myeloma
No specific instruction lesion can be on any Autoimmune hepatitis
part of the body but usually found on the trunk, More common in males than females.
lower limbs or face.
(b) What is your differential diagnosis?
APPROACH Autoimmune:
Rheumatoid vasculitis
Examine as for ulcers (see Case 1).
Infectious:
Tertiary syphilis
VITAL POINTS Amoebiasis
Iatrogenic:
Inspect
Warfarin necrosis
Unknown:
Ulcer with necrotic base
Behets disease.
Irregular bluish-red overhanging edges
Associated with surrounding erythematous (c) How would you treat this condition?
plaques with pustules.
Medical: treat underlying condition, saline
cleansing, high-dose oral or intralesional
Completion steroids cyclosporin
Surgical: serial allograft followed by
Say that you would like to: autologous skin graft or muscle flap
Take a history and examine the patient for coverage when necessary.
evidence of:
Ulcerative colitis (the presence of
pyoderma gangrenosum is related to Hulusi Behet (18891948). Professor to the Clinic
disease activity) of Dermatology and Syphilis, Turkey. Multi-organ
Crohns disease disease of unknown (?viral) aetiology causing arthritis,
Rheumatoid arthritis. pyoderma, ulcers in the mouth, scrotum and labia,
eye problems such as hypopyon (pus in the anterior
chamber of the eye) and iritis, and with central
QUESTIONS nervous involvement, e.g. meningoencephalitis. Most
common geographically along the Silk Road and is
This is likely to continue on to questions
linked with HLA-B51.
concerning inflammatory bowel disease,
particularly ulcerative colitis (Case 50).

FURTHER READING
ADVANCED QUESTIONS
Callen JP, Jackson JM: Pyoderma
(a) What other associations of pyoderma gangrenosum: an update. Rheum Dis Clin North
gangrenosum do you know of? Am 33(4):787802, vi, 2007.
Idiopathic (50%)
Case 41 Superficial lesions 61

CASE 41 VASCULAR MALFORMATIONS *

Can be secondary to skin irradiation (see


GENERAL
Case 32)
These are unlikely cases, but a few points are Can be part of hereditary haemorrhagic
described below in case they appear in your telangiectasia (OslerRenduWeber
examinations. Some of these conditions may syndrome) rare autosomal dominant
occur in paediatric short cases or OSCEs. disease (with incomplete penetrance) in
which overt and occult haemorrhage
can occur presenting as haematuria,
TYPES haemetemesis, malaena, epistaxis or
Capillary account for two-thirds of all cases iron-deficiency anaemia.
and include naevi, port-wine stains,
telangiectasia and spider naevi
Port-wine stain (also known
Predominantly venous venous angioma
as naevus vinosus)
Predominantly lymphatic lymphangioma
circumscriptum (see Case 130).
Purple-blue naevus found on face, lips and
mucous membranes of the mouth
COMMON FEATURES Present from birth and does not change in
size thereafter
Develop as an abnormal proliferation of the
embryonic vascular network Found on limbs in association with Klippel
Trenaunay syndrome (see Case 109)
Hamartomas
SturgeWeber syndrome is the association
May ulcerate
of a facial port-wine stain with a
May induce hyperkeratosis in the overlying corresponding haemangioma in the brain,
stratum corneum layer of the skin. leading to contralateral focal fits.

SPECIFIC LESIONS Strawberry patch


Campbell de Morgans spots (cavernous haemangioma)

Small red capillary naevus Bright-red raised strawberry-like lesion


Develops on the trunk in middle-age Present from birth, but 60% undergo
spontaneous resolution by the age of 3 years
No clinical significance.
Only treated if obscuring a visual field or
spontaneous resolution not occurring.
Spider naevus (also known as
naevus araneus)

Form of telangiectasis Campbell de Morgan (18111876). Full surgeon to the


Central arteriole with leg-like branches which Middlesex Hospital in London, who believed the spots
blanch on central pressure were a sign of cancer.
Found over upper torso, head and neck in Sir William Osler (18491919). Canadian-born
adults (thought to be within the drainage Professor of Medicine at McGill, Pennsylvania, Johns
area of the superior vena cava) Hopkins and Oxford. Also described Oslers nodes
Associated with chronic liver disease and (cutaneous nodules in infective endocarditis) and
pregnancy Osler-Vaquez disease (polycythaemia rubra vera).
More than five is considered as pathological Henry Jules Louis Marie Rendu (18441902). Parisian
in chronic liver disease. physician at the Necker Hospital.
William Allen Sturge (18501919). English physician
Telangiectasis and pathologist at the Royal Free Hospital.
Frederick Parkes Weber (18631962). English
Dilatation of normal capillaries physician.
This page intentionally left blank
2
SECTION

ABDOMEN AND TRUNK


42 Inguinal hernia *** 64
43 Abdominal examination general approach *** 68
44 Surgical jaundice *** 71
45 Stoma *** 73
46 Hepatomegaly *** 75
47 Incisional hernia *** 78
48 Umbilical/paraumbilical hernia *** 79
49 Splenomegaly *** 81
50 Inflammatory bowel disease *** 83
51 Examination of the scrotum general approach ** 86
52 Hydrocoele ** 87
53 Epididymal cyst ** 88
54 Varicocele ** 89
55 Right iliac fossa mass ** 90
56 Transplanted kidney ** 92
57 Ascites ** 93
58 Epigastric mass ** 95
59 Pleural effusion ** 96
60 Dysphagia ** 98
61 Enlarged kidney ** 100
62 Common surgical scars * 102
63 Breast examination general approach * 103
64 Breast lump * 105
65 Post-mastectomy breast * 106
66 Breast reconstruction * 107
67 Gynaecomastia * 109
68 Chest post-lobectomy/pneumonectomy * 110
69 Median sternotomy * 111
70 Testicular tumour * 112
71 Enterocutaneous fistula * 114
72 Mouth signs in abdominal disease * 116
73 Epigastric hernia * 117
74 Femoral hernia * 118
64 Abdomen and trunk Case 42

CASE 42 INGUINAL HERNIA ***

INSTRUCTION TOP TIP 2


Examine this gentlemans groin. Dont forget that the same opening instruction may
apply to the examination of the scrotum, and it may be a
scrotal lump rather than a hernia and if you cannot see a
APPROACH visible swelling. Proceed by asking the patient where the
lump is.
Expose the patient from umbilicus to knees.

VITAL POINTS
TOP TIP 3
TOP TIP 1 An inguinal hernia arises above and medial to the
pubic tubercle. By contrast, a femoral hernia arises below
Should the hernia be examined with the patient lying and lateral to it (see Fig. 22).
down or standing up? The answer is that it doesnt
matter. It is generally considered to be easier to define
the anatomy with the patient supine, and if the hernia can
be detected with the patient on the couch, then examine
them there. If no lump can be felt, or if no couch is TOP TIP 4
available, then stand the patient up first. Be completely conversant with the anatomy of the
inguinal region and the location of surface anatomy (Fig.
22 and Fig. 23).
Midinguinal point = point halfway along a line joining
the ASIS and the midline = location of femoral artery

Midpoint of
Midinguinal the inguinal
point Midline ligament

Inferior
epigastric
artery

Deep
Inguinal inguinal
hernia ring

Femoral hernia

Figure 22 Surface anatomy of groin examination for herniae.


Case 42 Abdomen and trunk 65

Midpoint of the inguinal ligament = halfway along by applying gentle pressure in the
inguinal ligament (i.e. between the pubic tubercle and direction of the inguinal canal. If you
ASIS) = deep inguinal ring. cannot reduce the lump it is either (1) not
a hernia or (2) incarcerated (irreducible).
Accordingly, the femoral pulse is located medial to
the deep inguinal ring.
Palpate

Begin by defining the anatomy (see Figs. 22


The objectives of the examination and 23)
are to: Palpate the pubic tubercle and the anterior
superior iliac spine. The former can reliably
1. Confirm that the lump is a hernia be located by palpating in the midline
2. Differentiate an inguinal from a femoral hernia downwards from the umbilicus until the
3. If it is an inguinal hernia, establish whether it symphysis pubis is reached (Fig. 24). Then
is direct or indirect. move laterally to identify the tubercle
Place one finger on each to demonstrate that
Inspect the inguinal ligament runs between the two
(Fig. 25)
Look at the groin for old surgical scars (is Ask the patient to cough for the second time
the hernia recurrent?) this requires very and demonstrate that the hernia arises above
careful inspection to identify old, small scars this line, medial to the pubic tubercle (see
that are frequently hidden in groin creases Fig. 22). This fulfils objective (2) (see above)
and/or by hair and confirms that the hernia is inguinal and
not femoral
If the hernia is obvious, then begin to
examine it If it wasnt obvious on inspection alone,
demonstrate that the lump has an expansile
If it cannot be seen then ask the patient to
cough impulse: place one hand over the
cough and observe for the appearance of a
lump and ask the patient to cough
lump
It is only possible to fulfil objective (3) above,
If it is still not obvious, ask: Have you
if the hernia is reducible. With the hernia fully
noticed a lump in your groin? and get the
reduced, try to control the hernia at the deep
patient to demonstrate it
inguinal ring. Redefine the inguinal ligament
Fulfil objective (1) above and confirm that the again and place one finger 1cm craniad to
lump is a hernia by: the midpoint of the inguinal ligament (halfway
Asking the patient to cough, while along the inguinal ligament) (Fig. 26). Ask the
observing for an expansile cough impulse patient to cough for the third and final time.
Getting the patient to reduce the hernia if If the hernia is controlled by your finger at
possible. If the patient cannot achieve the deep inguinal ring it is an INDIRECT
this, attempt to reduce the lump yourself inguinal hernia. If the hernia is not controlled

External iliac artery Anterior superior


iliac spine

Midpoint of inguinal ligament Inguinal ligament

Mid-inguinal point
Pubic tubercle

Femoral artery

Pubic
symphysis

Figure 23 Anatomy of the inguinal ligament.


66 Abdomen and trunk Case 42

Completion

Complete your examination by asking to


examine the scrotum for incidental scrotal
lumps and to examine the contralateral groin for
herniae.

QUESTIONS
(a) What is the difference between a direct
and an indirect inguinal hernia?
Figure 24 Palpation of the symphysis pubis.
Indirect inguinal herniae:
Are the remnants of a patent processus
vaginalis
Arise from the abdominal cavity lateral to the
inferior epigastric vessels at operation,
passing obliquely through the deep inguinal
ring and travelling through the inguinal canal
with the spermatic cord
May continue through the superficial inguinal
ring into the scrotum
Direct inguinal herniae:
Are the result of a weak posterior wall to the
Figure 25 Identification of the inguinal ligament using inguinal canal
bony landmarks. Arise medial to the inferior epigastric vessels
at operation
This weakness causes the abdominal
contents to bulge through the wall into the
inguinal canal but the hernia is not within the
spermatic cord.

(b) What are the contents of the


spermatic cord?
Three arteries:
Artery to vas deferens (from inferior vesicular
artery)
Testicular artery (from aorta)
Figure 26 Attempting to control the reduced hernia at Cremasteric artery (from inferior epigastric
the deep ring. artery)
Three nerves:

and reappears, then it is a DIRECT inguinal Ilioinguinal nerve (L1) on the front of the cord
hernia. Remember that the accuracy of Nerve to cremaster (from genitofemoral
clinical examination in distinguishing a direct nerve)
from an indirect hernia is low (e.g. 56% of Autonomic nerves (sympathetic fibres from
direct herniae were wrongly classified by T10)
consultant surgeons as indirect on clinical Three other structures:
examination in one such study, see Further Vas deferens
Reading)
Pampiniform plexus of veins (drains right
Other aspects of the lump (as for any lump) testis into inferior vena cava and left testis
may be defined at this stage, e.g. skin into renal vein)
changes
Lymphatics (drain the testis to the para-
Decide whether the lump is confined to the aortic lymph nodes).
inguinal region or descends into the scrotum.
Case 42 Abdomen and trunk 67

(c) What would you tell patients about their Bruising occurs in 30%
recovery from inguinal hernia repair? Pain often very severe and patients should
Early mobilization is important be discharged with adequate analgesia;
chronic groin pain persists in 5% of patients
They should keep the area clean and wash
carefully, especially after the clips/sutures Haematoma 10%
have been removed Infection 1%
They are able to bathe immediately Ischaemic orchitis 0.5% (caused by
They may need to be off work for 6 weeks if thrombosis of the pampiniform plexus
their job involves heavy lifting draining the testis)
They should avoid prolonged coughing Previous vasectomy is a predisposing
(control chronic obstructive pulmonary cause
disease preoperatively) Dissection beyond (more medial than) the
They should take laxatives if they get pubic tubercle is one operative risk and
constipated postoperatively. so this practice should be avoided
Recurrence should be <0.5%
Normally due to inadequate ring and
ADVANCED QUESTIONS posterior wall closure
Occasionally due to over-tight sutures.
(a) How would you perform a hernia repair?
Be prepared to discuss a method of hernia
repair you have learned. The main points to FURTHER READING
remember are: Cameron AE: Accuracy of clinical diagnosis of
Testicular damage should be mentioned as a direct and indirect inguinal hernia. Br J Surg
specific risk-factor 81(2):250, 1994.
The operation can be performed under local Liem MS, van Vroonhoven TJ: Laparoscopic
or general anaesthetic and often as a day inguinal hernia repair. Br J Surg 83(9):1197
case 1204, 1996.
The Royal College of Surgeons of England McGreevy JM: Groin hernia and surgical truth
has recommended the Lichtenstein mesh (editorial). Am J Surg 176(4):301304, 1998.
repair and the Shouldice repair
Laparoscopic repair is increasingly
recognised as having a role in the repair of
bilateral and/or recurrent herniae. Differential diagnosis of a lump in the groin
(b) What are the complications of inguinal Use the acronym L-SHAPE
hernia repair? Lymph node/Lipoma of the cord
Complications should be divided into immediate Sapheno-varix/Skin lesions (sebaceous cyt/lipoma,
(first 24h), early (within the first month) and late etc.)
(later than the first month) and further into Hernia inguinal/femoral
general for any procedure and specific for this
Aneurysmal dilatation of the femoral artery
procedure.
Psoas abscess/bursa
Specific complications to mention:
Ectopic/undescended testis.
Urinary retention
68 Abdomen and trunk Case 43

CASE 43 ABDOMINAL EXAMINATION GENERAL


APPROACH ***

of infusions/catheters/drains, etc. and then second, just


INSTRUCTION
prior to inspecting the abdomen itself, inspect more
Examine this patients abdominal system/ closely for scars, caput medusae, etc.
abdomen.

APPROACH VITAL POINTS


Inspect for peripheral stigmata of
TOP TIP 1 abdominal disease
Should i begin with the hands?
Hands
Listen carefully to the examiners instructions. If you Take the patients right hand and look for
are told to examine the ABDOMINAL SYSTEM, then begin signs of chronic liver disease (see Case 46)
with the hands and conduct a complete examination of and inflammatory bowel disease (see Case
the entire patient. By contrast, if you are told to examine 50)
the ABDOMEN, begin with the abdominal examination
Look for digital clubbing (see Case 28) by
itself (commence at Expose the patients abdomen
inspecting the angle between the nail bed
below); do not annoy your examiner by asking whether
and fold (Fig. 27) and by performing
you should begin with the hands!
Schamroths test (Fig. 28); a small diamond-
shaped window is normally apparent
between the nail beds when the distal
TOP TIP 2 phalanges are opposed. If this window is
obliterated, the test is positive and clubbing
Exposure of the patient.
is present. Clubbing is confirmed by the
The exposure of the abdomen is very important and following:
opinions differ as to how much of the abdomen needs to Fluctuation and softening of the nail bed
be exposed (and at what point) during the clinical (increased ballotability)
examination. Clearly, if there is an inguinal hernia or a Loss of the normal <165 angle between
scrotal lump, this will be missed if the external genitalia the nailbed and the fold
are not exposed. Equally, a patient who has an epigastric Increased convexity of the nail fold
hernia should not have his or her dignity compromised.
Thickening of the finger distally
You are not expected to say, I would expose the (resembling a drumstick)
patient from nipple to knees; this is both inappropriate to Shiny aspect and striation of the nail and
say in front of a patient and misses the point that the skin
degree of exposure depends on the case and how much
the examiner expects of you.
We would advocate beginning by positioning the
patient flat on the bed, but keeping the groin covered until
later on, starting your examination looking for peripheral
stigmata of gastrointestinal disease.

TOP TIP 3
Inspect and then inspect again.
Assuming that you have been instructed to undertake
an examination of the entire abdominal system, you
should inspect the patient TWICE: first, before you begin Figure 27 Examining for digital clubbing by inspecting
noting the general condition of the patient, the presence the angle between the nail bed and fold.
Case 43 Abdomen and trunk 69

Mouth
Look in the mouth (and smell) for oral
manifestations of chronic gastrointestinal
disease, e.g. hepatic foetor, pallor of the
mucous membranes (see Case 72 for full
description of mouth signs in abdominal
disease).
Neck
Palpate the neck for supraclavicular
lymphadenopathy Virchows node (also
known as Troisiers sign) is found in the
supraclavicular fossa between the sternal
and clavicular heads of the
sternocleidomastoid muscle.
Figure 28 Schamroths test to establish the presence of
Trunk
digital clubbing.
Inspect the rest of the arms and upper chest
wall for spider naevi, which are in the
distribution of drainage of the superior vena
cava (see Case 41)
Briefly inspect the back, by asking the
patient to roll towards and away from you.

Expose the patients abdomen

If it is a man, ask him to remove his shirt


If it is a woman, keep the bra on but expose
the chest otherwise
Expose down to the symphysis pubis,
keeping the genitalia covered.

Stand back from the couch


Figure 29 Examining for evidence of a liver flap by
extending the outstretched wrists. Stand at the end or to the side of the couch,
and inspect the abdomen thoroughly, looking
for:
Gastrointestinal disease associated with
digital clubbing includes inflammatory bowel Distension
disease, malabsorption and cirrhosis, Scars (see Case 62)
especially primary biliary cirrhosis Visible pulsation
Look for koilonychia (spoon-shaped nails in Ask the patient to take a deep breath in and
iron-deficiency anaemia) and pallor of the hold it, seeing the transmitted pulsation of an
palmar creases in anaemia abdominal aortic aneurysm
Check for a liver flap with both of the Ask the patient to cough or lift their head off
patients hands (you can ask the patient to the bed, demonstrating any herniae
put his hands out in front of him and cock especially important if scars are present.
his wrists back as if he is stopping traffic)
(Fig. 29); in practice, this is a late sign of
Palpate
hepatic encephalopathy and these patients
will not be in examinations.
Kneel down next to the patients right side and
Eyes ask if they have any pain anywhere, before
Look at the conjunctivae for pallor (anaemia beginning to palpate the abdomen. Look at the
can only be confirmed following haemoglobin patients face the whole time when attempting
measurement) and at the sclerae for jaundice to elicit any tenderness:
(see Case 14).
70 Abdomen and trunk Case 43

Figure 30 Auscultation of the abdomen.

Begin furthest away from you and palpate Finish your examination here
the nine regions of the abdomen with the
four fingers of your right hand held together, Make sure to cover the patient back up and
first lightly, testing particularly for tenderness ensure they are comfortable.
When you arrive in the epigastric region,
pause for pulsation, testing for an abdominal
aortic aneurysm (see Case 115) Completion
Continue with deep palpation in the same
nine regions, feeling for any masses often Say that you would like to:
this is done using two hands interlocked Review the observation chart (temperature,
above each other blood pressure, pulse and respiratory rate)
Palpate the liver (see Case 46) and spleen Examine the lower limbs for peripheral
(see Case 49) in turn oedema
Attempt to ballot the kidneys (see Case 61). Examine the external genitalia and groin (if
not already done)
Percuss Perform a digital rectal examination
Dipstick the urine.
If there is hepatosplenomegaly the spleen
and liver should be percussed
TOP TIP 4
If there is any abdominal distension you
should percuss for ascites (see Case 57). The term per rectal is incorrect; digital rectal
examination is more appropriate.

Auscultate (Fig. 30)

Over the liver for a bruit


In the left iliac fossa for bowel sounds
Over the abdominal aorta and iliac vessels C. E. Troisier (1844 1919). Professor of Pathology,
for bruits. Paris. Also described haemachromatosis (Troisier
syndrome).
R. L. K. Virchow (1821 1902). Professor of Pathology,
Continue Wurzberg and Berlin. Also described Virchow space
(perivascular space of VirchowRobin), Virchow cell
Continue if you have not found any abnormality (lepra cell) and the Virchow triad of thrombogenesis.
by exposing the external genitalia and
L. Schamroth (1924 1988). Professor of Cardiology,
examining the scrotum (see Case 51) and
Baragwanath Hospital, South Africa. A leading
inguinal canal for herniae (see Case 44). Palpate
authority on electrocardiography and cardiac
the femoral pulses and check for femoral
arrhythmias and originally demonstrated finger
herniae.
clubbing on himself!
Case 44 Abdomen and trunk 71

CASE 44 SURGICAL JAUNDICE ***

Examine the neck for Virchows node (see


INSTRUCTION 1
Case 43).
Ask this patient some questions about her
jaundice.
Inspect

APPROACH The abdomen may be distended with ascites


There may be distended veins around the
Direct your questions to finding out whether the
umbilicus if there is portal hypertension
cause of her jaundice is likely to be pre-hepatic,
(caput medusae, see Case 128).
hepatic or post-hepatic. Remember that in the
surgical short case, post-hepatic is the most
likely cause and this is the easiest to diagnose TOP TIP
from the history:
When examining the eyes for jaundice or pallor, only
Have you noticed any change in the colour
look into one eye (the sign will be bilateral if present). It is
of your urine?
easier to use your left thumb to lower the patients lower
Have you noticed any change in the colour eyelid and ask them to look towards the ceiling in order to
of your stools? get the best opportunity to inspect the sclera and
Have you noticed yourself feeling itchy? conjunctiva.
If the patient has noticed pale stools and dark
If you suspect a jaundiced sclera but you are unsure,
urine, then explore possible causes:
continue to examine the patients soft palate with a pen
Weight-loss, change in bowel habit, loss of torch. Bilirubin is avidly taken up by tissues that are rich
appetite and back pain are associated with in elastin, and therefore the soft palate is a sensitive
primary and secondary intra-abdominal indicator of the presence of jaundice.
malignancies
Younger age, previous biliary colic or
episodic right upper quadrant pains may
indicate gallstones.
Palpate
Continue by asking some questions
differentiating the other types of jaundice, and
Palpate the abdomen as previously (see
possibly to identify any risk-factors for hepatic
Case 43)
jaundice, such as foreign travel (hepatitis A),
blood transfusion (hepatitis B and C), sore Palpate carefully in the right upper quadrant
throat (EpsteinBarr virus), alcohol intake, and identifying any tenderness or masses,
use of certain drugs such as the oral remembering Courvoisiers law: in the
contraceptive pill and phenothiazines. presence of jaundice if a mass (the gall
bladder) is palpable in the right upper
quadrant, the cause is unlikely to be due
INSTRUCTION 2 to gallstones.

Examine this ladys abdominal system.


Finish your examination here

APPROACH
Expose the patient as previously (Case 43) and
begin by examining the hands. Completion

Say that you would like to:


VITAL POINTS
Complete the abdominal examination (see
Look for signs of chronic liver disease (see
Case 43) by checking the hernial orifices,
Case 46).
examining the external genitalia, performing
Confirm the presence of jaundice by looking a digital rectal examination and examining
at the sclera (see Top Tip) the lower limbs for peripheral oedema
72 Abdomen and trunk Case 44

Table 14 Causes of jaundice and effects on liver function tests


Pre-hepatic Hepatic Post-hepatic
Major causes Haemolysis Hepatitis Gall stones
Hereditary, e.g. Decompensated chronic liver disease Carcinoma head of pancreas
Gilberts syndrome Drugs Lymph nodes
Bili type Unconjugated Conjugated Conjugated
Bili increase ++ +++/++++ ++
ALT +/++ ++/+++ +/++
ALP /+ +/++ ++/+++
Bili, bilirubin; ALT, alanine aminotransferase (similar rises with aspartate aminotransferase, AST); ALP, alkaline
phosphatase.

Check the temperature to see whether the Liver function tests: see below
potential obstruction has been complicated Clotting: functional assessment of hepatic
by infection impairment.
Dipstick the urine for raised levels of Radiological investigations:
bilirubin.
Ultrasound will show:
Presence of underlying liver disease
QUESTIONS Degree of dilatation of the common bile
duct (>8mm is abnormal)
(a) How can jaundice be classified? Presence of gall stones
Jaundice is yellow discolouration of the skin Presence of lymphadenopathy or a
and mucous membranes caused by the pancreatic mass
accumulation of bile pigments. The causes can CT scan
be classified (Table 14) into: Endoscopic retrograde
Pre-hepatic cholangiopancreatography (ERCP)
Hepatic Magnetic resonance
Post-hepatic. cholangiopancreatography (MRCP).

(b) What level does the serum bilirubin need (d) How might the liver function tests help in
to rise to before jaundice can be detected on distinguishing the types of jaundice?
clinical examination? This is particularly important in OSCEs, where
Normal bilirubin is <17mmol/L and it usually you may be asked to interpret liver function test
has to reach at least three times this before the results (see Table 14).
sclera is discoloured (i.e. >50mmol/L). Very
high levels of bilirubin are usually associated
with hepatic jaundice.
ADVANCED QUESTIONS
(a) What are the causes of postoperative
(c) Assuming this lady has obstructive
jaundice?
jaundice, how should she be investigated?
Urine should be tested for raised bilirubin Pre-hepatic jaundice can occur due to
(Fig. 31). haemolysis, especially following a transfusion
Hepatic jaundice can result from the use of
Blood tests:
halogenated anaesthetics, sepsis or intra- or
Full blood count: evidence of anaemia in GI postoperative hypotension
malignancies or associated infection Post-hepatic jaundice can occur due to
Renal function: any evidence of hepatorenal biliary injury (such as in laparoscopic
syndrome cholecystectomy).
Case 45 Abdomen and trunk 73

Haemoglobin

Spleen and
Globin Haem reticuloendothelial
cells
Iron Billirubin

Unconjugated Conjugated Liver


bilirubin inside
hepatocytes
Conjugated
bilirubin
Small
Conjugated intestine
Systemic bilirubin
circulation
'Enterohepatic Bacterial
circulation' action

Kidney Hepatic
portal vein Reabsorbed

Large
intestine
Urobilinogen
conjugated bilirubin Stercobilinogen
Figure 31 Pathway for bilirubin excretion.

Nicolas Augustin Gilbert (18581927). French FURTHER READING


physician who worked on the classification of liver www.nlm.nih.gov/medlineplus/ency/
disease. article/000210.htm online medical
encyclopaedia explaining the various causes of
jaundice.

CASE 45 STOMA ***

INSTRUCTION APPROACH
Inspect this ladys abdomen and comment on Expose the patient as in Case 43. Do not start
what you see. at the hands, as you have been given a specific
direction to inspect the abdomen.
74 Abdomen and trunk Case 45

VITAL POINTS Finish your examination here


Inspect

Site (right iliac fossa, left iliac fossa, etc.)


(Fig. 32) Completion
Whether it is covered by a bag or whether
the bag has been removed Say that you would like to:
Appearance if the bag has been removed Continue to examine the rest of the
then comment on: abdomen to look for reasons why the stoma
Mucosal lining does it look healthy? may have been formed in the first place.
Presence of a spout or flush with the skin
End (one opening) or loop (an afferent QUESTIONS
and efferent portion of bowel with one
common opening or two separate (a) What are the indications for forming a
openings) stoma?
Contents if there is a bag covering the
Feeding, e.g. feeding gastrostomy/jejunostomy
stoma then describe the bag and comment
on: Lavage, e.g. appendicostomy
Urine Decompression bypass of an obstructing
bowel lesion distal to the stoma
Formed stool
Diversion
Semi-formed or liquid stool
1. Protection of a distal bowel anastomosis
Move on to describe the rest of the
abdominal wall and remember there is likely Previously contaminated peritoneal
to be a scar, which you should also describe cavity
(see Case 62) Anatomical considerations ileorectal
Are there any other drains/healed stoma anastomosis
sites? 2. Urinary diversion following cystectomy
Exteriorization
Perforated or contaminated bowel, e.g.
Palpate
distal abscess/fistula
Permanent stoma, e.g. abdomino-perineal
If you are only asked to inspect, then do not
resection of rectum.
touch the patient at any time, even to move the
stoma bag out of the way there may be a (b) How would you prepare a patient who is
good reason why you should not touch it.
going for surgery which will involve forming a
stoma?
Psychosocial and physical preparation
Explanation of indications and
complications
Percutaneous endoscopic This includes seeing a Clinical Nurse
gastrostomy Specialist in Stoma Care preoperatively, who
would normally mark the site
Marking of the stoma site with the patient
standing up as he or she must be able to
Tranverse loop see the stoma
colostomy The stoma must be within the rectus
End colostomy abdominus muscle
Ileal conduit
Away from scars or skin creases
Away from bony points or waistline of
Ileostomy
clothes
At a site that is easily accessible to
Mucous fistula the patient, e.g. not under a large
fold of fat.
Figure 32 Relevant sites for stomas on abdominal wall.
Case 46 Abdomen and trunk 75

Table 15
Ileostomy Colostomy
Site Right iliac fossa Left iliac fossa
Surface Spout (contents are erosive and Flush with skin
can damage local skin)
Contents Watery small bowel content Formed faeces
Examples of permanent stomas Post panproctocolectomy Abdominoperineal resection of rectum
Examples of temporary stomas Loop ileostomy over low Hartmanns procedure (end
anastomosis of anterior resection colostomy)

(c) What are the complications of forming a (d) How can you tell the difference between
stoma? an ileostomy and a colostomy?
Complications can always be divided into See Table 15.
specific to the procedure vs general to any
surgical procedure, and also into immediate (e) How would you rehabilitate a patient
(<24h), early (<1 month) and late (>1 month). following the placement of a stoma?
Specific complications: Diet should be normal
Ischaemia/gangrene Bag should be changed once or twice a day
Haemorrhage (needs to be emptied more frequently than
Retraction this if it is urine or fluid faeces)
Prolapse/intussusception Ileostomies should have the base plate under
the bag changed every 5 days and the bag
Parastomal hernia
changed daily
Stenosis
Psychological and psychosexual support.
Skin excoriation
General complications related to underlying
disease: FURTHER READING
Stoma diarrhoea related to water and Cheung MT: Complications of an abdominal
electrolyte imbalances, hypokalaemia being stoma: an analysis of 322 stomas. Aust N Z J
the commonest and most important Surg 65(11):808811, 1995.
consequence Shellito PC: Complications of abdominal stoma
Nutritional disorders surgery. Dis Colon Rectum 41(12):15621572,
Stones both gall stones and renal stones 1998.
increase in frequency following an ileostomy www.colostomyassociation.org.uk British
Psychosexual Colostomy Association website help and
Residual disease, e.g. Crohns and information for patients who are just about to
parastomal fistula. have a colostomy.

CASE 46 HEPATOMEGALY ***

INSTRUCTION VITAL POINTS


Examine this gentlemans abdominal system. Inspect

Look for the peripheral stigmata of chronic liver


APPROACH disease. In the hands:
Expose the patient as per Case 43 and begin Digital clubbing feature associated with
by examining the hands. cirrhosis of the liver, especially of primary
biliary cirrhosis (see also Cases 28 and 43)
76 Abdomen and trunk Case 46

Leukonychia white nails associated with


liver disease, fungal infection and can be
congenital
Terrys lines white nails with normal pink
tips seen in cirrhosis
Palmar erythema vasodilatation due to
non-metabolized oestrogens
Dupuytrens contracture (see Case 78)
Liver flap ask the patient to hyperextend
the hands, holding the arms straight out in
front of them and cocking their wrists back
as if you are trying to stop traffic look for
flapping of the hands (see Case 43). Figure 33 Palpation of the liver, beginning immediately
above the right anterior superior iliac spine, in the right
In the rest of the arms, and upper trunk:
iliac fossa.
Spider naevi
Tattoos (risk-factor for transmission of
hepatitis B and C virus)
Scratch marks icterus, or itch, is a sign of
post-hepatic jaundice
Gynaecomastia
In the face and neck:
Pale conjunctiva
Yellow sclera (see Case 44)
Inside the mouth smell for hepatic foetor
Palpate the supraclavicular fossa for
lymphadenopathy.

Inspection of the abdomen

The abdomen may be swollen due to Figure 34 Percussion of the upper edge of the liver.
ascites, and there might be a fullness in the
right upper quadrant At this point, auscultate the liver for a bruit
Note the presence of distended abdominal (heard in hepatocellular carcinoma and
veins, which may occur in portal hypertension alcoholic hepatitis)
(caput medusae, see Case 128). Check next for splenomegaly (see Case 49)
Check for ascites if the abdomen is
Examination of the liver distended (see Case 57).

Palpate the liver, beginning immediately


above the right anterior superior iliac spine, TOP TIP 1
in the right iliac fossa (Fig. 33). Ask the
Demonstrate to the examiner that the mass in the
patient to breathe in and move your hand
right upper quadrant is an enlarged liver using the
proximally between each breath in order to
acronym SPRUE (SPRUE = gastrointestinal disease =
detect the liver edge, coming down onto the
causes of gastrointestinal visceral enlargement). The
hand in inspiration
same acronym can also be used to confirm the presence
Define the distance in fingerbreadths from of splenomegaly (see Case 49).
the costal margin at which the liver edge first
appears Site of enlargement: from the right costal margin
Palpate the edge of the liver again, noting towards the right iliac fossa
the presence of nodules and whether the Percussion note: dull
edge is firm or smooth Respiration movement: it descends
Percuss the upper edge of the liver,
Unable to get above it
beginning at the top of the right hemithorax;
the percussion note usually becomes dull at Edge: may be smooth or irregular.
the level of the fifth rib (Fig. 34)
Case 46 Abdomen and trunk 77

Finish your examination here Malignant disease:


Primary/secondary solid tumours (secondary
are more common)
Completion Lymphoma
Leukaemia
Say that you would like to Congestive cardiac disease:
Complete the abdominal examination (see Right heart failure
Case 43) Tricuspid regurgitation (causes a pulsatile
Check for peripheral and sacral oedema liver)
(which occur in hypoalbuminaemia).
BuddChiari syndrome.

QUESTIONS
TOP TIP 2
(a) How would you investigate this patient?
Use the following acronym CHIASMA as an aide
Blood tests: memoire for the causes of hepato(-spleno)megaly. The
Full blood count e.g. raised white cell same causes are relevant for splenomegaly (see Case 49).
count in infection Congestive: cardiac failure
Liver function hypoalbuminaemia, evidence
Haematological: reticuloses
of hepatic dysfunction
Infection: viral, bacterial, protozoal
Clotting functional hepatic impairment
C-reactive protein/erythrocyte sedimentation Amyloid
rate increased in infection/inflammation Storage disorders: Wilsons disease,
and in malignancy haemochromatosis
Radiological investigations: Masses: primary/secondary neoplasia
Ultrasound is the first line radiological Autoimmune/alcohol (fatty liver/cirrhosis).
investigation used to define the liver
architecture, give an idea of the size and
may identify the pathology
Contrast-enhanced CT may also be useful,
especially to further investigate solid lesions.
ADVANCED QUESTIONS

(b) What are the causes of hepatomegaly? (a) What is the significance of an arterial bruit
or venous hum over the liver?
Physiological:
Reidels lobe An arterial bruit may indicate alcoholic hepatitis,
and carcinoma. A venous hum is associated
Hyperexpanded chest
with portal hypertension and if this is secondary
Infections: to cirrhosis with a patent umbilical vein (or
Viral: viral hepatitis, EpsteinBarr virus, varices in the falciform ligament), this is known
cytomegalovirus as the CruveilhierBaumgarten syndrome.
Bacterial: tuberculosis, liver abscess
Protozoal: malaria, histoplasmosis, (b) What is portal hypertension?
amoebiasis, hydatid, schistosomiasis Defined as portal vein pressure of more than
Alcoholic liver disease: 10mmHg (normal 510mmHg). Portal blood
Fatty liver (can also be caused by diabetes flow through the liver is greatly reduced or even
mellitus) reversed in the most severe cases. The causes
can broadly be divided into:
Cirrhosis (other causes of cirrhosis also l
ead to hepatomegaly but these are less Extrahepatic: caused by increased resistance
common) to flow, e.g. portal or splenic vein thrombosis
Metabolic diseases: Intrahepatic: due to cirrhosis, right heart
failure, sarcoidosis and schistosomiasis (the
Wilsons disease
latter is the most important cause worldwide
Haemochromatosis ova of the parasite colonize and obstruct
Cellular infiltration, e.g. amyloid the portal venules).
78 Abdomen and trunk Case 47

W. Baumgarten (18731945). Born in St Louis, FURTHER READING


Missouri, was one of founding members of the Garcia N Jr, Sanyal AJ: Portal hypertension. Clin
American Association of Physicians. Liver Dis 5(2):509540, 2001.
Jean Cruveilhier (17911874). French pathologist, www.british-liver-trust.org.uk patient-centred
who also described gastric ulcers. website with information about viral hepatitis
M. Epstein (1921 ). Professor of Pathology in Bristol, and regional liver support groups.
investigated Burkitts lymphoma and identified this
virus as important in its pathogenesis.
B. Riedel (18461916). German surgeon and
pathologist.
R. Terry. British physician.

CASE 47 INCISIONAL HERNIA ***

Determine whether the defect is the whole


INSTRUCTION
length of the scar or not
Examine this gentlemans abdomen. If the hernia was already present before the
patient coughed (typically a very large
defect), ask the patient whether he can push
APPROACH the lump back inside the abdomen.
Begin by exposing and examining the abdomen
itself.
Auscultate

VITAL POINTS Listen for bowel sounds if there is a large


incarcerated hernia.
Inspect
Finish your examination here
The patient may be overweight
There will be a scar over the abdominal wall
describe the scar (see Case 62) and be
sure to note the presence of any other scars, Completion
drain sites or old stomas
Ask the patient to lift his head off the bed Say that you would like to:
and note any bulging out of the scar Complete the rest of the abdominal
Ask the patient to cough and, again, tell the examination, and wait to see whether he
examiner that you have demonstrated an wants you to do so (implying there may be a
element of weakness associated with the second problem for you to identify).
scar.

QUESTIONS
Palpate
(a) What is an incisional hernia?
Begin by palpating the patients scar, asking Extrusion of peritoneum and abdominal
whether there is any tenderness contents through a weak scar or accidental
Note the presence of any nodularity and feel wound on the abdominal wall
for the presence of a defect under all, or part Represents a partial wound dehiscence
of, the length of the incision where the skin remains intact.
Ask the patient to cough and feel the
weakness in the scar allowing the intra-
abdominal contents to come out into
your hand
Case 48 Abdomen and trunk 79

(b) What are the complications of have concurrent medical problems (like obesity
incisional hernia? and chronic obstructive pulmonary disease)
increasing their anaesthetic risk.
Intestinal obstruction (often intermittent)
Incarceration Non-surgical:
Strangulation Use of a truss or corset
Skin excoriation Weight-loss and management of other
risk-factors
Persistent pain.
Surgical:
(c) What factors predispose to Prior to surgery
incisional hernia? Cardiac and respiratory disease should
Preoperative: be controlled first
Age Other risk-factors should be optimized
Immunocompromised state (including renal Preoperative weight-loss should be
failure, diabetes, steroid use) encouraged
Obesity Surgical treatment principles are:
Malignancy Dissection of the hernial sac from
surrounding tissues and definition of
Abdominal distension from obstruction or
tissue bordering the defect on all sides
ascites
to 23cm
Operative:
Closing the defect (if small) and/or
Poor technical closure of the wound using using a mesh overlapping adequately
too small bites or inappropriate suture (>5>8cm) over normal tissues to
material allow for mesh shrinkage/contraction
Placing drains through wounds during healing now the technique of
Postoperative: choice as it has been shown to be
Wound infection superior to suture repair
Wound haematoma Layered closure technique with sutures
(especially if there is no tissue-loss)
Early mobilization
Large hernia may require the placing
Postoperative atelectasis and chest infection.
of postoperative drains.

ADVANCED QUESTIONS FURTHER READING


(a) What are the treatment options for Khaira HS, Lall P, Hunter B et al: Repair of
this patient? incisional hernias. J R Coll Surg Edin 46(1):39
43, 2001.
Not every patient should undergo repair of his
or her incisional hernia. Some patients will Luijendijk RW, Hop WC, van den Tol MP, et al:
undergo operation and have a very high chance A comparison of suture repair with mesh repair
of wound haematoma and infection, dehiscence for incisional hernia. N Engl J Med 343(6):392
and recurrent hernia. In addition, many patients 398, 2000.

CASE 48 UMBILICAL/PARAUMBILICAL HERNIA ***

INSTRUCTION VITAL POINTS


Examine this gentlemans abdomen. Inspect

The patient may be overweight


APPROACH
From the side or end of the couch ask the
Expose the patient and begin examination of patient to lift his head off the bed and then
the abdomen itself. to cough noticing the bulge appearing
around or above the umbilicus
80 Abdomen and trunk Case 48

Note any associated ulceration or skin Skin dimple


damage
The normal
Note the presence of an overlying scar umbilicus
indicating a recurrent hernia
Point out the presence of a lump underlying Rectus abdominus
the umbilicus, pushing the umbilicus out muscle
from the abdominal wall. Scar in linea alba
tethered to the skin
Palpate Normal umbilicus

Try to determine the size of the defect


If there is a lump, ask the patient to push it
back in (do not attempt to do this yourself
they may be irreducible because loops of
adherent omentum divide the sac into True umbilical
multilocular cavities) hernia
Ask the patient to cough, demonstrating an
expansile cough impulse.

Finish your examination here

Scar stretched and umbilicus everts

Completion

Say that you would like to: Paraumbilical


Continue with the rest of the abdominal hernia
system examination.
Scar normal
QUESTIONS
(a) What is the pathogenesis of Hernial orifice at
umbilical herniae? side of umbilical scar
These are due to a defect through the linea alba Figure 35 Umbilical and paraumbilical herniae.
(the union of the rectus sheath in the midline)
adjacent to the umbilicus and usually due to
obesity stretching the fibres. (b) Tell me about umbilical herniae in children
True umbilical herniae occur through the Minor defects in neonates are common but
umbilical scar (cicatrix) (Fig. 35) and are usually usually repair spontaneously. In older children,
congenital in origin and particularly common in umbilical herniae are more common; they tend
patients of Afro-Caribbean origin. to have a narrow neck and folds of peritoneum
Paraumbilical herniae occur around the stuck within this neck, which can occasionally
umbilical scar (see Fig. 35). They are uncommon strangulate. Most cases resolve before puberty
before the age of 40 years and can become and should only be repaired in symptomatic
large. Peristalsis can be observed through the children.
skin when the defect is large.
(c) What differences are there in adults?
The neck of the sac is often tight and held with
Acquired umbilical herniae may be caused by:
a fibrous band this increases the rate of
strangulation and infarction of contained bowel. Pregnancy
Rarely, spontaneous discharge of the contents Ascites
(as an enterocutaneous fistula) can occur. Ovarian cysts
Fibroids
Case 49 Abdomen and trunk 81

Bowel distension The contents are reduced with or, more


They more commonly require surgical repair. commonly without, excision of the hernial
sac
The hernial defect is repaired:
ADVANCED QUESTIONS The lower edge of the rectus is sutured
behind the upper edge, so that the two
(a) How would this hernia be repaired?
flaps overlap using interrupted mattress
It is important to mention the importance of nonabsorbable sutures
treating concurrent medical problems prior to The use of a sublay (extraperitoneally
surgery; many of these patients have significant beneath the rectus sheath) mesh is
co-morbidity, which increases their anaesthetic preferable for large defects and is
risk. Where possible, the hernia should be associated with reduced recurrence rates
repaired as the risk of strangulation is high. compared to onlay/inlay meshes.
Surgical technique: Mayos vest-over-pants
operation is the most widely accepted repair for Charles Horace Mayo (18651939) and William James
these herniae: Mayo (18611939). Brothers who founded the Mayo
A horizontal ellipse of stretched supra or Clinic in Rochester, Minnesota.
infra-umbilical skin is excised, deepening the
incision to the rectus sheath and identifying
the fibrous band which is the neck of the sac FURTHER READING
The sac is dissected free from surrounding
Skinner MA, Grosfeld JL: Inguinal and umbilical
tissues, which may include release from the
hernia repair in infants and children. Surg Clin
cicatrix
North Am 73(3):439449, 1993.

CASE 49 SPLENOMEGALY ***

INSTRUCTION Palpate and percuss


Examine this gentlemans abdominal system.
Begin palpating for the spleen at the right iliac
fossa (i.e. to the right of and below the
umbilicus), moving your fingers towards the
APPROACH
costal margin on the left hand side as the
Expose the patient as for the abdominal patient breathes in each time (Fig. 36). The
examination (see Case 43). spleen is palpable below the costal margin on
the left. Characteristically, the defining features
confirming that a left upper quadrant mass is
VITAL POINTS
Peripheral stigmata

Pallor (pale nail beds/conjunctivae, skin


folds, mucous membranes)
Lymphadenopathy
There may also be stigmata of rheumatoid
disease (see Case 80).

Inspect

A fullness underneath the left costal margin


may be seen.
Figure 36 Palpating for the spleen beginning to the right
of and below the umbilicus.
82 Abdomen and trunk Case 49

due to splenomegaly are (using the acronym


SPRUE, see Case 46):
QUESTIONS
Site of enlargement from the left costal (a) What are the causes of splenomegaly?
margin towards the umbilicus
Infective:
Percussion note dull (remember that the
spleen underlies ribs 911 and while you are Acute: EpsteinBarr virus, cytomegalovirus,
remembering that fact, note also that it is 1 HIV, infective endocarditis
3 5 inches in size and that it weighs 7oz, Chronic: toxoplasmosis, malaria, brucella,
i.e. 1 by 3 by 5 by 7 by 911) leishmaniasis, schistosomiasis
Respiration movement it descends Haematological disease:
Unable to get above it or ballot it Haemolytic anaemia
(differentiating it from the kidney) Myeloproliferative disorders (especially
Edge a notch may be palpable on the myelofibrosis)
superomedial surface. Sickle cell disease/thalassaemia
Leukaemia (especially chronic myeloid
leukaemia, CML)
TOP TIP 1
Lymphoma
If the spleen is difficult to palpate, reach your left Portal hypertension:
hand around the lower left rib cage and lift forwards as
the patient breathes in this manoeuvre may make a Cirrhosis
slightly enlarged spleen more easily palpable (Fig. 37). Hepatic, portal or splenic vein thrombosis
Systemic diseases:
Amyloidosis
Sarcoidosis
Finish your examination here Rheumatoid arthritis (also remember Feltys
syndrome, see below).

Completion

Say that you would like to: TOP TIP 2


Examine the rest of the abdomen particularly Use the following acronym CHIASMA for the causes
for hepatomegaly of splenomegaly (The same acronym and causes are
Listen to the heart for murmurs, and examine relevant for hepatomegaly (see Case 46).
for other stigmata of infective endocarditis Congestive: portal hypertension, hepatic vein
Enquire about foreign travel and symptoms obstruction
of possible haematological malignancy.
Haematological: reticuloses
Infection: viral, bacterial, protozoal
Amyloid
Storage disorders: Gauchers disease
Masses: primary/secondary neoplasia
Autoimmune: Feltys syndrome.

(b) What are the causes of massive


splenomegaly?
Myelofibrosis
Chronic myeloid leukaemia
Malaria
Tropical splenomegaly
Figure 37 Attempting to make a spleen more easily
Kala-azar (visceral leishmaniasis)
palpable by insertion of the examining left hand around
the lower left rib cage.
Case 50 Abdomen and trunk 83

(c) What are the indications for splenectomy? (c) What are the appearances of the blood
Trauma film after a splenectomy?
Hypersplenism: Increased platelet count and large platelets
Autoimmune thrombocytopaenia/ Increased neutrophils
haemolytic anaemia Nucleated red cells with HowellJolly bodies
Hereditary spherocytosis and target cells
Thrombotic thrombocytopaenia Tend to mount more of a leukocytosis in
Sickle cell/thalassaemia response to infection.
Myelofibrosis, occasionally in CML,
Hodgkins. Augustus Roi Felty (18951963). American physician
who described the combination of splenomegaly,
lymphadenopathy and leucopenia.
ADVANCED QUESTIONS
Philippe Charles Ernest Gaucher (18541918). French
(a) What are the functions of the spleen? dermatologist. He originally described Gauchers
disease, the most common of the lysosomal storage
Produces IgM, to capture and process
diseases, in 1882. It is caused by a hereditary
foreign antigen
deficiency of the enzyme glucocerebrosidase (also
Filters especially capsulated microorganisms, known as acid -glucosidase), leading to an
e.g. pneumococcus accumulation of its substrate, the fatty substance
Sequesters and removes old red blood cells glucocerebroside (also known as glucosylceramide).
and platelets
Recycles iron
Pools platelets (30% of total platelets within
spleen).
FURTHER READING
(b) What immunizations would you need to
Baccarani U, Donini A, Terrosu G, et al:
organize in the event of performing a
Laparoscopic splenectomy for haematological
splenectomy? diseases: review of current concepts and
Protocol depends on local guidelines but opinions. Eur J Surg 165(10):917923, 1999.
essentially:
Farid H, OConnell TX: Surgical management of
Pneumococcal vaccine massive splenomegaly. Am Surg 62(10):803
Haemophilis influenzae type B vaccine 805, 1996.
Meningococcal vaccine Glasgow RE, Mulvihill SJ: Laparoscopic
Annual flu vaccine splenectomy. World J Surg 23(4):384388,
Consideration for lifelong penicillin or 1999.
penicillin as required when infection present
Warn about risk of malaria, especially
Plasmodium falciparum

CASE 50 INFLAMMATORY BOWEL DISEASE ***

INSTRUCTION VITAL POINTS


Examine this ladys abdominal system. Inspect for peripheral stigmata of
gastrointestinal disease
APPROACH
General signs of malnutrition or weight-loss
Expose the patient as in Case 43 and begin by
In the hands, look for:
examining the hands.
Digital clubbing
Pale skin creases if anaemic
84 Abdomen and trunk Case 50

In the eyes look for: Completion


Pale conjunctivae if anaemic, uveitis, iritis,
episcleritis Say that you would like to:
Around the mouth look for: Complete the abdominal system examination
Aphthous ulceration, often severe, deep (as in Case 43, inspection of the perineum
ulcers and the digital rectal examination is
If the patient is an inpatient, comment on particularly important)
the: Examine for systemic manifestations of
Intravenous lines, blood transfusions and inflammatory bowel disease:
fluids Large joint mono-arthritis and sacroiliitis
Central venous pressure line Pyoderma gangrenosum
Urinary catheter. Erythema nodosum (usually over the
extensor surfaces of the limbs).
Inspect abdominal signs
QUESTIONS
Comment on all the signs on the abdominal
wall, including: (a) What investigations would you perform?
Scars (see Case 62) in cases of Stool tests:
complicated Crohns disease these may be
multiple, and not typical just describe the A stool culture should be performed in new
anatomical location of each scar cases/exacerbations of inflammatory bowel
disease to exclude infection
Stomas (see Case 45) or healed stoma sites
Blood tests:
Enterocutaneous fistulae (more common in
Crohns disease) Full blood count may show anaemia and
Abdominal drains or healed drain sites. leukocytosis
Electrolytes may show evidence of
dehydration or hypokalaemia
Palpate Liver function tests
C-reactive protein and erythrocyte
In acute exacerbations the abdomen may be
sedimentation rate may be raised, which can
distended and tense
also be used to monitor progress of the
There may be a mass, most commonly in the disease
right iliac fossa (see Case 55)
Endoscopy:
Note the site of any tenderness
An ileocolonoscopy with regional biopsies
The patient may have hepatomegaly.
throughout the entire bowel is the
investigation of choice
Percuss Radiology:
Depending on the exact symptoms, a small
If the abdomen is distended the percussion bowel follow through study is useful in
note is likely to be hyper-resonant, reflecting assessing jejunal/ileal Crohns disease.
dilatation of sub-acutely obstructed (small) Occasionally, a barium enema is useful for
bowel. assessing the large bowel
Abdominal MRI is increasingly used for the
Auscultate assessment of Crohns disease, especially in
younger patients, in an attempt to reduce
Bowel sounds may be increased in acute ionizing radiation exposure.
exacerbations likely to be normal in the exam
(b) What is the definition of severe
itself.
exacerbation of inflammatory bowel disease?
Truelove classification:
Finish your examination here
Gastrointestinal symptoms:
Passage of bloody stools >6 times per
day
Case 50 Abdomen and trunk 85

Systemic signs: Sclerosing cholangitis


Tachycardia (>90bpm) Cholangiocarcinoma.
Pyrexia (>37C)
(b) What are the surgical options for
Laboratory findings:
managing ulcerative colitis?
Anaemia (Hb <10.5g/dL
C-Reative protein >30. Subtotal colectomy with ileostomy ( mucous
fistula) formation:
(c) What are the indications for surgery in Operation of choice for acute severe colitis
inflammatory bowel disease? All of the colon is resected except the distal
Acute severe ulcerative colitis sigmoid and rectum, which is retained. If the
rectosigmoid stump is very diseased, the
Use the following acronym MPS (as in Medical
surgeon may choose to bring it up into the
Protection Society, whom you may be calling
wound to form a mucous fistula, particularly
if you forget these indications!) as an aide
if it is too fragile to be closed safely or if the
memoire.
general condition of the patient is poor. A
(Toxic) Megacolon (transverse diameter of compromise to the latter is to close the
colon of at least 6cm on a plain abdominal stump but to locate it in the subcutaneous
X-ray) high risk of perforation and faecal tissues so that if it blows and leaks, there is
peritonitis no peritoneal contamination or its associated
Perforation rare in absence of toxic physiological complications
dilatation and raises possibility of Crohns Proctocolectomy and permanent ileostomy
disease. The mortality is 40%! (= panproctocolectomy):
Severe gastrointestinal bleeding. Rectum and anus excised with all of the
Chronic ulcerative colitis colon
Use the 3 Ms: As it involves the construction of a
permanent stoma, it is only performed for
Medical management failure to control patient choice or when the patient (and
symptoms specifically the anus) is not suitable for a
Malignant transformation, including severe restorative procedure (e.g. advanced age,
dysplasia on biopsies impaired anal sphincter function, etc.)
Maturation failure in children. Restorative proctocolectomy:
Surgery for Crohns disease This is the procedure of choice for most
Essentially, to treat complications not amenable patients as it avoids a permanent stoma
to medical therapy: The surgery may involve three stages,
although increasingly these may be
Intra-abdominal abscesses that cannot be
combined
drained radiologically
Neo-rectum is created in a pelvic reservoir
Entero-cutaneous fistulae (but see notes in
Case 71) Stage 1: resection of colon and/or rectum
(many patients have already undergone
Stenosis causing obstructive symptoms
subtotal colectomy and ileostomy formation
Control of acute/chronic bleeding.
as an emergency procedure, in which case
this stage involves completion proctectomy
ADVANCED QUESTIONS only)
Stage 2: construction of an ileal reservoir,
(a) What are the hepatobiliary complications which is anastomosed to the anus (ileal
of inflammatory bowel disease? pouch anal anastomosis) this is usually
covered with a diverting loop ileostomy
Liver: proximal to the pouch, although some
Fatty change surgeons, under the right circumstances,
Chronic active hepatitis omit the diverting stoma and the need for
Cirrhosis another operation
Amyloid deposition Stage 3: closure of a diverting loop ileostomy
Gall bladder and bile ducts: if constructed following satisfactory water
soluble contrast study.
Gallstones
86 Abdomen and trunk Case 51

(c) What are the surgical options for Crohns Occasionally a subtotal colectomy and
disease? permanent end ileostomy may be needed
In surgery for the small intestine, as much Pouch surgery is generally contraindicated in
bowel should be preserved after the Crohns disease.
operation as possible
Up to 80% of patients with distal ileal Burrell Bernard Crohn (born 1884). US physician
disease require surgical intervention within a working in New York who became the president of the
5-year period. The operation of choice is a American Gastroenterology Society, presenting a
limited ileocaecectomy, including only a few paper in 1932 which described Crohns disease.
centimetres of macroscopically normal bowel
at each end
Intra-abdominal abscesses should be FURTHER READING
drained www.nacc.org.uk website of the UK national
Colonic defunctioning using a loop ileostomy association for colitis and Crohns disease, a
may be needed for patients who have failed charity for patients with inflammatory bowel
medical therapy disease.

CASE 51 EXAMINATION OF THE SCROTUM


GENERAL APPROACH **

INSTRUCTION Inspect
Examine this gentlemans scrotum.
The key distinction in these cases is whether
the problem arises from the groin (is it an
indirect inguinoscrotal hernia?) or is of scrotal
APPROACH
origin:
It is important to listen to the stem of the
Inspect the groin and scrotum
question, as there will be a clue as to whether
Scrotal incisions may be difficult to see as
the problem is in the groin or in the scrotum
they are frequently made in the median
itself. If asked to examine the groin then begin
raphe in between the two hemiscrotums
with the inguinal hernia examination (see Case
42), unless there is an obvious mass or swelling Check in the groins, identifying any oblique
in the hemiscrotum. groin incisions, which may have been used
to approach the testes.
If the patient is lying on a bed then examine him
supine, remembering to ask him to stand up at
the end to ensure that you do not miss a Palpate
varicocele. If he is standing or sitting in a chair,
then examine him standing. Ask the patient if he has any pain and watch his
face while palpating the scrotum:
Ensure that the swelling is confined to the
VITAL POINTS scrotum by demonstrating that you can get
The objectives of the examination are to: above it
Confirm that the swelling is confined to the Attempt to palpate both testes one at a time,
scrotum (i.e. you can get above it) commencing on the side with the lump if
Establish whether the testis and epididymis evident on inspection. Establish whether the
are identifiable testis and epididymis are separate from the
Determine whether the lump transilluminates. lump or undefinable
When palpating the testis, place the fingers
of one hand behind the testis, supporting it,
while examining the surface of the testis with
the thumb
Case 52 Abdomen and trunk 87

Palpate the normal contour of the testis, Completion


identifying the epididymis and the ductus
deferens as well Say that you would like to:
The surface of the testis is normally firm and
Continue to examine the rest of the
regular
abdomen and groin (see specific cases)
Lumps and irregularity, and especially any
The lymph drainage of the testes are to
hard masses, are abnormal and should
the para-aortic nodes which are
precipitate further investigation. If a lump is
retroperitoneal and unless extremely large
identified attempt to transilluminate it.
will not be palpable
Lumps in the groin can easily be distinguished
Inguinal lymphadenopathy is not likely to
by answering the objectives of the examination,
be a response of testicular problems, but
as set out above (see Cases 52, 53, 54 and 70
the lymph drainage from the skin of the
for details).
scrotum and penis is to the inguinal
nodes, and if there is pathology involving
Finish your examination here the scrotal skin or a squamous cell
carcinoma of the penis, then these nodes
may be enlarged.

CASE 52 HYDROCOELE **

INSTRUCTION Finish your examination here


Examine this gentlemans scrotum.

Completion
APPROACH
See Case 51. Say that you would like to:
Examine the contralateral scrotum.

VITAL POINTS
QUESTIONS
Inspect
(a) What is a hydrocoele?
The scrotum may be very swollen if the Excess accumulation of fluid in the processus
hydrocoele is large. vaginalis. During the descent of the testis from
the posterior abdominal wall in utero, it carries a
Palpate fold of peritoneum, the processus vaginalis. This
normally forms the tunica vaginalis, one of the
The identifying characteristics of the mass are: adult coverings of the testis, and the rest of the
connection from the abdomen is obliterated.
Distinct from the superficial inguinal ring (you Should this obliteration not occur, and fluid
can get above the mass) accumulate in any part of this peritoneum-
Usually inseparable from the testis (although derived covering, a hydrocoele forms.
a hydrocoele of the cord will be separate)
and uniformly enlarged (b) What is the anatomical
Firm often tense classification hydrocoeles?
Usually it is possible to transilluminate. Vaginal hydrocoele: fluid accumulates in the
Hydrocoeles vary enormously in size and some tunica vaginalis which surrounds the testis
patients who come along to examinations have but does not extend up into the cord
chronic hydrocoeles which may be very large Hydrocoele of the cord: fluid accumulates
it may be that some other medical problem around the spermatic cord and therefore the
makes surgical intervention hazardous. mass appears around the ductus deferens.
88 Abdomen and trunk Case 53

This may be very difficult to distinguish from plicated with a series of interrupted sutures
an irreducible inguinal hernia, as it may to the junction of the testis and epididymis
extend up to and beyond the superficial Jaboulays operation the sac is everted
inguinal ring into the groin. If in doubt, through a longitudinal incision, excess sac is
traction on the testis causes a hydrocoele of excised and the remainder replaced behind
the cord to be pulled downwards the cord.
Congenital hydrocoele: the proximal part of
the processus vaginalis has not obliterated, (b) What is a secondary hydrocoele?
the sac communicates directly with the Although most hydrocoeles are the result of a
peritoneum and the hydrocoele is filled with patent processus vaginalis, the vaginal type can
peritoneal fluid be secondary to a number of local pathologies:
Infantile hydrocoele: a situation in between Testicular tumours
the congenital hydrocoele and hydrocoele Torsion
of the cord; the processus vaginalis is
Orchitis
obliterated at the deep ring and so the
hydrocoele does not communicate with the Trauma
abdomen but it remains patent in both the Following inguinal hernia repair.
cord and scrotum.
Peter Lord. Contemporary surgeon at Wycombe
ADVANCED QUESTIONS General Hospital, England, also named the Lords
stretch, for treatment of anal fissure (now obsolete
(a) What are the treatment options? due to unacceptable risk of anal sphincter injury), and
Lords directors, instruments used to assist
Non-surgical: knot-tying within the abdominal cavity.
Watch and wait a small hydrocoele may
require no treatment other than reassurance,
but an underlying malignancy should be
excluded (clinically and with an ultrasound)
FURTHER READING
Aspiration the hydrocoele fluid can be
aspirated to relieve symptoms; tends to Davenport M: ABC of general paediatric
reaccumulate surgery. Inguinal hernia, hydrocele, and the
undescended testis. BMJ 312(7030):564567,
Surgical:
1996.
Lords plication small incision through the
scrotum to lift out the testis; the sac is

CASE 53 EPIDIDYMAL CYST **

INSTRUCTION Palpate
Examine this gentlemans scrotum.
The identifying characteristics of the mass are:
Distinct from the superficial inguinal ring (you
APPROACH can get above the mass)
Separate from the testis within the
As in Case 51.
epididymis
Firm, and may be loculated
VITAL POINTS May be brilliantly transilluminable, unless
they contain sperm (spermatocele) in which
Inspect case they do not transilluminate.

Unless the cyst is unusually large the


scrotum will appear normal. Finish your examination here
Case 54 Abdomen and trunk 89

Completion (b) How should they be managed?


Non-surgical:
Say that you would like to: If the cyst is not troublesome, it should not
Examine the contralateral hemiscrotum. be removed, especially in younger men,
because there is risk of operative damage
and postoperative fibrosis causing subfertility
QUESTIONS Surgical:
(a) How are epididymal cysts caused? Very large or painful cysts can be removed
and occasionally excision of the entire
They are often multiple and most commonly epididymis is indicated to prevent frequent
arise in the head of the epididymis. Occasionally recurrence of painful cysts.
they occur as a complication of vasectomy, in
which case they are full of sperm and are
termed spermatoceles.

CASE 54 VARICOCELE **

INSTRUCTION Completion
Examine this gentlemans scrotum.
Say that you would like to:
Examine the contralateral hemiscrotum.
APPROACH
As in Case 51. QUESTIONS
(a) What is the aetiology of varicoceles?
VITAL POINTS
Varicoceles are dilated tortuous varicose
Inspect veins in the pampiniform plexus, the network
of veins that drains the testis (draining
The scrotum will usually appear normal but the eventually into the testicular vein)
testis on the side of the varicocele may hang They usually occur in up to 15% of younger
lower than the other side. men, often around puberty, and are thought
to have an anatomical basis
If they appear suddenly in older men,
Palpate
underlying retroperitoneal disease should be
sought, including renal carcinoma extending
The varicocele does not usually appear until the
into the left renal vein clinically these may
patient is standing up; all scrotal examinations
be suggested by varicoceles that do not
should include an examination of the patient
disappear on lying supine.
standing to exclude a varicocele. Ask the
patient to cough while palpating the varicocele
The mass is characterized by: ADVANCED QUESTIONS
Distinct from the superficial inguinal ring (you (a) Why are 98% of varicoceles left-sided?
can get above the mass)
Being separate from the testis The left spermatic vein is more vertical where
Bag of worms feel it connects to the left renal vein
Non-transilluminable The left renal vein can be compressed by the
colon
May have a palpable cough impulse.
The left testicular vein is longer than the right
It frequently lacks a terminal valve which
Finish your examination here serves to try to prevent back-flow in the
vein.
90 Abdomen and trunk Case 55

(b) What are the treatment options? Inguinal approach similar principle with
Non-surgical: ligation of the veins in the inguinal canal
Laparoscopic ligation is also possible.
Transfemoral radiological embolization of the
testicular vein, using either a spring coil or
sclerosant FURTHER READING
Surgical:
Cornud F, Belin X, Amar E, et al: Varicocele:
Surgical treatment is often advised as the strategies in diagnosis and treatment. Eur
problem usually gets worse with age and Radiol 9(3):536545, 1999.
there is a risk of infertility
Jarow JP: Effect of varicocele on male fertility.
Palomo operation exposure of the Hum Reprod Update 7(1):5964, 2001.
testicular vein by the high retroperitoneal
approach, through an incision above and www.netdoctor.co.uk/diseases/facts/hydrocele
medial to the anterior superior iliac spine and review of both hydrocele and varicocele.
ligation of all the surrounding veins

CASE 55 RIGHT ILIAC FOSSA MASS **

abdominal mass especially note the presence


INSTRUCTION
of scars indicating renal transplantation (see
Examine this ladys abdomen. Case 56).

APPROACH Palpate
Expose the patient and begin, as in Case 43, by
Begin palpating the abdomen as in Case 43.
examining the hands.
When you locate the mass, differentiate the
mass before continuing with the rest of the
VITAL POINTS abdominal examination. Note the:
Size
Inspect peripheral signs
Edge well defined or poorly defined
Surface smooth/irregular/nodular
In the hands look for:
Relations does it arise from the pelvis or
Digital clubbing (inflammatory bowel disease) are you able to place a hand between the
Pale skin creases (anaemia, e.g. chronic pelvis and the mass
bleeding from colonic carcinoma) Attachment to skin
Arteriovenous fistula at the wrist Attachment to the abdominal wall muscles
(transplanted kidney) ask the patient to lift their head up off the
In the eyes look for: bed while palpating the mass.
Pale conjunctivae (anaemia)
Sclera (jaundice) Finish your examination here
In the neck, palpate:
Lymphadenopathy, especially noting the
presence of a Virchows node in the left
supraclavicular fossa (Case 43).
Completion
Inspect abdominal signs Further examination would depend on your
diagnosis but say that you would like to:
Note the presence of any scars from previous
Complete the rest of the abdominal system
surgery and asymmetry may suggest an
examination.
Case 55 Abdomen and trunk 91

Arising from the gynaecological organs:


QUESTIONS
Ovarian tumours (benign and malignant)
(a) What are the causes of a mass in the right Fibroid uterus
iliac fossa? Arising from the male reproductive system:
The best way to classify this answer is to think Incompletely descended testis (Fig. 38)
of the different anatomical layers and structures Ectopic testis (Fig. 39)
within the right iliac fossa this avoids leaving Arising from the urological system:
out any important causes.
Transplanted kidney
Arising from the skin and soft tissues: Ectopic kidney
Sebaceous cyst Bladder diverticulum
Lipoma Arising from blood vessels:
Sarcoma External iliac or common iliac artery
Arising from the bowel: aneurysm
Carcinoma of the caecum Lymphadenopathy.
Crohns mass in the terminal ileum
(b) What radiological investigations would be
Tuberculosis of the terminal ileum
helpful in distinguishing the possible causes?
Appendicular mass or abscess
Ultrasound would be the first investigation
this would distinguish a bowel mass from

Posterior
abdominal wall

Inguinal Deep inguinal ring


canal
Superficial inguinal
ring

Neck of scrotum

Figure 38 Incompletely descended testis.

Superficial
inguinal pouch
('inguinal')

Femoral triangle
Base
('femoral')
of penis
('penile')

Perineum ('perineal')

Figure 39 Ectopic testis.


92 Abdomen and trunk Case 56

an ovarian or uterine mass, and would


Differential diagnosis of a mass in the left
identify any lymph nodes or abnormal blood
iliac fossa
vessels
Abdominal wall masses are better seen with This is a very similar list to the one above; the only
CT scan, and this would also be useful in change is in the arising from the bowel section:
looking at the extent of intra-abdominal Diverticular mass (often tender)
malignant disease Carcinoma of the colon
Intravenous contrast-enhanced CT scanning
Faecal mass.
would clarify lower abdominal and pelvic
vasculature.

CASE 56 TRANSPLANTED KIDNEY **

internal oblique and transversus abdominus


INSTRUCTION
muscles. It should only be palpated very lightly.
Examine this gentlemans abdomen.

Finish your examination here


APPROACH
Expose the patient as in Case 43 and begin by QUESTIONS
examining the hands.
(a) What are the major indications for
renal transplantation?
VITAL POINTS
Renal transplantation is indicated in end stage
Inspect peripheral signs renal failure, the commonest reasons in the
UK are:
There may be signs of anaemia (pale palmar Diabetes mellitus
skin creases, pale conjunctiva) Hypertensive renal disease
A scar may be visible over the wrist at the Glomerulonephritis
site of a BresciaCimino arteriovenous fistula
Polycystic kidney disease.
(see Case 131)
There may be signs of steroid use (e.g.
bruising, thin skin). ADVANCED QUESTIONS
(a) How is matching of transplanted
Inspect the abdomen kidneys performed?
Note the swelling in the right or left iliac Matching is performed at two levels:
fossa ABO compatibility
There will be a specific scar over the iliac HLA compatibility, matching at the HLA-DR
fossa; a curved inguinal incision is used to locus has the greatest importance followed
perform the transplant by matching at the HLA-B locus and then at
Note also the presence of previous the HLA-A locus
nephrectomy scars and points of access of In patients who are HLA and ABO matched the
old (peritoneal) dialysis catheters. 1-year donor kidney survival rate is 90%. Blood
transfusions prior to transplant should be
Palpate avoided as this carries the risk of HLA
sensitization.
Note the mass in the right or left iliac fossa
(b) What occurs in transplant rejection?
the mass is superficial and well defined as the
transplanted kidney is placed outside the Rejection is genetically modified and also relates
peritoneum, covered only by the external and to HLA incompatibility. It can be divided into:
Case 57 Abdomen and trunk 93

Kidney

External iliac vein Renal artery

External
iliac artery

Renal
vein
Ureter

Figure 40 Vascular supply of the transplanted kidney.

Hyperacute: within hours of surgery due to (d) Describe the vascular supply of the
pre-formed antibodies in a sensitized transplanted kidney
recipient
The donor renal artery is anastomosed to
Accelerated acute: 14 days postoperatively either the internal or external iliac artery
due to a secondary immune response as (Fig. 40)
a consequence of activation of memory T
The donor renal vein is anastomosed to the
cells
external iliac vein
Acute 5 days to 2 weeks after surgery
The ureter is anastomosed separately to the
cell-mediated immunity related; renal
patients bladder
epithelial cells are destroyed by a
lymphocyte interstitial infiltrate The renal pelvis is the most anterior
structure, then artery and the vein most
Chronic humoral mechanisms more
posterior.
important, tubular atrophy and interstitial
fibrosis are the histological features.
FURTHER READING
(c) How might you be aware that transplant
rejection is occurring? Paduch DA, Barry JM, Arsanjani A, et al:
Indication, surgical technique and outcome of
The features that may be expected are:
orthotopic renal transplantation. J Urol
Tenderness over the graft 166(5):16471650, 2001.
Reduction in urine output Tejani A, Emmett L: Acute and chronic rejection.
Rising creatinine. Semin Nephrol 21(5):498507, 2001.

CASE 57 ASCITES **

INSTRUCTION APPROACH
Examine this gentlemans abdomen. Expose the patient and begin to examine the
abdomen as in Case 43.
94 Abdomen and trunk Case 57

VITAL POINTS TOP TIP


Inspect It can be extremely difficult to palpate the liver in
patients with ascites. If possible, percuss the abdomen for
The abdomen may be distended if the fluid before continuing to examine for organomegaly the
ascites is gross distension will tend to be examiner may stop you at this point.
lateral, as fluid accumulates in the paracolic
gutters when the patient is supine
Begin at the hands, noting any peripheral
stigmata of chronic liver disease (see Finish your examination here
Case 46).

Specific tests for ascites Completion

Flank dullness percussion over the flanks is Say that you would like to:
dull because of accumulated fluid in the Examine the rest of the abdomen looking for
paracolic gutters other problems, and in particular evidence of
Shifting dullness define the margin where intra-abdominal malignancy
the percussion note first becomes dull in the Continue to look for ankle and sacral
right flank; then ask the patient to roll away oedema (signs of hypoalbuminaemia)
from you, keeping your finger on the same Examine the chest for signs of right heart
point on the abdomen, wait for the fluid to failure.
resettle and then demonstrate the percussion
note has become resonant again (Fig. 41)
Fluid thrill with large volumes of ascites, a
Ask patient to place
transmitted thrill can be felt. Ask the patient
hand in midline
to place his hand parallel to the body over
the umbilicus, resting firmly on the abdomen
(Fig. 42). Tap gently (with a percussing Gently but rapidly
action) with your right hand onto his left Feel other side apply pressure to
flank, feeling the transmitted pulsation with for transmitted one side of the
your left hand resting on the right flank pulse abdomen
(Fig. 43).

Figure 42 Fluid thrill.

Figure 41 Shifting dullness to examine for the presence Figure 43 Testing for the presence of a fluid thrill in the
of ascites. presence of ascites.
Case 58 Abdomen and trunk 95

Table 16
QUESTIONS
Transudate (Protein Exudate (Protein
(a) What are the causes of ascites?
<30g/L) >30g/L)
Common: Cardiac failure Cirrhosis
Chronic liver disease Tricuspid regurgitation Malignancy
Right heart failure Constrictive pericarditis Lymphatic rupture
Intra-abdominal malignancy or damage
Hypoalbuminaemia
Uncommon:
Nephrotic syndrome (b) With which conditions would an exudate
Tuberculosis be expected?
Chylous ascites. See Table 16.

(c) What are the indications for the use of a


ADVANCED QUESTIONS shunt in the management of ascites?
(a) How would you perform an ascitic tap? The mainstay of treatment of ascites is to treat
the underlying condition and to place the
The procedure should be performed under patient on a weight reduction program, with the
sterile conditions and if the ascites is not help of diuretics, and a low-sodium diet. In
clinically apparent or easy to locate, it should diuretic-resistant ascites, shunting may be
be done by a radiologist under ultrasound performed in a number of ways:
guidance to prevent inadvertent injuries to
Peritoneovenous shunting (LeVeen shunt),
intra-abdominal structures:
where a subcutaneous silastic catheter is
Local anaesthetic is infiltrated and the site used to drain the fluid into the jugular vein
marked. A safe site is in the iliac fossa lateral The Denver shunt is a modification, adding a
to the linea semilunaris (this avoids the small subcutaneous pump that can be
inferior epigastric vessels) and should be in compressed externally
an area dull to percussion (fluid rather than
Transjugular intrahepatic portosystemic stent
air underneath)
shunt (TIPS), a side-to-side shunt stenting a
A narrow-gauge needle should be introduced channel between a branch of the portal vein
first to check the position before a larger- and the hepatic vein
gauge cannula is inserted into the abdomen
Note that interventional radiology techniques
When in position, a plastic tube can be have largely replaced the surgically difficult
connected to a urine bag in order to collect open techniques.
the ascitic fluid from the abdominal cavity
Samples of fluid are taken for:
Cytology (presence of any malignant cells) FURTHER READING
Protein (difference between exudate and Suzuki H, Stanley AJ: Current management and
transudate an exudate has protein novel therapeutic strategies for refractory
content of >30g/L) ascites and hepatorenal syndrome. QJM
Microbiology (to exclude bacterial 94(6):293300, 2001.
peritonitis as a complication). Yu AS, Hu KQ: Management of ascites. Clin
Liver Dis 5(2):541568, viii, 2001.

CASE 58 EPIGASTRIC MASS **

INSTRUCTION APPROACH
Examine this ladys abdominal system. Expose the patient and begin, as in Case 43,
with the hands.
96 Abdomen and trunk Case 59

VITAL POINTS Completion


Inspect peripheral signs Say that you would like to:
Carry on to complete the rest of the
Look for signs of anaemia in the hands and
abdominal examination.
eyes
Look for evidence of jaundice
Palpate the supraclavicular fossa for QUESTIONS
lymphadenopathy (especially for a Virchows
node in the left supraclavicular fossa). (a) What is the differential diagnosis?
As with right iliac fossa masses (see Case 55),
Inspect abdominal signs the best way to think about this answer is to
consider the possible diagnoses anatomically.
Comment on the presence of any scars You are less likely to forget any of the potential
answers.
There may be epigastric fullness.
Arising from the skin and soft tissues:

Palpate Sebaceous cysts


Sarcoma
Begin palpating as in Case 43, but stop when Lipoma
you find the mass and describe the mass fully Hernia (epigastric)
before moving on. Comment on the: Arising from the gastrointestinal tract (begin with
Size the stomach and move distally):
Surface Carcinoma of the stomach
Edge Hepatomegaly
Consistency Carcinoma of the pancreas (remember
Relations to the skin, to the costal margin, Courvoisiers law a palpable gall bladder in
to the abdominal muscles the presence of jaundice is not likely to be
Could the mass be hepatomegaly or due to gall stones)
splenomegaly? Pancreatic pseudocyst
Arising from the vascular system:
Finish your examination here Abdominal aortic aneurysm (see Case 115)
Retroperitoneal lymphadenopathy.

Ludwig Georg Courvoisier (18431918). Professor of


Surgery, Basle, Switzerland.

CASE 59 PLEURAL EFFUSION **

INSTRUCTION VITAL POINTS


Examine this gentlemans respiratory system. Inspect

In the hands and wrists, look for:


APPROACH
Digital clubbing
Expose the patient from the waist up and sit
Nicotine (tar) staining of the fingers
him at 45 on the bed. Begin by examining the
hands for peripheral stigmata of chronic Pale palmar skin creases secondary to
pulmonary disease. anaemia
Hypertrophic pulmonary osteoarthropathy
Case 59 Abdomen and trunk 97

In the neck, note the: Vocal resonance will also be reduced


Position of the jugular venous pulse Bronchial breathing may be heard if there is
Presence of supraclavicular associated consolidation of the lung
lymphadenopathy parenchyma.
Whether the trachea is central
Inspect the chest wall for: Finish your examination here
Scars
Abdominal breathing
Note the respiratory rate while you are Completion
completing the peripheral examination.
Say that you would like to:

Palpate Examine the sputum pot


Check the temperature
Check expansion of the chest wall, noting Examine for potential causes of a pleural
whether it is equal bilaterally. effusion (see below).

Percuss QUESTIONS

Percuss the chest wall from the upper zone (a) How may pleural effusions be classified?
down, comparing the percussion note on The protein content of a sample of effusion fluid
both sides is measured and the classification depends on
Repeat the process on the posterior chest this value:
wall (where effusions will be easier to hear) Transudate = protein <30g/L
The percussion note is stony dull on the Exudate = protein >30g/L
side of the effusion.
(b) What are the causes of a pleural effusion?
Auscultate See Table 17.

Auscultate using the bell over the apices and


the diaphragm elsewhere
Diminished breath sounds will be heard over
the effusion

Table 17

Transudate Exudate
Cardiac failure Malignancy
Medical disorders leading Primary lung tumour
to hypoalbuminaemia: Secondary (especially breast, GI, ovary)
Cirrhosis Lymphoma
Nephrotic syndrome Chylothorax secondary to malignant infiltration of lymph
Cardiovascular:
Pulmonary embolus/infarct
Dresslers syndrome (post-myocardial infarct)
Infections:
Pneumonia
Tuberculosis
Subphrenic abscess
Systemic diseases:
Rheumatoid arthritis
Systemic lupus erythematosus
98 Abdomen and trunk Case 60

ADVANCED QUESTIONS (b) Under what situations would a chest drain


be required to manage a pleural effusion?
(a) How would you diagnose and treat a Exudates that recur after aspiration require
pleural effusion? drainage and they may be placed on low
suction (2.55kPa); unlike drainage of a
When the diagnosis has been made and
pneumothorax, these drains may be interrupted
confirmed with a plain radiograph of the chest,
periodically to allow mobilization. The drain is
a sample should be taken for:
left until the volume of fluid is <100mL/day and
Biochemistry (including protein) there is radiological re-expansion of the lung.
Microbiology
Cytology
Pleural taps are most easily performed in the William Dressler (18901969). Cardiologist,
mid-scapular line with the patient leaning Maimonides Hospital, New York. He described fever,
forward over a table within the zone of clinical chest pain, pericardial and pleural rub developing
signs (stony dull to percussion/reduced breath 210 weeks after a myocardial infarction. It is thought
sounds etc.). Closed-needle biopsy of the to be due to an antibody reaction to heart muscle.
pleura can also be performed combined with
cytology, this will diagnose 90% of malignancies
and 75% cases of tuberculosis.
Treatment: FURTHER READING
If the patient remains symptomatic the fluid Ferrer J, Roldan J: Investigation of pleural
should be drained with a 14 gauge cannula effusion. Eur J Radiol 34(2):7686, 2000.
Occasionally the pleural space may be Peak GJ, Morcos S, Cooper G: The pleural
obliterated (pleurodesis) using talcum cavity. BMJ 320:13181321, 2000.
powder.

CASE 60 DYSPHAGIA **

INSTRUCTION Do you ever get any pain when you


swallow or at other times?
Ask this lady a few questions about her
Have you had any weight-loss?
swallowing.
Have you had a chest infection recently?

APPROACH QUESTIONS
It is useful to group your questions in terms of
aetiologies and to let the examiner know you (a) What are the causes of dysphagia?
are conscious of the possible more serious See Table 18.
pathologies.
(b) Which of these conditions are predisposing
factors for the development of oesophageal
VITAL POINTS
carcinoma?
Are you having difficulty swallowing liquids,
Barretts oesophagus
or solids, or both?
Strictures (especially chemical)
Did the problem start suddenly or was the
onset gradual? Achalasia
Do you ever regurgitate food? PlummerVinson syndrome.
Can you eat a full meal? (c) How do carcinomas of the oesophagus
How long have you had this problem for? present?
Where does the food stick in the back of
The characteristic presentation is insidious
the throat, bottom of the neck or bottom of
with progressive weight-loss and dysphagia
the chest?
Case 60 Abdomen and trunk 99

Table 18 Causes of dysphagia


Mechanical obstruction Coordination abnormalities
Within the lumen: Motility disorders:
Foreign body Diffuse oesophageal spasm
Oesophageal web (e.g. in scleroderma) Achalasia
PlummerVinson syndrome (PatersonBrownKelly syndrome) Neurological disease:
In the wall: Myaesthenia gravis
Carcinoma of the oesophagus Bulbar palsy including motor neurone
Oesophagitis (due to burns or chronic reflux) disease
Barretts oesophagus Cerebrovascular accident (with
Benign oesophageal stricture involvement of the 9th, 10th or
Post-radiation/chemical strictures 12th cranial nerves or a
Outside the wall: coordination difficulty)
Retrosternal goitre
Lung carcinoma
Pharyngeal pouch

The patient initially has difficulty swallowing a rats tail appearance on the barium
solids and often describes the food getting swallow and there is no gas bubble in the
stuck in the lower part of the oesophagus stomach
They may also describe odynophagia pain Endoscopy:
on swallowing If the diagnosis is in doubt, endoscopy with
Occasionally they present with aspiration biopsies and brushings should be performed
pneumonia. to exclude a carcinoma
Test of physiological function:
(d) What other conditions cause odynophagia?
24-hour oesophageal pH studies (to exclude
Infections within the oesophagus (especially
reflux) and manometry.
candidiasis, herpes simplex)
Pharyngitis
Occasionally ulceration over the lower third D. R. Paterson (18631939). ENT surgeon at the
of the oesophagus. Cardiff Royal Infirmary, described the association of
glossitis, anaemia and dysphagia.
A. Brown-Kelly (18651941). British ENT surgeon; also
ADVANCED QUESTIONS described congenital stenosis as well as problems
with motility.
(a) Which investigations would you use in a
patient whom you thought might be suffering H. S. Plummer (18741936). US physician working at
from a dysmotility problem? the Mayo clinic, who investigated the therapeutic use
of oxygen in respiratory disease, and was interested in
A barium swallow may be useful: the diagnostic and therapeutic use of bronchoscopy
In diffuse oesophageal spasm, a motor and endoscopy.
disorder of smooth muscle, below the aortic P. P. Vinson (18901959). US physician also working
arch, normal coordinated peristalsis is at the Mayo clinic.
replaced by multiple spontaneous
contractions and this gives the characteristic
corkscrew oesophagus appearance
Achalasia is a motility disorder due to loss of
FURTHER READING
ganglia in the myenteric plexus, causing
incomplete relaxation of the lower Owen W: Dysphagia. BMJ 323:850853, 2001.
oesophageal sphincter; the oesophagus has
100 Abdomen and trunk Case 61

CASE 61 ENLARGED KIDNEY **

INSTRUCTION
Examine this gentlemans abdominal system.

APPROACH
Expose the patient and begin with the hand
(Case 43).

VITAL POINTS
Inspect

Inspection is likely to be normal.


Figure 44 Balloting the right kidney.

TOP TIP 1
Demonstrate to the examiner that the mass is an
enlarged kidney using the acronym SPRUE as used for
hepato-/splenomegaly (see Cases 46 and 49).
Site of enlargement: from the costal margin towards
the ipsilateral iliac fossa
Percussion note: resonant due to gas in the overlying
colon
Respiration movement: it descends
Unique features: ballottable and bimanually palpable
it, reflecting the
Edge: in most instances only the lower pole of the
kidney is palpable. This is smooth and hemi-ovoid.

Figure 45 Balloting the left kidney.


Palpate

In advanced renal tumours, there may be Finish your examination here


supraclavicular lymphadenopathy
Note the presence of a mass in the left or QUESTIONS
right loins or upper quadrants
This mass can be balloted between one (a) What is the differential diagnosis for an
hand on the anterior abdominal wall and the enlarged kidney?
other behind the patient in the renal angle
Congenital:
(Fig. 44 and Fig. 45). It relies on the fact that
the kidney is reducible into the loin. Cystic disease (including polycystic kidney
disease)
Horseshoe kidney
Hypertrophic single kidney
Acquired:
Diseases specific to the kidney:
Solitary cysts
Tumours
Case 61 Abdomen and trunk 101

Table 19 Polycystic kidney disease


Adult Infantile
Inheritance Autosomal dominant Autosomal recessive
Incidence 1 in 500 1 in 500040000
Genetics Chromosomes 4,16 6
Age of presentation 30s50s Perinatal
Pattern of presentation Hypertension Oligohydramnios
Haematuria Large liver and kidneys
Loin pain Chronic renal failure
Pattern of enlargement Asymmetrical Symmetrical
Liver involvement Adult liver cysts common Always congenital hepatic fibrosis
Sometimes biliary ectasia
Other systemic involvement Intracranial aneurysms None
Colonic diverticulae
Mitral regurgitation
Prognosis Often require dialysis but good prognosis All die by age 20, but often in
neonatal period

Hydronephrosis Raised erythrocyte sedimentation rate


Pyonephrosis Hypercalcaemia
Perinephric abscess Left-sided varicocele.
Renal vein thrombosis
As part of systemic disease:
ADVANCED QUESTIONS
Diabetes
Amyloidosis (a) Simple cysts are found in 33% of patients
Systemic lupus erythematosus. by the age of 60. How should they be
managed?
(b) What are the differences between infantile
and adult polycystic kidney disease? History and clinical examination:

See Table 19. They usually present incidentally but


occasionally with a renal mass or haematuria
(c) What is the normal mode of presentation Investigations:
of renal cell carcinomas? Urine cytology
Usually occur in the over 50s, although it Blood tests would be expected to be normal
may affect younger patients. Indeed, it is the A renal ultrasound scan shows a cyst with a
commonest cause of a renal mass in women smooth outline, sharply defined thin wall and
of childbearing age. no internal echoes (which imply solid
Classic presentation is a triad (Becks triad) components)
of: Treatment:
Haematuria The major differential diagnoses would be
Mass with a renal tumour and adult polycystic
Loin pain kidney disease and if there is any doubt of a
Other presentations: tumour, then the cyst fluid may be sent for
Incidentally found on abdominal cytological analysis.
ultrasound or CT scans performed for
(b) What radiological features would make
another reason (very common)
you suspicious of an occult renal cell
Pyrexia of unknown origin
carcinoma?
Anaemia of chronic disease
More worrying features for a tumour would
Polycythaemia (due to erythropoietin
include:
secretion by the tumour)
102 Abdomen and trunk Case 62

Thick, or irregular wall Godley PA, Taylor M: Renal cell carcinoma. Curr
Extensive calcification within the cavity or Opin Oncol 13(3):199203, 2001.
wall of the cyst Tomson CR: Recent advances: nephrology.
Multilocular cysts. BMJ 320(7227):98101, 2000.
www.pkdcure.org website of the polycystic
foundation, a worldwide organization devoted to
FURTHER READING
determining the cause and treatment for
Gladman MA, Macdonald D, Webster JJ, et al: polycystic kidney disease.
Renal cell carcinoma in pregnancy. J R Soc
Med 95:199201, 2002.

CASE 62 COMMON SURGICAL SCARS *

INSTRUCTION VITAL POINTS


Inspect this gentlemans abdomen. Inspect

Comment on the presence of any surgical scars


APPROACH (Fig. 46):
Introduce yourself and expose the patients Use the correct technical names for the
abdomen, positioning him flat on the bed. Leave scars where possible, if not describe the
his external genitalia covered at this point to anatomical position of the scar and indicate
maintain his dignity but expose the whole of the whether it looks well healed or recently
top half of his body down to the symphysis formed
pubis. If the examiner indicates the patient
You should not guess at the operation the
should leave his shirt on, then expose from
patient underwent in order to produce the
above the xiphisternum to the symphysis pubis.
scar unless asked specifically to do so by
the examiner

Thoraco-abdominal

Kocher's
(subcostal)
Rooftop

Right
paramedian

Midline

Gridiron

Inguinal hernia scar

Transverse Pfannenstiel
(Lanz)
Figure 46 Common surgical scars. Remember to examine the flanks for the loin incision of a nephrectomy.
Case 63 Abdomen and trunk 103

Finish your examination here

QUESTIONS

(a) Which operation do you think this patient


may have had?
Although you should not voluntarily make a
guess at which operation the patient may have
had in order to produce the scar, in the short
cases and OSCEs the examiner may ask you
for an educated opinion.

Figure 47 A midline laparotomy scar. There may be a few pointers:


Is there evidence of a new or old stoma site
(from a bowel operation)?
A midline laparotomy scar is the most Is there evidence of a small incision to one
common incision to be revealed in side of the scar (from a drain this may
examinations (Fig. 47). have been due to a bowel operation)?
Are there also scars in the groins (perhaps
ilio-femoral segment surgery in a patient who
TOP TIP has also had an abdominal aortic aneurysm
When you see a scar on the abdomen, always think repaired through a midline incision)?
of the presence of an incisional hernia (see Case 47). Are there striae gravidarum (Pfannenstiel
From the end of the bed, ask the patient to raise their incision may have been for a Caesarean
head off the bed and cough in order to demonstrate this. section)?
Continue to examine each scar more carefully during the
palpation section to rule out incisional herniae.
Herman Johannes Pfannenstiel (18621909).
Gynaecologist from Breslau who described the popular
curved suprapubic incision.

CASE 63 BREAST EXAMINATION GENERAL


APPROACH *

INSTRUCTION VITAL POINTS


Examine this ladys breasts. Inspect

Begin by asking the patient if they have noticed


APPROACH a lump in the breast, and which breast this is in.
Before beginning this examination, whether Stand away from the couch and look at the
in a long case or short case, you must be patients breasts from the front.
accompanied by a chaperone, and a nurse
will often be available for this purpose in the Ask the patient to lift both her hands above
examination. Increasingly in OSCE her head, stretching the skin and
examinations, models wearing breast emphasizing any tethering of a breast tumour
prostheses, containing pathological signs, are to the skin
used rather than actual patients for obvious Next, ask her to place both her hands on her
reasons. hips, and while watching her breasts, ask her
to press firmly into the hips with both hands;
Expose the patient from the waist up and lie her
this emphasizes any attachment of a breast
at 45 on the couch.
tumour to the underlying pectoralis major
muscle, which contracts with this manoeuvre
104 Abdomen and trunk Case 63

Move closer to the patient and look more Imagine the breast as a clock face and make
closely at the breasts: sure that each area is palpated
Inspect the nipple and areola (see Top Tip Pay particular attention to the axillary tail and
below) underneath the nipple where masses are
Inspect the rest of the breasts for: frequently missed
Asymmetry in size or shape When palpating the abnormal breast, ensure
that the area of abnormality that you have
Skin changes or subcutaneous nodules
found is the same that the patient has
Previous scars from excision of benign or
noticed
malignant lumps.
The location of the lump within the breast
should be named according to the quadrant
TOP TIP (upper outer, lower outer, upper inner, lower
inner)
The seven Ds of nipple signs
Describe any lump as in Case 1, noting
When inspecting the nipple, or taking a history of whether it is attached to the skin or the
nipple symptoms, look for: underlying muscle
Palpate the five areas of lymph nodes in the
Discolouration
axilla medial, lateral, anterior, posterior and
Discharge apical (Fig. 48) palpating the left axilla with
Depression (often referred to as inversion) your right hand and vice versa
Deviation Palpate the supraclavicular fossa for
lymphadenopathy (Fig. 48).
Displacement
Destruction
Finish your examination here
(Duplication unlikely in the exam).

Palpate
Completion
Ask the patient to position her hands in her lap.
Say that you would like to continue with:
Ask her to tell you where the lump in the breast
is first, but start by palpating the normal breast. Percussing and auscultating the chest
Palpate systemically round the breast, using Palpating the abdomen for hepatomegaly
the tips of your fingers held together Percussing the axial spine for tenderness
Retract the breast with the left hand and use Completing a general physical examination
the right hand to palpate each of the four to determine the patients fitness for surgery.
quadrants

Axillary chain
Apical group
Medial group
Pectoralis minor
Anterior group Axillary
Internal chain
mammary Posterior group
chain Lateral group
Axillary vein

Figure 48 Lymph nodes and lymphatic drainage of the breast.


Case 64 Abdomen and trunk 105

CASE 64 BREAST LUMP *

INSTRUCTION (b) How would you classify breast disease?


Generally, disease of the breast is classified as
Examine this ladys breasts.
follows:
Malignant disease:
APPROACH Ductal carcinoma (also referred to as cancer
Expose the patient to the waist to adequately of no specific type), accounts for ~70% of
expose the breasts and chest wall. cancers
Lobular carcinoma, accounts for ~20% of
cancers
VITAL POINTS Other (mucinous, tubular, medullary),
accounts for ~10% of cancers
Inspect
Benign disease:
The lump may be tethered to the skin or Congenital abnormalities (supernumerary
underlying muscle nipples, hypoplasia, etc.)
There may be associated nipple changes, or Aberrations of normal development and
changes to the skin of the breast involution (ANDI)
There may be scars from previous surgery. Fibroadenomas
Breast cysts
Palpate Sclerotic/fibrotic lesions
Non-ANDI conditions:
As in Case 63, begin with the normal breast, Infective
examining with the patients hands in her lap. Lipomas
When you have identified the lump, describe the Fat necrosis
lump in detail: Single lumps in the breast are most likely to be:
Site (position): name the quadrant the lump Fibroadenomas
is located within Breast cysts
Size: measure the lump approximately Fat necrosis
Surface: smooth/irregular/nodular Breast cancer.
Edge: well/poorly defined
Consistency: soft/firm/hard (c) What features of the lump would make you
Tenderness suspicious that it is a breast cancer?
Fluctuation Surface: irregular or nodular
Fixation: to skin or the underlying chest wall Edge: poorly defined, with areas which are
Continue as in Case 63 by palpating the axilla more like normal breast tissue in between
and supraclavicular fossa and complete the more abnormal areas
examination as in Case 63. Consistency: breast tumours are usually firm,
rather than hard
Tenderness: usually non-tender
Finish your examination here
Fluctuation: usually not fluctuant
Fixation: to skin or the underlying chest wall
QUESTIONS Any involvement of the nipple in the lump or
concurrent nipple changes
(a) How would you investigate this patient? Also mention the presence of any lymphaden-
You must mention a triple assessment, which opathy, or features in the history or physical
consists of: examination, that suggest disseminated disease.
Clinical: history and physical examination
Radiological: ultrasound or mammography
Pathological: cytology (fine-needle aspiration)
or histological (Tru-Cut biopsy).
106 Abdomen and trunk Case 65

Drukker BH: Fibrocystic change of the breast.


FURTHER READING
Clin Obstet Gynecol 37(4):903915, 1994.
Brennan M, Houssami N, French J: Management Houssami N, Cheung MN, Dixon JM:
of benign breast conditions. Part 2 breast Fibroadenoma of the breast. Med J Aust
lumps and lesions. Aust Fam Physician 174(4):185188, 2001.
34(4):253255, 2005.

CASE 65 POST-MASTECTOMY BREAST *

INSTRUCTION QUESTIONS
Examine this ladys breasts.
(a) What are the indications for mastectomy?
Modern oncological surgery involves breast
APPROACH conservation wherever possible and usually
Expose the patient and begin examining the involves wide local excision and assessment of
breasts as in Case 63. the axillary contents (sentinel node biopsy,
axillary node sampling or clearance).
There are some occasions where mastectomy is
VITAL POINTS still considered, although these indications are
Inspect by no means absolute and are increasingly
challenged by surgical oncologists:
Note the asymmetrical chest wall and Patient preference
describe the location of the scar Clinical evidence of multifocal/multicentric
Look at the surrounding skin and into the disease
axilla determining whether there has also Large lump in small breast tissue (depends
been radiotherapy to the surrounding area on the size of the breast but often defined as
Ask the patient to press her hands into her a lump >4cm)
hips, ascertaining whether the pectoralis Large area (>4cm) ductal carcinoma in situ
major remains underneath the mastectomy. (DCIS)
Involvement of the nipple.
Palpate
ADVANCED QUESTIONS
It is likely that you would not have to
continue to palpate the breast, but if the (a) What different types of mastectomy can be
examiner wants you to continue, you should performed?
examine the remaining breast as in Case 63
Simple mastectomy removal of the breast
Palpate the axilla and supraclavicular fossa
alone
for lymphadenopathy.
The following procedures are rarely performed
now as survival benefit has not been
Finish your examination here demonstrated:
Modified radical mastectomy (Patey)
removal of the breast, pectoralis minor and
Completion the axillary structures
Radical mastectomy (Halsted mastectomy)
Say that you would like to: removal of the breast, pectoralis major and
Examine the abdomen, neck, lung fields and minor, and the axillary contents
spine Extended radical mastectomy as for the
Examine the ipsilateral arm for radical procedure but also removing the
lymphoedema. internal mammary nodes (between the 2nd
and 4th anterior intercostal spaces).
Case 66 Abdomen and trunk 107

The disadvantage with simple mastectomy, The option of reconstructive surgery (see
although least disfiguring, is that the axilla still Case 66) should be discussed.
needs to be managed following surgery (for
instance with radiotherapy), whereas the other (c) When should the drains be removed
types involve surgical dissection of the axillary post-surgery?
nodes. Postoperative radiotherapy is more Often surgeons place two drains, one in the
common for limited resections. axilla and one at the site of surgery within
the breast tissue
(b) How would you prepare a patient prior to The drains are usually left for 35 days, or
breast surgery? until the drainage volume is <50mL in 1 day
Physical preparation: Patients can safely be sent home with drains
Mark the side prior to anaesthetic in place and district nurse support, as
Explanation of the procedure should include otherwise they may spend a week in
the use of a suction drain to close the cavity hospital.
following surgery and decrease the risk of
scar and haematoma formation William Stewart Halsted (18521922). First Professor
An anaesthetized patch of skin in the axilla of Surgery at Johns Hopkins Medical School,
and the upper medial part of the arm will Baltimore, where Harvey Cushing was his assistant.
follow division of the intercostobrachial nerve He also introduced rubber gloves to surgery (made for
(T1), which is divided as it emerges from the him by the company Goodyear) and was one of the
chest wall just posterior to the origin of the first to use regional anaesthesia with cocaine.
pectoralis minor muscle
Anaesthetic work-up should include chest
X-ray to exclude pulmonary metastasis
Psychological preparation: FURTHER READING
All patients should see the breast care nurse Eatock J: Counselling in primary care: past,
preoperatively and the reasons for present and future. Br J Guidance & Counselling
mastectomy should be discussed fully 28(2):161173, 2000.

CASE 66 BREAST RECONSTRUCTION *

INSTRUCTION Flap reconstruction


Examine this ladys chest wall.
More extensive surgical scarring
Scars extend over the back or abdominal
APPROACH wall
Look at the patients back and see the
Expose the patient as in Case 63.
recess where the latissimus dorsi has been
removed
VITAL POINTS Ask the patient to lift their head off the bed
(when lying flat) to see the recess in the
Inspect rectus abdominis muscle.

Note the asymmetry of the chest wall


Implant reconstruction
The prosthesis can be identified by the
presence of surgical scars and by a different
Shape is rounder than a normal breast
shape from the normal breast contour.
Lie of the breast is usually higher
A Becker implant may have a palpable
subcutaneous filling port in the axilla.
108 Abdomen and trunk Case 66

Finish your examination here Pedicled/free transverse rectus abdominis


myocutaneous (TRAM) flap
Free deep inferior epigastric perforator
(DIEP) flap: spares rectus muscle/
TOP TIP destruction of abdominal wall
Do not be embarrassed to ask the patient for Free superficial inferior epigastric artery
permission to examine the reconstructed breast and the (SIEA) flap: vessel absent in one-third of
other breast. Do so with confidence. In this situation it is patients
useful to inform the examiner of your findings on Latissimus dorsi (LD) flap.
inspection prior to palpation.
(b) What are the advantages and
disadvantages of using an implant?
See Table 20.
ADVANCED QUESTIONS
(c) What are the advantages and
(a) What are the possible types of breast disadvantages of myocutaneous flaps?
reconstruction surgery? See Table 21.
Timing:
Immediate (advantages: single operation; FURTHER READING
preservation of/better quality skin flaps)
Cordeiro PG:. Breast reconstruction after
Delayed (allows increased patient decision
surgery for breast cancer. N Engl J Med
time, avoidance of delay/detrimental effects
359(15):15901601, 2008.
of adjuvant therapy)
Malata CM, McIntosh SA, Purushotham AD:
Technique:
Immediate breast reconstruction after
Tissue expansion reconstruction (implants) mastectomy for cancer. Br J Surg 87(11):1455
Subcutaneous prosthesis 1472, 2000.
Submuscular implant Malyon AD, Husein M, Weiler-Mithoff EM: How
Autologous tissue reconstruction many procedures to make a breast? Br J Plast
(myocutaneous flaps) Surg 54(3):227231, 2001.

Table 20 Using an implant


Advantages Disadvantages
Technique simpler than flaps Cosmetic result less satisfactory than using a flap
Place under the pectoralis muscles to reduce the Requires plenty of available skin following surgery
incidence of contraction of the capsule
Can be performed at the time of the mastectomy or at Lies above the natural inframammary fold, leaving
a later date the breast higher than the other one

Table 21 Myocutaneous flaps


Advantages Disadvantages
Useful where remaining skin and muscle May need to be performed in combination with plastic surgeon
in short supply, e.g. following extensive Greater blood loss
surgery Greater operating time and operative complications
Cosmetic results can be very good Use of rectus abdominus may be impossible if the patient has
Suitable for use post-mastectomy had previous abdominal surgery
Suitable for salvage after local recurrence Late complications include flap necrosis and infection
Case 67 Abdomen and trunk 109

CASE 67 GYNAECOMASTIA *

Testicular atrophy (bilateral


INSTRUCTION
cryptorchidism; post orchitis/bilateral
Look at this gentlemans chest. torsion)
Klinefelters syndrome
Hyperprolactinaemia
APPROACH
Renal failure
Expose the patient as for the chest examination Androgen resistance
and position him at 45.
Testicular feminization
Increased oestrogens:
VITAL POINTS Increased secretion
Inspect Testicular tumours
Lung carcinoma
Note the presence of unilateral or bilateral Increased peripheral aromatization
breast swellings. They may be simply small Liver disease
breast buds, or more significant amounts of Adrenal disease
breast tissue may be present. Thyrotoxicosis.
Potions (i.e. drugs):
Finish your examination here Recreational drugs: marijuana,
amphetamines, diazepam
Gastrointestinal drugs: cimetidine, ranitidine
Completion Cardiovascular drugs: digoxin, angiotensin
converting enzyme inhibitors (e.g. captopril,
Say that you would like to: enalapril), spironolactone, nifedipine,
Look for a cause of the gynaecomastia, verapamil
including examination of the external Antibiotics: metronidazole, isoniazid,
genitalia, examining for clinical signs of ketoconazole.
thyroid dysfunction, and for signs of liver
disease
Ask the patient some directed questions,
ADVANCED QUESTIONS
especially about the use of prescription or (a) How might you investigate this patient?
recreational drugs.
The level of investigation depends very much on
the clinical situation, and some patients may
QUESTIONS require no investigations at all. Possible useful
investigations would include:
(a) What are the causes of gynaecomastia?
Plasma alpha feto-protein and beta-human
Causes may be classified as the 3 Ps: chorionic gonadotrophin raised levels may
indicate a testicular tumour
Physiological:
Testosterone and luteinizing hormone levels
Particularly common at puberty where the to demonstrate hypogonadism
patient may have noticed unilateral or Thyroid function tests.
bilateral gynaecomastia may enlarge to
considerable size but usually disappear (b) What would make you concerned the
before adulthood. patient may have a breast cancer?
Pathological (due to relative hyperoestrogenism): Male breast cancer is responsible for 1% of all
Decreased androgens: cases of breast cancer in the UK. Features that
would be suspicious include:
Reduced androgen production (i.e.
hypogonadism) Older age
Unilateral gynaecomastia
110 Abdomen and trunk Case 68

Firm or hard nodules within the breast tissue


Harry Fitch Klinefelter Jr (born 1912). Associate
(the texture is normally rubbery or soft)
Professor of Medicine at Johns Hopkins University
Remember to examine the axillary and
the syndrome in its full form is known as Klinefelter
supraclavicular fossae for lymphadenopathy.
ReifensteinAlbright syndrome.
If in doubt, the patient should undergo triple
Fuller Albright (19001969). Professor of Medicine,
assessment (see Case 64) including imaging
Harvard Medical School who contributed a huge
and pathological evaluation.
amount to the study of metabolic diseases.
E.C. Reifenstein also worked with Albright. Reifenstein
FURTHER READING syndrome is male pseudohermaphroditism.
Neuman JF: Evaluation and treatment of
gynecomastia. Am Fam Physician 55(5):1835
1844, 18491850, 1997.

CASE 68 CHEST POST-LOBECTOMY/


PNEUMONECTOMY *

INSTRUCTION Percuss
Examine this gentlemans respiratory system.
Percussion note over the side of surgery is
hyper-resonant.
APPROACH
Position the patient and begin to examine the Auscultate
chest as in Case 59, beginning with the hands,
but expect that the examiner may move you Breath sounds are harsher over the side of
directly on to examining the chest wall. the pneumonectomy.

Finish your examination here


VITAL POINTS
Inspect QUESTIONS
Note the lateral thoracotomy scar over the (a) What are the indications for
chest wall the scar begins at the sternal lung resections?
end of the 5th or 6th intercostal space and
curves posteriorly and upwards, ending 90% of lung resections in the Western world
midway between the spine of the scapula are performed for bronchial carcinoma
and the thoracic vertebral spines Other indications include:
Scars at the sites of chest drains may be Traumatic injury
present Bronchiectasis
Muscle bulk over the side of the scar may be Chronic infection including tuberculosis
reduced as parts of serratus anterior and Benign tumours (e.g. carcinoid)
latissimus dorsi may have been removed. Metastatic tumours.

Palpate (b) What are the types of lung resection?


Lobectomy is the excision of a single lobe of
The trachea is deviated away from the side the lung
of surgery Pneumonectomy is the excision of an entire
Expansion will be reduced over the side of lung
surgery. Non-anatomical resections are often
performed for traumatic injury
Case 69 Abdomen and trunk 111

Sleeve resection is the resection of a lobe mortality of pneumonectomy. Risk of operative


including its bronchial origin with mortality is higher with:
re-anastomosis of the proximal and distal Higher American Society of
bronchus. Anaesthesiologists score (preoperative
morbidity)
ADVANCED QUESTIONS Age >70 years
Poor respiratory function, especially FEV1/
(a) What is the operative mortality for FVC ratio of <55%.
lung resections?
A retrospective study looked at 442 patients FURTHER READING
who had undergone lobectomy and
Ferguson MK, Karrison T: Does
pneumonectomy over an 18-year period (see
pneumonectomy for lung cancer adversely
Further Reading). The operative mortality for
influence long-term survival? J Thorac
lobectomy was 7% and for pneumonectomy
Cardiovasc Surg 119(3):440448, 2000.
12%, but no difference in long-term survival
was detected. Techniques such as sleeve Groenendijk RP, Croiset van Uchelen FA, Mol
lobectomy and bronchoplasty have been SJ, et al: Factors related to outcome after
developed as a result of the high operative pneumonectomy: retrospective study of 62
patients. Eur J Surg 165(3):193197, 1999.

CASE 69 MEDIAN STERNOTOMY *

INSTRUCTION Finish your examination here


Examine this gentlemans chest.
QUESTIONS
APPROACH (a) What are the indications for
Expose the patient to examine the chest as in median sternotomy?
Case 59. Emergency procedures, e.g. following
penetrating chest trauma
Cardiac surgery
VITAL POINTS Resection of lung cancer.
Inspect There is some evidence that median sternotomy
may be as efficient an approach to the lung as
The median sternotomy scar runs from the the lateral thoracotomy (see Further Reading).
suprasternal notch vertically in the midline to
the xiphisternum
Subtotal median sternotomy involves an
ADVANCED QUESTIONS
incision from the sternomanubrial junction to (a) What are the principles of
the 5th or 6th intercostal space.
cardiopulmonary bypass?
The aim of bypass is to provide a systemic
Palpate
circulation while the heart is stopped and
emptied of blood (Fig. 49)
Occasionally there is non-union of the two
Blood is drained by gravity from the right
sides of the manubrium sternum when a
heart to a circuit where, after gas and
click can be felt on palpation
temperature exchange, it is returned to the
Unless there is co-morbidity, the examiner
arterial side of the circulation
wants you to pick up that the rest of the
The components of the perfusion circuit are:
thoracic examination is likely to be normal.
An oxygenator: oxygen added and carbon
dioxide removed from the blood
112 Abdomen and trunk Case 70

Aorta

Arterial line

Right Cardiotomy suction


atrium

Air bubble
detector Cardiotomy
reservoir

Oxygenator

Arterial Heat
filter exchanger

O2
Roller pump

Hot/cold
water
Roller pump
Figure 49 Components of a cardiopulmonary bypass circuit.

A heat exchanger: initially blood is cooled, Cerebral damage due to ischaemia (12%)
later in the procedure it is warmed again Microemboli, e.g. to kidneys, terminal
Cardiotomy suction: pericardial sump arteries in limbs, retina.
suckers return spilt blood to the
circulation
Roller pump: returns blood to the aorta
FURTHER READING
Arterial line filter: removes debris and Asaph JW, Handy JR Jr, Grunkemeier GL, et al:
matter from the circulation. Median sternotomy versus thoracotomy to
resect primary lung cancer: analysis of 815
(b) What are the major complications cases. Ann Thorac Surg 70(2):373379, 2000.
of bypass?
Systemic activation of inflammatory
mediators causes coagulopathy

CASE 70 TESTICULAR TUMOUR *

INSTRUCTION APPROACH
Examine this gentlemans scrotum. See Case 51.
Case 70 Abdomen and trunk 113

Occasionally a long-standing hydrocoele


VITAL POINTS
may develop calcification and become
Inspect harder, clinically similar to a tumour
Tumours occasionally grow locally to
The scrotum may look normal but often an become adherent to the inside of the scrotal
enlarged testis is visible. skin, again it is possible for a chronic
hydrocoele to mimic this, but the level of
suspicion of a tumour would be high.
Palpate
(b) How do testicular tumours usually
The mass is: present?
Usually inseparable from the testis The commonest presentation is a painless
Hard, irregular, nodular lump or a dull ache in one testis in a young
Non-tender man
Not transilluminable Occasionally there is a history of trauma
Distinct from the superficial inguinal ring (you accompanying the discovery of the mass
can get above the mass) 10% present with an acutely painful testis
It may have an associated hydrocoele and there (which must be distinguished from a
may be some thickening of the spermatic cord testicular torsion)
due to malignant infiltration. If para-aortic nodes have become infiltrated
with metastases, the patient may complain
of back pain.
Finish your examination here
(c) How is a testicular tumour removed?
Through an inguinal approach, with early
Completion clamping of the testicular artery and vein within
the spermatic cord before the testis is mobilized
Say that you would like to: out of the scrotum this prevents intraoperative
seeding of tumour up the testicular vein.
Examine the contralateral scrotum
Continue to examine for abdominal
lymphadenopathy ADVANCED QUESTIONS
Perform an examination of the abdomen (for
hepatomegaly) and chest (for thoracic (a) What is the classification of testicular
metastates). malignancies?
Almost all are seminomas or teratomas (Table
QUESTIONS 22), other types are:
Embryonal carcinoma (arising from a very
(a) What is the differential diagnosis? primitive germ cell)
Testicular tumours can be mimicked by Choriocarcinoma
chronic or old infection leading to scarring, Yolk sac tumour
such as in orchitis or tuberculosis Leydig cell tumours associated with
gynaecomastia, but only 10% are malignant

Table 22

Teratoma Seminoma
Age of presentation 2030 years 3040 years
Tumour markers AFP and hCG raised in 90% Usually normal
Treatment of early disease Chemotherapy (often only two cycles) Radiotherapy to the para-aortic
nodes single dose of cisplatin
Treatment of advanced disease Combination chemotherapy Adjuvant chemotherapy, either
single dose or in combination
114 Abdomen and trunk Case 71

Sertoli cell tumours also produce FURTHER READING


gynaecomastia
Lymphoma most commonly in patients Dearnaley D, Huddart R, Horwich A: Regular
who have generalized lymphoma elsewhere review: Managing testicular cancer. BMJ
and is generally associated with a poor 322(7302):15831588, 2001.
prognosis. Oliver RT: 2001 Testicular cancer. Curr Opin
Oncol 13(3):191198.

Franz von Leydig (18211908). German histologist www.icr.ac.uk/everyman/about/testicular.html


who first described the androgen-producing Leydig guide to testicular self-examination for patients.
cell.
Enrico Sertoli (18421910). Professor of Experimental
Physiology, Milan.

CASE 71 ENTEROCUTANEOUS FISTULA *

Drips and parenteral nutrition


INSTRUCTION
Catheters/central venous pressure lines.
Inspect this gentlemans abdomen and
comment on what you can see.
Finish your examination here

APPROACH
Expose the patient as in Case 43 and inspect Completion
the abdominal wall; do not begin with the hands
as you have been given a specific instruction to Say that you would like to:
inspect the abdomen. Examine the patient looking for an underlying
aetiology (see below).
VITAL POINTS
Inspect QUESTIONS
(a) What is the definition of an
Describe the appearance of the fistula: enterocutaneous fistula?
Site
A fistula is an abnormal connection between
Size
two epithelial or endothelial surfaces
Discharge (fluid/solid/colour) material may
An enterocutaneous fistula is an abnormal
be bile or faeces, etc.
connection between the gastrointestinal tract
Surrounding skin (may be damaged by and the skin.
irradiation, inflammatory bowel disease or
chemical irritation from small intestine (b) What is the aetiology of enterocutaneous
contents) fistulae?
Describe the rest of the abdominal wall: Inflammation:
Presence of recent scar; an anastomotic leak Inflammatory bowel disease, especially
may have led to the fistula Crohns disease
Previous surgery especially for malignancy Diverticular disease
or inflammatory bowel disease
Tuberculosis
Presence of a stoma, healed stoma or drain
Malignancy:
sites
Comment on any other clues around the bed: Often following spontaneous rupture and
abscess formation by the tumour
General condition of the patient (anaemic,
cachectic, etc.)
Case 71 Abdomen and trunk 115

Radiotherapy: A fistulogram is the injection of contrast


Pelvic irradiation can damage the intestine material into the fistula opening in order to
see (using screening) where the fistula
Trauma:
connects to the bowel.
Penetrating wounds to the abdomen,
especially involving the perforation of several (c) What are the principles of treatment?
separate loops of bowel with significant Early intervention to close fistulae should be
contamination and sepsis avoided due to the high mortality due to sepsis,
Post-surgery: electrolyte imbalance and malnutrition.
Anastomotic leak, often following primary Therefore, non-operative therapy (see Top Tip,
anastomosis in contaminated conditions, e.g. below) is favoured in the first instance,
sepsis or distal obstruction. particularly as the majority will close
spontaneously. In the absence of diseased
bowel (inflammation/malignancy) or distal
ADVANCED QUESTIONS obstruction, the majority will close
spontaneously within approximately 6 weeks. If
(a) How may the anatomical locations of these it has not closed by 12 weeks, it is unlikely to
fistulae be classified? do so and definitive surgery should be planned.
A high intestinal fistula involves the stomach,
duodenum, jejunum and ileum and results in
high volume fluid losses; a low fistula involves TOP TIP
the large intestine and fluid losses are lower in The acronym SNAPP summarizes the principles of
volume. management of enterocutaneous fistulae:

(b) What investigations are required? Sepsis elimination: open or percutaneous drainage of
collections; administration of appropriate
Blood tests: antimicrobials
Full blood count anaemia may be caused Nutritional resuscitation/optimization: patients may be
by haemorrhage and sepsis raises white cell fluid and electrolyte depleted (losing litres of
count electrolyte-rich fluid through the fistula) and
Blood cultures should be taken prior to malnourished. Resuscitation with intravenous fluid
commencement of antibiotics with supplementary potassium with careful monitoring
Electrolytes, especially to check the patient and consideration for nutritional optimization, which
is not severely hypokalaemic may involve parenteral administration
Inflammatory markers (C-reactive protein, Anatomy: predominantly defined using radiological
ESR) investigations (see above)
Liver function tests reduced albumin Protect surrounding skin from intestinal enzymes with
indicates malnutrition stoma appliances
Radiological investigations:
Plan definitive surgery if remains unhealed.
Enhanced CT scanning with administration of
oral contrast is the investigation of choice.
This will (usually) reveal the site of the fistula
within the bowel, whether there is distal
progression of contrast beyond the fistula FURTHER READING
and whether there are any associated Berry SM, Fischer JE: Classification and
intra-abdominal collections or underlying pathophysiology of enterocutaneous fistulas.
pathology of the bowel Surg Clin North Am 76(5):10091018, 1996.
A barium follow-through or enema Metcalf C: Enterocutaneous fistulae. J Wound
(depending on the site) contrast study may Care 8(3):141142, 1999.
provide additional information, but will not
identify extraluminal collections
116 Abdomen and trunk Case 72

CASE 72 MOUTH SIGNS IN ABDOMINAL DISEASE *

surrounding erythema; they are common in


INSTRUCTION
childhood and are associated with infection
Look inside this patients mouth, or as part of and minor trauma to the oral cavity. In
Examine this gentlemans abdominal system. inflammatory bowel disease and coeliac
disease, the ulcers tend to be more
persistent and much larger; in Crohns
VITAL POINTS disease, the mouth takes on a cobblestone
Inspect appearance due to frequent ulceration,
healing with some fibrosis
Abnormalities within the mouth can be grouped Behets disease: a rare autoimmune disease
according to the appearances seen. most common in young men, causing a triad
of genital and oral ulceration and anterior
uveitis
Abnormal pigmentation Herpes simplex: small vesicles with an
erythematous base on the lips and inside of
Addisons disease: the mouth and lips are the mouth; diagnosed with scrapings of the
hyperpigmented base of the lesions and usually responsive to
Lichen planus: white lines and streaks inside topical acyclovir.
the mouth
PeutzJeghers disease: pigmented freckles Lip disorders
around the lips and inside the mouth;
associated with intestinal intussusception Angular stomatitis: chapping and splitting of
and gastrointestinal bleeding from colonic the corners of the lips is normal, but can
polyps. The patients have a higher incidence also occur in:
of concurrent gastrointestinal (stomach/
Herpes simplex and candidal infections
pancreatic/colonic) and extragastrointestinal
(breast/gonad) malignancy Iron, folate, vitamin B and C deficiencies.
Hereditary telangiectasia (RenduOsler
Weber disease): multiple telangiectasia Infections
(clusters of dilated capillaries and venules)
around the mouth and on the tongue and Herpes simplex causes both stomatitis and
lips; associated with gastrointestinal bleeding ulceration (above)
due to arteriovenous malformations; a group Oral candidiasis is the most common
of clustered autosomal dominant conditions abnormality; causes creamy white patches
Acanthosis Nigricans: black discolouration of that may be rubbed off, and may be seen in:
the skin, associated with carcinoma of the Patients prescribed inhaled steroids for
stomach and oesophagus, lymphomas and the treatment of asthma and chronic
with endocrine disorders (acromegaly, obstructive airways disease (the patient
Cushings, diabetes complicated by severe information leaflet advises patients to
insulin resistance). wash their mouths out after using these
inhalers)
Ulceration Oral antibiotic use
Immunocompromised patients including
Aphthous ulcers are small, round and those who are diabetic, and those on oral
shallow, and have a shallow yellow base with steroids.
Case 73 Abdomen and trunk 117

Henry Jules Louis Marie Rendu (18441902). Parisian Halushi Behet (18891948). Turkish dermatologist.
physician. Thomas Addison (17931860). Physician at Guys
Sir William Osler (18491919). Tremendously hospital, who was by reputation an excellent
significant medical educator, who was Professor of diagnostician and lecturer. Known as the Founder of
Medicine at Johns Hopkins and Oxford Universities, Endocrinology.
and was responsible for the formation of the John Law Augustine Peutz (18861957). Dutch
Association of Physicians of Great Britain and Northern physician; Chief of Internal Medicine, St Johns
Ireland, and for setting up full-time chairmen of Hospital, The Hague.
medicine in London hospitals.
Harald Jos Jeghers. Professor of Medicine, New
Frederick Parkes Weber (18631962). London Jersey College of Medicine and Dentistry, Jersey City.
physician.
Harvey Williams Cushing (18691939). The Founder
of Neurosurgery and Professor of Surgery, Harvard.

CASE 73 EPIGASTRIC HERNIA *

INSTRUCTION TOP TIP


Examine this gentlemans abdomen. If there is no scar but there is a longitudinal bulging
of the abdominal wall in the midline when the patient lifts
his head off the bed or coughs, consider divarication of
APPROACH the recti.
Expose the patient and begin to examine as for
Case 43.

Finish your examination here


VITAL POINTS
Inspect
Completion
When asking the patient to lift his head off
the bed, a lump may appear in the epigastric Say that you would like to:
region, in the midline Complete the rest of the abdominal
Ask him to cough and see if this lump examination.
becomes more prominent in the abdominal
wall.
QUESTIONS
Palpate (a) What is an epigastric hernia?

Palpate the area of the hernia carefully An abnormal protrusion of abdominal contents
(usually extraperitoneal fat, but occasionally
It can be very difficult to find the hernia, ask
peritoneal contents) through a defect in the linea
the patient to help by again lifting the head
alba, usually halfway between the xiphoid
off the bed and coughing
process and umbilicus.
Try to identify the borders of the defect and
the size of the neck. (b) What symptoms might the patient have
complained of at presentation?
The symptoms are commonly confused with
other upper gastrointestinal pathologies and
include:
118 Abdomen and trunk Case 74

Epigastric pain, which may increase after Surgical:


meals The principles of surgery are that the sac is
May be acutely painful after physical excised completely or inverted, and the
exercise defect in the linea alba repaired
Nausea and early satiety The fat contained within the hernia can be
Reflux and non-ulcer dyspepsia. excised or reduced
The site of the defect should be marked with
the patient lying supine preoperatively, as it
ADVANCED QUESTIONS may not be possible to find when the patient
(a) How would you treat this patient? is anaesthetized.

Non-surgical: the same principles as when


managing an incisional hernia (see Case 47) FURTHER READING
would apply. The patient may present with Coats RD, Helikson MA, Burd RS: Presentation
non-specific upper gastrointestinal symptoms; it and management of epigastric hernias in
is particularly important that other causes are children. J Pediatr Surg 35(12):17541756,
considered and ruled out; possible 2001.
investigations would include:
www.surgerydoor.co.uk patient-centred
Liver function tests and a biliary tree information on how to prepare for over a
ultrasound scan hundred common operations.
Helicobacter pylori serology and upper
gastrointestinal endoscopy

CASE 74 FEMORAL HERNIA *

INSTRUCTION Palpate
Examine this ladys groin. Identify the anterior superior iliac spine and
the pubic tubercle, demonstrating the
inguinal ligament between the two (see Case
APPROACH
42)
Expose the patient as for the inguinal hernia Femoral herniae are found below the inguinal
examination (Case 42), remembering to examine ligament compared with inguinal herniae
them lying down if they are presented on a which lie above (Fig. 50)
couch, and standing up if they are sitting in a Palpate the femoral pulse the lump lies
chair (see Top Tip, Case 42). medial to the pulse (Fig. 51)
Ask the patient to cough femoral herniae
VITAL POINTS usually do not have a cough impulse
Ask the patient if they can push the lump
These cases are seen only infrequently in the back femoral herniae are usually irreducible
clinical examination as they are often repaired
Ask if there is any pain and palpate the lump
surgically due to the risk of complication. It is
for the characteristic features of a femoral
important to know the essential differences from
hernia:
an inguinal hernia and the differential diagnosis
of a lump in the groin. Shape: usually round
Surface: smooth
Edge: well defined
Inspect Consistency: firm
Temperature: same as surrounding skin
There may be a marble-shaped lump in the
groin Tenderness: may or may not be tender
There may be a scar from previous surgery. Transilluminability: not transilluminable
Pulsatility: not pulsatile
Case 74 Abdomen and trunk 119

Femoral
vein
Femoral
artery

Inguinal
Femoral ligament
canal
Femoral
sheath Femoral
Pubic nerve
tubercle

Femoral hernia
exits through
saphenous
opening in thigh

Figure 50 Femoral hernia.

Compressibility: not compressible


Fluctuance: not fluctuant.

Iliacus Inguinal ligament


Finish your examination here

Nerve
Femoral Artery Completion
Vein Say that you would like to:
Sac of
femoral hernia Examine the contralateral groin for herniae.

Femoral sheath
QUESTIONS

(a) How can you tell this lump is a femoral


hernia (rather than an inguinal hernia)?
See Table 23.
Figure 51 NAVY acronym for arrangement of femoral
(b) What is the differential diagnosis for a
structures (Nerve, Artery, Vein, Y-fronts).
femoral hernia?
Skin and soft tissue masses:
Sebaceous cyst
Lipoma
Sarcoma
120 Abdomen and trunk Case 74

Table 23

Inguinal hernia Femoral hernia


Above the inguinal ligament Below the inguinal ligament
Usually reducible Usually not reducible
M:F 6:1 M:F 1:2 (but note inguinal herniae are still commoner in women
than femoral herniae)
Risk of strangulation low Risk of strangulation high
Cough impulse present Cough impulse usually absent

Vascular masses: Best option for elective procedures but


Saphena varix risk of narrowing the femoral vein when
closing the femoral canal
Femoral aneurysm
Transinguinal (Lotheissen) repair:
Inguinal lymphadenopathy
The posterior wall of the inguinal canal is
Other herniae:
opened to access the femoral canal from
Inguinal hernia above
Obturator hernia (rarely actually palpable) Best approach if the nature of the hernia
Others: is uncertain as inguinal hernias can also
Psoas bursa be repaired
Ectopic testis. Recurrent inguinal hernias are common
Use the acronym L-SHAPE (see also Case 42) High approach (McEvedy):
to help remember these causes: Involves a transverse incision above the
Lymph node/Lipoma of the cord inguinal ligament and dissection to
expose the preperitoneal space
Sapheno-varix/Skin lesions (sebaceous cyt/
lipoma etc.) Conversion to a Pfannenstiel incision
allows repair of bilateral hernia
Hernia: inguinal/femoral
Useful for repair in the emergency setting
Aneurysmal dilatation of the femoral artery
as it allows inspection of the peritoneal
Psoas abscess/bursa
contents (bowel) with easy conversion to
Ectopic/undescended testis. laparotomy if bowel resection is
necessary
ADVANCED QUESTIONS The femoral canal can be closed from
above without breeching the peritoneum.
(a) What are the surgical options for
management of a femoral hernia?
FURTHER READING
The surgical principles are: Chammary VL: Femoral hernia: intestinal
Reduction of the contents of the sac obstruction is an unrecognised source of
Excision of the sac morbidity and mortality. Br J Surg 80(2):230
Repair of the defect taking care not to 232, 1993.
narrow the femoral vein while tightening up Cheek CM, Black NA, Devlin HB, et al: Groin
the femoral canal (see Fig. 51) hernia surgery: a systematic review. Ann R Coll
Possible surgical techniques are: Surg Engl 80(Suppl 1):S1S80, 1998 (meta-
Low approach (Lockwood) (easiest and most analysis of results from hernia surgery in the
commonly used for elective repair): UK).
An incision is made directly over the www.nlm.nih.gov/medlineplus/ency/
hernia, below the medial half of the article/001136.htm#treatment patient-focused
inguinal ligament discussion of the causes and treatments of
The sac is ligated and the femoral canal femoral hernia.
closed with non-absorbable sutures or a
plug
3
SECTION

MUSCULOSKELETAL
AND NEUROLOGY
75 Orthopaedic history taking general approach *** 122
76 Osteoarthritis of the hip *** 123
77 Osteoarthritis of the knee *** 130
78 Dupuytrens contracture *** 137
79 Carpal tunnel syndrome *** 139
80 Rheumatoid hands *** 142
81 Osteoarthritis in the hands *** 144
82 Ulnar nerve lesions *** 145
83 Hallux valgus *** 147
84 Hammer toes *** 149
85 Mallet toes *** 150
86 Claw toes *** 151
87 Mallet finger *** 152
88 Trigger finger ** 153
89 Ingrowing toenail ** 154
90 Ligamentous and cartilaginous knee lesions ** 155
91 Radial nerve lesions ** 157
92 Examination of the shoulder ** 159
93 Gait ** 165
94 Osteochondroma ** 166
95 Popliteal fossa swellings ** 167
96 Hallux rigidus ** 169
97 Casts ** 170
98 Simulated reduction of fractures ** 171
99 Lumbar disc herniation ** 172
100 Brachial plexus lesions * 175
101 Ivory osteoma * 178
102 Chondroma * 179
103 Charcots joints * 180
104 Winging of the scapula * 181
105 External fixators * 182
106 Intramedullary nails * 183
107 Pagets disease of bone * 183
108 Achondroplasia * 185
122 Musculoskeletal and neurology Case 75

CASE 75 ORTHOPAEDIC HISTORY TAKING


GENERAL APPROACH ***

Use of walking aids, e.g. walking stick(s),


INSTRUCTION
Zimmer frame, crutches
Ask this patient some questions about her Walking distance (ask them how long or far
painful hip. they can walk before stopping because of
their joint problem)
Use of stairs
APPROACH
Upper limb (general):
The candidate may be asked to take a history
Feeding, washing, dressing, brushing hair,
from a patient both in OSCEs and short cases.
writing
In orthopaedics, the key elements are:
Specific to the lumbar spine:
Pain
History of injury
Loss of function
Radiation of pain particularly looking for
Stiffness
dermatomal distribution
Deformity
Associated neurological symptoms (e.g.
Swelling. numbness and paraesthesia) and their
distribution the two most commonly
VITAL POINTS involved nerve roots in lumbar disc prolapse
are shown in Table 24.
Introduction Sphincter disturbance bladder and bowel
symptoms secondary to cauda equina
Ask the patients age compression (unlikely to be present in
Ask their occupation patients used for examination purposes)
Ask about hand dominance (for upper limb Specific to the hip:
cases) Assess the stiffness and pain arising from
Ask which joints are symptomatic. the hip joint
Ask specifically about ability to:
Pain Care for their feet/pedicure
Get in and out of the bath
Site: remembering that superficial pain tends Specific to the knee:
to be recognized at the site but pain may be Locking of the knee is an intermittent inability
referred, so pain from the hip may radiate to to fully extend the knee and suggests a
the groin, anterior thigh, knee or shin mechanical block Does your knee ever get
Intensity: such as a pain severity score, stuck when you are trying to straighten it?
where the patient allocates a mark out of 10 Giving way a sign of either a patellofemoral
for the pain problem, loose body, meniscal flap tear or
Frequency: early morning pain is a hallmark ligamentous laxity Does your knee ever
of inflammation; pain that is relieved at night give way when you walk?
is often mechanical in nature; night pain is a Specific to the feet:
very important symptom as it indicates the
Ask the patient about back pain (pain on the
severity of pain and may also raise suspicion
sole of the foot may be due to a L5/S1 disc
of an underlying malignant process
prolapse)
Analgesic requirement.
Pins and needles may be due to lumbar
spine pathology, nerve entrapment (such as
Loss of function tarsal tunnel syndrome which is caused by a
posterior tibial nerve palsy) or peripheral
Impact on patients life, e.g. work, sleep. neuropathy
Lower limb (general): Pain present when the patient is barefoot is
Going to the shops suggestive of metatarsalgia
Case 76 Musculoskeletal and neurology 123

Table 24
Prolapsed Involved Distribution of sensory symptoms Distribution of motor signs Involved
disc nerve root reflexes
L4/L5 L5 Lateral aspect of the leg and Weakness of big toe extension None
dorsum of the foot and ankle dorsiflexion
L5/S1 S1 Lateral aspect of the foot and heel Weakness of ankle Ankle jerk
plantarflexion and foot eversion

Ask about the patients shoes do they have Swelling


to have special footwear designed?
May be noticed by patients in superficial
Stiffness joints (such as the hip and ankle)
Enquire as to whether the swelling is
Distinguish between early morning stiffness permanent or transient (e.g. following injury)
(suggestive of an inflammatory condition) Is there an inflammatory component, e.g.
and stiffness following activity (suggestive of erythema?
a mechanical condition).
Past medical history
Deformity
This essentially refers to fitness for surgery/
As fixed deformities develop, patients (or contraindications to anaesthesia and the
their friends or family) may notice a change presence of other underlying diseases.
in shape of their spine or lower limbs.

CASE 76 OSTEOARTHRITIS OF THE HIP ***

which are the two most common surgical


INSTRUCTION
approaches to the hip
Examine this gentlemans right hip. Look for muscle wasting (particularly the
gluteal muscles)
Look from the side at the patients posture
APPROACH
increased lumbar lordosis may indicate a
Expose the patients legs but keep his fixed flexion deformity at the hip
underwear on Look from the back at the spine scoliosis
This examination should be divided clearly may indicate a fixed adduction deformity
into: Perform the Trendelenburg test with the
Examining with the patient standing patient standing.
Watching the patient walk
Examining with the patient supine. Trendelenburg test (Fig. 52)

VITAL POINTS Ask the patient to stand on his good leg and
flex the other leg at the knee as you face him
Examining with the patient standing and place your hands on his pelvis, while he
(from the front, side and back) places his hands on your shoulders (this allows
you to feel what happens to the patients pelvis
Comment on the presence of any walking during the test and how the patient responds
aids with respect to weight transfer). This manoeuvre
Look at the hip for scars or sinuses look is then repeated on the bad leg. The test is
particularly for lateral and posterior scars, positive if the pelvis on the unsupported side
124 Musculoskeletal and neurology Case 76

Negative Positive Figure 54 Measuring apparent leg length from a fixed


Trendelenburg test Trendelenburg test midline bony point (the xiphisternum).
(normal) (abnormal)
Figure 52 Trendelenburg test. of his body over the affected hip in order to
compensate for his loss of balance due to the
pelvic dip on the contralateral side.

Watching the patient walk

Begin by asking the patient to stand in front


of you
Watch the patient walk (see Case 93) ask
him to walk away from you and then back
towards you; look specifically for:
A Trendelenburg gait, due to abductor
weakness (Fig. 52) characterized by the
presence of a sideways lurch of the trunk
to bring the patients body weight over
the affected limb
An antalgic gait (due to pain) decreased
stance phase and increased swing phase.

Examining with the patient lying down

Ask the patient to lie on the couch


Measure the real and apparent leg lengths
Figure 53 Performing the Trendelenburg test. using a tape measure
This is a difficult concept but one which may
(i.e. the side where the knee is flexed) sags be asked in the examination
down (Fig. 53). A positive Trendelenburg test In both cases, the good leg should be
typically occurs in patients with hip abductor measured first, then the abnormal leg,
weakness this can be due to chronic hip pain, comparing one side with the other
multiple surgeries, underlying structural The apparent leg length is measured from
abnormalities such as developmental hip the xiphisternum (a fixed midline bony point)
dysplasia or neuromuscular diseases such (Fig. 54) to the medial malleolus (Fig. 55),
as polio. while the patient is lying supine with the legs
A positive Trendelenburg lurch may be also parallel. With a fixed adduction deformity of
seen in the patient he throws the upper part the hip, the apparent leg length will be
Case 76 Musculoskeletal and neurology 125

shorter on the affected side, while with a with their pelvis at 90 to the bodys long
fixed abduction deformity, the apparent leg axis (Fig. 56). If you are unable to square the
length will be greater (remembering that you pelvis, correct the deformity by placing the
are measuring from the leg to a fixed midline normal leg in the same position as the
point) abnormal leg. The real leg length is
Now ensure that the patient is square on measured from the anterior superior iliac
the couch, in other words that they are lying spine to the medial malleolus of the
ipsilateral ankle (Fig. 57). If the measurement
is different between the two legs, it is due to
a real difference in the length of the bones.
Real leg length discrepancies can be:
In the femur either above the greater
trochanter (i.e. in the hip joint) or below it
Above or below the knee
A quicker screen for leg length discrepancy
(and one wed recommend for the clinical
examination) is Galeazzis test. Flex both
knees to 90, ensure the heels are together
and assess the position of the tibial
tuberosities. If the tibial tuberosity of the
Figure 55 Measuring apparent leg length to a fixed shorter leg lies distal and inferior when
bony point (the medial malleolus).

Figure 56 Squaring the pelvis.

Figure 57 Measuring real leg length.


126 Musculoskeletal and neurology Case 76

looking from the side, the tibia is shorter. If it much as possible and then assessing how
lies proximal and inferior, the femur is shorter. much further flexion is possible passively
(Figs 5962). Fig. 63 diagrammatically
Palpate summarizes Thomas test.

With the patient still supine, feel over the


greater trochanter for any tenderness
(trochanteric bursitis)
Remember that the joint itself is deep and
that the femoral head can only be palpated
with deep pressure over the midpoint of the
inguinal ligament (see Case 42) this may be
uncomfortable for the patient and usually
does not add further information.

Thomas test for fixed flexion


deformity

This measures a loss of extension at the hip


(fixed flexion deformity) Figure 59 Flexing the good hip while feeling for
Begin by placing one hand in the small of obliteration of the lumbar lordosis.
the patients back (Fig. 58), feeling for the
lumbar lordosis and assisting them to flex
their good hip as far as possible, feeling for
flattening of the lumbar lordosis (Fig. 59)
Maintain flexion in one hip (ask the patient to
hold onto the knee) while asking the patient
to flex the other hip as far as possible (Fig.
60) and then to extend it. By maintaining the
flexion of the other hip, the lumbar lordosis
is obliterated
In the presence of any fixed flexion deformity,
there will be a point at which further
extension ceases and the residual flexion (i.e.
fixed flexion) is measured from the horizontal
Repeat with the other leg (Fig. 61)
These movements can also be combined
with measuring range of movement of the Figure 60 Ask the patient to hold on to the good hip
hips in a cycling manoeuvre and passive while flexing the affected hip.
and active flexion can also be assessed by
asking the patient to actively flex the hip as

Figure 58 Feeling for the lumbar lordosis in Thomas


test. Figure 61 Repeat with the other leg.
Case 76 Musculoskeletal and neurology 127

Table 25 Movements of the hip


Hip Muscle group Expected
movement movement
Flexion Iliopsoas, rectus 140
femoris, tensor fascia
lata, quads
Extension Gluteus maximus and 10
hamstrings
Abduction Gluteus medius and 45
minimus
Adduction Adductors (longus, 30
brevis, magnus)
Internal Gluteus medius, 40
rotation minimus, iliopsoas
Figure 62 Assessing how much further passive flexion External Gluteus maximus 40
can be obtained from the affected hip. rotation

Normal

Positive Thomas test:


fixed flexion deformity in right hip Figure 64 Testing hip abduction.

Figure 63 Thomas test. say that you would assess the patient prone
normally and the examiner will often let you
move on to testing other movements)
Ask the patient to straighten the leg and with
Movements of the hip the hip fully extended, measure abduction
and adduction. Detect any tilting of the
Carefully watch the patients face at all times pelvis by placing one hand on one of the
during this examination and be sure to ask the anterior superior iliac spines (it is best to feel
patient to tell you if there is any discomfort the spine which is furthest away from the hip
before attempting any passive joint movements. being assessed) (Fig. 64 and Fig. 65)
Assess each muscle group in turn, Bring the hip back to 90 of flexion, at right
remembering that the movements of the hip angles to the couch. Keeping the knees
are flexion/extension, abduction/adduction flexed, measure internal and external rotation
and internal/external rotation (Table 25) (Fig. 66 and Fig. 67)
Flexion has already been assessed during You can also measure internal and external
Thomas test rotation with the knee in extension by
Extension can only be assessed by turning comparing movements of the patella (Fig. 68
the patient prone (as this is time consuming, and Fig. 69).
128 Musculoskeletal and neurology Case 76

Figure 65 Testing hip adduction.

Figure 68 Measuring hip internal rotation in extension.

Figure 66 Measuring hip external rotation in flexion.

Figure 67 Measuring hip internal rotation in flexion. Figure 69 Measuring hip external rotation in extension.
Case 76 Musculoskeletal and neurology 129

Finish your examination here improve their symptoms and may delay the
need for a total hip replacement
Occupational therapy: fitting of suitable
Completion devices to aid mobility (such as walking
sticks, frames, etc.) and more importantly
practical advice on how to use them
Say that you would like to:
Analgesic therapy: using the pain ladder (see
Examine the back and knee (the joints above
Case 118) beginning with paracetamol and
and below the hip, as pain in one joint may
non-steroidal anti-inflammatories
be referred to the next)
Surgical options include:
Examine the neurology of the limb
Osteotomy
Examine the vascular supply of the limb
Arthroplasty (i.e. hip resurfacing or hip
Offer to help the patient dress.
replacement)
Arthrodesis
QUESTIONS The US National Institute of Health concluded
in 1994 that the indications for total hip
(a) How should this patient be investigated? replacement are:
There is no specific laboratory test for Instability
osteoarthritis usually the diagnosis is made Severe pain or disability that is not
with a combination of clinical features and X-ray substantially relieved by an extended course
appearances. However, tests are performed to of non-surgical management
exclude other systemic diseases that may cause Rest pain or pain with movement
hip pain, particularly rheumatological disorders:
Loss of mobility.
Blood tests include:
Haematological: full blood count, erythrocyte
sedimentation rate ADVANCED QUESTIONS
Biochemical: baseline renal and liver (a) What are the complications of total hip
function, particularly if long-term non-
replacement?
steroidal anti-inflammatory medication is
being considered Complications can be divided into
Immunological: rheumatoid factor, intraoperative, immediate (within 24h), early
antinuclear antibody (within 30 days) and late (later than 30 days).
Radiological tests used are plain anteroposterior Specific complications include:
and lateral X-rays of the hip and pelvis. Intraoperative:
Fracture of the acetabulum or femur
(b) What are the X-ray features of Immediate:
osteoarthritis of the hip?
Dislocation (due to malalignment of the
Use the mnemonic LOSS to remember these prosthetic components)
features:
Early:
Loss of the joint space
Deep vein thrombosis (DVT) and pulmonary
Osteophyte formation embolus (PE)
Subchondral sclerosis Sciatic nerve palsy (more common in the
Subchondral cysts. posterior surgical approach to the hip joint)
Infection
(c) What are the treatment options?
Fat embolism syndrome
This is a classic examination question which
Late:
should be answered in a clearly structured way.
Infection
Non-surgical options would include:
Loosening (septic or aseptic)
Lifestyle modifications: diet and exercise are
Heterotopic ossification
important, including weight loss if
appropriate, and patients may need referral Leg-length discrepancy
to appropriate services Periprosthetic fractures
Physiotherapy: some patients will respond to Thigh pain.
personalized exercise regimens which will
130 Musculoskeletal and neurology Case 77

(b) How do you prevent postoperative deep


Friedrich Trendelenburg (18441924). Professor of
vein thrombosis (DVT) following a total hip Surgery, Bonn and Leipzig, Germany.
replacement?
Hugh Owen Thomas (18341891). General practitioner
DVT is the commonest complication following in Liverpool who founded orthopaedic services in the
total hip replacement, with a peak incidence at city and also designed a range of splints as he
510 days postoperatively. believed in the power of complete rest to heal
Prevention is impossible but measures that can fractures many of these were used in the First
be taken are classified according to: World War.
Preoperative: thromboembolic deterrent Ricardo Galeazzi (18561952). Italian surgeon at the
(TED) stockings fitted preoperatively Instituto de Rachitici in Milan, known for his extensive
Perioperative: TED stockings, minimizing the work experience on congenital dislocation of the hip.
length of surgery, using compression boots
and foot pumps
Postoperative: low dose or low molecular
FURTHER READING
weight heparin can reduce incidence of DVT,
early mobilization of patients with the help of Hoaglund FT, Steinbach LS: Primary
physiotherapists. osteoarthritis of the hip: etiology and
epidemiology, J Am Acad Orthop Surg
9(5):320327, 2001.
Zhang W, Moskowitz RW, Nuki G, et al: OARSI
recommendations for the management of hip
and knee osteoarthritis, Part II: OARSI evidence-
based, expert consensus guidelines,
Osteoarthritis Cartilage 16(2):137162, 2008.
http://orthoinfo.aaos.org/topic.
cfm?topic=A00213 information for patients on
osteoarthritis of the hip.

CASE 77 OSTEOARTHRITIS OF THE KNEE ***

INSTRUCTION if the examiner asks you to quantify this)


or is there fixed flexion or hyperextension
Examine this ladys right knee. present? Look particularly for varus and
fixed flexion deformities in patients with
osteoarthritis
APPROACH
Obvious quadriceps wasting?
Adequately expose both lower limbs, keeping Scars look particularly for:
the ladys underwear on (patients in exams will Arthroscopic portal scars (see Case 90)
often be wearing appropriate clothing, such as
Meniscectomy scars (see Case 90)
shorts).
Total or unicondylar knee replacement
scars midline longitudinal incision
VITAL POINTS Swellings especially popliteal fossa
?Bakers cyst (see Case 95).
Inspect with the patient standing
(front and back)
Gait (see Case 93)
Comment on the presence of any walking
aids Antalgic: if the knee is painful seen in
osteoarthritis
Shape:
Stiff knee gait: pelvis rises to allow the leg
Alignment is the knee in varus or valgus
clearance during the swing phase seen in
(you can measure intermalleolar distance
severe osteoarthritis
Case 77 Musculoskeletal and neurology 131

Figure 70 Measuring quadriceps wasting.

Instability (thrust) gait: may be mechanical or


neuropathic.
Figure 71 Palpating the tibial tuberosity.
Ask the patient to lie down on
the couch

Measure

Quadriceps wasting measure thigh


circumference at a set distance (e.g. 15cm)
above the tibial tuberosity (fixed bony point)
and compare with the opposite side (Fig. 70).

Feel

Temperature using the dorsum of the hand


from the proximal thigh to the distal leg
Tenderness start with the knee in extension
and feel around the margins of the patella:
Grind test move patella up and down
while pressing it gently against the femur Figure 72 Palpating the patella ligament.
painful grating is indicative of
patellofemoral compartment pathology,
e.g. patellofemoral osteoarthritis (PFOA)
Clarkes test press the patella
backwards and distally onto the
patellofemoral groove, and ask the patient
to gently contract his quadriceps muscle
pain at this point is indicative of
patellofemoral compartment pathology,
e.g. PFOA
Note that the two tests described above
can be very painful and may have to be
omitted in your exam routine
Next flex the knee to 90 and ensure the foot
is flat on the couch
Feel the tibial tuberosity (Fig. 71), patella
ligament (Fig. 72), medial joint line (Fig.
73) and lateral joint line (Fig. 74) of the
knee there may be tenderness of the Figure 73 Palpating the medial joint line.
132 Musculoskeletal and neurology Case 77

Figure 74 Palpating the lateral joint line. Figure 76 Patella hollow test.

Figure 75 Palpating the posterior aspect of the knee for Figure 77 Patella swipe test: emptying the medial
popliteal swellings. aspect of the knee.

joint line of involved compartments in


osteoarthritis)
Feel the posterior aspect of the knee for
any popliteal fossa swellings (Fig. 75) (see
Case 95)
Effusion:
Patellar hollow test (for very small
quantities of fluid) as the normal knee is
flexed, a hollow appears medial to the
patella (Fig. 76) and disappears with
further flexion with intraarticular fluid,
Figure 78 Patella swipe test: swiping across the lateral
the hollow fills and disappears at lesser
aspect of the knee.
angles of flexion you may even notice
that the hollows have disappeared
altogether compared with the normal
knee then sharply swipe across the lateral
Bulge test (for small quantities of fluid) aspect of the knee (Fig. 78) and observe
empty the medial compartment by for a ripple or bulge to appear on the
pressing on that side of the joint (Fig. 77), medial aspect of the knee
Case 77 Musculoskeletal and neurology 133

Cross-fluctuation (for moderate quantities patella can be felt to hit the femoral
of fluid) use your left hand to compress condyles and to bounce off.
and empty the suprapatellar pouch (Fig.
79), and with your right hand, empty the
Move
medial side of the knee by sweeping the
back of the hand up the medial side, then
sweep down the lateral side and observe Extension ask the patient to press her
the fluid impulse transmitted across thigh into the couch and note any presence
the joint of hyperextension (if no hyperextension, the
range of movement is 0 extension) and then
Patellar tap (for large amounts of fluid)
ask her to lift her leg straight up in the air
use the index finger of your right hand to
(Fig. 80), looking for an extensor lag due to
push the patella sharply downwards the
weak quadriceps a lag exists if there is
loss of extension but this can be passively
corrected (Fig. 81); if this cannot be
passively corrected, then there is a fixed
flexion deformity
Flexion normally the knee flexes until the
calf meets the hamstring this is around
140 during this movement, place your
hand over the patella and joint lines, noting
any clicks or crepitus (Fig. 82)
You may elicit a fixed flexion deformity and
decreased flexion in patients with
osteoarthritis the movements may also be
painful, so it is important to be very gentle.

Figure 79 Cross-fluctuation test.

Figure 80 Assessing for extensor lag.


134 Musculoskeletal and neurology Case 77

Figure 83 Anterior drawer test.


Cruciate ligaments (ensure that the
quadriceps and hamstrings are relaxed):
Posterior sag flex both knees to 90,
keeping the feet flat on the couch, and
look across at the anterior profile of both
knees if there is a drop back or sag of
the upper end of the tibia, or the upper
end can be gently pushed back, this
indicates a tear of the posterior cruciate
Figure 81 Correcting passively for extensor lag. ligament (PCL) (the sag sign)
Anterior drawer flex the knee to 90,
ensure the hamstrings are relaxed and
use both hands to pull the upper end of
the tibia forwards (= anterior drawer) (Fig.
83). A positive test indicates an anterior
cruciate ligament (ACL) injury. The test
can be graded according to the distance
forward that the tibia moves (1+=05mm,
2+=610mm, 3+=1115mm and
4+=>15mm) and according to the feel
of the endpoint of the movement (firm =
ACL intact, marginal or soft). Note that a
positive posterior sag can give you a
false-positive anterior drawer test
Posterior drawer performed in a similar
manner to the anterior drawer test except
that the upper end of the tibia is pushed
backwards. A positive test is indicative
of a PCL injury. It is graded in a similar
manner to the anterior drawer test
Lachman test (most sensitive for ACL
Figure 82 Palpating for joint line crepitus. injury) flex the knee to 30 and holding
distal thigh firmly with your left hand,
lift the proximal tibia forward with your
Special tests right hand with your thumb on the
anteromedial joint line. If you find this
This is part of the routine knee examination difficult, either because you have small
and should be carried out in the exam unless hands or the patient has big thighs, an
the examiner stops you. In patients with alternative method is to flex the patients
osteoarthritis, the most common finding with knee over your thigh, press down on the
these tests is a degenerate meniscal tear, which distal thigh of the patient and lift the
may give you a positive McMurrays test. proximal tibia forward. The test can be
Case 77 Musculoskeletal and neurology 135

Figure 84 Standard test for collateral ligament laxity.

Figure 86 Modified McMurrays test.

Repeat the varus and valgus stress test


with the knee in full extension. Instability
in extension signifies a combined
collateral and cruciate ligament injury
Menisci:
McMurrays test this test, if done
accurately, can help to differentiate
between medial and lateral meniscal
tears, but often does not add much
information to your clinical examination.
Flex the knee maximally, and grasp the
knee with one hand and the foot with the
Figure 85 Modified test for collateral ligament laxity. other. To test the medial meniscus,
palpate the posteromedial joint line and
graded according to the distance forward externally rotate the leg and for the lateral
that the tibia moves and according to the meniscus, palpate the posterolateral joint
feel of the endpoint of the movement, as line and internally rotate the leg. Now
in the anterior drawer test slowly extend the knee and feel for a
Collateral ligaments: palpable click, which is indicative of a
Tuck the patients foot under your arm meniscal tear. Pain during the test is also
and flex the knee to 2030, then apply suggestive of a meniscal tear
valgus and varus stresses to the knee A click that is palpable from moving from
alternately while feeling the joint line for full flexion to 90 flexion is suggestive of
opening (Fig. 84). This may be difficult as a posterior tear of the meniscus, while a
this manoeuvre often results in rotation at click that is palpable moving from 90
the hip rather than true stress testing of flexion to full extension is suggestive of a
the knee. The test can be alternatively middle or anterior tear of the meniscus.
performed by using one hand to hold the The test can also be repeated with valgus
knee from beneath feeling the joint lines (testing medial meniscus) and varus
(Fig. 85) and the other to hold the heel, (testing lateral meniscus) stresses applied
although this does require more strength. Modified McMurrays test this is far
Opening of the medial joint line on valgus easier to perform in a surgical exam. Flex
stress testing signifies a medial collateral the knee maximally palpating both medial
ligament injury, while opening of the and lateral joint lines, then bring the knee
lateral joint line on varus stress testing slowly into extension rotating internally and
signifies a lateral collateral ligament injury externally and feeling for a click (Fig. 86).
136 Musculoskeletal and neurology Case 77

Although not specific to either meniscus, Medical therapy using the pain ladder (see
this is the quickest way to rapidly assess Case 118) beginning with paracetamol and
the menisci. non-steroidal anti-inflammatories
Intra-articular steroid injections may
Finish your examination here provide temporary relief, but repeated
injections may lead to progressive cartilage
Completion and bone destruction
Viscosupplementation intra-articular
Say that you would like to: injections of hyaluronic acid may provide
benefit
Examine the hip and ankle (the joints above
Surgical options include:
and below the knee joint)
Assess the neurovascular status of the limb Arthroscopic debridement and washout
may give temporary relief and is of use in
Ask the patient some questions to ascertain
younger patients as a temporizing procedure
how much the problem affects her life,
before subsequent arthroplasty; degenerate
particularly activities of daily living, the
meniscal tears and osteophytes can be
presence of night pain and his mobility.
trimmed
Patellectomy indicated in rare cases where
QUESTIONS osteoarthritis is confined only to the
patellofemoral joint; may result in decreased
(a) What are the X-ray changes of extensor mechanism power, and if total knee
osteoarthritis of the knee? replacement is needed later results in less
predictable pain relief
Remember the mnemonic LOSS for radiological
features of osteoarthritis: Realignment osteotomy useful particularly
in younger patients (under 50 years) with
Loss of the joint space medial compartment osteoarthritis, in whom
Osteophyte formation a high tibial valgus osteotomy redistributes
Subchondral sclerosis weight to the lateral side of the joint
Subchondral cysts Unicompartmental or total knee arthroplasty
More specifically, ensure that the X-rays have (i.e. knee replacement) indicated in older
been taken with the patient standing and patients with progressive joint destruction. If
bearing weight, so that even small degrees of the disease is confined to one compartment,
articular cartilage thinning can be seen. The a unicompartmental knee replacement can
tibiofemoral joint space is diminished (usually be performed as an alternative to osteotomy.
medial compartment) and the lateral X-ray may Arthrodesis indicated if there is a strong
show patellofemoral osteoarthritis. contraindication to arthroplasty (e.g. previous
sepsis) or as a salvage procedure for a failed
(b) How do you treat osteoarthritis of arthroplasty.
the knee?
Non-surgical: QUESTION
Lifestyle modifications diet and exercise
are important, including weight-loss if What are the complications of a total
appropriate, and patients may need referral knee replacement?
to appropriate services Intraoperative:
Physiotherapy many patients will respond
Fracture of the tibia or femur
to personalized exercise regimens that will
Immediate:
improve their symptoms and may delay for
many years the need for a knee replacement. Vascular injuries superficial femoral,
Strengthening the quadriceps muscles is popliteal and genicular vessels
very important Early:
Occupational therapy fitting of suitable Deep vein thrombosis (DVT) and pulmonary
devices to aid mobility (such as walking embolus (PE)
sticks, frames, etc.) and more importantly Peroneal nerve palsy (1%)
practical advice on how to use them. Even a
Infection
simple elastic support may help, probably by
Fat embolism syndrome
improving proprioception in an unstable knee
Case 78 Musculoskeletal and neurology 137

Late: Instability manifests as increased


Infection anteroposterior glide and lateral wobble.
Loosening (septic or aseptic) (b) What are the surgical options for
Patellar instability/fractures/disruption of rheumatoid arthritis of the knee?
extensor mechanism
Synovectomy and debridement for failed
Periprosthetic fractures.
medical treatment. This procedure can be
performed arthroscopically and involves
ADVANCED QUESTIONS removing the articular pannus and cartilage
Supracondylar osteotomy useful if the knee
(a) Rheumatoid arthritis is another disease is stable and pain-free but can be
that can affect the knee. What do you know of complicated by valgus and flexion deformity
the clinical features of rheumatoid arthritis of Total knee arthroplasty for advanced joint
the knee? destruction.

Stage 1 proliferative:
Palpable effusions and thickened synovium FURTHER READING
but stable joint Cole BJ, Harner CD: Degenerative arthritis of
Posterior capsule at risk of rupture the knee in active patients: evaluation and
Acute rupture of Bakers cysts (see Case 95). management, J Am Acad Orthop Surg 7(6):389
Stage 2 destructive: 402, 1999.
Increasing instability of the knee joint Samson DJ, Grant MD, Ratko TA, et al: 2007.
Marked muscle wasting Treatment of primary and secondary
osteoarthritis of the knee, Evid Rep Technol
Some loss of flexion and extension.
Assess (Full Rep) 157:1157.
Stage 3 reparative:
www.aafp.org/afp/20000315/1795.html a very
Severe pain and instability there may be useful online paper published by the American
marked stiffness or severe instability Family Physician with approaches to managing
Commonest deformities are fixed flexion and hip and knee osteoarthritis.
valgus

CASE 78 DUPUYTRENS CONTRACTURE ***

INSTRUCTION Look for involvement of the thumb and the


first web space (a sign of more aggressive
Examine this patients hands. disease)
Ask the patient to turn his hands over to look
for Garrods pads (thickening of the
APPROACH
subcutaneous tissues) over the PIPJ.
Expose to elbows and ask the patient to place
his hands palm upwards on a pillow (if available).
Feel

VITAL POINTS Palpate the swelling, particularly noting its


fixation to skin
Look Does the other palm have similar thickening?

Describe any tethering or pitting of the skin


on the palmar aspect of the hand, and also Move
note the appearance of any visible cords
Look for scars from previous surgery Assess the range of motion in the involved
fingers
Describe any flexion deformities at the
metacarpophalangeal and proximal Note the presence of fixed deformities by
interphalangeal joints (MCPJ and PIPJ) of the passively moving the involved joints.
involved fingers
138 Musculoskeletal and neurology Case 78

*A group of disorders characterized by diffuse


TOP TIP
fibrosis, which include such diverse conditions
Asking the patient to lay the involved hand flat as desmoid tumours, Reidels thyroiditis,
against a hard surface, e.g. a table may facilitate retroperitoneal fibrosis and Ledderhose disease
assessment of the deformities and allow you to judge the (fibrosis of the plantar aponeurosis seen in 5%
extent to which they are fixed (table-top test). of patients with Dupuytrens).

(c) What are the surgical options available?


Finish your examination here Operative management is considered when
MCPJ or PIPJ contracture exceeds 30:
Fasciotomy for prominent bands
Completion Partial fasciectomy (with Z-plasty to lengthen
wound) in conjunction with postoperative
Say that you would like to: physiotherapy (early active-flexion range-of-
motion exercises for grip strength) and
Enquire about causes and associations (see
night-time splintage in extension
below)
Dermofasciectomy (with full-thickness skin
Assess the patients function, e.g. writing
grafting) associated with the lowest risk of
and dressing
recurrence
Look for other features of diffuse
Arthrodesis/amputation for late
fibromatosis.
presentations and repeated recurrences.

QUESTIONS
ADVANCED QUESTIONS
(a) What is your differential diagnosis?
(a) What is the underlying pathophysiology of
The differential diagnosis includes: the condition?
Skin contracture look for scar from Local microvessel ischaemia is thought to result
previous wound in increased activity of xanthine oxidase,
Tendon contracture thickened area, which resulting in superoxide free radical production
moves on passive flexion of involved finger that in turn stimulates myofibroblast proliferation
Congenital contracture of the little finger and type III collagen formation. Specific
affects PIPJ platelet-derived and fibroblast growth factors
Ulnar nerve palsy ring and little fingers are also play a role in the aetiology. Allopurinol,
hyperextended at MCPJ and flexed at PIPJ. which inhibits xanthine oxidase, may help to
reduce symptoms.
(b) What conditions are associated with The process of chronic inflammation is thought
Dupuytrens contracture? to be essential to the subsequent fibrosis (see
We have found the following mnemonic Further reading).
helpful to remember the associations
DEAFEST PAIL: Baron Guillaume Dupuytren (17771835). Surgeon in
Diabetes mellitus Chief, Hotel-Dieu, Paris. He described the condition as
Epilepsy permanent retraction of the fingers and was also
Age (positive correlation) Surgeon to Louis XVIII and Charles X during the
Family history (autosomal restoration of the Bourbon monarchy. He was a cold,
dominant)/Fibromatoses* rude, ambitious and arrogant man, earning him the
epithet the Napoleon of Surgery. He died following a
Epileptic medication (e.g. phenobarbitone)
stroke and was described as first among surgeons;
Smoking last among men.
Trauma and heavy manual labour
Sir A. E. Garrod (18571936). English physician,
Peyronies disease (fibrosis of the corpus St Bartholomews Hospital, London, later succeeding
cavernosum seen in 3% of patients with William Osler as Regius Professor at Oxford.
Dupuytrens)
G. Ledderhose (18551925). German surgeon.
AIDS
Idiopathic (most common) Francois Gigot de la Peyronie (16781747). French
surgeon.
Liver disease (secondary to alcohol).
Case 79 Musculoskeletal and neurology 139

Shaw RB Jr, Chong AK, Zhang A, et al:


FURTHER READING
Dupuytrens disease: history, diagnosis, and
Rayan GM: Dupuytren disease: Anatomy, treatment, Plast Reconstr Surg 120(3):44e54e,
pathology, presentation, and treatment, J Bone 2007.
Joint Surg Am 89(1):189198, 2007. www.patient.co.uk/showdoc/23068725/
patient-centred information.

CASE 79 CARPAL TUNNEL SYNDROME ***

INSTRUCTION TOP TIP 1


This lady is complaining of a tingling sensation The autonomous sensory areas of the hand (i.e. only
in the thumb and index fingers of the right hand. one nerve always supplies this area) are as follows:
Examine her hands and tell me what you think
Median nerve distal phalanges of index and middle
the diagnosis is.
fingers
Ulnar nerve middle and distal phalanges of little finger
APPROACH Radial nerve over first dorsal interosseus muscle
Expose to elbows and ask the patient to place between first and second metacarpals (but usually
her hands palm upwards on a pillow (if there is no autonomous zone)
available). Testing these three areas confirms restriction of pathology
to one nerve only.

VITAL POINTS
The clues are in the instruction. The examiner is Motor assessment
expecting you to perform a directed
neurological assessment of the hands. He is Test the power of muscles innervated by the
leading you towards to the diagnosis with his median nerve (LOAF) (Fig. 87):
question, but is interested in seeing how you Lateral two lumbricals difficult to test
approach the task.
Opponens pollicis oppose the patients thumb
and the little finger and ask her to stop you
Look pulling the fingers apart
Abductor pollicis brevis place dorsum of hand
Wasting of the thenar muscles (in advanced on a flat surface and ask the patient to lift
cases) her thumb to the ceiling against resistance,
Scar from previous surgery over the feeling the thenar eminence for the power of
transverse carpal ligament. abductor pollicis brevis
Flexor pollicis brevis not an autonomous
muscle (innervation varies).
Sensory assessment
Test only for abductor pollicis brevis in the
exam (see Top Tip 2).
Test light touch over the palmar aspects of the
thumb, index and middle fingers of the involved
hand deficiency implies median nerve TOP TIP 2
involvement. Compare this with the other
fingers, proceeding to other sensory modalities The autonomous motor supply of the hand are:
such as pain only if the examiner wishes you to. Median nerve abductor pollicis brevis (as above)
Ulnar nerve palmar interossei (adduction of the fingers)
Radial nerve metacarpophalangeal extensors (extension
of the fingers at the knuckles).
140 Musculoskeletal and neurology Case 79

Median
nerve

Pronator teres
Flexor carpi radialis Anterior
Palmaris longus interosseous
Flexor digitorum nerve
superficialis Flexor digitorum
profundus I & II
Flexor pollicis
longus
Pronator quadratus
Abductor pollicis
brevis
Flexor pollicis
brevis
Opponens
pollicis Second lumbrical

First lumbrical

Figure 87 Muscles supplied by the median nerve.

Finish your examination here nerve at the wrist reproduces symptoms (if
symptoms appear within 20s, sensitivity =
82% and specificity = 99%).
Assess the effect of the symptoms on the
Completion patients quality of life, e.g. symptoms are
usually worse at night and first thing in the
Say that you would like to perform the following morning sleep quality may be affected.
special tests:
Look for underlying causes and associations
Tinels sign tapping over the median nerve (see below).
at the wrist reproduces tingling sensation in
the distribution of the nerve
Phalens test maximal flexion of the wrist QUESTIONS
for 1min exacerbates symptoms which are
promptly relieved when flexion is (a) What are the causes of carpal tunnel
discontinued syndrome?
Flexion compression test (also known as
The most common cause is idiopathic. The
Durans test) maximal flexion of wrist and
other causes can be classified as follows:
direct digital compression of the median
Case 79 Musculoskeletal and neurology 141

Anatomical abnormalities: skin incision is made on the ulnar side of the


Bone previous wrist fractures, e.g. palmar crease
Colles fracture, acromegaly Failure to relieve symptoms if the
Soft tissues lipomas, ganglia retinaculum is incompletely divided.
Physiological abnormalities:
Inflammatory conditions rheumatoid ADVANCED QUESTIONS
arthritis, gout
Alterations of fluid balance pregnancy, (a) What are the boundaries of the carpal
menopause, hypothyroidism, obesity, tunnel?
amyloidosis, renal failure
Ulnar aspect: pisiform (where flexor carpi
Neuropathic conditions diabetes
ulnaris attaches) and hook of hamate
mellitus, alcoholism.
Radial aspect: scaphoid and trapezium
(b) Name one investigation you might perform Volar aspect: transverse carpal ligament.
before offering this lady treatment?
(b) Where else is the median nerve likely to
Nerve conduction studies:
be compressed?
Symptoms of carpal tunnel syndrome can be
The following causes are rare:
mimicked by higher (more proximal) lesions
of the median nerve. These high lesions are Pronator syndrome compression of the
characterized by loss of sensation over the median nerve by the ligament of Struthers
thenar eminence due to involvement of (fibrous band arising from the medial
the palmar cutaneous branch, and loss of epicondyle of the humerus that passes
the relevant forearm flexors (especially flexor medially and upwards to attach to a
pollicis longus) supratrochlear spur on the lower anterior
Symptoms may also be due to cervical nerve humerus), pronator teres muscle or the
root lesions (e.g. secondary to a cervical disc proximal arch of the flexor digitorum
herniation) or thoracic outlet syndrome superficialis
Nerve conduction studies also assist in Anterior interosseous syndrome
determining the severity of the lesion. entrapment of the anterior interosseous
branch of the median nerve, usually at the
(c) How would you treat this lady? origin of the deep head of pronator teres.
The nerve supplies flexor pollicis longus,
Non-surgical removal of underlying causes,
pronator quadratus and the radial side of
splinting of the wrist in a neutral position
flexor digitorum profundus, leading to loss of
(especially at night-time), and local steroid
precise pinch (inability to make the OK
injections just proximal to the carpal
sign), but there are no sensory signs.
tunnel
Surgical carpal tunnel decompression (c) What is the pathophysiology underlying
(division of the flexor retinaculum under carpal tunnel syndrome (or any compression
tourniquet control) can be performed either
syndrome)?
as an open or endoscopic procedure.
The primary change is thought to be vascular in
(d) What complications would you warn this origin. Pressure on the nerve results in blood-
lady about if you were offering her surgery? flow obstruction in the vasa nervorum, resulting
in venous congestion and oedema. With time,
Scar formation high-risk area for keloid or
fibroblast proliferation occurs in the nerve,
hypertrophic scars
leading to inefficiency of cell transport
Scar tenderness can occur in up to 40% of mechanisms and the sodium pump, resulting in
patients impairment of nerve conduction.
Wound infection
Nerve injury palmar cutaneous branch of
the median nerve (which lies superficial to
the retinaculum) and the motor branch to the
thenar muscles (which usually leaves the Jules Tinel (18791952). French neurologist.
radial side of the median nerve towards
George S. Phalen. American orthopaedic surgeon who
the distal extent of the standard incision).
worked at the Cleveland Clinic in Ohio.
The risk of nerve injury is decreased if the
142 Musculoskeletal and neurology Case 80

Verdugo RJ, Salinas RA, Castillo JL, et al:


FURTHER READING
Surgical versus non-surgical treatment for
Cranford CS, Ho JY, Kalainov DM, et al: Carpal carpal tunnel syndrome, Cochrane Database
tunnel syndrome, J Am Acad Orthop Surg Syst Rev 4:CD001552, 2008.
15(9):537548, 2007. www.patient.co.uk/showdoc/23068696/ guide
to carpal tunnel syndrome for patients.

CASE 80 RHEUMATOID HANDS ***

Boutonniere deformity (hyperextension of the


INSTRUCTION
DIPJ and flexion of the PIPJ) (Fig. 88)
Examine this ladys hands. Dorsum:
Wasting of the interossei, best seen in the
APPROACH first dorsal webspace.

Expose the hands and the forearms to the


elbows (to examine for rheumatoid nodules later Palpate
in the case) and place on a white pillow or
blanket, palm upwards. Palpate over any swollen joints to detect the
warmth and tenderness of acutely inflamed
joints
VITAL POINTS Finally palpate the elbows (over the
Almost all the clinical signs are elicited on subcutaneous border of the ulna) for
inspection alone. rheumatoid nodules present in 25% of
patients, especially in active seropositive
disease. You may also find rheumatoid
Look nodules at other pressure areas, e.g. the
pulps of the fingers and the radial side of the
Wrist: index finger.
Radial deviation of the wrist
Volar subluxation of the wrist joint
Piano-key sign subluxation of the radio-
ulnar joint causes the head of the ulna to
pop up on the dorsum of the wrist where it
can be jogged up and down
Thumb:
The Z-thumb appearance, with flexion of
the interphalangeal joint and hyperextension
of the metacarpophalangeal joint (MCPJ)
Palm:
The presence of palmar erythema
Fingers:
Ulnar deviation of the fingers, involving the
more lateral digits in more advanced cases
Metacarpophalangeal joint volar subluxation,
most commonly over the index and middle
fingers
Swelling of the proximal interphalangeal
joints (PIPJ)
Swan neck deformity (flexion of the DIPJ and
hyperextension of the PIPJ) (Fig. 88) Figure 88 Swan neck (above) and boutonniere (below)
deformities of the rheumatoid hand.
Case 80 Musculoskeletal and neurology 143

Sensory and motor assessment (b) What investigations can be used to confirm
the diagnosis of rheumatoid arthritis?
See the section under carpal tunnel syndrome Blood tests:
(Case 79) the median nerve may be involved
Haematological
in rheumatoid arthritis if carpal tunnel syndrome
Anaemia of chronic disease (due to
is a complication.
decreased production of red blood cells,
increased destruction of red cells or
Functional assessment ineffective erythropoiesis)
Raised erythrocyte sedimentation rate
Ask the patient to perform simple tests such as Immunological
unbuttoning a shirt and writing with a pen, in Rheumatoid factor is positive in 75%
order to assess the function of the hands, and HLA-DR3/DR4 present in approximately
determine the need for specific treatments. one-third of patients
Antinuclear antibody (ANA) is raised in
Finish your examination here 30%
X-rays (see below).

(c) What are the treatment options for


Completion
rheumatoid arthritis?
Say that you would like to: Non-surgical (hand therapy/physiotherapy):

Ask the patient what other joints in the body Splinting (static and dynamic)
are affected by rheumatoid arthritis Active hand and wrist exercises
Examine the rest of the patient for other Household aids and personal aids (orthoses)
features of rheumatoid arthritis (see below) Injections:
Ask the patient how the condition affects Local injections of corticosteroid and local
her life. anaesthetic for persistent synovitis of a few
joints or tendon sheaths.
Pharmacological:
QUESTIONS
First-line drug treatment with non-steroidal
(a) What are the extra-articular anti-inflammatory drugs (considering the
manifestations of rheumatoid arthritis? need for a gastric mucosal protective agent
such as the addition of a proton pump
Ophthalmic: inhibitor, or a prostaglandin analogue such
Episcleritis as misoprostol) or COX-2 inhibitors, which
Scleritis specifically inhibit cyclooxygenase 2,
Keratoconjuctivitis sicca reducing the incidence of gastrointestinal
Respiratory: bleeding (which is mediated through COX-1)
Second-line treatment is with disease-
Pleural effusions
modifying anti-rheumatic drugs (DMARDS)
Pulmonary fibrosis such as sulfasalazine, methotrexate,
Cardiac: penicillamine or gold
Pericarditis Newer (but more expensive and need to be
Reticuloendothelial: injected) DMARDS include anti-TNF alpha
Lymphadenopathy drugs such as etanercept, and infliximab and
IL-1 inhibitors such as anakinra
Splenomegaly
Surgical:
Feltys syndrome (see Case 49)
Neurological: Operative options include soft tissue
procedures (e.g. synovectomy, carpal tunnel
Multifocal neuropathies decompression and tendon repairs/transfers)
Carpal tunnel syndrome and bone/joint procedures (e.g. arthrodesis
Vascular: and arthroplasty). You would not be
Vasculitis. expected to know the details of these
procedures for finals or the MRCS.
144 Musculoskeletal and neurology Case 81

ADVANCED QUESTIONS (b) What are the radiological stages of


rheumatoid arthritis?
(a) What are the clinical stages of rheumatoid Stage 1 soft tissue swelling and
arthritis of the hand? periarticular osteoporosis
Stage 1 proliferative: synovitis of the joints Stage 2 joint space narrowing and small
(swelling of MCPJ and PIPJ) and of tendon periarticular erosions (most common at
sheaths (flexor and extensor tenosynovitis, MCPJs and styloid process of the ulna)
the former leading to carpal tunnel Stage 3 marked articular destruction, seen
syndrome) most commonly at MCPJs, PIPJs and wrist
Stage 2 destructive: joint and tendon joints.
erosions, e.g. drop finger and mallet thumb
due to extensor tendon ruptures, radial FURTHER READING
deviation of the wrist and ulnar deviation of
the fingers Majithia V, Geraci SA: Rheumatoid arthritis:
Stage 3 reparative: leading to established diagnosis and management, Am J Med
deformities such as marked ulnar deviation 120(11):936939, 2007.
of the fingers, volar dislocation of the www.arc.org.uk/arthinfo/patpubs/6033/6033.asp
MCPJs, and multiple swan-neck and patient information on rheumatoid arthritis
boutonniere deformities. from the Arthritis Research Campaign.

CASE 81 OSTEOARTHRITIS IN THE HANDS ***

INSTRUCTION Functional assessment


Examine this gentlemans hands.
Ask the patient to perform simple tests such as
unbuttoning a shirt, in order to assess the
APPROACH function of the hands.

Expose the hands and the forearms to the


elbows and place on a white pillow or blanket, Finish your examination here
palm upwards.

Completion
VITAL POINTS
Almost all the clinical signs are elicited on Say that you would like to:
inspection alone. Examine other joints for arthritis. In
comparison with rheumatoid arthritis, the
hips, lumbar spine and knees are more
Look
commonly affected than the hands.
The distal interphalangeal joints (DIPJ) are
swollen (Heberdens nodes) and may be QUESTIONS
fixed in flexion
Bouchards nodes are bony swellings at the (a) Which joints in the hands are most
proximal interphalangeal joints (PIPJ) in frequently affected by osteoarthritis?
osteoarthritis
DIPJ (Heberdens nodes)
Square hand appearance see below.
PIPJ (Bouchards nodes) these are strongly
associated with polyarticular osteoarthritis,
Palpate i.e. arthritis at other joints in the body,
including carpometacarpal arthritis
Continue to test active and passive Carpometacarpal joint (CMCJ) of the thumb
movements of the affected joints to define which sometimes leads to the square
degree of reduction of movement. hand or metacarpal bossing appearance.
Case 82 Musculoskeletal and neurology 145

(b) What are the treatment options for Heberdens and Bouchards nodes rarely
osteoarthritis of the hands? require surgical management, but sometimes
an arthrodesis is required if the joint
Non-surgical:
becomes unstable or very painful.
Physiotherapy may help to maintain
functional ability, and especially with thumb William Heberden (17101801) also described angina,
involvement, splints may also be used chicken pox and night blindness. Heberden Disease
Pain relief (using analgesic ladder is another name for angina pectoris. He was physician
Case 118): paracetamol, aspirin or other to George III and attended to Dr Johnson during his
non-steroidal anti-inflammatory drugs for last illness.
symptomatic relief
C.J. Bouchard (18371925). French physician who
Surgical: was also one of the initial physicians to describe
Joint arthrodesis, and in the case of thumb spider naevi.
CMCJ involvement, trapeziectomy can be
performed with tendinous interpositional
graft (from flexor carpi radialis) FURTHER READING
Arthroplasty (with the Swanson silicone Van Heest AE, Kallemeier P: Thumb carpal
trapezium implant) has largely been metacarpal arthritis, J Am Acad Orthop Surg
abandoned due to the problems of 16(3):140151, 2008.
dislocation and silicone-induced synovitis www.medicinenet.com/script/main/art.
asp?articlekey=20167 information for patients.

CASE 82 ULNAR NERVE LESIONS ***

Ask the patient to turn his hands over and


INSTRUCTION
observe the guttering between the
Examine this gentlemans hands. metacarpals as the interossei are wasted
(best seen in the first dorsal webspace).

APPROACH
Sensory assessment
Within surgical short cases, the likely reason
for weakness of the hand will be a specific
Test the autonomous area (see Case 79)
neurological lesion of the median, ulnar or radial
over the middle and distal phalanges of the
nerves. Beginning with the Top Tips (see Case
little finger.
79) will allow the candidate to get swiftly to the
diagnosis without wasting time.
Expose to elbows and ask the patient to place Motor assessment
his hands palm upwards on a pillow (if available).
Test the palmar interossei (which adduct the
fingers) by asking the patient to hold a piece
VITAL POINTS of paper between two fingers while you
attempt to pull it away you have now
Inspect tested the autonomous motor supply (see
Case 79)
Note the claw hand appearance, with Continue to test the dorsal interossei (which
paralysis of lumbricals and interossei, and abduct the fingers) by asking the patient to
unopposed action of the long flexors and spread his fingers and prevent you from
extensors, causing flexed, deformed little pushing them together
and ring fingers (see difference between high
Assess for weakness of flexor digitorum
and low lesions below)
profundus to the ring and little fingers (see
Examine the palm, noting the wasting of the Case 88, Trigger finger).
hypothenar eminence (all muscles here are
supplied by the ulnar nerve) (Fig. 89)
146 Musculoskeletal and neurology Case 82

Ulnar nerve

Flexor carpi
ulnaris
Flexor digitorum
profundus
III & IV

Adductor pollicis
Flexor pollicis Abductor
brevis
Opponens Digiti
All dorsal minimi
interossei Flexor
All palmar
interossei
Fourth lumbrical
Third lumbrical

Figure 89 Muscles supplied by the ulnar nerve.

Special tests within 1 minute this test may be quite


uncomfortable for the patient, and therefore
Froments sign: this test is based on the fact you are better off describing it first to the
that adductor pollicis is supplied by the ulnar examiner.
nerve, and if there is an ulnar nerve palsy,
the only way to adduct the thumb is by
TOP TIP
using flexor pollicis longus to compensate
(this muscle is supplied by the median When examining the peripheral nerves of the upper
nerve). Ask the patient to hold a piece of limb, begin at the hand and move proximally. Continue
paper between the thumb and radial aspect examining one nerve in its entirety before moving onto the
of the index finger and as you pull this piece next nerve, by which time the examiner will usually have
of paper away from the patient, observe the stopped you. This allows a precise definition of the
thumb you will see the distal phalanx flex if anatomical location of the nerve lesion described.
the patient is using flexor pollicis longus to
hold the piece of paper
Elbow flexion test: with the elbow fully
flexed, the patient will complain of numbness Finish your examination here
and tingling in the ring and little fingers, often
Case 83 Musculoskeletal and neurology 147

Completion component of the clawing is less prominent


this is known as the ulnar paradox
Say that you would like to: Decreased sensation over ulnar border of the
hand
Examine the neck and all the other peripheral
nerves of the affected limb. Otherwise as for low lesion.

QUESTIONS ADVANCED QUESTIONS

(a) What causes of ulnar nerve palsies (a) How do you treat ulnar nerve palsies?
do you know? Non-surgical:
The causes can be divided up according to For patients with mild, intermittent symptoms
broad aetiological categories: and no significant neurological deficits
avoid repetitive flexion-extension motions
Anatomical: cubital tunnel syndrome at the
and prolonged elbow flexion, and use of
elbow (due to repeated elbow flexion leading
night splintage with the elbow in extension
to traction injury recurrent subluxation of
the nerve within the tunnel) Surgical:
Trauma: anywhere along the course of the For patients with persistent, significant
nerve, e.g. supracondylar fractures and symptoms or neurological deficit, surgical
dislocations of the elbow (also late sequelae options include:
of trauma can lead to ulnar nerve palsy, e.g. Ulnar nerve decompression
cubitus valgus deformity at the elbow) (decompression of the roof of the cubital
Degenerative arthritis: with compressing tunnel at the elbow)
proliferative synovitis and osteophytes, or Ulnar nerve anterior transposition
loose bodies subcutaneous or submuscular
Rare causes: compression from tight fascia transposition
or ligaments, tumour masses, aneurysms, Medial epicondylectomy.
vascular thromboses, or anomalous muscles
(e.g. anconeus epitrochlearis). Jules Froment (18781946). Neurologist and Professor
of Clinical Medicine, Lyons, France.
(b) How do you clinically differentiate between
a high and a low ulnar nerve lesion?
Low lesions (below elbow): FURTHER READING
More marked clawing as flexor digitorum Elhassan B, Steinmann SP: Entrapment
profundus to ring and little fingers still neuropathy of the ulnar nerve, J Am Acad
functioning Orthop Surg 15(11):672681, 2007.
High lesions (above elbow): Toussaint CP, Zager EL: Whats new in
Paralysis of flexor digitorum profundus to common upper extremity entrapment
ring and little fingers leads to less marked neuropathies, Neurosurg Clin N Am 19(4):573
clawing of these fingers as the flexion 581, 2008.

CASE 83 HALLUX VALGUS ***

INSTRUCTION the patient or kneeling on the ground


opposite her.
Examine this ladys feet.

VITAL POINTS
APPROACH
Look
Expose both ankles and feet and begin by
describing any obvious deformities. You should
Unilateral or bilateral?
position yourself either sitting on a chair facing
148 Musculoskeletal and neurology Case 83

Estimate degree of valgus of the big toe


Is there rotation (pronation) of the big toe
(nail faces medially)? C
Is there a bunion present? A bunion is a
prominence of the medial aspect of the first
metatarsal head with or without an overlying
bursa
Are there visible signs of inflammation of the
bursa, e.g. erythema?
Is there a hammered or retracted second toe A
(see Case 84)? B
Look at the soles of the feet are there any
callosities present?

Feel

Is there any inflammation of the bunion, e.g.


warmth, tenderness?
Localize any areas of tenderness, e.g.
osteoarthritis of the first metatarsophalangeal
joint (MTPJ).

Move

Assess range of movement of the first MTPJ,


including noting the presence of any D
hypermobility (indicating instability).
Figure 90 Important radiological angles in hallux valgus.
A, hallux valgus angle; B, distal metatarsal articulation
Finish your examination here angle; C, interphalangeal angle; D, intermetatarsal angle.

Completion First/second intermetatarsal angle (IMA) and


distal metatarsal articular angle (DMAA) (Fig.
Say that you would like to: 90)
Presence of osteoarthritis of the first MTPJ.
Assess the range of motion of the other toe
joints (b) What do you know of the aetiology of
Watch the patient walk (gait) this allows hallux valgus?
inspection of the heel and ankle from the
Essentially unknown
back, again noting any gross deformities
Strong familial trait
Examine her shoes abnormalities of
weightbearing will be reflected in the pattern Increased incidence in people who wear
of wear enclosed footwear; rarely seen in those who
have never worn shoes
Ask her questions to assess the effect of the
condition on her life. Associated with rheumatoid arthritis
Secondary to metatarsus primus varus,
which itself may be congenital or secondary
QUESTIONS to loss of muscle tone with age.

(a) Name one investigation you would (c) What treatment options are available in
perform in order to further assess the hallux valgus?
condition? Non-surgical:
Plain weightbearing X-rays in order to assess: Appropriate footwear, e.g. wide shoe with
Degree of valgus deformity soft upper, wide toe box and protective
padding over prominences (follow the patient
Case 84 Musculoskeletal and neurology 149

up after a couple of months in order to In a low-demand elderly patient Kellers


examine the footwear and note wear procedure (becoming less popular)
patterns) Non-arthritic joint:
Physiotherapy IMA <15, e.g. Chevron osteotomy
Surgical: IMA >15, e.g. scarf osteotomy.
Several options are available, depending on the
wishes of the patient, her level of activity and
the state of her peripheral vascular system. Colonel William Keller (18741959). Director of
These include: Professional Services of the American Expeditionary
Forces during the First World War. He first devised the
Bunionectomy
procedure in Manila during the Philippine War.
First metatarsal realignment osteotomy
Excision arthroplasty (Kellers procedure)
essentially a proximal hemiphalangectomy
Fusion for degenerative joint disease.
FURTHER READING
Easley ME, Trnka HJ: Current concepts review:
hallux valgus part I: pathomechanics, clinical
ADVANCED QUESTION assessment, and nonoperative management,
Foot Ankle Int 28(5):654659, 2007.
(a) How would you decide which surgical
techniques to use in a specific patient? Easley ME, Trnka HJ: Current concepts review:
hallux valgus part II: operative treatment, Foot
If the examiner moves you on to specific Ankle Int 28(6):748758, 2007.
surgical options, a rough outline of treatment www.bofas.org.uk/PublicArea/PatientAdvice/
can be given, although decisions are tailored to Halluxvalgusbunion/tabid/101/Default.aspx
the individual. information for patients.
Arthritic joint:
In an active patient first MTP arthrodesis or
prosthetic arthroplasty

CASE 84 HAMMER TOES ***

INSTRUCTION The distal interphalangeal joint (DIPJ) can be


in any position, but extension is most
Examine this ladys right foot. common (Fig. 91)
Neutral or extension at the metatarsophalan-
geal joint (MTPJ), although hammer toes
APPROACH
deformities are primarily confined to the
Expose both ankles and feet by removing socks interphalangeal joints (compared with claw
and shoes, and begin by describing any obvious toes, see Case 86)
deformities. Note any associated callosities.

VITAL POINTS Feel


Look Any tenderness of the affected toe(s).

Affects the lesser toes, most commonly the


second toe Move
May be associated with hallux valgus
Note whether deformity is fixed or mobile.
Flexion deformity at the proximal
interphalangeal joint (PIPJ) of the involved
toe(s) this is the major deformity to note
150 Musculoskeletal and neurology Case 85

QUESTIONS
(a) What is the aetiology of hammer toes?
Imbalance between intrinsic (lumbricals and
interossei) and extrinsic (long flexors and
extensors) muscles of the lesser toes
More common in:
Females than males
The elderly
Patients with rheumatoid arthritis.

(b) How would you treat this ladys condition?


Non-surgical:
Appropriate footwear, e.g. high, wide
toeboxes and semirigid longitudinal arch
supports with metatarsal pads to evenly
distribute plantar pressure
Surgical:
Mobile deformity: consider flexor-to-extensor
tendon transfer (split flexor digitorum longus
transfer to the extensor hood Girdlestone
Taylor procedure)
Fixed deformity: consider:
Resection of the proximal phalangeal
Figure 91 Hammer toe (top) and claw toe (bottom). head and neck with possible flexor and
extensor release
Proximal phalangectomy
PIPJ arthrodesis
Painful callosities: consider terminal
Finish your examination here phalangectomy.

Gathorne Robert Girdlestone (18811950). First


Professor of Orthopaedic Surgery in Britain (at Oxford).
Completion

Say that you would like to: FURTHER READING


Watch the patient walk Coughlin MJ: Lesser toe abnormalities, Instr
Examine her shoes Course Lect 52:421444, 2003.
Ask her questions to assess the effect of the www.patient.co.uk/showdoc/40002322/
condition on her life. information for patients.

CASE 85 MALLET TOES ***

INSTRUCTION VITAL POINTS


Have a look at this ladys right foot. Look

Affects the lesser toes, commonly the


APPROACH second toe
Expose both ankles and feet by removing socks Flexion deformity at the distal interphalangeal
and shoes, and begin by describing any obvious joint (DIPJ) of the involved toe(s)
deformities.
Case 86 Musculoskeletal and neurology 151

No proximal interphalangeal joint (PIPJ) or


metatarsophalangeal joint (MTPJ)
QUESTIONS
involvement (a) What is the aetiology of mallet toes?
Note any associated callosities.
As with hammer toes, imbalance between
intrinsic (lumbricals and interossei) and
Feel extrinsic (long flexors and extensors)
muscles of the lesser toes
Any tenderness of the affected toe(s). More common in females than males, the
elderly and patients with rheumatoid arthritis
Move and diabetics with peripheral neuropathy.

(b) How would you treat this ladys condition?


Note whether deformity is fixed or mobile.
Non-surgical:
Appropriate footwear: usually unhelpful
Finish your examination here
Surgical:
Mobile deformity, consider flexor digitorum
longus tenotomy (if extensor digitorum
Completion
longus tendon intact)
Fixed deformity, consider:
Say that you would like to:
Flexor tenotomy with resection of the
Watch the patient walk
middle phalangeal head and neck
Examine her shoes
Fusion of the DIPJ
Ask her questions to assess the effect of the
Amputation of distal half of distal phalanx
condition on her life.
to include nail and matrix.

FURTHER READING
Coughlin MJ: Lesser toe abnormalities, Instr
Course Lect 52:421444, 2003.
www.patient.co.uk/showdoc/40002322/
information for patients.

CASE 86 CLAW TOES ***

INSTRUCTION May be bilateral


Flexion deformity at the proximal
Examine this ladys right foot. interphalangeal joint (PIPJ) and distal
interphalangeal joint (DIPJ) (see Fig. 91)
APPROACH Involves the metatarsophalangeal joint
(MTPJ) note MTPJ hyperextension
Expose both ankles and feet (by removing Note any associated callosities plantar to
socks and shoes) and begin by describing any the metatarsal heads and dorsal to the
obvious deformities. PIPJs.

VITAL POINTS Feel


Look
Any tenderness of the affected toe(s).

Affects the lesser toes frequently all four


involved
152 Musculoskeletal and neurology Case 87

Move supports with metatarsal pads to evenly


distribute plantar pressure
Note whether deformity is fixed or mobile. Surgical:
Mobile deformity, consider flexor-to-extensor
Finish your examination here tendon transfer (see Case 84, Hammer toes)
Fixed deformity, consider:
Resection of the proximal phalangeal
head and neck with possible flexor and
Completion extensor release
Extensor tenotomy for contractures of the
Say that you would like to:
MTPJ
Watch the patient walk Resection of the metatarsal heads of the
Examine her shoes lesser toes.
Ask her questions to assess the effect of the
condition on her life. Jean-Martin Charcot (18251893). See Charcots
joints (Case 103).
QUESTIONS Pierre Marie (18531940). Professor of Pathological
Anatomy and Clinical Neurology, Salpetriere Hospital,
(a) What is the aetiology of claw toes? Paris.
As with hammer and mallet toes, imbalance Howard Henry Tooth (18561926). English physician
between intrinsic (lumbricals and interossei) and neurologist.
and extrinsic (long flexors and extensors) CharcotMarieTooth syndrome (peroneal muscular
muscles of the lesser toes atrophy) presents in puberty or early adult life and
More common in females than males, the causes progressive weakness of the peroneal
elderly and patients with rheumatoid arthritis muscles, the foot and toe dorsiflexors, the plantar
May be secondary to neurological disorders flexors and the intrinsic muscles of the foot, leading to
such as peripheral neuropathy (diabetes, claw toes, and varus and cavus deformities of the
CharcotMarieTooth disease), lower motor forefoot and hindfoot. It may also affect the median
neurone disease (poliomyelitis) and upper and ulnar nerves.
motor neurone disease (cerebral palsy,
multiple sclerosis, stroke).
FURTHER READING
(b) How would you treat this ladys condition? Coughlin MJ: Lesser toe abnormalities, Instr
Non-surgical: Course Lect 52:421444, 2003.
Appropriate footwear, e.g. high, wide www.patient.co.uk/showdoc/40002322/
toeboxes and semirigid longitudinal arch information for patients.

CASE 87 MALLET FINGER ***

INSTRUCTION VITAL POINTS


Examine this gentlemans hand. Look

Note the flexion deformity of the distal


APPROACH phalanx of one or more of the fingers.
Expose the patient as for any hand examination,
asking him to place his hands palms upwards
on a white pillow.
Move

Test for active movement of the finger the


terminal phalanx cannot be actively extended
Case 88 Musculoskeletal and neurology 153

On passive movement of the joint the digit (b) How is mallet finger managed when
can be moved back into the normal position presenting acutely?
(unless chronic).
The finger should be X-rayed to exclude a
fracture. The finger should be splinted for
Finish your examination here 6 weeks with the distal interphalangeal joint
in extension in a mallet splint to allow
reattachment of the tendon. If the avulsed flake
QUESTIONS of bone is greater than a third of the width of
the joint space on the lateral X-ray, it should be
(a) What is the aetiology of mallet finger? repositioned with a fine Kirschner (K) wire or
There has been damage (usually division) of the other internal fixation device.
extensor tendon to the terminal phalanx of the
finger involved. This may occur if a flake of Martin Kirschner (18791942). A pupil of Trendelen-
bone is avulsed from the base of the distal burg who became Professor of Surgery in Heidelberg,
phalanx (thus the term baseball finger, as this Germany.
avulsion may occur in catching a ball).
Interestingly, note that in rheumatoid arthritis,
rupture of the central slip of the extensor
FURTHER READING
tendon more proximally leads to the classic Tuttle HG, Olvey SP, Stern PJ: Tendon avulsion
Boutonnieres deformity (see Case 80). injuries of the distal phalanx, Clin Orthop Relat
Res 445:157168, 2006.
www.bssh.ac.uk/patients/
commonhandconditions/malletfingerinjury
information for patients.

CASE 88 TRIGGER FINGER **

overlying the flexor tendon sheath as it enters


INSTRUCTION
the digit. The nodule is usually approximately at
Examine this gentlemans hand. the level of the proximal transverse palmar
crease.

APPROACH
Move
Expose the patient as for any hand examination,
asking him to place his hands palms upwards Test for active movement of the finger and note
on a white pillow. that on gentle forced extension of the finger,
there may be a characteristic snap as the distal
finger passes the obstruction.
VITAL POINTS
Look TOP TIP
Note flexion of one or more of the fingers. Examination of the flexor tendons of the hand.
However, the finger does not necessarily have Place the patients hand flat on a hard surface with
to be fixed in flexion the whole time for it to be the palms facing upwards.
triggering. Trigger finger most frequently affects
the middle or ring fingers. Flexor digitorum profunda tendon is tested by active
flexion of the distal interphalangeal joint, with the PIPJ
fixed in full extension
Feel
Flexor digitorum superficialis tendon is tested by
active flexion of the PIPJ when the examiner fixes the
Ask the patient if there is any pain and then other fingers in full extension.
palpate carefully over the palm proximal to the
finger involved, as there may be a small nodule
154 Musculoskeletal and neurology Case 89

Finish your examination here when other signs of rheumatoid arthritis of the
hands would be expected.

QUESTIONS (b) How is trigger finger treated?


If a nodule is present over the flexor tendon
(a) What is the aetiology of trigger finger? sheath then a steroid injection may be of benefit
Trigger finger is caused by fibrosis and but often a tendon release, by incising the
thickening of the flexor tendon sheath as the sheath, is required.
tendon enters the digit. They may be idiopathic,
or may follow trauma or some unusual activity.
Occasionally they can be congenital and FURTHER READING
present in children. The other name for this Ryzewicz M, Wolf JM: Trigger digits: principles,
condition is stenosing tenovaginitis. A similar management, and complications, J Hand Surg
triggering may occur in rheumatoid arthritis, (Am) 31(1):135146, 2006.

CASE 89 INGROWING TOENAIL **

INSTRUCTION QUESTIONS
Look at this gentlemans feet and tell me what
(a) What treatments are available for
you would do about his problem.
ingrowing toenails?
Non-surgical: good nail care, with the help of a
APPROACH chiropodist, trimming nail transversely, using
Expose both feet for comparison. cotton wool packs to lift up the nail and keeping
the foot clean and dry.
Surgical options include:
VITAL POINTS Simple nail avulsion
Inspect Best treatment for acutely infected toes
as nail-bed treatment carries the risk of
Most commonly affects the lateral aspect of osteomyelitis in the presence of infection;
the great toenail recurrence and regrowth are common
Lateral aspect of the nail seen to be digging When combined with the use of phenol,
into the substance of the toe recurrence is less common, but this
Look for signs of inflammation such as increases the risk of postoperative
swelling and erythema infection
Look for evidence of serous or purulent Wedge excision
discharge. Excision of the involved aspect (lateral or
medial) of the nail and nail-bed with a
wedge of the nail fold down to the
Finish your examination here periosteum of the phalanx distal to
the joint
Zadeks procedure
TOP TIP Total excision of the nailbed including the
germinal matrix.
If this diagnosis is obvious, tell the examiner
immediately what is wrong and move on to discuss the (b) What are the complications of ingrowing
treatment options.
toenail surgery?
Wound infection
Regrowth
Osteomyelitis and septic arthritis.
Case 90 Musculoskeletal and neurology 155

Rounding C, Bloomfield S: Surgical treatments


FURTHER READING
for ingrowing toenails, Cochrane Database Syst
Heidelbaugh JJ, Lee H: Management of the Rev 18(2):CD001541, 2005.
ingrown toenail, Am Fam Physician 79(4):303 www.bofas.org.uk/PublicArea/PatientAdvice/
308, 2009. Ingrowingtoenail/tabid/105/Default.aspx
information for patients.

CASE 90 LIGAMENTOUS AND CARTILAGINOUS


KNEE LESIONS **

INSTRUCTION Feel (lying supine)


Examine this gentlemans right knee.
Tenderness
Medial joint line indicates likely medial
APPROACH meniscal tear or osteochondral defect
Lateral joint line indicates likely lateral
Proceed with the routine knee examination (see
meniscal tear or osteochondral defect
Case 77).
Effusion may be a sympathetic effusion
secondary to a cruciate ligament or meniscal
VITAL POINTS injury.

Inspect standing (front and back)


Move
Scars look particularly for:
Block to extension = locked knee (indicates
Arthroscopic portal scars: anteromedial
meniscal, cruciate or osteochondral injury).
and anterolateral scars found in the
triangle between the patella tendon, tibial
plateau and femoral condyle Special tests
Meniscectomy scars: transverse/oblique
scar over anteromedial aspect of knee Cruciate ligaments
joint for open medial meniscectomy and Posterior sag posterior cruciate
transverse/oblique scar over anterolateral ligament (PCL) injury
aspect of knee for open lateral Anterior drawer anterior cruciate
meniscectomy ligament (ACL) injury
Valgus or varus deformity of the knee: may Posterior drawer PCL injury
be secondary to an old meniscectomy
Positive Lachman test (most sensitive)
resulting in secondary osteoarthritis.
ACL injury
Collateral ligaments
Gait Medial joint line opens up on stress
testing medial collateral ligament (MCL)
Antalgic if the knee is painful, e.g. injury
secondary to a meniscal tear or recent injury. Lateral joint line opens up on stress
testing lateral collateral ligament (LCL)
Measure injury
Menisci
Quadriceps wasting may be demonstrated Positive modified McMurrays test
(see Case 77). medial or lateral meniscal tear.

Finish your examination here


156 Musculoskeletal and neurology Case 90

Completion Surgical:
Arthroscopic or open
Say that you would like to: Partial meniscectomy
Examine the hip and ankle (the joint above Meniscal repair
and below the knee joint) Meniscal transplant and meniscal
Assess the neurovascular status of the limb replacement (novel, unproven techniques).
Ask the patient some questions to ascertain
how much the problem affects his life, e.g.
ADVANCED QUESTIONS
work, sports, etc.
(a) What do you know of the anatomy of
QUESTIONS the menisci?
Medial meniscus:
(a) Cruciate ligament injuries are one cause of
Semicircular
haemarthrosis of the knee. What other causes
Anterior horn attaches to the anterior
can you think of?
intercondylar fascia of the tibia anterior to
Primary spontaneous haemarthrosis: the ACL tibial insertion
Occurs without trauma Posterior horn attaches posteriorly to the
May be secondary to disorders of intercondylar fascia between the PCL tibial
coagulation or vascular malformations insertion and the posterior insertion of the
Secondary haemarthrosis: lateral meniscus
Secondary to trauma Bound to the joint capsule peripherally
80% are due to ACL injury Bound to the femur and tibia at its midportion
by the deep medial collateral ligament
10% are secondary to patellar dislocation
Lateral meniscus:
10% follow tears in the peripheral third of
the menisci (where the meniscus is Nearly circular
vascularized), capsular tears and Covers a greater area of the tibial articular
osteochondral or osteophyte fractures. surface than the medial meniscus
Anterior horn attaches to the tibial eminence
(b) What factors in the history point to an behind the ACL tibial insertion
ACL injury? Posterior horn attaches behind the tibial
Most commonly associated with valgus/ eminence anterior to the posterior edge of
external rotation, hyperextension, the medial meniscus
deceleration and rotational movements Loosely attached to its respective tibial
Patient hears a pop or feels something tear plateau by a capsular apron known as the
in >50% of cases coronary ligament
Inability to continue sport or activity Medial and lateral menisci are connected
Effusion (haemarthrosis) developing within to each other anteriorly via the transverse
46h. ligament.

(c) What are the problems associated with (b) What do you know of the anatomy of
ACL rupture? the cruciate ligaments?
Abnormal knee movements occur leading to: Anterior cruciate ligament:
Meniscal tears Intracapsular
Collateral ligament injury Originates from the medial aspect of the
Progressive premature osteoarthritis. lateral femoral condyle
Inserts into the anterolateral aspect of the
(d) How do you treat a meniscal tear? medial tibial plateau
Treatment depends on age, chronicity of injury, Stops tibia moving forward (anteriorly) in
activity requirements and location, type and relation to the femur (and also resists tibial
length of tear, but there are options available. rotation and varus-valgus angulation)
Non-surgical: no intervention; treat Consists of two bundles, the anteromedial
symptomatically (tight in flexion) and posterolateral (tight in
extension) bands
Case 91 Musculoskeletal and neurology 157

Posterior cruciate ligament: Discoid meniscus


Intraarticular but extrasynovial Pathology in the hip
Broad origin forming a semicircle on the Osteochondritis dissecans
lateral aspect of the medial femoral condyle Adolescent causes are:
Inserts in a depression 1cm inferior to the Meniscal tear
articular surface on the posterolateral aspect Cruciate ligament injury
between the medial and lateral tibial plateaus
Osteochondritis dissecans
Stops tibia moving backwards (posteriorly) in
Synovial chondromatosis
relation to the femur
Adult:
Consists of two functional components, the
anterolateral group (tight in flexion) and the Meniscal tear
anteromedial group (tight in extension). Cruciate ligament injury
Loose body
(c) How do you treat ACL ruptures? Osteochondral fracture
Non-surgical: intensive physiotherapy Synovial chondromatosis
Re-education of quadriceps and hamstrings; Elderly:
note that the hamstrings restrict the amount Meniscal tears
of forward movement of the tibia in relation
Loose body
to the femur
Intraarticular tumour (rare).
The results of physiotherapy may be
predicted by Noyes rule of thirds a third
will compensate and pursue normal FURTHER READING
recreational sports, a third will reduce
sporting activities with avoidance of jumping Beynnon BD, Johnson RJ, Abate JA, et al:
and pivoting exercises, and a third will do Treatment of anterior cruciate ligament injuries,
poorly and develop instability with sporting part I, Am J Sports Med 33(10):15791602,
and activities of daily living, thus needing 2005.
early surgery Beynnon BD, Johnson RJ, Abate JA, et al:
Surgical: Treatment of anterior cruciate ligament injuries,
part II, Am J Sports Med 33(11):17511767,
Intraarticular reconstruction: commonly
2005.
autologous hamstring tendon or bone-
patellar tendon-bone graft Iobst CA, Stanitski CL: Acute knee injuries. Clin
Extraarticular reconstruction: e.g. MacIntosh Sports Med 19(4):621635, 2000.
tenodesis www.arthroscopy.com/sp05018.htm pictorial
Combination of both the above. explanation of anterior cruciate ligament
reconstruction.
(d) What are the causes of a locked knee?
Causes can be classified by age. Causes in
childhood include:

CASE 91 RADIAL NERVE LESIONS **

INSTRUCTION neurological lesion, of the median, ulnar or


radial nerves. Beginning with the Top Tips
This gentleman is complaining of weakness of (see Case 79) will allow you to get swiftly to
his right hand. Have a look at his hands and tell the diagnosis without wasting time.
me what you think the problem may be.
Expose to elbows and ask the patient to
place his hands palm upwards on a pillow
APPROACH (if available). To observe wrist drop, ask him
to keep his hands out in front of him, palms
Within surgical short cases, the likely reason for downward.
weakness of the hand will be a specific
158 Musculoskeletal and neurology Case 91

VITAL POINTS Motor assessment


Inspect Begin with the metacarpophalangeal
extensors (Top Tip, Case 79) and note that
There is unlikely to be any wasting of the extension is lost (Fig. 92)
hand muscles but the hand is held with the Fix the MCPJ extensors and demonstrate
wrist and fingers flexed (drop wrist) that extension at the PIPJ is preserved as
If the radial nerve has been damaged at the the lumbricals and interossei are supplied by
origin in the brachial plexus the whole arm is the median (lateral 2 lumbricals) and ulnar
deformed (as in Erbs palsy, see Case 100). (rest of muscles) nerves
Finally, test triceps (extension of the elbow)
Sensory assessment to demonstrate weakness in high radial nerve
lesions.
Note the loss of sensation over the first
dorsal interosseus, which is on the dorsum
of the hand between the thumb and index
finger
Test also for sensory loss over the dorsal
aspect of the forearm.

Triceps, long head

Triceps, lateral head Triceps,


medial head

Radial nerve
Brachioradialis
Extensor carpi radialis
longus
Extensor carpi radialis
brevis
Supinator Posterior
Extensor carpi ulnaris interosseous
Extensor digitorum nerve
Extensor digiti minimi
Abductor pollicis longus
Extensor pollicis longus
Extensor pollicis brevis
Extensor indicis

Figure 92 Muscles supplied by the radial nerve.


Case 92 Musculoskeletal and neurology 159

TOP TIP Middle lesions:


Occur in the spiral groove of the humerus,
When examining the peripheral nerves of the upper
usually as a result of a fracture of the middle
limb, begin at the hand and move proximally. Continue
third of the humerus, sometimes secondary
examining one nerve in its entirety before moving onto the
to the use of a tourniquet
next nerve, by which time the examiner will usually have
stopped you. This allows a precise definition of the In some instances the elbow is preserved, as
anatomical location of the nerve lesion described. the supply to the elbow leaves the main
trunk of the nerve before it enters the groove
Low lesions:
These occur at the elbow, due to local
Finish your examination here wounds, surgery or fracture/dislocations
Only the posterior interosseus branch of the
nerve is involved and sensation is preserved.
Completion The only lesion will be loss of extension at
the MCPJ.
Say that you would like to:
Examine the neck and all the other peripheral ADVANCED QUESTIONS
nerves of the affected limb.
(a) What is Wartenbergs syndrome?
QUESTIONS Compression of the sensory component of the
radial nerve as it exits from under brachioradia-
(a) What are the likely causes of a radial lis at the wrist. This is an unusual situation
nerve lesion? where an even more distal lesion can actually
cause a sensory loss over the 1st dorsal
High lesions:
interosseus.
Occur at the level of the brachial plexus,
usually by crutches or Saturday night palsy
(secondary to falling asleep with arm leaning FURTHER READING
over the top of a chair, thus compressing the
Toussaint CP, Zager EL: Whats new in
radial nerve in the axilla usually secondary
common upper extremity entrapment
to a state of alcoholic stupor!)
neuropathies, Neurosurg Clin N Am 19(4):573
This causes loss of extension of the elbow, 581, 2008.
wrist drop and loss of sensation over the 1st
dorsal interosseus webspace

CASE 92 EXAMINATION OF THE SHOULDER **

INSTRUCTION APPROACH
Examine this ladys right shoulder. She is You need to be able to see both shoulder joints
complaining of pain during movement. ask her to undress down to her bra (or
suitable top) to be able to see both shoulders
(you may wish to ask for a chaperone). Male
Note patients should be asked to take off their shirt
to adequately expose both upper limbs.
The three most common pathologies of the
shoulder that you may encounter in the exam
are rotator cuff tears, impingement syndrome VITAL POINTS
and frozen shoulder (adhesive capsulitis). The
latter two appear less commonly as they can be Look (from front and behind)
very painful.
Skin scars, sinuses
160 Musculoskeletal and neurology Case 92

Symmetry looking particularly for wasting Move


of:
Deltoid: loss of contour of shoulder Stand opposite the patient and ask her to
Supraspinatus: look at the muscle bulk imitate you in order to test active movements.
above the spine of the scapula from Normal ranges of movement are:
behind Forward flexion to 165 Lift both your
Infraspinatus: look at the muscle bulk arms forwards (Fig. 95)
below the spine of the scapula from Abduction to 180 (of which the first 90 is
behind glenohumeral, although scapulothoracic
Shape, looking particularly for: movement starts at 3040 abduction) Lift
Prominent sternoclavicular joint (SCJ) both your arms up to the side (Fig. 96)
due to subluxation or osteoarthritis If there is difficulty initiating abduction or
Clavicular deformity due to old fracture the humeral head rises up, consider
Prominent acromioclavicular joint (ACJ) rotator cuff tear
due to subluxation.

Feel

Skin: increased temperature


Bones and joints systematically work your
way laterally starting at the SCJ, feeling for
tenderness:
SCJ
Clavicle
ACJ
Acromion
Humeral head
Greater tuberosity
Coracoid (Fig. 93)
Spine of the scapula (Fig. 94)
Tendons:
Long head of biceps can be palpated Figure 94 Palpating the spine of the scapula.
for tenderness anteriorly in the bicipital
groove as the shoulder is internally and
externally rotated
Supraspinatus can be palpated just
under the anterior edge of the acromion
as the shoulder is held in extension.

Figure 93 Palpating the coracoid. Figure 95 Assessing forward flexion of the shoulder.
Case 92 Musculoskeletal and neurology 161

Figure 96 Assessing abduction of the shoulder.

Figure 97 Assessing external rotation of the shoulder.

If there is a painful arc from 60120,


think of rotator cuff tendinitis (also known
as impingement syndrome) or a minor
rotator cuff tear
If there is a painful arc from 140180 i.e.
a painful high arc, think of osteoarthritis
of the ACJ
External rotation to 60 (with the elbow
flexed to 90 and tucked into the side of the
body) (Fig. 97)
This movement is most commonly
affected in frozen shoulder
Internal rotation measured as thumb
reaching mid-thoracic level (T6) put your
hand behind your back and reach up as far
as you can (Fig. 98).
You can then test the passive range of the
same movements, but prevent scapulothoracic
movement by anchoring the scapula (press Figure 98 Assessing internal rotation of the shoulder.
162 Musculoskeletal and neurology Case 92

firmly down on top of the shoulder with one


TOP TIP
hand).
This is followed by testing for power, particularly The mnemonic SITS can be used to remind you of
of: the tendons of the rotator cuff:

Deltoid: test abduction against resistance Supraspinatus (abductor)


(Fig. 99) an axillary nerve palsy can result Infraspinatus (external rotator)
in decreased deltoid power, with loss of
Teres minor (external rotator)
sensation in the regimental badge (British)
area of skin of the shoulder Subscapularis (internal rotator).
Serratus anterior: pushing against a wall may
demonstrate winging of the scapula
secondary to a long thoracic nerve palsy
(see Case 104). Special tests

You can test each muscle of the rotator cuff


(see the mnemonic) individually for pain (in
rotator cuff tendinitis) and weakness (in rotator
cuff tears):
Supraspinatus: resisted abduction with arm
in maximum internal rotation with 20
abduction and 20 flexion (Fig. 100) if torn,
the patient cannot initiate abduction
Infraspinatus/teres minor: resisted external
rotation with elbow flexed to 90 (Fig. 101)
Subscapularis: the most sensitive test is
Gerbers lift-off test where the shoulder is
internally rotated with the arm behind the
back and the hand is resisted from being
lifted off posteriorly (Fig. 102); resisted
internal rotation can be tested with the elbow
flexed to 90, but this does not isolate
subscapularis.
Figure 99 Testing the power of deltoid. If you have time, you can also test specifically
for biceps tendon pathology:

Figure 100 Testing the power of supraspinatus.


Case 92 Musculoskeletal and neurology 163

Assess the neurovascular status of the limb


Ask the patient some questions to assess
how the condition affects her life.

QUESTIONS
(a) What are the causes of a painful shoulder?
These can be divided up into:
Tendon (rotator cuff) disorders:
Tendinitis
Rupture
Frozen shoulder
Joint disorders:
Glenohumeral arthritis
Acromioclavicular arthritis
Referred pain:
Cervical spondylosis
Figure 101 Testing the power of the external rotators Cardiac ischaemia
(infraspinatus/teres minor). Mediastinal pathology
Instability:
Dislocation
Subluxation
Bone lesions:
Infection
Neoplasms
Nerve lesions
Suprascapular nerve entrapment.

(b) What is the aetiology of rotator cuff


impingement?
Repetitive rubbing of rotator cuff tendons
under the coracoacromial arch (the
coracoacromial ligament forms the roof of
Figure 102 Testing the power of subscapularis. this arch and runs between the coracoid
process anteriorly and the anterior third of
the acromion posteriorly)
Resisted elbow flexion with forearm in
The greatest amount of wear occurs in the
neutral rotation: biceps bulge if the long
impingement position abduction, slight
head is ruptured (Popeye bulge)
flexion and internal rotation
Speeds test: resisted forward flexion of the
Site of impingement: critical area of
shoulder with the elbow extended and the
decreased vascularity in the supraspinatus
forearm supinated
tendon about 1cm proximal to its insertion
Yergasons test: resisted supination with the
into the greater tuberosity
elbow flexed to 90.
Contributing factors to impingement:
Bone: osteoarthritic thickening of ACJ;
Finish your examination here osteophytes anterior edge of acromion
Tendon: rotator cuff swelling (in
inflammatory disorders such as
Completion rheumatoid arthritis and gout)
Bursa: subacromial bursitis (in
Say that you would like to: inflammatory disorders as above).
Examine the neck (joint above) and elbow
(joint below)
164 Musculoskeletal and neurology Case 92

(c) How do you treat impingement syndrome? Extrinsic causes (painful disorders leading to
Non-surgical: decreased movements of the shoulder):
Referred pain from cervical radiculopathy
Eliminate aggravating activity/avoid
impingement position Post-hand, wrist or elbow surgery
Physiotherapy Post-breast surgery (especially when
axillary node dissection has been
Short courses of analgesia, e.g. non-steroidal
performed)
anti-inflammatory drugs
Post-myocardial infarct.
Subacromial corticosteroid injections for pain
relief (b) How do you treat a frozen shoulder?
Surgical:
Non-surgical:
Open or arthroscopic subacromial
Pain relief analgesic ladder (see Case 118),
decompression.
interscalene blocks
(d) How do you treat rotator cuff tears? Physiotherapy especially pendulum
exercises
Non-surgical:
Manipulation under anaesthesia and steroid/
Physiotherapy to improve overall shoulder local anaesthetic injections once acute
muscle strength pain has settled
Surgical: Surgical:
Open or arthroscopic cuff repair (if amenable Surgery has an ill-defined role
to repair) and subacromial decompression
Reserved for prolonged and disabling
Open or arthroscopic cuff debridement (if not restriction
amenable to repair) and subacromial
Open or arthroscopic rotator interval,
decompression.
coracohumeral ligament release and excision
of the coracoacromial ligament.
ADVANCED QUESTIONS
(a) What are the causes of a frozen shoulder? FURTHER READING
Ben Kibler BW, Sciascia A: What went wrong
Primary frozen shoulder (adhesive capsulitis):
and what to do about it: pitfalls in the treatment
Often idiopathic of shoulder impingement. Instr Course Lect
Strong associations with diabetes and 57:103112, 2008.
Dupuytrens contracture Dias R, Cutts S, Massoud S: Frozen shoulder.
Global contracture of the shoulder joint, but BMJ 331(7530):14531456, 2005.
maximally in the rotator interval area and
Rees JL: The pathogenesis and surgical
around the coracohumeral ligament
treatment of tears of the rotator cuff. J Bone
Histologically, the contracture is made of a Joint Surg Br 90(7):827832, 2008.
dense collagen matrix, with numerous
fibroblasts and myofibroblasts this active http://orthoinfo.aaos.org/topic.
fibroblast and myofibroblast proliferation is cfm?topic=a00065 patient information website
similar to the histology of Dupuytrens on shoulder pain.
contracture (see Case 78) www.nismat.org/orthocor/exam/shoulder.html
The frozen shoulder may remain frozen due how to examine the shoulder with excellent
to slow remodelling as a result of high levels anatomical models showing the various joints
of tissue inhibitors of metalloproteinases and tendons around the shoulder girdle and
(TIMPSs), which inhibit matrix clinical photographs.
metalloproteinases (MMPs)
Secondary frozen shoulder:
Intrinsic causes:
Chronic rotator cuff injuries
Post-traumatic scarring following fractures
around the shoulder, e.g. surgical neck or
tuberosity fractures
Case 93 Musculoskeletal and neurology 165

CASE 93 GAIT **

INSTRUCTION QUESTIONS
This gentleman has an abnormal gait. Watch
him walk and tell me what you make of it.
(a) What are the phases of gait?
Four phases make up the gait:
Heel strike (more correctly known as initial
APPROACH
contact)
This is a potentially difficult case, as often the Stance, when foot is on the ground and the
different abnormal gaits can be problematic to centre of gravity of the body moves forward
distinguish.
Toe off, as the foot begins to lift off the
Expose the patients lower limb keeping his ground from the heel forward
underwear on but ask him to remove his socks Swing, as the foot moves forward while the
and shoes. contralateral foot supports the weight of the
body.
VITAL POINTS (b) What are the common abnormalities of
Inspect gait?
Table 26 summarizes the various abnormalities
Ask the patient to walk towards a given point you would be expected to know about.
at the other side of the room and then back
towards you
Note if he has difficulty in initiating
FURTHER READING
movement or other signs of Parkinsons Devinney S, Prieskorn D: Neuromuscular
disease (unlikely in a surgical exam) examination of the foot and ankle, Foot Ankle
Look to see if the patient grimaces as if with Clin 5(2):213233, 2000.
pain (does he have an antalgic gait?).

Finish your examination here

Completion

Say that you would like to:


Examine the back and hip joints, including
performing a Trendelenburg test (see
Case 76)
Measure the leg lengths (in short leg gait)
(see Case 76)
Examine the lower limb neurology, looking
specifically for footdrop (see below).
166 Musculoskeletal and neurology Case 94

Table 26
Type of gait Description Reason for abnormality
Antalgic Decreased stance and Pain
increased swing phase
Trendelenburg Hip dips instead of rising when Abductor weakness
(see Case 76) foot is lifted off floor, shoulders
also lurch to opposite side
Parkinsonian Small shuffling steps Parkinsons disease
(=festinant gait)
Broad-based Reels, lurches to one side Cerebellar lesions
Short leg Ipsilateral hip drops when Previous fracture or congenital shortening
weight is on short leg
High stepping Foot lands flat or on ball Foot drop inability to dorsiflex the foot secondary to
instead of on heel, with foot damage to L5 (which supplies extensor hallucis longus,
slapping to the ground extensor digitorum longus and tibialis anterior) this is
most commonly due to common peroneal nerve palsy
(e.g. trauma to fibular head, tight casts) or sciatic nerve
palsy (e.g. gunshot wounds, posterior hip dislocations,
posterior approach during surgery to the hip joint)
Spastic Jerky, feet in equinus, hips Likely to be an upper motor neurone cause such as cord
adducted (scissoring) compression, multiple sclerosis or cerebral palsy

CASE 94 OSTEOCHONDROMA **

INSTRUCTION Palpate
Examine this lump and tell me your diagnosis.
Smooth surface
Narrow base
APPROACH Osteochondromas point away from the joint
May be bony-hard in consistency or soft if
Expose the relevant area note that osteochon-
there is an overlying bursa
dromas are adjacent to the epiphyseal line in
the diaphyseal side of the bone and that they Move the adjacent joint while palpating the
are most commonly found around the knee joint lump and assess the relationship with
(lower end of the femur or upper end of the adjacent muscle and tendons, and degree of
tibia). interference with joint movement.
Examine as for lumps (see Case 2).
QUESTIONS
VITAL POINTS (a) What is an osteochondroma?
Inspect Lump of cancellous bone with a covering of
cortical bone and a cartilaginous cap
Hemispherical lump They originate from the separation of small
Solitary or multiple (the latter may be part of pieces of metaphyseal cartilage from the
hereditary multiple exostoses). main cartilaginous physis these small
pieces escape remodelling and carry on
growing and ossifying
Usually start to grow rapidly in adolescence.
Case 95 Musculoskeletal and neurology 167

ADVANCED QUESTIONS (b) What is the surgical treatment?


The osteochondroma should be excised if
(a) What is multiple hereditary symptomatic (such as pressure symptoms or
osteochondromatosis? cosmesis) or if it increases in size or becomes
more painful (suspicion of malignancy).
An autosomal dominant condition,
characterized by multiple osteochondromas
particularly in the limb bones (previously FURTHER READING
known as diaphyseal aclasis)
Kitsoulis P, Galani V, Stefanaki K, et al:
All bones that ossify in cartilage can be
Osteochondromas: review of the clinical,
affected except the spine and skull
radiological and pathological features, In Vivo
It is thought to be a failure of growth-plate 22(5):633646, 2008.
modelling
There is a small incidence of sarcomatous
change.

CASE 95 POPLITEAL FOSSA SWELLINGS **

INSTRUCTION Finish your examination here


Examine this gentlemans right knee.

Completion
APPROACH
Approach as for knee examination (see Say that you would like to:
Case 77). Continue with the rest of the knee
examination (the examiner may allow you to
proceed if there are further signs to elicit,
VITAL POINTS e.g. osteoarthritis of the knee)
You should aim to elicit features of a lump (see Perform a neurological and peripheral
Case 2). The vital points are outlined below. vascular examination, including the
peripheral pulses
Examine the joint above (the hip) and the
Look from behind and from the side joint below (the ankle).
(with the patient standing)

Visible swelling in the popliteal fossa QUESTIONS


(proceed with the following routine, leaving
gait and measure until you have fully (a) What is your differential diagnosis?
described this swelling) Skin and subcutaneous tissues lipoma (see
Skin involvement sebaceous cyst. Case 2), sebaceous cyst (see Case 3)
Artery popliteal artery aneurysm (see Case
Palpate from behind 117)
Vein saphena varix (at the saphenopopliteal
Pulsatile (swelling overlying popliteal artery) junction, see Case 109), deep vein
or expansile (popliteal artery aneurysm) thrombosis
Fluctuant (cystic swelling) Nerve neuroma (e.g. tibial nerve)
Compressible (saphena varix of the Enlarged bursae, e.g. associated with
saphenopopliteal junction) semimembranosus and medial head of
gastrocnemius medially and with popliteus
Transilluminable (cystic swelling).
and lateral head of gastrocnemius laterally
(Fig. 103)
168 Musculoskeletal and neurology Case 95

Found in children and young adults (in


adults, may be associated with pathology in
the knee)
Twice as common in boys compared
with girls
Semimembranosus Usually unilateral
bursa Becomes more prominent when knee
Above the joint line extended and disappears in flexion
Firm and can be transilluminated
Joint line If the cyst is not found in the medial part of
the popliteal fossa or has a solid component
(on ultrasound), rule out tumour with a CT or
Baker's cyst MRI scan
Below the joint line Cysts can either be aspirated and sent for
and deep to cytology if the patient (or the childs parents)
gastrocnemius are concerned, or surgically excised (with a
high rate of recurrence)
Figure 103 Anatomical difference between a
Although spontaneous remission is not to be
semimembranosus bursa and a Bakers cyst.
expected in all cases, asymptomatic
popliteal cysts in children can be treated
non-surgically with good results.
Cysts Bakers cyst (associated with
degenerative changes in the knee joint) and
popliteal cyst (enlargement of the popliteal ADVANCED QUESTIONS
bursa but the knee joint is normal).
(a) What are the boundaries of the popliteal
(b) What is a Bakers cyst? fossa?
Described by Baker as a posterior herniation Superomedial: semimembranosus and
of the capsule of the knee joint semitendinosus tendons
Leads to escape of synovial fluid into one of Superolateral: biceps tendon
the posterior bursae, stiffness and knee Inferomedial: medial head of gastrocnemius
swelling
Inferolateral: lateral head of gastrocnemius
Often associated with degenerative knee
Roof: fascia lata
joint disease
Floor: (from proximal to distal) popliteal
Diagnosis is confirmed by ultrasound
surface of the femur, capsule of the knee
examination identification of fluid between
joint and popliteus muscle covered by its
the semimembranosus and medial
fascia.
gastrocnemius tendons in communication
with a posterior knee cyst indicates Bakers
cyst with 100% accuracy
Aspiration is possible, although recurrence is William Morrant Baker (18391896). English surgeon,
common St Bartholomews Hospital, London. Private assistant
Important to rule out deep vein thrombosis, to Sir James Paget.
but the two can co-exist.

(c) What is a popliteal cyst?


Usually located on the medial side of FURTHER READING
popliteal fossa, just distal to the flexion Fritschy D, Fasel J, Imbert JC, et al: The
crease of the knee, under the medial head of popliteal cyst, Knee Surg Sports Traumatol
gastrocnemius Arthrosc 14(7):623628, 2006.
Case 96 Musculoskeletal and neurology 169

CASE 96 HALLUX RIGIDUS **

INSTRUCTION QUESTIONS
Examine this gentlemans right foot.
(a) What is hallux rigidus?
Painful loss of motion (particularly
APPROACH dorsiflexion) of the first MTPJ secondary to
Expose both ankles and feet and begin by degenerative joint disease
describing any obvious deformities. May be due to bony degeneration, which
can be either primary, as in osteoarthritis, or
secondary to degenerative conditions, such
VITAL POINTS as gout
May be due to capsular damage and
Look (and ask)
contraction.

Likely to be unremarkable if you notice no (b) What are the radiological changes
abnormalities on initial inspection, ask the associated with hallux rigidus?
gentleman where exactly the problem is, in
Initially normal
order to focus your examination on the big
toe (this will stop you wasting time Degenerative osteoarthritic changes seen
examining all the joints of the foot) later, particularly joint space narrowing and
marginal osteophytes (especially dorsally and
Note that he is likely to be complaining of
laterally).
pain in the right big toe on weightbearing,
particularly during the push-off phase of gait. (c) How would you treat this gentlemans
condition?
Feel Non-surgical:
Appropriate footwear, e.g. stiff-soled shoes
First metatarsophalangeal joint (MTPJ)
to limit dorsiflexion of the first MTPJ or
tenderness, particularly on the dorsal surface
rocker-bottom soles
Unilateral versus bilateral involvement.
During acute exacerbations use of
non-steroidal anti-inflammatories (NSAIDs) or
Move intra-articular steroid injections for temporary
relief this is technically difficult if the
Limited first MTPJ dorsiflexion spaces between the joints has narrowed
May be associated with crepitus (see below). Surgical:
Early disease (good range of movement and
Finish your examination here little loss of joint space) cheilectomy
(excision of dorsal segment of
metatarsal head)
Advanced disease consider silastic
Completion interposition arthroplasty or arthrodesis.

Say that you would like to:


Watch the patient walk ADVANCED QUESTIONS
Examine his shoes (a) What other diagnosis should you be aware
Ask him questions to assess the effect of the of when managing a patient with hallux
condition on his life. rigidus?
Diffuse joint osteoarthritis characterized by
tenderness of the sesamoid bones and more
crepitus than in hallux rigidus.
170 Musculoskeletal and neurology Case 97

(b) What is the optimal position for fusion of FURTHER READING


the MTPJ during arthrodesis?
Yee G, Lau J: Current concepts review: hallux
The position of fusion should be tailored to the rigidus, Foot Ankle Int 29(6):637646, 2008.
individual, but generally:
www.bofas.org.uk/PublicArea/PatientAdvice/
1015 of dorsiflexion (with respect to the Halluxrigidus/tabid/102/Default.aspx
floor) information for patients.
15 of valgus
Neutral rotation.

CASE 97 CASTS **

(see Notes) if there is any suspicion that


INSTRUCTION
the cast is too tight, remove it
Patients with casts can easily be recruited for Loosening of the cast leading to failure of
the clinical examination. Be prepared to fixation
describe casts and to discuss their advantages Skin complications:
and complications.
On application pressure sores can occur
particularly over bony prominences and are
Description of the cast best prevented by adequate padding
On removal abrasions and lacerations can
Plaster of Paris or newer cast material (see occur, especially if an electric saw is used
below) Foreign body may fall into the cast
Complete or incomplete (the latter is also necessitating replacement
known as a slab, or backslab) Allergic reactions (rare).
Positioning of the cast, such as above or
below the joint. Remember that ideally the
joints above and below the fracture should
Advantages of newer cast materials
be immobilized in order to prevent
displacement of the fracture Lighter
Special casts, such as patella tendon Stronger
bearing (Sarmiento) casts which allow More radiolucent
movement of the knee while controlling the Waterproof
rotation of tibial fractures Resistant to wear and tear
Has functional cast bracing been used? Different colours available (for children)
These are casts with hinges attached to Less untidy to apply (than plaster of Paris).
allow some movement of the joint in a
controlled manner. They are most commonly
used for fractures of the tibia or femur. Disadvantages of newer cast materials
Functional cast braces can be:
Simple, such as for the elbow Expense
Complex, e.g. polyaxial at the knee. Skin irritation (gloves should be worn by the
applicant)
Less deformation after setting, therefore
Complications increased risk of neurovascular compromise
if the limb swells
Tight casts: More difficult to apply
Due to tight casting, ridges in the cast due Can be very difficult to remove in an
to poor application or limb swelling. This emergency situation without a proper
may lead to vascular compression and plaster saw.
compartment syndrome. Warn patients of
the symptoms of compartment syndrome
Case 98 Musculoskeletal and neurology 171

Plaster of Paris points of note Highly reactive material which is packaged in


resistant containers.
Use first described by Antonius Mathijsen,
a Flemish military surgeon, in 1854 for war
NOTES
injuries
The name originates from the Montmartre Compartment syndrome is defined as
region of Paris where it was first mined elevated compartment pressures within a
soft-tissue envelope (either due to decreased
Consists of open-weave cloth or muslin
space or increased pressure)
strip impregnated with dehydrated calcium
sulphate Remember all the Ps for symptoms and
signs
On addition of water, a latent heat of
crystallization reaction occurs, calcium Pain out of Proportion to injury
sulphate becomes hydrated and then rapidly Pain on Passive flexion
sets Palpate tense compartment
Cloth is made stiffer using dextrose or starch Pallor, Paralysis, Paraesthesia and
Hardening rates can be altered using Pulselessness (these are all seen late in
accelerators or inhibitors. the condition)
The diagnosis is made clinically and can be
confirmed with compartment pressure
Newer cast materials points of note monitoring with the latter, urgent treatment
is needed if the absolute compartment
Knitted material of cloth or glass fibre pressure is greater than 30mmHg or if the
impregnated with a monomer or polymer of compartment pressure is within 30mmHg of
polyurethane with substituted isocyanate the diastolic blood pressure
terminal groups
The treatment is emergency fasciotomy to
On addition of water the compound decompress the involved compartments.
polymerizes and releases carbon dioxide

CASE 98 SIMULATED REDUCTION OF


FRACTURES **

INSTRUCTION VITAL POINTS


You may be asked to demonstrate the reduction Initiation
of a common fracture (e.g. fracture of the distal
radius) on a patient model your understanding Place yourself on the same side of the
of the principles of fracture reduction is being patient as the injury
examined, e.g. This lady has presented to you
Explain the fracture to her and the need for
in Accident and Emergency with a fracture of
reduction
the right distal radius which you have confirmed
Explain that you will provide pain relief with a
on X-ray views. I would like you to demonstrate
combination of inhaled nitrous oxide and
how you would reduce this fracture in A&E.
local haematoma block with local anaesthetic
You may be shown an X-ray of the fracture at
(or your own preferred choice of analgesia).
this point.

Assistance
APPROACH
Introduce yourself to the patient and begin by Tell the examiner that you would use him as
describing the fracture on the X-ray (the site an assistant and position his hands proximal
of the fracture, any associated displacement, to the fracture site in order to provide
deviation or shortening, involvement of the counter-traction (usually just distal to the
ulna, etc.). flexed elbow).
172 Musculoskeletal and neurology Case 99

Traction a complete cast as swelling may occur


post-application)
Demonstrate the way that traction is A broad arm sling provides additional
performed, but do not pull with any force comfort.
do not harm the patient
Explain that the first manoeuvre is linear X-rays
traction by yourself from distal to the fracture
site traction serves to relax the Anteroposterior and lateral X-rays are taken
musculature around the fracture site due to in order to check the reduction.
the inverse stretch reflex.

Follow-up
Reduction
A fracture clinic appointment is made in a
The manoeuvres performed essentially serve few days time in order to check the
to reverse the direction in which the reduction and complete the cast once the
fragment displaced at the time of injury and swelling has reduced
will depend upon the fracture itself, e.g. with
It is particularly important to X-ray the wrist
dorsal displacement of the fragment, an
at 710 days as this is the most common
opposing volar force is needed to effect the
time-point for the fracture to re-displace
reduction. Sometimes it is necessary (while
Say that you would like to give the patient an
applying longitudinal traction) to increase the
information leaflet on the care and
angulatory deformity in order to disimpact
complications of casts
the fracture, and to then angle it back into
the opposite direction. Explain that the fracture would normally
take 46 weeks to heal, after which she
may need physiotherapy for any resulting
Hold stiffness.

The fracture is held post-reduction in this


case by a below elbow dorsal backslab (not

CASE 99 LUMBAR DISC HERNIATION **

INSTRUCTION VITAL POINTS


This lady is complaining of pain in her lower Important features to extract are:
back radiating down to her right leg. Ask her a Age
few questions, examine her and tell me what
Occupation
you think the problem is.
Features of the pain, especially site,
radiation, any history of injury, and relieving
APPROACH and exacerbating factors
Neurological symptoms (e.g. weakness,
The ideal patient for this case would be one
numbness and paraesthesia) and their
with a known lumbar disc prolapse in whom
distribution
there are focal neurological signs to be elicited.
Sphincter disturbance bladder and bowel
Be as gentle as possible when examining the
symptoms (unlikely in patients used for
patients back in order to minimize the chances
examination purposes)
of hurting her.
Effect on patients lifestyle, e.g. work, sleep
Previous treatments, e.g. use of analgesia,
physiotherapy, caudal epidurals, operative
intervention
Case 99 Musculoskeletal and neurology 173

Explore other causes of back pain, e.g. Straight leg raising (SLR)
diseases of the pancreas, abdominal aortic
aneurysm, loin pain from renal causes. Demonstrates lumbosacral nerve root
irritation
EXAMINATION With the knee fully extended, gently flex the
hip and record the angle at which there is
It is essential to examine the patient in her onset of pain. This angle is normally 80 and
underwear so that the whole back and lower Lasgues sign is positive if pain is felt in the
limbs are exposed. Start with the patient back, buttock and thigh at less than 60
standing facing away from you and examine her
A feeling of tightness in the hamstring is not
gait first, then her back and finally, examine her
significant
lower limbs after asking her to lie down on the
examination couch. Crossed SLR if SLR on unaffected side
produces pain on the affected side, this
suggests L4/5 lumbar disc protrusion (this is
Gait the most specific sign for lumbar disc
herniation).
Half-shut knife position patient leans forward
with a painful, partially flexed back. Sciatic stretch test (SST)

Look Having extended the hip as above,


dorsiflexion of the foot should induce
Skin scars, sinuses further pain
From the side loss of normal lumbar Flexing the extended knee relieves the pain.
lordosis
Posture the sciatic list is an involuntary Neurological examination of
attempt by the patient to reduce nerve root the lower limbs
irritation by leaning to one side in an effort to
open up the neural foramen. Tone
Power: test in particular for movements
Feel affected by the involved nerve roots
Reflexes: knee/ankle/plantars
Muscle erector spinae muscle spasm/ Sensation: looking for specific dermatomal
tenderness. loss (see Table 27 and Fig. 104).

Move Finish your examination here


Forward flexion ask the patient to touch
her toes; normally her fingertips should be
able to reach to within 5cm of the floor
Completion
(estimate distance from bony landmarks, e.g.
ankle or knee) Say that you would like to:
Extension normally 30 Examine the patient prone in particular,
Lateral flexion ask the patient to slide hand perform a femoral stretch test (FST) to
down outside of leg; normally 30 to each exclude a L2/3/4 root lesion
side Examine the peripheral pulses to exclude a
Rotation ask the patient to sit down (to fix vascular cause
the pelvis) and fold her arms across her Examine the abdomen to exclude intra-
chest; normally 40 to each side. abdominal pathology that may cause back
pain
Perform a digital rectal examination to
Lie the patient supine check anal tone, perianal sensation and the
anal reflex to exclude cauda equina
compression (not likely to be encountered in
the clinical examination!).
174 Musculoskeletal and neurology Case 99

Table 27
Prolapsed Involved Distribution of sensory Distribution of motor signs Involved
disc nerve root symptoms reflexes
L4/L5 L5 Lateral aspect of the leg and Weakness of big toe extension None
dorsum of the foot and ankle dorsiflexion
L5/S1 S1 Lateral aspect of the foot and heel Weakness of ankle plantarflexion Ankle jerk
and foot eversion

Poor strength of spinal extensor and


abdominal muscles
L1 Decreased spinal mobility
T10
Environmental:
S4
T11 L2 Smoking
S5 S3
T12 Occupational:
Heavy physical work
L1
Frequent bending, lifting, pushing, pulling
S3 and twisting
S2 Repetitive work postures
S4 Static work postures
L2
Vibration
Psychological and psychosocial work
L3 factors.

(b) How would you treat a lumbar disc


L3
S2 herniation?
Non-surgical:
L4 Short period of bed rest (not more than 2
L5 days) with appropriate analgesia
(combination of muscle relaxant, nonsteroidal
anti-inflammatories and opioids) for acute
L5 S1 attacks
L4 Physiotherapy (back school) aimed at
muscle strengthening and stabilization, with
emphasis usually on extension exercises,
which strengthen back extensors and are
S1 less likely to increase intradiskal pressures
Epidural analgesia:
S1 L5
L5 Injection of a long-acting steroid with an
epidural anaesthetic results in 6085%
Figure 104 Dermatomes of the lower limb. short-term pain relief rate which falls to
3040% at 6 months
Surgical: indicated with a herniated disc where
there is a progressive neurological deficit and/or
QUESTIONS severe incapacitating pain (or failure of
non-surgical treatment).
(a) What factors increase the risk of
developing symptomatic disc degeneration Options include:
and herniation? Chemonucleolysis of the disc with
chymopapain success rate ~70% but out
Physiological: of favour due to complications such as
Increasing age severe postoperative pain and spasms, and
Poor posture rarely, transverse myelitis and anaphylactic
Poor overall aerobic fitness reactions
Case 100 Musculoskeletal and neurology 175

Percutaneous discectomy decompresses (b) What is the pathology of disc herniation?


the nerve root by removing disc material Pre-existing degeneration contributes to disc
from the centre of the disc space; ~50% herniation
success rate
A disc consists of an outer annulus fibrosus
Endoscopic discectomy success rates and an inner nucleus pulposus
reported to be ~80% but technique still in
Herniation is the end process of progressive
evolution
rupture of the nucleus pulposus through the
Open hemilaminotomy and discectomy posterior annulus and herniation of this
gold standard with 85% success rate and material into the spinal canal
performed with the use of loupe
The initial pathology is asymptomatic disc
magnification or operating microscope.
fissuring and fragmentation
This is followed by progressive annular
ADVANCED QUESTIONS disruption from the inner to the outer layers
In some patients, this finally results in a
(a) What is the difference between sciatica complete annular rupture with herniation of
and referred pain? disc material into the canal.
The term sciatica should only be used to
refer to nerve root pain in a specific FURTHER READING
dermatomal distribution
Referred pain is pain caused by any lesion in Awad JN, Moskovich R: Lumbar disc
a spinal motion segment which radiates herniations: surgical versus nonsurgical
distally in a non-dermatomal pattern, e.g. to treatment, Clin Orthop Relat Res 443:183197,
the buttocks, thighs, hips and occasionally 2006.
the lower leg from the back. This pain is www.bbc.co.uk/health/conditions/back_pain/
usually poorly localized, dull and less information for patients.
superficial than nerve root pain.

CASE 100 BRACHIAL PLEXUS LESIONS *

Clinical consequences enquire about pain/


INSTRUCTION sensory loss/paraesthesia/weakness
This gentleman suffered an injury to his right Functional consequences use of the limb
upper limb 5 years ago in a road traffic for daily activities
accident. Ask him a few questions first, then Previous treatments ask about operative
examine him and tell me what you think. interventions, e.g. contracture release, nerve
repair, tendon transfers
Effect of the condition on the gentlemans
APPROACH
quality of life and activities of daily living.
Ensure both upper limbs are fully exposed ask
him to take off his shirt if he has not already
undressed. Note that the patient may have a
Examination
mixed picture of upper and lower brachial
plexus injuries (Fig. 105). You are essentially performing a neurological
examination in order to distinguish between the
various lesions.
VITAL POINTS
Mechanism of injury: Inspect
If traction injury to the abducted arm,
suspect lower brachial plexus lesion Limb position
If fall onto tip of shoulder, suspect upper Waiters tip position in Erbs palsy
brachial plexus lesion (ErbDuchenne injury) C5 and C6
involvement adducted shoulder,
176 Musculoskeletal and neurology Case 100

Terminal branches Cords Divisions Trunks Roots

Dorsal C5
scapular C5

US = upper subscapular C6
TD = thoracodorsal Suprascapular C6
LS = lower subscapular
er
Upp C7
Lateral Nerve to C7
pectoral subclavius

ral dle
Mid C8
Late
T1

er
terio
r Low T1
Axillary Pos
Musculocutaneous US
TD
LS
ial
Radial Med

Median Medial
pectoral
MCNA
MCNF
MCNA = medial cutaneous nerve of arm
MCNF = medial cutaneous nerve of forearm
Ulnar

Figure 105 Organization of the nerves within the brachial plexus.

extended elbow, and forearm pronated


TOP TIP 1
and internally rotated
Claw hand in Klumpkes palsy (Dejerine Nerve roots responsible for movements of the upper
Klumpke injury) C8 and T1 involvement limb.
paralysis of the small/intrinsic muscles Shoulder abduction: C5
of the hand
Shoulder adduction: C5C7
Scars: traumatic/operative
Elbow flexion: C5C6
Muscle wasting/atrophy
Fasciculations of involved muscles Elbow extension: C7
Inspect the face for evidence of ipsilateral Wrist flexion: C7C8
Horners syndrome (ptosis, anhydrosis, Wrist extension: C7
miosis and enophthalmos) (see Case 19).
Finger flexion: C7C8
Finger extension: C7C8
Tone
Finger abduction: T1.

Flaccidity and hypotonia lower motor


neurone lesion.

Power TOP TIP 2


medical research council grading of muscle power.
Compare power of muscle groups in affected
limb to contralateral limb note that in C5/6 M0 No contraction
lesions, there is decreased shoulder M1 Flicker or trace of contraction
abduction and elbow flexion, while in C8/T1 M2 Active movement with gravity eliminated
lesions, there is decreased wrist flexion, and
M3 Active movement against gravity
finger abduction and adduction.
M4 Active movement against gravity and resistance
M5 Normal power.
Case 100 Musculoskeletal and neurology 177

Reflexes Table 28
3 from root Long thoracic nerve (C5,6,7)
Hyporeflexia or absent reflexes in involved Nerve to subclavius (C5)
roots, e.g. biceps and supinator reflexes
affected in C5/6 lesions. Dorsal scapular nerve (C5,6)
1 from Suprascapular nerve
upper trunk
TOP TIP 3
0 from
A way to remember limb reflexes (12345678!) division
S1/2 Ankle reflex 3 from Lateral pectoral nerve
L3/4 Knee reflex lateral cord Musculocutaneous nerve
C5/6 Biceps and supinator reflexes Lateral contribution to median nerve
C7/8 Triceps reflex. 5 from Medial cutaneous nerve of arm
medial cord Medial cutaneous nerve of forearm
Medial pectoral nerve
Sensation Medial contribution to median nerve
Ulnar nerve
Test light touch (cotton wool) and pain 5 from Upper subscapular nerve
(orange stick) in all dermatomes compared posterior Lower subscapular nerve
with the unaffected side (Fig. 105) cord Thoracodorsal nerve
In C5/6 lesions, the outer aspect of the
arm (over the insertion of deltoid) is Axillary nerve
affected Radial nerve
In C8/T1 lesions, the ulnar side of the
hand and forearm are affected.
Lateral cord anterior divisions of upper
Completion and middle trunks
Medial cord anterior division of lower
Say that you would like to: trunk
Posterior cord all three posterior
Determine whether the level of injury is more
divisions
proximal than C5 test trapezius (C3/4 root
value) by asking the patient to shrug his Nerve branches: remember the 3:1:0:3:5:5
shoulder against resistance rule (Table 28).
Examine the neck to exclude compression (b) How would you differentiate between
from thoracic outlet syndrome, especially if
pre-ganglionic and post-ganglionic injuries?
there is no history of trauma (see Case 126).
Pre-ganglionic injuries have a worse prognosis,
as the nerve roots have been avulsed directly
QUESTIONS from the spinal cord and, as there is no
proximal peripheral nerve tissue, surgical repair
(a) What are the branches of the brachial is difficult. The features of a pre-ganglionic
plexus? (Fig. 105) injury are:
The brachial plexus is formed by the anterior Bruising in the posterior triangle of the neck
rami of C5, 6, 7, 8 and T1 nerve roots Pain in an insensate hand
(five roots) Loss of sensation above the clavicle
Three trunks: upper, middle and lower lying Ipsilateral Horners syndrome (avulsion of
in the posterior triangle of the neck C8/T1 nerve roots)
Six divisions: anterior and posterior division Loss of muscle function of branches direct
of each trunk at the level of the clavicle from the roots of the brachial plexus, e.g.
Three cords: lateral, medial and posterior long thoracic nerve (C5/6/7 winging of the
formed as the 6 divisions enter the axilla scapula) and phrenic nerve (C3/4/5
(name of the cord denotes relationship to the elevated hemidiaphragm seen on chest
axillary artery) X-ray).
178 Musculoskeletal and neurology Case 101

continue to treat expectantly, if not, nerve


ADVANCED QUESTIONS exploration
(a) How would you assess the prognosis of a Stage III nerve exploration and/or repair
brachial plexus injury? Stage IV from time of exploration and
repair to 12 years later (involves active
Three types of injury pattern have been hand therapy)
described which provide a guide to prognosis:
Stage V (at 2 years) final assessment of
Root avulsion: direct avulsion of roots from recovery made and adjunct procedures
spinal cord which is not amenable to surgical considered, such as tendon transfers,
repair arthrodesis and amputation.
Rupture: of the plexus outside the vertebral
column and although the injury is unlikely to
heal spontaneously, surgery may be of some Wilhelm Erb (18401921). German neurologist and
benefit Professor of Medicine at Leipzig and Heidelberg.
Nerve damage without rupture: improvement Guillame Benjamine Amand Duchenne (18061875).
likely to occur spontaneously. French neurologist.
Joseph Jules Dejerine (18491917). French
(b) What do you know of the staged
neurologist and psychiatrist, and Professor of
management of brachial plexus injuries? Neurology at the Salpetriere, Paris.
Staged management depends upon: Johann Friedrich Horner (18311886). Professor of
Mechanism of injury, e.g. gunshot injuries Ophthalmology, Zurich, Switzerland.
debride and treat as closed injury (see Madame Dejerine-Klumpke ne Auguste Klumpke
below) (18591927). American-born neurologist married to
Whether the injury open or closed if open, Joseph Dejerine.
primary epineural repair and if closed, staged
management
Staged management is as follows:
FURTHER READING
Stage I (3 months) treat expectantly and
assess clinically and electrophysiologically Songcharoen P: Management of brachial plexus
(electromyographic and nerve conduction injury in adults, Scand J Surg 97(4):317323,
studies) 2008.
Stage II (36 months) if clinical or www.patient.co.uk/showdoc/40001318/
electrophysiological improvement, information for patients.

CASE 101 IVORY OSTEOMA *

INSTRUCTION Inspect
Look at this gentlemans forehead and tell me
Sessile, flat mounds.
the diagnosis.

Palpate
APPROACH
For the spot diagnosis type question, simply Smooth surface
introduce yourself to the patient as the examiner Bony hard in consistency
wants a quick answer to his question. Ivory Can move the superficial layers of the scalp
osteomas are commonly found on the vault of across the top of the lump.
the skull and frequently the forehead.

VITAL POINTS
Examine as for any lump (see Case 1).
Case 102 Musculoskeletal and neurology 179

TOP TIP QUESTIONS


Layers of the scalp (a) What is an ivory osteoma?
Remember the mnemonic that spells the word Ivory osteomas are the most common benign
SCALP: tumours of the skull vault. They arise from
Skin cortical bone and radiologically may resemble
sclerotic reaction produced by a meningioma
Connective tissue
(a CT scan may be needed to differentiate
Aponeurotic muscle the two).
Loose areolar tissue
Periosteum. (b) How should they be managed?
They should not be resected unless they are
symptomatic.

CASE 102 CHONDROMA *

INSTRUCTION (b) What is a chondroma?


Examine this lump and tell me your diagnosis. Benign cartilaginous tumours within or on
the surface of long bones
X-rays show well-defined lucent area in the
APPROACH medulla and characteristic specks of
calcification.
Examine the relevant area note that
chondromas are usually found in the tubular
bones of the hand or feet (e.g. phalanges). ADVANCED QUESTIONS
(a) What is the surgical treatment?
VITAL POINTS
Surgical excision or curettage with bone
Examine as for lumps (see Case 1). Note that grafting.
an enchondroma grows from the centre of the
bone and an ecchondroma (periosteal (b) What is Olliers disease?
chondroma) grows over the surface of the bone.
Multiple chondromas (also known as
dyschondroplasia) is a sporadic disorder with
Inspect equal sex distribution where pathological
fractures and growth arrest occur
Visible solitary or multiple swellings The rare combination of enchondromas with
May be fusiform (enchondroma) or sessile cutaneous haemangiomas is known as
lumps (ecchondroma). Maffuccis syndrome
Patients with Olliers disease have a 30%
chance of developing malignant
Palpate transformation to chondrosarcomas, and
also have an increased risk of visceral
Smooth surface malignancies.
Hard consistency.

QUESTIONS
(a) What is the differential diagnosis? A. Marfucci (18471903). Professor of Pathology, Pisa,
Italy.
Benign cysts (no calcification)
L. Ollier (18301900). French surgeon who was senior
Chondrosarcoma (older patients, especially
surgeon at the Hotel Dieu in Lyon in 1860.
in large bones).
180 Musculoskeletal and neurology Case 103

CASE 103 CHARCOTS JOINTS *

INSTRUCTION QUESTIONS
Examine this gentlemans ankle.
(a) What is a Charcots joint?
It is a progressive destructive joint arthropathy
APPROACH secondary to a disturbance of sensory
Compare both ankles, ideally by exposing the innervation to a joint. The end result is a
entire lower limbs, keeping the patients painless deformed joint resulting from repetitive
underwear on. minor trauma.

(b) What are the causes of a Charcots joint?


VITAL POINTS Peripheral:
Look Diabetic neuropathy
Peripheral nerve injuries
Look for swelling of the affected joint Leprosy
Colour of the overlying skin is usually normal. Central:
Syringomyelia
Feel Cauda equina syndrome, e.g. secondary to
myelomeningocoele
Joint is not tender or warm (although it may Tabes dorsalis.
have been in the early stages)
Crepitus may be felt
The normal contours of the joint are lost and Jean-Martin Charcot (18251893). French neurologist
the joint may be hypertrophic or atrophic and son of a Paris coachbuilder. Professor of
The joint may be subluxed or dislocated. Pathological Anatomy, Salpetriere Hospital in 1872.
Subsequently the Professor of Nervous Diseases in
Move 1882. His pupils included Freud and Babinski.

Instability of the joint may be demonstrated


by abnormal movements and hypermobility.
FURTHER READING
Pinzur MS: Current concepts review: Charcot
Finish your examination here arthropathy of the foot and ankle. Foot Ankle Int
28(8):952959, 2007.
www.diabetes.usyd.edu.au/foot/Chartec1.html
Completion the radiological features of Charcots
arthropathy.
Say that you would like to:
Perform a neurological examination of the
affected limb, particularly looking for loss of
sensation and joint position sense
Dip the urine for sugar (to exclude diabetes
mellitus).
Case 104 Musculoskeletal and neurology 181

CASE 104 WINGING OF THE SCAPULA *

INSTRUCTION QUESTIONS
Look at this gentlemans back and tell me your
(a) What is the most common cause of
diagnosis.
winging of the scapula?
Weakness of the serratus anterior muscle
APPROACH secondary to:
Expose fully to waist and ask the patient to turn Damage to the long thoracic nerve (anterior
around and face away from you. rami of C5, 6 and 7) which supplies serratus
anterior, e.g. secondary to axillary surgery
Upper brachial plexus injury
VITAL POINTS
Viral infections of C5, 6 and 7 nerve roots
This case is essentially a spot diagnosis, and Certain types of muscular dystrophy, e.g.
if asked to look at the back do not touch fascioscapulohumeral dystrophy (Dejerine
the back. Landouzy syndrome).

Look ADVANCED QUESTIONS


Asymmetry of the shoulders (this may not be (a) What other causes are you aware of?
obvious until the patient pushes against a
Trapezius palsy secondary to injury to the
wall see below)
spinal accessory nerve (at risk of iatrogenic
Any obvious scapular winging
injury in the posterior triangle of the neck)
Bilateral involvement.
Post-glenohumeral fusion
Abduction contracture of the deltoid.
Move
(b) How would you treat this condition?
Ask the patient to abduct his arm above the Non-surgical if disability minimal
horizontal (there may be difficulty in Surgical tendon transfers or
performing this action) scapulothoracic fusion.
Ask him to stand up facing a wall and to
push firmly with both hands against the wall
(this makes winging more prominent and it is Sir Charles Bell (17741842). Scottish surgeon,
clinically easy to determine). physiologist and painter
J. J. Dejerine (18491917). French neurologist
Finish your examination here L. T. J. Landouzy (18451917). French physician.

Completion
FURTHER READING
Say that you would like to: Duralde XA: Evaluation and treatment of the
Examine the upper limb musculature to winged scapula, J South Orthop Assoc
exclude muscular dystrophy. 4(1):3852, 1995.
182 Musculoskeletal and neurology Case 105

CASE 105 EXTERNAL FIXATORS *

Circular: Ilizarov frame (frame circles around


INSTRUCTION
the limb)
You may be shown an external fixator on a Hybrid: supports one segment using rings
patient and asked to comment. The following and other segments using uni-axial
notes are a guide to external fixators. constructs
Pinless: metal clamps that tighten onto the
bone and therefore avoid interfering with the
PRINCIPLES
medullary canal when later definitive internal
Focus: this is the area of bone being fixation is being considered.
supported, i.e. the fracture
Segment: this is the area of bone into which
the external fixator is inserted in order to INDICATIONS
control the focus. Multiple trauma there is evidence that adult
The external fixator device is made up of pins and respiratory distress syndrome may
frame (the former connect the frame to the bone). complicate intramedullary nailing when there
has also been a concurrent chest injury,
therefore external fixators are an alternative
CLASSIFICATION OF PINS Peri-articular fractures
External fixators are inserted into bone by: Intra-articular fractures
Open fractures
(a) Screw-threaded half pins Pelvic fractures to reduce life-threatening
Applied to one side of the bone haemorrhage
Advantage is the low risk of neurovascular Bone transport (Ilizarov technique) to
damage encourage fracture union and replace
But there is a risk of loosening. lost bone.

(b) Transfixing pins


Passes through the bone from one aspect of
COMPLICATIONS
the limb to the other Pin-track infections
Provides better control of the fracture Chronic pain
Disadvantages include possible damage to Pin loosening and breakage
neurovascular structures as the pins pass Neurovascular damage
through the bone, and risk of loosening Joint stiffness.
Therefore transfixing pins are only used in
very simple constructs such as joint fusions
and temporary stabilization of severely
traumatized limbs.
Professor Gabriel Abramovitch Ilizarov graduated from
(c) Tensioned fine wires medical school in the Soviet Union in 1943, near the
Also pass all the way through the limb end of the Second World War. After graduation, he
This support is made of multiple small, thin was assigned to practice in Kurgan, a small town in
wires placed into the bone at different western Siberia. He was the only physician within
orientations which are then tightened to hundreds of miles and had little in the way of supplies
support the bone and medicine. Faced with numerous cases of bone
deformities and trauma victims due to the war,
Are less traumatic to the blood supply of the
Professor Ilizarov used the equipment at hand to treat
limb and tend not to loosen but are more
his patients. Through trial and error, with handmade
expensive and complicated to apply.
equipment, this self-taught orthopaedic surgeon
created the Ilizarov technique of distraction
CLASSIFICATION OF FRAMES osteogenesis. This refers to the formation of new bone
between two bone surfaces that are pulled apart in a
Uni-axial: half pin (frame is adjacent to one
controlled and gradual manner.
side of the limb only)
Case 107 Musculoskeletal and neurology 183

CASE 106 INTRAMEDULLARY NAILS *

INSTRUCTION forming a fixed construct, whereas in the


latter, some element of movement at the
You may be shown an intramedullary nail, fracture site is allowed by locking one end of
particularly one that has been removed from a the nail or placing one screw through an
patient (the nail is likely to be lying next to a oblong hole
patient or may be shown as a prop in between Dynamization this is the process of
cases). You should be able to recognize the fact removing one or more screws in order to
that it is an intramedullary nail and know some allow collapse, which increases the loading
basic principles. of the fracture site and hastens union.

MATERIAL CONTROVERSIES OF REAMING


Most nails are made from stainless steel, Enlarges medullary canal and therefore
titanium, or titanium alloys (e.g. titanium permits insertion of the stronger, wider nail
aluminiumvanadium) this may not be necessary with modern
Titanium and its alloys are stronger, lighter, materials
more resistant to infection and Weakness due to the loss of endosteal bone
osseointegrated better than stainless steel. Disturbance of medullary blood supply may
cause cortical necrosis
LOCKING Time-consuming
May cause embolism of debris into
Screws inserted proximally and distally:
medullary veins this is implicated in the
Provide longitudinal and rotational stability aetiology of adult respiratory distress
May be static or dynamic in the former, the syndrome in severe trauma.
screws are inserted through round holes thus

CASE 107 PAGETS DISEASE OF BONE *

INSTRUCTION Finish your examination here


Have a look at this gentlemans legs.

Completion
APPROACH
Expose the patients legs but preserve his Say that you would like to:
dignity by keeping his underwear on. Examine for the typical appearance of the
skull enlarged due to increased skull
diameter (note that more than 55cm is
VITAL POINTS abnormal)
Ask the patient if he has any hearing
Look
difficulties (either a conduction defect due to
involvement of the ossicles or neural due to
Anterior bowing of the tibia (also known as
compression of the 8th nerve)
sabre tibia)
Ask about osteoarthritic involvement of joints
There may be lateral bowing of the femur.
(e.g. limitation of hip abduction and fixed
flexion deformity of the knees)
Feel Inspect the neck for a raised jugular venous
pressure (cardiac failure secondary to a
The affected bone for warmth. hyperdynamic circulation)
184 Musculoskeletal and neurology Case 107

Inspect the spine for kyphosis (and Vertebrae picture frame appearance due
auscultate over the vertebral bodies for to sclerotic margins
bruits secondary to hyperdynamic Pelvis brim-sign due to thickening of the
circulation). iliopectineal line, enlargement of the ischial
and pubic bones
QUESTIONS Long bones increased trabeculation (note
that bone scans are more sensitive for
(a) What is Pagets disease? assessing the extent of disease).

Pagets disease is a remodelling disease of (b) What do you know about the aetiology of
isolated skeletal areas there is increased bone Pagets disease?
turnover, increased numbers of osteoblasts and
Several theories exist including:
osteoclasts, and bone enlargement, deformity
and weakness. Genetic: exact mechanisms unclear; may be
linked to gene coding for the protein
(b) What are the biochemical features of sequestosome 1
Pagets disease? Infectious: may be due to slow viral infection,
Normal serum calcium and phosphate e.g. paramyxovirus.
(occasionally hypercalcaemia)
(c) What changes can be seen on fundoscopy
Markedly raised serum alkaline phosphatase
in patients with Pagets disease?
(due to increased osteoblastic activity)
Increased urinary hydroxyproline secretion Optic atrophy
(due to increased bone resorption). Angioid streaks known as Terry syndrome.

(c) What are the complications of Pagets


disease?
Bone: increased risk of fractures, Sir James Paget (18141899). English surgeon, St
sarcomatous change (affects 1% of patients Bartholomews Hospital, London and Professor of
who have the disease for more than 10 years) Surgery, Royal College of Surgeons. Described Pagets
Neurological: cranial nerve palsies, cord disease of bone as osteitis deformans. Also described:
compression due to basilar invagination, Pagets disease of the nipple intra-epidermal,
nerve root lesions due to vertebral damage, intraductal cancer involving the areola and/or the
headache, fits nipple characterized by large cells with a clear
Cardiac: high-output cardiac failure. cytoplasm
Pagets disease of the skin skin cancer involving
(d) How would you treat Pagets disease? the apocrine glands characterized by large cells
Symptomatic: simple analgesics, progressing with a clear cytoplasm
to bisphosphonates (e.g. alendronate), which PagetSchroetter syndrome idiopathic axillary
inhibit osteoclast-mediated bone resorption. venous thrombosis
Calcitonins, which act by reducing
osteoclastic activity, are less effective than Pagets sign (on testing whether a lump is
bisphosphonates fluctuant) rest two fingers of one hand on
opposite sides of the lump and press the middle of
Surgical, e.g. hip arthroplasty for
the lump with the index finger of your other hand
symptomatic osteoarthritis.
if the fingers are moved apart, the lump is
fluctuant and you have demonstrated Pagets sign
ADVANCED QUESTIONS (see Case 1)
T. L. Terry (18991946). US ophthalmologist.
(a) What are the radiological features of
Pagets disease?
The hallmarks on plain X-rays are localized bony
enlargement and patchy cortical thickening with FURTHER READING
sclerosis, osteolysis and deformity. More Ralston SH, Langston AL, Reid IR: Pathogenesis
specific features include: and management of Pagets disease of bone.
Skull honeycomb and cottonwool Lancet 372(9633):155163, 2008.
appearance with underlying osteoporosis www.arc.org.uk/arthinfo/patpubs/6031/6031.asp
circumscripta information for patients.
Case 108 Musculoskeletal and neurology 185

CASE 108 ACHONDROPLASIA *

INSTRUCTION Completion
Have a look at this gentleman and describe
Say that you would like to:
what you see.
Take a family history (see below)
Assess the effect of the symptoms on the
APPROACH patients quality of life.
Introduce yourself, step back and talk
systemically through the physical appearance of QUESTIONS
the patient. Remember not to lay a hand on the
patient unless the examiner prompts you to. (a) What is achondroplasia?
Commonest form of disproportionate short
VITAL POINTS stature with proximal shortening of long
bones
Height Equally common in males and females
Prevalence is between 0.5 and 1.5 in 10000
Reduced (dwarfism, but avoid use of this live births.
word in the exam)
Normal trunk size (b) What treatment options are available for
Shortened extremities. the problems associated with achondroplasia?
Non-surgical:
Hands Subcutaneous human growth hormone to
increase height
Short and broad Surgical:
Wedge-shaped gap between middle and ring Limb lengthening using distraction devices
fingers (trident hands). Correct body proportion and axial
deviation
Skull Improve appearance, body-image and
self-esteem
Macrocephaly Region-specific surgery, e.g.
Frontal bossing (prominent forehead) Spinal surgery correction of
Saddle nose (depression of the root of thoracolumbar kyphosis, decompression
the nose) for spinal stenosis
Maxillary hypoplasia Correction of genu varum (by fusion of
Mandibular prognathism (protrusion of the fibular epiphysis or osteotomy).
the jaw).
ADVANCED QUESTIONS
Spine
(a) Do you know of any conditions resembling
Thoracolumbar kyphosis achondroplasia?
Excessive lumbar lordosis. Hypochondroplasia similar to mild
achondroplasia
Knees Pseudochondroplasia similar to
achondroplasia but normal head and face.
Genu varum.
(b) What do you know about the genetics of
achondroplasia?
Finish your examination here Autosomal dominant with complete
penetrance
186 Musculoskeletal and neurology Case 108

Around 80% of cases represent new


FURTHER READING
mutations
Mutations in transmembrane domain of Baujat G, Legeai-Mallet L, Finidori G, et al:
fibroblast growth factor receptor 3 (FGFR-3) Achondroplasia, Best Pract Res Clin Rheumatol
mapped to chromosome 4p16.3. 22(1):318, 2008.
www.bbc.co.uk/health/conditions/
achondroplasia1.shtml information for
patients.
4
SECTION

CIRCULATION AND
LYMPHATIC SYSTEMS
109 Varicose veins *** 188
110 Venous ulcer *** 193
111 Peripheral arterial system examination *** 194
112 Vascular effects of the diabetic foot *** 199
113 Amputations *** 200
114 Peripheral arterial system history *** 202
115 Abdominal aortic aneurysm *** 203
116 Carotid artery disease *** 205
117 Popliteal aneurysm ** 207
118 Ischaemic ulcer ** 208
119 Post-phlebitic limb ** 210
120 Gangrene ** 211
121 Raynauds phenomenon ** 212
122 Neuropathic ulcer ** 214
123 Lymphoedema ** 215
124 Hyperhidrosis ** 217
125 False aneurysm ** 218
126 Thoracic outlet obstruction * 219
127 Thromboangiitis obliterans (Buergers disease) * 221
128 Superior vena cava obstruction * 222
129 Carotid artery aneurysm and dilated common
carotid artery * 223
130 Lymphangioma * 224
131 Arteriovenous fistula * 224
132 Coarctation of the aorta * 226
133 Atrial fibrillation * 227
188 Circulation and lymphatic systems Case 109

CASE 109 VARICOSE VEINS ***

Ask the patient to turn around and


INSTRUCTION
inspect for varicosities in the distribution
Examine this ladys varicose veins. of the short saphenous system (SSV)
(Fig. 106)
Try to decide whether the varicosities
APPROACH are long or short saphenous in origin,
Expose the patient up to the groin, maintaining commenting that the distinction may be
her dignity by keeping her external genitalia difficult below the knee
covered. Examine the groins for the presence of
a saphena varix. This will be located at
the saphenofemoral junction (SFJ) (see
VITAL POINTS below)
Skin for changes and scars
Look for scars indicating previous
TOP TIP 1
surgery, especially hidden in the groin
When performing vascular examinations is it useful to creases;
remember that there are 3 objectives to be completed for Determine whether there are signs of
both inspection and palpation. This also applies to chronic venous insufficiency (see below);
examination of arterial supply (see Case 111). When If there is evidence of ulceration, describe
assessing the venous system of the lower limbs these are its characteristics fully (see Case 110)
as follows:
Swelling of the ankle
Inspection for the 3 Ss: Look for asymmetry between the lower
limbs and establish the height and
Site and size of varicosities, including the presence of
severity of the swelling.
a saphena varix
Skin for changes and scars
Swelling of the ankle TOP TIP 2
Use the acronym LEGS to help you remember the
Palpation for the 3 Ss:
signs of chronic venous insufficiency:
State of the skin/subcutaneous tissues
Lipodermatosclerosis
Sites of fascia defects
Eczema
Site of incompetence (including the Trendelenburg and
Gaps in the skin (i.e. ulceration) active and healed
tourniquet tests).
(the latter causing a white patch called atrophie
blanche)
Swelling (pedal oedema)

Inspect Inspect specifically around the medial malleolus (the


gaiter area) for evidence of these changes.
Ask the patient to stand (veins are collapsed
when the patient is supine) and while kneeling
in front of the patient, look for the 3 Ss:
Site and size of varicosities, including the Palpate
presence of a saphena varix:
Establish that any visible veins are State of the skin/subcutaneous tissues
varicosities (dilated and tortuous) as Palpate the skin for the presence of
opposed to physiological (dilated only, pitting oedema;
as in athletes)
Feel along the course of the long and
Determine whether there are varicosities short saphenous veins, determining
in the distribution of the long saphenous whether there is induration of the
vein (LSV) (Fig. 106) subcutaneous tissues.
Case 109 Circulation and lymphatic systems 189

Superficial Superficial
epigastric external
vein pudendal
vein
Superficial circumflex
iliac vein
Saphenofemoral
opening
Saphenofemoral
junction
Femoral vein Popliteal vein
Mid-thigh (Hunterian) Saphenopopliteal
perforating vein junction
Long saphenous vein
Short saphenous
Posterior arch vein vein
Mid-calf
perforator
Dorsal venous arch
Passes posterior
to lateral malleolus
Dorsal venous
Passes anterior to Direct calf arch
medial malleolus perforating
veins
Figure 106 Anatomy of the superficial venous system of the leg.

Sites of fascia defects incompetent. Should the veins fill, then


Feel along the medial aspect of the leg the SFJ may or may not be competent,
for tenderness in the fascial defects at the but there are certainly distal incompetent
site of perforator veins; perforators
Site of incompetence The tourniquet test employs a similar
Ask the patient to turn to face you again principle but uses a tourniquet to control
and palpate the saphenofemoral junction the junction rather than fingers. Again
(SFJ), which is located 3.5cm empty the leg in the supine position and
(approximately 2 finger breadths) below apply the tourniquet (in the interests of
and lateral to the pubic tubercle (Fig. 107). infection control, a single-use examination
glove is frequently employed for this
Feel for the smooth swelling and palpable
purpose) (Fig. 110). Then ask the patient
thrill of a saphena varix. If present, the
to stand and inspect for filling and
cough test may be positive (Cruveilhiers
collapse above and below the position of
sign)
the tourniquet (see Top Tip 3). The
The Trendelenburg test is performed with
tourniquet test has the advantage that
the patient first lying down. Elevate the
perforators can be examined below the
leg to gently empty the veins (Fig. 108).
groin. To do this, this tourniquet should
Palpate the SFJ and ask the patient to
be positioned in the mid-thigh (Fig. 111),
stand while maintaining direct pressure
above (Fig. 112) and below knee regions
over the SFJ with one finger (Fig. 109). If
(Fig. 113)
the veins do not refill then the SFJ is
190 Circulation and lymphatic systems Case 109

Figure 107 Anatomy of the saphenofemoral junction.

TOP TIP 3
The tourniquet test is designed to reveal the presence
and site of incompetent veins, especially at the sites of
connection between the superficial and deep venous
systems (Fig. 106). The test is interpreted by examining
the condition of the veins below the level of the tourniquet
as follows:
Collapsed veins below the tourniquet indicate that the
incompetent vein is at/above the level of the
tourniquet
Rapid filling of veins below the tourniquet indicate that
the incompetent vein is below the level of the
tourniquet.

Figure 108 Emptying the veins in preparation for the Finish your examination here
Trendelenburg test.

Completion

Use a hand-held Doppler (if provided) to Say that you would like to:
identify SFJ/popliteal fossa reflux by Perform a tap test (Chevriers tap sign)
squeezing the muscle of the thigh or calf,
Auscultate the vein for bruits (indicating the
listening proximally as blood flows up the
presence of arteriovenous fistulae)
leg (normal) and then for a second
swoosh in incompetent veins as blood Examine the abdomen for masses (including
refluxes down the leg when the probe is a digital rectal examination) to ascertain
positioned over the junction. whether the varicose veins are primary or
secondary.
Case 109 Circulation and lymphatic systems 191

Figure 110 The tourniquet test in the upper thigh.

Figure 111 The tourniquet test in the mid-thigh.

Figure 109 The Trendelenburg test with direct pressure


over the saphenofemoral junction with one finger.

QUESTIONS
(a) What are the indications for preoperative
Duplex ultrasound scanning?
Some surgeons would advocate that all patients
should undergo Duplex scanning of the leg
veins before any surgery is undertaken. Others
would consider indications to be:

Figure 112 The above knee tourniquet test.


192 Circulation and lymphatic systems Case 109

Ligation of incompetent perforating vessels


(with site of incompetence preoperatively
marked with Duplex)
Subcutaneous endoscopic perforator surgery
(SEPS).

(c) What would you tell this lady about the


proposed surgery?
Procedure usually performed as day case
Need to wear tight-fitting stockings for 6
weeks postoperatively
No driving for 1 week
Does not alter the skin changes, including
skin flares
May not improve symptoms such as aching
Risk of recurrent veins (20% at 5 years).

ADVANCED QUESTIONS
Figure 113 The below knee tourniquet test. (a) What do you know about the
pathophysiology of varicose veins?
Fibrous tissue invades the tunica intima and
media of the vein and breaks up the smooth
muscle, preventing the maintenance of
adequate vascular tone. These changes are
Previous history of deep vein thrombosis patchy and may not affect adjacent segments
(DVT) of vein.
Any signs of chronic venous insufficiency
(raises possibility of deep venous disease (b) What syndromes are associated with
with the varicose veins acting as a collateral varicose veins?
return of blood) KlippelTrenaunayWeber syndrome consists
Recurrent varicose veins of a triad of varicose veins, port wine stains
Difficulty in deciding whether the SSV or LSV and bony and soft-tissue hypertrophy of the
is incompetent. limbs. This may present with varicose veins
in an unusual position, classically over the
(b) How would you treat varicose veins? lateral aspect of the thigh. Peripheral
Conservative: oedema is often significant, as the deep
venous system may be abnormal
Graduated elastic compression stockings
ParkesWeber syndrome is characterized by
(grade II compression)
multiple arteriovenous fistulae, with limb
Encourage weight loss and regular exercise hypertrophy. The AV fistulae may be so
Sclerotherapy: severe as to cause high-output cardiac
Injection sclerotherapy with 1% sodium failure.
tetradecyl sulphate (high recurrence rate)
indicated for:
Postoperative recurrence of veins M. Klippel (18581942). French psychiatrist and
Below knee varicosities if the LSV and neurologist.
SSV are not involved P. Trenaunay. French neurologist.
Surgical:
Professor F. Trendelenburg (18441924). German
Ligation of the incompetent SFJ or SPJ with surgeon, Leipzig.
stripping of the involved vein and stab
F. P. Weber (18631962). British physician.
avulsion of varicosities
Case 110 Circulation and lymphatic systems 193

Pleass HC, Holdsworth JD: Audit of introduction


FURTHER READING
of hand-held Doppler and duplex ultrasound in
Critchley G, Handa A, Maw A, et al: the management of varicose veins, Ann R Coll
Complications of varicose vein surgery, Ann R Surg Engl 78(6):494496, 1996.
Coll Surg Engl 79(2):105110, 1997.
Houghton AD, Panayiotopoulos Y, Taylor PR:
Practical management of primary varicose veins,
Br J Clin Pract 50(2):103105, 1996.

CASE 110 VENOUS ULCER ***

INSTRUCTION Surrounding skin


Examine this ladys ankle and tell me the
Look for signs of primary varicose veins
diagnosis.
The signs of chronic venous disease are
usually present: induration, pigmentation,
and brown discoloration of
APPROACH lipodermatosclerosis
The patient should be exposed from the groin Oedema, spider veins and telangiectasia
to the toes, preserving her dignity. may be present.

Feel
VITAL POINTS
Look Feel the adjacent skin for temperature it
may be warmer than the rest of the leg
Observe the following characteristic features of (compared with the ischaemic ulcer where
a venous ulcer (see Case 1). the surrounding skin will be cold).

Site Finish your examination here

The venous ulcer is most commonly found Completion


over the lower third of the medial aspect of
the leg, immediately above the medial Say that you would like to:
malleolus (gaiter area).
Examine the limb properly for varicose veins
(see Case 109)
Shape Perform an ankle brachial pressure index
measurement (see Case 111) as this must be
The size varies enormously, and they can be >0.8 before compression bandaging can be
extremely large. used.

Edge and base QUESTIONS


The edge is sloping and pale purple/brown in (a) What are the causes of venous ulcers?
colour
Any cause of deep venous insufficiency can
The base is usually covered with pink-
lead to ulceration:
coloured granulation tissue but there may
also be some white fibrous tissue Valvular disease:
They are usually rather shallow and often Varicose veins
have a seropurulent discharge. Deep vein reflux (such as post-DVT)
194 Circulation and lymphatic systems Case 111

Communicating vein reflux (post- Surgical:


thrombotic or non-thrombotic) If the ulcer fails to heal, careful consideration
controversial (unusual) should be given to excluding other causes
Outflow tract obstruction: (such as a malignant Marjolin ulcer) and the
Often post-DVT area may need to be biopsied (2% of chronic
Muscle pump failure: leg ulcers are malignant)
Primary stroke, neuromuscular disease Otherwise, a split skin graft should be
Secondary due to musculoskeletal considered with excision of the dead skin
pathology/injury of the ankle. and the graft attached to healthy granulation
tissue
(b) How are venous ulcers treated? If ulceration is due to primary varicose veins,
Non-surgical: surgery to the superficial veins is required.
High success 5070% will heal at 3
months, 8090% at 12 months
The patient should be warned to avoid
Rene Marjolin (18121895). Surgeon in Paris who
trauma to the affected area
described the formation of a carcinoma in a chronic,
Four-layer compression bandaging non-healing ulcer.
comprising:
Non-adherent dressing over ulcer plus
wool bandage
Crepe bandage FURTHER READING
Blue-line bandage
Fletcher A, Cullum N: A systematic review of
Adhesive bandage to prevent the other
compression treatment for venous leg ulcers.
layers from slipping
BMJ 315:376380, 1997.
Encourage rest and elevation of leg
London NJ, Donnelly R: ABC of arterial and
Once healed, grade II compression stockings
venous disease. Ulcerated lower limb. BMJ
should be fitted and continued for life
320:15891591, 2000.

CASE 111 PERIPHERAL ARTERIAL SYSTEM


EXAMINATION ***

INSTRUCTION TOP TIP 1


Examine this gentlemans legs (from the point As with the assessment of the venous system (see
of view of the peripheral arterial system). case 109), examination of arterial supply requires that
three objectives be completed for both inspection and
palpation. These are as follows:
APPROACH
Inspection for:
Expose the patients legs from the groin to the
Colour changes
toes (Fig. 114), preserving his dignity by keeping
his underwear on. Trophic changes
Vascular angle

VITAL POINTS Palpation for:


Temperature
Capillary refill
Peripheral pulses.
Case 111 Circulation and lymphatic systems 195

Aorta

Common iliac
artery Internal iliac
artery
External iliac
artery Inguinal
ligament
Profunda
femoris artery
Superficial
femoral artery

Popliteal artery

Anterior tibial
artery

Peroneal artery
Posterior tibial artery
Dorsalis pedis artery

Figure 114 Anatomy of the arteries in the leg.

circumscribed, may be very tender, and


Inspect the surrounding skin is cold
They may vary considerably in size but
Most of the pathology will be around the feet are usually smaller than venous ulcers
and toes. Begin by looking carefully at the feet. There is no granulation tissue but may be
Observe the following features: a thin layer of slough at the base,
Colour changes: otherwise the base is flat and pale
The skin of the lower limb may be red They may be very deep and penetrate
(vasodilatation of the microcirculation due surrounding tissue like bone
to tissue ischaemia), white (advanced The commonest differential is with a
ischaemia or purple/blue (excess neuropathic ulcer and if there is any
deoxygenated blood in the tissues) doubt, limb neurology should be examined
Trophic changes: Be sure to look between each of the toes
Loss of hair and small non-healing sores on both feet
may be evident on the lower limbs/feet Ask the patient for permission, then lift
Gangrene (especially between and at the the foot up to observe the heel for ulcers
tips of the toes) (neuropathic ulcers are commonest here)
Loss of digits, due to previous gangrene/ and the sole of the foot for ulceration of
amputation the metatarsal heads
Arterial (ischaemic) ulcers are found Vascular (Buergers) angle:
typically in the least well-perfused areas Lift the leg until it becomes white as the
and over the pressure points, such as perfusion drops
lateral aspect of foot and malleoli The angle between the horizontal and the
The lesions are punched out (because leg when it becomes white is Buergers
there is no attempt at healing) and well angle (Fig. 115)
196 Circulation and lymphatic systems Case 111

Figure 116 Palpation to assessing the capillary refill


time.

Figure 115 Examining for the vascular (Buergers)


angle.

Venous guttering can also be observed


A normal leg can be raised to 90 and still
remain perfused; if the angle is less than
20, this indicates severe ischaemia
Assisting the patient to drop their leg over
the side of the bed causes the diseased
leg to become purple-red in colour due to
reactive hyperaemia this is the second
part of Buergers test. It represents Figure 117 Palpation to assess and compare limb
dysfunction of the microcirculation temperature.
(perhaps secondary to sympathetic
dysfunction and loss of vasoconstrictive
tone) due to chronic ischaemia.
examiners, but such knowledge will allow you to
surgically expose such vessels with confidence!
Palpate You should say that you wish to examine all
peripheral pulses. However, if you have been
Temperature asked to examine just the lower limbs, as in this
Feel for skin temperature, staying at the case, it is sufficient to examine the radial pulse
end of the bed next to the patients feet (to check its rate and rhythm), before
use the back of the hand, comparing concentrating on the lower limbs.
both sides simultaneously for any
difference (Fig. 116)
Capillary refill TOP TIP 2
Examine the toes for capillary refill use Vascular surgeons palpate the peripheral pulses using
the thumb to push hard over the pulp of the following technique:
the big toe on both sides (Fig. 117),
normally the toe blanches but then Use more than one finger to palpate the pulse if the
returns to the normal colour within 2s. artery is big enough
Any longer than this is abnormal Position your fingers in the direction of the artery to
Palpation of the peripheral pulses. optimize your chances of feeling it
It is imperative that you know the surface Palpate both sides simultaneously to detect subtle
anatomy of all the named peripheral arteries. differences.
Not only is this a favourite question of
Case 111 Circulation and lymphatic systems 197

Mid inguinal point Mid inguinal point

Figure 120 Palpation of the popliteal pulse.

Figure 118 Surface anatomy of the femoral pulses.


The pulse is most easily palpated by
compressing it against the posterior
aspect of the tibia
Ask the patient to bend the knee slightly,
and hold the knee between your hands.
Use the pulps of your four fingers of both
hands held alongside each other to feel
the two heads of the gastrocnemius as
they join (marking the lower borders of
the popliteal fossa). The pulse lies
between these two heads (Fig. 120)
It is not possible to compare both sides
Figure 119 Palpation of the femoral pulse with the simultaneously, so palpate each side in
fingers parallel to the direction of the artery. Auscultation turn before moving on to the foot pulses
of the abdomen. Foot pulses:
Go to the bottom end of the examination
couch to palpate the foot pulses
If you cannot palpate the pulse having Do not be tempted to palpate the dorsalis
employed all of these measures, then it is pedis (DPA) and posterior tibial (PTA)
(almost certainly!) absent. For the purposes of pulses simultaneously. Instead, examine
exams, peripheral pulses should be specifically each pulse in turn using 2 or 3 fingers,
and confidently reported to your examiner as: while simultaneously comparing it with
(1) present; (2) reduced; or (3) absent. Nothing the contralateral side
annoys examiners more than hearing I dont To examine the DPA ask the patient to
think that I can feel the pulse. bring the big toe towards the head (thus
demonstrating the tendon of extensor
Femorals:
hallucis longus). The artery lies
The femoral pulse lies at the mid inguinal immediately lateral to this tendon.
point, halfway between the anterior Examine both DPAs simultaneously. It is
superior iliac spine and the pubic easier to do this crouching down (Fig.
symphysis (Fig. 118). It should be 121)
palpated with the fingers parallel to the
The PTA lies one fingers breadth below
direction of the artery (Fig. 119)
and behind the medial malleolus. Position
Popliteals: your fingers in line with the artery as it
Next, move down to palpate the popliteal curves forwards towards the dorsum of
pulse, which is often quite difficult to the foot (Fig. 122). Examine both PTAs
palpate and one that is easily palpable simultaneously
should raise the suspicion of aneurysmal Remember that the foot pulses may be
dilatation absent in 2% of normal subjects!
198 Circulation and lymphatic systems Case 111

Figure 121 Palpation of both dorsalis pedis pulses simultaneously.

The cuff is then placed over the calf


When the dorsalis pedis pulse has been
located with the Doppler, the cuff is inflated
until the pressure is high enough to occlude
the artery and thus the Doppler sound
disappears
Slowly lower the cuff pressure until the
Doppler sound restarts; this is the ankle
pressure
The index is the ankle pressure divided by
the brachial pressure.
Figure 122 Palpation of the posterior tibial pulse in the
line of the artery. (b) What is the significance of the ankle
brachial pressure index?
Normal index is 1
Listen As the perfusion of the leg begins to
decrease in a patient with peripheral vascular
Check for a bruit over the femoral artery and disease, the ratio begins to fall
in the subsartorial canal. Patients with intermittent claudication have
an index of approximately 0.50.8
Finish your examination here Patients with rest pain have an index <0.5
An absolute pressure of less than 50mmHg
at the ankle is used in some definitions of
critical ischaemia.
Completion

Say that you would like to:


Examine the rest of the peripheral arterial
Leo Buerger (18791943). Austrian who lived in the
system
USA all his life. Surgeon and urologist who became
Examine the abdomen for aneurysmal
Professor of Urology in New York and subsequently
dilatation of the aorta (see Case 115)
Los Angeles. Also named Buergers disease
Measure the ankle brachial pressure indices (thromboangiitis obliterans) (see Case 127).
on each side.

QUESTIONS
(a) How would you demonstrate this
gentlemans ankle brachial pressure index?
The pressure cuff is inflated over the upper
arm and the systolic pressure measured at
the brachial artery using a Doppler probe
Case 112 Circulation and lymphatic systems 199

CASE 112 VASCULAR EFFECTS OF THE DIABETIC


FOOT ***

INSTRUCTION Examine the neurological system. Diabetic


neuropathy is much commoner in Type 1
This gentleman has diabetes. Examine his foot diabetics and can form a glove-and-stocking
and describe your findings. type sensory or mixed neuropathy. Isolated
peripheral nerve lesions and autonomic
neuropathy may also be present
APPROACH
Examine the abdomen and the rest of the
Expose the patients legs from the groin to the peripheral arterial system. In addition, say
toes, preserving his dignity. that you would like to perform fundoscopy
and urinalysis for proteinuria
(microalbuminuria is an early marker of renal
VITAL POINTS impairment).
Look
QUESTIONS
Inspect the foot as for any vascular case (see
Case 111). Diabetic peripheral vascular disease (a) Why are diabetics particularly prone to
is usually evident below the knee. Note foot pathology?
especially:
The diabetic foot syndrome encompasses a
Presence of bilateral disease number of pathologies, including: diabetic
Any previous surgical scars including neuropathy, peripheral occlusive arterial
excision of metatarsal heads or digits for disease, Charcot neuroarthropathy,
gangrene osteomyelitis, foot ulceration and the potentially
Charcots joints are more common in preventable end-point of amputation.
diabetics with neuropathy (see Case 103) Diabetic neuropathy and peripheral occlusive
Signs of damage to the foot if the patient arterial disease are the major aetiological factors
has a sensory neuropathy and cannot feel for the development of ulceration and may act
injuries such as hot bath-water or nails alone, together or in combination with other
digging into the feet factors such as microvascular disease,
Evidence of ulceration. biomechanical abnormalities, limited joint
mobility and increased susceptibility to infection.
Feel
(b) What is the aetiology of diabetic
The pulses may be preserved until very late
foot ulcers?
in the disease and may in fact feel quite Neuropathic (4560% of ulcers)
prominent due to calcification of arteries Ischaemic due to peripheral occlusive arterial
Take care when palpating the joints of the disease (10% of ulcers)
foot as they may be painful Mixed neuroischaemic (2545% of ulcers).
Test for sensation over the foot, and if any Opinion is divided as to whether diabetic
abnormality is detected, work proximally to neuropathy occurs due to (1) microvascular
find the level at which sensation returns to disease leading to nerve hypoxia or (2) direct
normal. effects of hyperglycaemia on neuronal
metabolism.
Finish your examination here Atherosclerotic occlusive arterial disease is
present (albeit in subclinical form in some) in
virtually all long-term diabetics. Vascular disease
being responsible for 70% of diabetic deaths.
Completion The distribution of peripheral occlusive arterial
disease is different in diabetics, predominantly
Say that you would like to: affecting below knee vessels.
200 Circulation and lymphatic systems Case 113

(c) It is known that the pulses are preserved in debridement. All diabetics should be seen
the diabetic why is this? regularly by the chiropodist with a view to
preventing complications.
Calcification of the walls of the vessels
preserves the pulses until late in the natural
(b) Are there any problems with diabetics
history of disease, and prevents the
sphygmomanometer from compressing the
undergoing angiography?
vessels. This tends to lead to an abnormally They may have a degree of renal impairment
(and reassuringly) high ankle brachial pressure which can be dramatically worsened following a
index measurement. A similar effect is seen in dose of intra-arterial contrast. Patients should
peripheral vascular disease caused by chronic be kept well hydrated with intravenous fluids
renal failure. peri-procedure. If they are on metformin this
has to be stopped prior to the procedure, as
lactic acidosis has been reported.
ADVANCED QUESTIONS
(a) What differences in management are FURTHER READING
there in diabetics? Caputo GM, Cavanagh PR, Ulbrecht JS, et al:
The abnormal ankle brachial pressure index, Assessment and management of foot disease in
plus the fact that patients often have occlusions patients with diabetes, N Engl J Med
at multiple levels, means that earlier recourse to 331(13):854860, 1994.
intra-arterial digital subtraction angiography is Frykberg RG, Armstrong DG, Giurini J, et al:
indicated. Any infections should be treated Diabetic foot disorders: a clinical practice
aggressively with bed rest and intravenous guideline. American College of Foot and Ankle
antibiotics, together with meticulous foot care. Surgeons, J Foot Ankle Surg 39(5 Suppl):S160,
Sepsis should be treated with surgical 2000.

CASE 113 AMPUTATIONS ***

INSTRUCTION Comment on the stump healthy stumps are


cylindrical and the skin wound has healed
Examine this gentlemans legs. completely
Amputations are traditionally formed 12cm
above the knee or 14cm below the knee
APPROACH
Below-knee amputations can be performed
Expose the patients legs from the groin to in two ways:
the toes, keeping his underwear or a gown in Long posterior flap of Burgess posterior
place calf muscles are used to cover the
The patient may be presented on a chair if transected bone, but the muscle should
it is clear the examiner only wants you to be debrided carefully so the flap is not
look at the amputation, then the other leg too bulky to fit within the skin
should not be exposed. Skew flap of Kingsley Robinson the
suture line runs obliquely (anterior
VITAL POINTS posterior)
In above-knee amputations, equal anterior
Look and posterior semicircular flaps are formed
the vastus lateralis is sutured to the
Examine the amputation stump first adductors, and the other quadriceps
Describe the anatomical level of the muscles to the hamstrings.
amputation, usually above-knee or below-
knee Feel
If the patient has a partial foot amputation,
then describe the level of amputation (Ray, The soft tissue under the skin should move
transmetatarsal, Symes) freely over the bone beneath.
Case 113 Circulation and lymphatic systems 201

Move any operation, and also into immediate


(within 24h), early (up to 1 month) and late
Ask the patient to actively flex and extend (beyond 1 month)
the knee joint immediately above the Mention at the beginning that these patients
amputation to demonstrate the fixed flexion often have other medical problems,
deformity that often follows a below-knee especially cardiovascular disease, putting
amputation if good physiotherapy has not them at particularly high risk
prevented it Operative mortality is 20% and one-year
Manipulate the joint passively if the patient survival is 50%.
cannot move it at all Specific early complications:
If the patients prosthesis is available, ask to Psychological and social implications
look at it, and then ask the patient to fit the Haematoma and wound infection, including
appliance and walk with it in place. gas gangrene (rare)
Deep vein thrombosis and pulmonary
Finish your examination here embolus
Phantom limb pain due to the sensory
cortex believing the limb is still present
Completion Skin necrosis (caused by poor perfusion of
the stump) requires refashioning, usually at a
higher level
Say that you would like to:
Specific late complications:
Examine the rest of the limb and the other
limb for signs of peripheral vascular disease. Osteomyelitis infection transmitted to the
bone through the stump
Stump ulceration can be caused by
QUESTIONS pressure from the prosthesis
Stump neuroma swelling of the distal nerve
(a) What are the indications for
as it tries to regrow following division; during
an amputation? the initial procedure the nerve should be cut
Indications for amputation can be remembered back far enough to prevent a neuroma from
as the 4 Ds: forming
Dead (ischaemic): Fixed flexion deformity of the knee,
especially with long-term disease
Peripheral vascular disease (8090% of
all cases) Difficulty in mobilizing
Thromboangiitis obliterans (see Case 127) Spurs and osteophytes in the underlying
bone.
Arteriovenous fistulae (see Case 131)
Damaged (trauma):
Unsalvageable limbs
Burns James Syme (17991870). Professor of Surgery,
Frostbite Edinburgh and University College Hospital, London,
Dangerous (malignancy) and father-in-law to Joseph Lister.
Bone
Soft tissue
Damn nuisance (infection/neuropathy)
Osteomyelitis FURTHER READING
Necrotizing fasciitis Dormandy J, Heeck L, Vig S: Major
Charcot neuroarthropathy. amputations: clinical patterns and predictors,
Semin Vasc Surg 12(2):154161, 1999.
(b) What are the complications of Robinson KP, Hoile R, Coddington T: Skew flap
amputations? myoplastic below-knee amputation: a
Complications should be divided into preliminary report, Br J Surg 69(9):554557,
specific to the amputation and general for 1982.
202 Circulation and lymphatic systems Case 114

CASE 114 PERIPHERAL ARTERIAL SYSTEM


HISTORY ***

Intensity:
INSTRUCTION
The pain is always felt in the muscles as it is
This patient is describing some pain in the calf due to increased oxygen demand from
on walking. Ask him some questions to help actively contracting muscle
you define the cause.
When the demand is not met due to
ischaemia in the afferent arterioles, anaerobic
APPROACH metabolism takes over and lactic acidosis
occurs
Within a short case or OSCE, you may be asked The pain is due to anoxia, acidosis and the
to take a history from a patient, and vascular build-up of metabolites
long cases are extremely common in the final
Precipitating and relieving factors:
MB examination.
The pain comes on during exercise and
typically after a fixed distance
VITAL POINTS It comes on more rapidly after walking up a
The history should be structured to answer hill rather than on the flat
three basic questions, summarized in Table 29. The pain is relieved after a few minutes of
resting.

Introduction
Rest pain
Ask the patients age
Site:
Ask their occupation.
Rest pain is described in the least well
perfused areas of the leg, over the toes and
Pain of intermittent claudication forefoot. Thus, calf pain at night in the
absence of foot pain is unlikely ischaemic in
Site: origin
Stenosis of the lower aorta and common iliac Intensity:
artery cause buttock claudication (and may The pain is very severe, aching in nature and
be associated with impotence) typically wakes the patient from sleep
External iliac artery stenosis causes thigh Precipitating and relieving factors:
claudication
The pain comes on at night when the patient
Superficial femoral artery stenosis leads to
is lying flat in bed
calf claudication

Table 29
Vascular symptoms Risk factors for arterial disease Fitness for surgery
Intermittent claudication Smoking Previous medical history
Rest pain Diabetes Anaesthetic history
Critical ischaemia Hypertension Drug history and allergies
Cholesterol Social history (related to
Previous history (especially heart disease or stroke) postoperative
Family history rehabilitation)
(Renal failure)
(Hypothyroidism)
(Gout)
Case 115 Circulation and lymphatic systems 203

It is relieved by getting up and walking on a


cold floor and sometimes the patients
QUESTIONS
describe that they have to sleep sitting up to (a) What is the differential diagnosis of
prevent the pain from coming on
intermittent claudication?
Characteristically, the pain is also relieved by
hanging the foot over the edge of the bed. The causes of pain in the leg can be divided
into:

Critical ischaemia Musculoskeletal: pathologies of the knee,


ankle or hip (such as osteoarthritis)
It is important to distinguish immediately Neurological: spinal stenosis (leading to
whether there is any feature in the history which spinal claudication)
suggests more severe (limb-threatening) Vascular: intermittent claudication, deep vein
disease. Ask specifically about the presence of thrombosis.
ulcers or gangrene (trophic changes).
(b) Why do patients with rest pain typically get
Critical limb ischaemia is defined by the
more severe pain at night?
European working group as:
The pain is caused by a reduced blood supply
1. Presence of arterial ulcers or gangrene, or
to the distal aspects of the limb. The pain gets
2. Rest pain that lasts for two weeks or more worse at night because the perfusion of the limb
and is only relieved by opiate analgesia, and is further reduced when the patient is lying
3. An absolute ankle pressure (see Case 111) of down. This is due to:
<50mmHg.
Decreased cardiac output at night
Reduced effect of gravity, which normally
Function acts to increase relative blood supply to the
legs
As for the orthopaedic history: Relative dilatation of the skin vessels due to
Impact on patients life, e.g. work, sleep the warmth of the bedclothes.
Going to the shops
Walking aids
Limp.

CASE 115 ABDOMINAL AORTIC ANEURYSM ***

INSTRUCTION Palpate
Examine this gentlemans abdomen.
Hand examination is likely to be normal, and
the examiner will probably move you on
immediately to palpation of the abdomen
APPROACH
Gentle palpation of the nine abdominal areas
Expose the patient as for the abdominal may be normal
examination (see Case 43).
A pulsatile mass may be identified on deeper
palpation in the epigastric region
VITAL POINTS The mass should be measured by bringing
the lateral sides of the index fingers of both
Inspect hands together to identify the borders of the
aneurysm and estimating the distance
A midline pulsating mass may be visible, between your fingers (in cm)
especially in deep inspiration this is easier An expansile mass moves your fingers
to identify in thin patients laterally with each pulse; aneurysms are
Note the presence of any abdominal scars. expansile as well as pulsatile (a transmitted
pulsation is not expansile, see Case 1)
204 Circulation and lymphatic systems Case 115

Take care not to palpate too firmly


QUESTIONS
Palpate over the course of the common iliac
arteries (a) In which patients are abdominal aortic
Continue the examination by palpating the aneurysms most common?
femoral arteries in the groin and the popliteal
arteries (which may also be aneurysmal in Aneurysms are most common in:
nature). Men
Aged >60 years
Auscultate Smokers
Hypertensive patients
For aortic and iliac bruits. Often strong family history.

(b) Which patients should have their


Finish your examination here aneurysms repaired?
The reason for repairing abdominal aortic
aneurysms is to avoid complications
TOP TIP The following aneurysms should be repaired:
Remember that the abdominal aorta lies slightly to Symptomatic aneurysms (back pain,
the left of the midline and bifurcates into the common tenderness over the aneurysm on
iliac arteries at the level of L4, approximately at the level palpation, distal embolic events, ruptured/
of the umbilicus. A common error is to try to palpate for leaked aneurysms)
an abdominal aortic aneurysm (AAA) too low. Begin by Asymptomatic aneurysms (5.5cm
palpating in the epigastric region to avoid missing the diameter; increase of diameter of 1cm
pulsation (Fig. 123). per year, suggesting rapidly expanding
aneurysm)
The UK Small Aneurysm Trial suggests that if
the aneurysm is between 4.0 and 5.5cm in
diameter, open surgical repair is not
Completion recommended, and for those greater than
5.5cm in diameter, the patient will benefit
Say that you would like to examine the heart, from surgery
carotid vessels and legs for concurrent cardiac, The risk of rupture of a >5.5cm aneurysm is
carotid or peripheral vascular disease. 10% per year, increasing with the size of the
aneurysm.

(c) What is the operative mortality of


AAA repair?
The elective mortality from open AAA repair
is 5%, but this figure may be lower in
specialist centres
If the patient suffers a ruptured aneurysm
and reaches the hospital, their operative
mortality rises to 50%, but only 50% of
patients reach hospital alive
Mortality is usually from haemorrhage,
subsequent myocardial infarction or renal
failure.

ADVANCED QUESTIONS
(a) Are there any other options other than
open AAA repair?
Endovascular repair (EVAR), using grafts
Figure 123 Examination of the aorta in the abdomen. placed into the abdominal aorta from the
Case 116 Circulation and lymphatic systems 205

femoral artery by a vascular surgeon and a Syphilitic aneurysms have been consigned to
radiologist is increasingly performed history, although recent increasing rates of
Although the operative mortality is lower, infection may cause a resurgence in the future.
there is no long-term data to suggest that
outcome is better from this procedure and
there is a significant failure rate
FURTHER READING
(approximately 25%), and medium-term Ashton HA, Buxton MJ, Day NE, et al;
complications like endoleaks are of Multicentre Aneurysm Screening Study Group:
increasing concern The Multicentre Aneurysm Screening Study
Laparoscopic repair of abdominal aneurysms (MASS) into the effect of abdominal aortic
is the subject of current clinical trials. aneurysm screening on mortality in men: a
randomised controlled trial, Lancet
(b) Should we be screening for AAA? 360(9345):15311539, 2002.
A screening programme in the UK has never Scott RA, Vardulaki KA, Walker NM, et al: The
been agreed, and opinions differ as to the long-term benefits of a single scan for
value of screening on a population basis abdominal aortic aneurysm (AAA) at age 65, Eur
The MASS trial revealed a significantly J Vasc Endovasc Surg 21(6):535540, 2001.
reduced prevalence of aneurysm-related The UK Small Aneurysm Trial Participants:
death in the screened male population Mortality results for randomised controlled trial
between 65 and 74 years of age, with a 53% of early elective surgery or ultrasonographic
reduction in those who attended for surveillance for small abdominal aortic
screening. aneurysms, Lancet 352(9141):16491655, 1998.

(c) Do you know of any infectious agents


associated with AAA? Note
Salmonella typhi is the most common
infective organism The actor George C. Scott died from a ruptured
Mycotic aneurysms sometimes occur as a abdominal aortic aneurysm in September 1999,
result of staphylococcal infection as did Sir John Hunter (see Case 117).

CASE 116 CAROTID ARTERY DISEASE ***

and medial to the sternocleidomastoid in the


INSTRUCTION
anterior triangle of the neck
Listen to this gentlemans neck. The bruit is best heard in expiration
Tell the examiner that you would listen over
the precordium to ensure this is not a
APPROACH transmitted aortic stenosis murmur (heard as
Expose the patients neck as for the neck exam an ejection systolic murmur in the aortic area
(see Case 6). second intercostal space immediately to
the right to the sternum).

VITAL POINTS Finish your examination here


This is a direct instruction and you should
proceed immediately to auscultation.
TOP TIP

AUSCULTATE The subject of carotid bruits and cerebrovascular
Notice the bruit over one or both carotid events may also be brought up by asking you to
arteries question the patient with regards to his bruit
The bruit is best heard over the course of the In this case, you should ask about previous transient
common carotid artery, which runs behind ischaemic attacks or stroke, asking about temporary
206 Circulation and lymphatic systems Case 116

or resolving neurological symptoms such as weakness angiography (MRA) is an alternative, which


or paraesthesia does not carry such risk
Also ask about amaurosis fugax, the visual sensation An echocardiogram would also be an option,
of a curtain being drawn down slowly in front of one especially if the patient had a precordial bruit
eye A CT or MRI scan of the brain may be
Neurological symptoms occur on the contralateral side performed, demonstrating lacunar infarcts.
but amaurosis fugax is ipsilateral to the side of the
(b) What is the consequence of carotid
carotid stenosis.
stenosis?
Stroke is the third leading cause of death in
the west and 85% of strokes are
thromboembolic, caused by atherosclerosis
Completion at the carotid bifurcation or proximal
(23cm) internal carotid artery
You would perform a neurological examination Atherosclerosis can also affect the
to look for signs of a previous cerebrovascular intracranial circulation, particularly the circle
event, and would also check for signs of of Willis and the vertebrobasilar system
atherosclerosis elsewhere (heart, abdominal
Transient ischaemic attacks (neurological
aorta and peripheral vascular system).
symptoms resolving completely within 24h)
and amaurosis fugax are usually caused by
repeated microemboli from the plaque,
QUESTIONS consisting of clusters of platelets and
cholesterol
(a) How would you investigate a patient who
The same symptoms can be caused by
was referred with a carotid bruit?
microemboli from the heart and aortic arch.
The patient should have a full workup for
atherosclerosis:
ADVANCED QUESTIONS
General investigations:
Urinalysis for proteinuria, marker of (a) Which patients might be considered for
atherosclerotic renal disease carotid endarterectomy (Fig. 124)?
Blood tests Symptomatic carotid stenosis of 70%
Haematology: full blood count for Both the North American (NASCET,
anaemia, which might precipitate North American Symptomatic Carotid
symptoms
Biochemistry: renal function for possible
undetected renal disease
Glucose: exclude diabetes
Internal carotid
Cholesterol: to identify artery
hypercholesterolaemia
Electrocardiogram: To look for evidence of
atrial fibrillation, cardiac disease, previous
infarction, ischaemia or left ventricular
dysfunction
Special investigations: External carotid
artery
A carotid duplex scan, looking for
atherosclerotic plaques
The report would detail the size and location
of plaques, the diameter of the patent lumen
remaining, and may also provide some Javid shunt
information on the plaque friability or
likelihood of embolization
Carotid angiography to identify the anatomy
of the carotid arteries in more detail is also a Common carotid
possibility, but this procedure in itself carries artery
a 2% risk of stroke. Magnetic resonance Figure 124 Carotid endarterectomy.
Case 117 Circulation and lymphatic systems 207

Endarterectomy Trial) and European (ECST,


Thomas Willis (16211675). English physician and
European Carotid Surgery Trial) trials set up
anatomist who described the sweet taste of diabetic
to look at evidence-based reasons for
urine, myasthenia gravis, general paralysis of the
carotid endarterectomy demonstrated the
insane, whooping cough and identified the intercostal,
value of surgery in patient with symptomatic
spinal and spinal accessory nerves. He gave names to
stenosis of 70% or more
reflex and neurology and was buried in Westminster
These trials demonstrated that for patients
Abbey.
with severe stenosis (over 70%) surgery
reduced the relative risk of disabling stroke
or death by 48%
The benefit in asymptomatic patients has not FURTHER READING
yet been proven. Cina CS, Clase CM, Haynes RB: Carotid
endarterectomy for symptomatic carotid
(b) What would you warn the patient of in
stenosis, Cochrane Database Syst Rev
consenting them for an endarterectomy? 2:CD001081, 2000.
The advantages of having surgery are a
European Carotid Surgery Trial (ECST):
six-fold reduction in the rate of stroke at
Randomised trial of endarterectomy for recently
3 years
symptomatic carotid stenosis Final results of
The operative risk of stroke is 2% and the MRC ECST, Lancet 351(9113):13791387.
operative mortality 12%
North American Symptomatic Carotid
Specific risks of haematoma, hypoglossal
Endarterectomy Trial (NASCET) investigators:
nerve injury, and numbness of the ipsilateral
National Institute of Neurological Disorders and
earlobe should also be mentioned.
Stroke and Trauma Division. Clinical alert:
benefit of carotid endarterectomy for patients
with high-grade stenosis of the internal carotid
artery, N Engl J Med 22(6):816817, 1991.

CASE 117 POPLITEAL ANEURYSM **

Note the expansile pulsation behind the knee


INSTRUCTION
The aneurysmal pulse is relatively easy to
Examine the pulses in this gentlemans legs. find and the artery does not need to be
compressed against the tibia
Slide the fingers of your two hands apart and
APPROACH comment on the diameter of the vessel. A
Expose the patients legs from the groin to the popliteal aneurysm is 2cm or greater in
toes, preserving his dignity. If asked to examine diameter
the pulses, rather than the limb, be sure to The pulsating mass does not alter with
begin by palpating the femoral pulse you have change in position of the knee
not been asked to examine the feet for signs of The ankle and foot pulses may not be
peripheral arterial disease. palpable if the aneurysm is thrombosed
50% are bilateral dont forget to examine
VITAL POINTS the other knee.

Palpate the leg pulses Finish your examination here

Begin with the femoral pulses, comparing


one side with another (see Case 111) Completion
Comment on whether the pulses are present
or absent and whether the character of the Say that you would like to:
pulse is normal Examine the rest of the limb and the other
Move down to the popliteal pulses limb for signs of peripheral vascular disease
208 Circulation and lymphatic systems Case 118

Examine the abdomen as 50% will also have Those greater than 2cm
an abdominal aortic aneurysm. The aneurysm is surgically repaired by either an
excision bypass, where the popliteal artery is
ligated above and below the diseased segment
QUESTIONS and a graft interposed, or a simple resection
(a) How might a patient with a popliteal and anastomosis without the use of a graft
(Hunters ligation).
aneurysm present?
Acute ischaemia caused by thrombus can be
Popliteal aneurysms represent 80% of all treated with thrombolysis.
peripheral (non-aortic) aneurysms
The patient may have presented with a lump
behind the knee if the aneurysm has grown
to such a size that it has expanded beyond Sir John Hunter (17281793). Scottish surgeon and
the popliteal fossa anatomist. Also described Hunters canal (subsartorial
50% present with distal limb ischaemia adductor canal) and Hunterian chancre (syphilitic
caused by thrombosis or embolism chancre). Interestingly, he died of a ruptured
abdominal aortic aneurysm and was buried in
Patients may present with an acutely
Westminster Abbey.
ischaemic leg
Less than 10% rupture.

(b) Under what circumstances would they


be treated? FURTHER READING
Surgery is indicated for: Thompson MM, Bell PR: ABC of arterial and
venous disease. Arterial aneurysms, BMJ
Symptomatic aneurysms
320(7243):11931196, 2000.
Those containing thrombus

CASE 118 ISCHAEMIC ULCER **

Shape:
INSTRUCTION
The size of the ulcer varies from a few
Examine this gentlemans feet.
millimetres on the tip of the toes to several
centimetres over the lower leg
APPROACH Edge and base:
The edge is punched out, clean cut because
Again, ideally expose the whole of the legs from
there has been no partial healing of the
the groin, maintaining the patients dignity, but if
wound
the patient is in an environment where other
patients are present, this would be inappropriate The base may contain slough and may be
and you should just comment on the feet. infected but there is no healthy red
granulation tissue as the blood supply is too
poor
VITAL POINTS The ulcer may be very deep and penetrate
down to bone and the underlying joints the
Begin examining the legs as for the peripheral
bone may be exposed at the base
arterial system examination (see Case 111).
Surrounding skin:
The skin around the ulcer is a grey/blue
Look colour.

Observe the following characteristic features of


an ischaemic ulcer. Feel
Site:
Palpate for temperature using the back of
Characteristically over the tips of the toes the hands note the surrounding skin is cold
and over the pressure areas
Case 118 Circulation and lymphatic systems 209

compared with the proximal limb and the (c) What other non-surgical treatments
contralateral foot are available?
Check the peripheral pulses, noting the most Risk-factor modification:
distal pulse that is still palpable
Stopping smoking
Check the pulses of the other leg.
Good diabetic and hypertensive control
Optimized serum lipid levels
Finish your examination here Symptom modification:
Avoidance of drugs which might worsen
symptoms, such as beta blockers
TOP TIP Commencement of low-dose aspirin (75mg/
day), which reduces the incidence of cardiac
Almost all of the examination is Look and you should and cerebrovascular events in high-risk
spend as much time as possible commenting on the patients
features without progressing to Feel as this will score
Intravenous prostaglandins act by inhibiting
marks very quickly.
platelet aggregation, stabilizing leucocytes
and endothelial cells, and are vasodilators.
They can have some effect in healing ulcers,
relieving rest pain and reducing the risk of
amputation
QUESTIONS Lumbar sympathectomy reduces
(a) What are the causes of ischaemic ulcers? sympathetic-mediated vasoconstriction and
improves perfusion by allowing for
These can be divided into large and small unopposed vasodilatation of the skin
vessel arterial disease: vessels. This is often unsuccessful in
Large vessel diabetics who may have autonomic
Atherosclerosis neuropathy causing autosympathectomy.
Thromboangiitis obliterans (see Case 127)
Different types of leg ulcers
Small vessel
Diabetes mellitus (See Table 31).
Polyarteritis nodosa
Rheumatoid arthritis. FURTHER READING
(b) What kinds of analgesia would be London NJ, Donnelly R: ABC of arterial and
venous disease. Ulcerated lower limb, BMJ
appropriate for this patient?
320(7249):15891591, 2000.
Ischaemic ulcers can be extremely painful
Sarkar PK, Ballantyne S: Management of leg
and even removing the bandages from
ulcers, Postgrad Med J 76(901):674682, 2000.
around the ulcer can cause pain that lasts
for several hours
Note
Consider the analgesic ladder (Table 30),
remembering that combinations of drugs About 400 years BC, Hippocrates wrote, In
administered regularly in a variety of different case of an ulcer, it is not expedient to stand,
formulations (oral, intramuscular, etc.) can be especially if the ulcer be situated on the leg.
more effective. Hippocrates himself had a leg ulcer.

Table 30 The analgesic ladder


Stage Analgesia
I Simple, oral agents such as paracetamol or non-steroidal anti-inflammatory drugs (NSAIDs)
like ibuprofen taking great care in the elderly and patients with renal impairment
selective cyclooxygenase 2 (COX-2) inhibitors such as rofecoxib are also available which
reduce the incidence of gastrointestinal bleeding
II Stronger oral agents, such as a mixture of orally acting opioids like codeine and paracetamol
Stronger NSAIDs such as diclofenac
III Intramuscular, stronger oral or intravenous opioids such as morphine, diamorphine
210 Circulation and lymphatic systems Case 119

Table 31 Comparison of different types of leg ulcer


Venous Ischaemic Neuropathic
Site Gaiter region over medial Tips of toes and Heel, underneath metatarsal
malleolus of ankle pressure areas heads (pressure bearing areas)
Shape Variable, usually irregular Regular outline Regular outline, follows skin
contour
Size Can be very large Varying size, few mm Several cm
to several cm
Edge Usually sloping pale purple/ Punched out, clean Clean
brown
Base Pink granulation tissue or white Bone may be exposed, Often exposing bone
fibrous tissue characteristic no granulation tissue
Surrounding skin Chronic venous signs, e.g. Grey/blue Normal
lipodermatosclerosis
Skin temperature May be warmer Cold Normal
Pulses Present Absent Present

CASE 119 POST-PHLEBITIC LIMB **

Venous eczema
INSTRUCTION
Lipodermatosclerosis
Examine this gentlemans legs. Venous ulceration or evidence of previous
ulceration.
APPROACH
Expose the patients legs, maintaining his
Feel
dignity and keeping his underwear on. Ensure
that you can see his feet and position him lying Compare the temperature of both legs
comfortably on the couch. Check for pitting oedema (watching the
patients face at all times).

VITAL POINTS Finish your examination here


Look
Completion
Note the features of chronic venous
insufficiency, comparing one side with the other: Say that you would like to:
Swelling Test for deep venous occlusion Perthes
Dilated superficial veins (as blood cannot test place a high tourniquet around the top
return to the inferior vena cava through the of the patients thigh and ask them to walk.
deep veins) If the deep venous system is occluded, the
Skin pigmentation, possibly restricted to the leg will become swollen and blue with dilated
medial malleolus (ankle flare) superficial veins distal to the tourniquet.
Case 120 Circulation and lymphatic systems 211

ADVANCED QUESTIONS (c) What are the surgical options available for
deep venous occlusion/reflux?
(a) What is venous gangrene? Reflux:
Venous gangrene is a rare complication of deep Trahere transplantation use a segment of
vein thrombosis in the iliofemoral segment and axillary vein with valve and insert it into the
presents in three phases: deep venous system of the leg, wrapping it
in a PTFE cuff
1. Phlegmasia alba dolens (white leg)
Kistners operation valvuloplasty of
2. Phlegmasia cerulea dolens (blue leg)
damaged valves
3. Gangrene occurs as a consequence of
Obstruction:
acute ischaemia and may be restricted to the
foot or spread up the leg Palma operation use contralateral long
saphenous vein (LSV) and anastomose to the
(b) What investigations are appropriate for femoral vein to bypass iliofemoral
deep venous disease? obstruction
Duplex shows areas of reflux and deep Warren bypass use LSV to bypass deep
venous occlusion venous blockage no longer used.
Venography:
Ascending identifies deep venous Notes
patency and perforator incompetence
Descending identifies areas of reflux Post-phlebitic limbs:
Varicography: shows sites of communication 90% are due to reflux following DVT
Ambulatory venous pressures. 10% are due to obstruction following DVT.

FURTHER READING
Hopkins NF, Wolfe JH: ABC of vascular
diseases: deep venous insufficiency, BMJ
304:107, 1992.

CASE 120 GANGRENE **

Wet gangrene usually has an ill-defined,


INSTRUCTION
spreading edge
Examine this gentlemans legs. Skin blistering may occur
A line of demarcation gradually appears
between the viable and dead tissue if this
APPROACH
has occurred, the gangrene is dry and
Expose the patient and examine the legs (see the dead tissue may eventually fall off
Case 111). (autoamputation)
Continue to comment on the other features
of peripheral vascular disease (see Case
VITAL POINTS 111).
Look
Feel
Note the appearance of gangrene which
often begins between the toes Palpate the peripheral pulses and check for
Comment on whether the gangrene is wet or temperature differences between the legs (see
dry Case 111).
Wet gangrene is due to either acute
ischaemia or local trauma, and may be
complicated by infection
Finish your examination here
212 Circulation and lymphatic systems Case 121

QUESTION ADVANCED QUESTION

(a) What are the causes of gangrene? (a) What is Fourniers gangrene?
Gangrene is the result of irreversible tissue Rare necrotizing subcutaneous infection
necrosis and has a number of causes: involving the scrotum, penis and perineum
Diabetes (the commonest cause) Scrotum is red and swollen with crepitus on
Embolus and thrombosis both leading to palpation due to dermal gangrene
acute limb ischaemia, mesenteric infarction, Organisms responsible are usually coliforms
critical limb ischaemia, trashing of feet and anaerobes.
Raynauds syndrome see Case 121
Thromboangiitis obliterans (Buergers Jean Alfred Fournier (18321914). French dermatolo-
disease) see Case 127 gist who specialised in the study of venereal diseases.
Ergot poisoning Fourniers gangrene was actually first described by
Vessel injury secondary to extreme cold, Baurienne in 1764 but was named after Fournier
heat, trauma or pressure following five cases he presented in clinical lectures in
Drug-induced, e.g. warfarin. 1883. His name is associated with the two other
medical terms:
Fourniers sign: Scars on the mouth following the
healing of lesions in congenital syphilis
Fourniers tibia: Fusiform thickening and anterior
bowing of the tibia in congenital syphilis.

CASE 121 RAYNAUDS PHENOMENON **

INSTRUCTION VITAL POINTS


Look at this ladys hands and ask her few Ask questions at the same time as looking at
questions. the hands:
What is the main problem you have with
your hands?
APPROACH
When does this symptom occur?
The examiner will often use a leading question Is it precipitated by any specific weather
like this in order to stimulate a spot diagnosis conditions?
and lead to some supplemental questions. In
Can you describe the colour changes your
this situation, the key is to ascertain the
fingers go through during these episodes?
presence of the central clinical features (in this
case of Raynauds) and then to try to identify
any precipitating features. Look

Note that the pathology is usually bilateral


TOP TIP 1 In between acute attacks, the skin may be
Be clear about terminology: dry and red, especially around the tips of the
fingers, and the nails brittle
Raynauds phenomenon: characteristic cold-induced
Also note any ulcers or gangrene on the
changes associated with vasospasm
pulps.
Raynauds disease: primary disease occurring in
isolation
Feel
Raynauds syndrome: secondary Raynauds
phenomenon associated with other diseases (see
The radial pulse is normal.
below).
Case 121 Circulation and lymphatic systems 213

TOP TIP 2 TOP TIP 3


The acronym WBC may help you recall the order of The secondary causes can be remembered using the
the skin colour changes of the fingers seen in Raynauds: acronym BADCaT:
White blanching of digits Blood disorders, e.g. polycythaemia
Blue cyanosis and pain Arterial, e.g. atherosclerosis, thromboangiitis
Crimson reactive hyperaemia fingers turn red in obliterans
colour. Drugs, e.g. beta blockers, oral contraceptive pill
Connective tissues disorders, e.g. rheumatoid arthritis,
systemic lupus erythematosus, scleroderma,
polyarteritis nodosa
Finish your examination here Trauma, e.g. vibration injury.

Completion
ADVANCED QUESTION
Say that you would like to:
Ask about symptoms and look for signs of What are the treatment options for
the secondary causes of Raynauds. Raynauds?
Non-surgical:
QUESTIONS Use of gloves and discontinuing any
predisposing drugs, e.g. beta blockers
(a) What is the pathogenesis of Raynauds Using warm pads in gloves and socks in the
phenomenon? winter
If the vessels are normal in calibre, the Encourage patients to stop smoking
clinical features may be caused by relatively Medical (used with variable success):
overactive alpha receptors in the wall, Calcium channel blockers, e.g. nifedipine
leading to abnormal smooth muscle Prostacyclin analogues
contraction or changes in elasticity
Alpha blockers
Alternatively, there may be a fixed
5HT antagonists
obstruction in the vessel wall, which reduces
the distal flow and thus renders the digits Surgical:
susceptible to the effects of cold. Cervical sympathectomy and amputation of
the affected phalanges
(b) What are the predisposing factors? Cervical sympathectomy may not be a
The causes can be divided into primary and permanent solution and may only relieve
secondary: symptoms for 2 years or less
Primary Raynauds (Raynauds disease) is Amputate only if digits are threatened with
due to vasomotor malformation gangrene.
Secondary Raynauds occurs as a
consequence of pathology affecting the
vessel wall. Maurice Raynaud (18341881) was a physician in
In general the secondary causes, especially Paris and he described the differences between
when related to connective tissue diseases, primary Raynauds disease and secondary Raynauds
cause more severe problems with necrosis and phenomenon in his MD thesis at the age of 28.
gangrene.

FURTHER READING
Block JA, Sequeira W: Raynauds phenomenon,
Lancet 357(9273):20422048, 2001.
www.nhlbi.nih.gov/health/public/blood/other/
raynaud.htm information for patients.
214 Circulation and lymphatic systems Case 122

CASE 122 NEUROPATHIC ULCER **

INSTRUCTION TOP TIP


Examine this ladys feet. It can be difficult to distinguish between an ischaemic
and neuropathic ulcer, however, neuropathic ulcers are:

APPROACH Painless
Associated with normal appearance of the surrounding
As previously (see Case 110).
skin
Associated with local sensory loss.
VITAL POINTS
Look

Observe the characteristic features of a QUESTIONS


neuropathic ulcer.
Site: (a) What are the causes of neuropathic
ulcers?
They are usually found over the pressure
areas, over the metatarsal heads on the sole They can be caused by any disease that leads
of the foot and the balls of the toes; they can to a peripheral sensory neuropathy, or by
also occur on the heel causes of spinal cord disease. Causes of
Shape: peripheral neuropathy include:
Irregular, correspond to the shape of the Systemic diseases:
pressure point that has become exposed Diabetes by far the most relevant cause
Edge and base: in clinical practice
Clean edge Vasculitis (SLE)
Base may be deep, with exposure of bone Hypothyroidism
and tendon Vitamin B12 deficiency
Surrounding skin: Drugs and toxins:
The surrounding skin has a normal blood Prescribed drugs (amiodarone/
supply and therefore looks normal. metronidazole, etc.)
Alcohol
Toxins
Feel
Infections:
Feel the temperature of the surrounding skin, TB, leprosy
which is expected to be normal HIV
The peripheral pulses are usually normal Carcinomas, especially in lung cancer and
Test the sensation over the dermatomes using polycythaemia rubra vera
light touch and pinprick, note the absence of Idiopathic (5060%).
sensation around the ulcer and describe the
extent of the sensory abnormality. (b) Why do these ulcers form?
Peripheral neuropathy has several effects:
Finish your examination here Slowly progressive sensory loss, with
numbness and tingling of the feet and
sometimes also hands. The sensory loss is
often glove-and-stocking in distribution and
Completion may also be associated with motor
impairment. The impact of the sensory loss
Say that you would like to: is that damage over the pressure areas is
Perform a complete neurological not noticed by the patient
examination, including cranial and peripheral Motor neuropathy may result in wasting of
nerves. the intrinsic foot muscle and an altered foot
Case 123 Circulation and lymphatic systems 215

shape, with claw toes and prominent Tuberculosis:


metatarsal heads Undermined edge
Autonomic neuropathy reduces sweating, Shallow ulcer
leading to a dry foot susceptible to cracking.
Pyoderma gangrenosum:
Therefore, the insensitive, mechanically
Undermined edge
abnormal, dry foot is at risk from unperceived
external trauma (e.g. from shoes) and from Violaceous
repetitive (and often relatively minor) painless Necrotic ulcer with hypertrophic margins
injury (e.g. foreign body in shoe). Progressive Syphilis:
skin loss and ulceration may occur. Gumma of tertiary syphilis has a typical
punched-out ulcer, over the anterior surface
of the lower leg and has a yellow coloured
FURTHER READING wash leather base
Phillips TJ: Successful methods of treating leg Scalloped border
ulcers. The tried and true, plus the novel and Arteriovenous fistulae:
new, Postgrad Med 105(5):159174, 1999.
Ulcer is distal to the fistula
www.skinwound.com/online_training_manual/
Shallow indolent ulcers
neuropathic_wounds.htm guide to neuropathic
Rheumatoid arthritis:
ulcers.
Necrotizing vasculitis
Purpuric, haemorrhagic bullae
Note: Rarer causes of leg ulceration
Squamous cell carcinoma:

The following causes of leg ulcers may also be Rolled or raised edge
encountered in the clinical examination, a Often on sun-damaged skin
couple of characteristics are listed for each Sickle cell disease:
type: Small, punched-out ulcers
Often over medial aspect of lower leg.

CASE 123 LYMPHOEDEMA **

(hyperkeratosis, lichenification and peau


INSTRUCTION
dorange)
Examine this ladys legs. Yellow discoloration of nails.

APPROACH Palpate
Expose the patients legs, preserving her
Determine whether or not the oedema is
dignity.
pitting in nature
Initially the oedema is characteristically
VITAL POINTS pitting but later it stops pitting as tissue
resistance increases
Look Palpate the groin for inguinal
lymphadenopathy (which may be present).
The legs may be grossly swollen, with no
particular distribution
Tends to be bilateral
Finish your examination here
Note the loss of contour at the ankle which
causes a buffalo hump appearance on the
dorsum of the foot Completion
There may be lichenified fronds on the toes
and the skin looks thick and indurated Say that you would like to:
216 Circulation and lymphatic systems Case 123

Examine the jugular venous pulse, heart and Infections, e.g. filiaris (infection by the
lungs to exclude right-sided cardiac failure Wuchereria Bancrofti worm), tuberculosis
Palpate the liver to identify hepatomegaly Post surgery or radiotherapy such as axillary
Ask the patient some questions to determine dissection in breast surgery and inguinal
any hereditary conditions that predispose to irradiation.
lymphoedema.
(b) What are the treatment options?
Non-surgical:
TOP TIP
Grade III compression stockings to apply
The commonest cause of unilateral ankle oedema is 40mmHg pressure at the ankles
venous disease; lymphoedema is much more commonly Intermittent pneumatic compression device
bilateral. Cellulitis should be treated
Advise patient to elevate their leg as much
as possible and stress the importance of
cleanliness and careful chiropody.
Limb elevation reduces intravascular hydrostatic
QUESTION
pressure and the stockings increase
(a) What is the differential diagnosis of extracellular hydrostatic pressure, together
swollen legs? reducing the level of tissue oedema. These
measures can be very successful but patient
Lymphoedema can be similar in appearance to motivation is key and it may take some time for
any other cause of swollen legs, but tends to be the results to become apparent.
bilateral: Surgical:
Central causes include right heart failure, Used rarely: the results tend overall to be
hypoalbuminaemia, nephrotic syndrome and poor
hypothyroidism
More likely to be successful where there is
Peripheral (local) causes are usually venous discrete occlusion of the lymphatics
disease such as deep vein thrombosis,
Options include debulking or bypass
KlippelTrenaunay syndrome, chronic venous
procedures:
insufficiency or post-phlebitic limb (see
Cases 109, 110 and 119) Direct lymphovenous anastomosis
Rare causes are angio-oedema, Stripping a piece of small intestine
arteriovenous malformations (ParkesWeber mucosa, exposing the rich submucosal
syndrome, multiple AV fistulae) and hemi- plexus this can then be used to replace
hypertrophy. a leg lymph node which then forms new
connections with distal lymphatics in
order to drain the leg
ADVANCED QUESTIONS Debulking to reduce the volume of the leg
Homans procedure is an example of
(a) What is the difference between primary such an operation. Flaps are raised above
and secondary lymphoedema? and below the knee (beginning on the
medial side and then returning to surgery
Primary lymphoedema refers to congenital
later if required to complete the lateral
disease or primary lymphatic failure. It is three
flap) and strips of subcutaneous tissue
times more common in women and the
are removed before the flap is sutured. If
pathology originates from within the lymphatics.
the skin is in poor condition, a different
It is also known as Milroys disease.
operation, which excises the skin in
Secondary lymphoedema can be classified addition to the soft tissues, can be
according to the cause: performed and the skin covered with a
Malignancy: infiltration of nodes; may also split skin graft (Charles procedure).
cause a chylothorax or chylous ascites when
this occurs in nodes in the thorax and
abdomen
Case 124 Circulation and lymphatic systems 217

Joseph Bancroft (18361894). Surgeon to the General FURTHER READING


Hospital, Brisbane, Australia. Cohen SR, Payne DK, Tunkel RS: Lymphedema:
John Homans (18771954). Professor of Clinical strategies for management, Cancer 92(4
Surgery, Harvard Medical School, Boston. He also Suppl):980987, 2001.
described Homans sign, which occurs when passive Rockson SG: Lymphedema, Am J Med
dorsiflexion of the foot gives pain in the calf in the 110(4):288295, 2001.
presence of a deep vein thrombosis.
www.lymphoedema.org/lsn/ information and
WF Milroy (18551942). North American physician. support network for patients.
www.cancerbacup.org.uk/info/lymphoedema.
htm online booklet from CancerBACUP about
lymphoedema.

CASE 124 HYPERHIDROSIS **

Hyperhidrosis erythematosus traumatica


INSTRUCTION
rare occupational form where a vibratory
Examine this patients hands. surface, (e.g. capstan lathe) produces
excessive sweating of the skin on contact
Phaeochromocytoma.
APPROACH
Expose to the elbows and ask the patient to (b) How do you treat this condition?
place his hands palm upwards on a pillow (if Reassurance: if symptoms are not
available). distressing to the patient
Medical: aluminium hexachloride solution
painting for axillary hyperhidrosis
VITAL POINTS Surgical:
Describe the presence of excessive sweat on Axillary excise hair-bearing skin/
the palmar surface of both hands, confirming intradermal Botulinum A neurotoxin
this by palpation. (Botox) this has a 62% cure rate
Palmar cervical sympathectomy (T2T4)
Finish your examination here via thoracoscopic approach this has a
98% cure rate
Plantar lumbar sympathectomy.

Completion (c) What side-effects would you warn this


patient about if considering cervical
Say that you would like to: sympathectomy?
Examine the axillae, groins and soles of the Excessive dryness of skin
feet for excessive sweating
Compensatory sweating around trunk (in up
Enquire about the social effects of the to 50% of patients)
symptoms
Horners syndrome (a consequence of
Exclude underlying causes (see below). damage to the stellate ganglion) 0.1%
Pneumothorax/haemothorax
QUESTIONS Important to warn of the risks of a general
anaesthetic for what may be largely a
(a) What is the differential diagnosis? cosmetic problem.
Anxiety
Hyperthyroidism
218 Circulation and lymphatic systems Case 125

ADVANCED QUESTIONS FURTHER READING


Chiou TS, Chen SC: Intermediate-term results
(a) What other part of the body can be
of endoscopic transaxillary T2 sympathectomy
affected by hyperhidrosis?
for primary palmar hyperhidrosis, Br J Surg
The face can be affected in patients with: 86(1):4547, 1999.
Syringomyelia Glogau RG: Botulinum A neurotoxin for axillary
Freys syndrome (see Case 21). hyperhidrosis. No sweat Botox, Dermatol Surg
24(8):817819, 1998.

CASE 125 FALSE ANEURYSM **

mid-inguinal point as it should be closely


INSTRUCTION
associated with the transition from external
Examine this gentlemans groin. iliac artery to common femoral artery as at
passes under the inguinal ligament.
Continue to palpate the distal peripheral
APPROACH pulses.
Expose the patients groin and begin your
examination (see Case 42).
Auscultate

VITAL POINTS There may be a bruit over the swelling.

Finish your examination here


TOP TIP 1
Most false aneurysms in exams affect the common
QUESTIONS
femoral artery, occurring following radiological puncture
during arteriography.
(a) What is the difference between a false and
a true aneurysm?
An aneurysm (see Case 115) is an abnormal
dilatation of a blood vessel (Fig. 125)
Inspection A true aneurysm involves all layers of the
arterial wall
Note the presence of a mass in the groin. It
A false aneurysm follows a partial laceration
is usually obvious that it is pulsatile from
of the vessel wall, causing blood to leak out
inspection alone
of the vessel into the surrounding tissues
Note the presence of any surgical scars or
A false aneurysm is the same as a pulsating
(much more subtly) puncture sites in the
haematoma and is most common in the
groin and accurately describe their position.
common femoral artery
Scars will usually be longitudinal, indicating
previous exposure of the common femoral Fibrous tissue forms around the haematoma
artery. and then contracts, producing a false sac
which contains thrombus but remains
connected to the lumen of the damaged
Palpate vessel
Pulsation transmitted from the artery tends
Describe the pulsatile swelling underneath to increase the size of the cavity with time.
one of the scars
Fully describe all the characteristics of the (b) What are the causes of a false aneurysm?
swelling Traumatic
Define the anatomical position of the Iatrogenic
swelling, which is usually located at the
Case 126 Circulation and lymphatic systems 219

Following angiography, blood continues


to leak from the puncture site (easy to
repair with suture to arterial wall)
Following bypass, e.g. femoropopliteal, is
often associated with infection (more
complex and often need vein patch to
reconstruct).
Fusiform aneurysm
(c) What are the treatment options?
Ultrasound compression of the false
aneurysm
Thrombin injection
Surgical repair
Observation and review.

Saccular aneurysm

False aneurysm

A cavity in a haematoma
which connects with the
lumen of the artery
Figure 125 The types of aneurysm.

CASE 126 THORACIC OUTLET OBSTRUCTION *

appearance or worsening of these symptoms


INSTRUCTION
on exercise
Examine this ladys right arm. Inspect the hand especially carefully, with
the hand resting on a white pillow noting the
possible arterial complications of thoracic
APPROACH outlet syndrome:
Expose the patients arm and shoulder, also Patchy gangrene of the tips of the fingers
taking care to expose the contralateral arm. and palm
Fingertip necrosis
Continue by examining for wasting of the
VITAL POINTS
small muscles of the hand (T1 distribution), a
Look feature caused by the neurological deficit
from the obstruction.
Inspect the arm from the anterior and
posterior Palpation
Note the presence of oedema, cyanosis or
pallor due to reduced venous outflow from Palpate the neck, in thin people there may
the arm the patient may describe the be a bony swelling of the cervical rib above
the clavicle in the supraclavicular fossa
220 Circulation and lymphatic systems Case 126

Brachial
Scalenus plexus
medius
Groove for plexus
and
Scalenus subclavian
anterior artery

Clavicle

Groove for
subclavian vein
Figure 126 Anatomy of the first rib.

A pulsatile mass might be present (due to Arterial symptoms (fingertip gangrene, necrosis)
post-stenotic dilatation) are more commonly due to:
If there is any evidence of oedema, palpate Raynauds phenomenon (see Case 121)
this and note that it is characteristically Thromboangiitis obliterans (see Case 127)
pitting in nature
Takayasus arteritis
The radial pulse is usually present and
Venous symptoms (oedema, cyanosis or pallor
normal.
of the arm) may be caused by:
Axillary vein thrombosis
Auscultation Damage to axillary drainage following
surgery (such as axillary dissection in breast
There might be a bruit over the subclavian surgery)
artery (Fig. 126). Neurological symptoms may be due to:
Cervical spondylosis
Sensation Pancoasts tumour
Cervical disc protrusions
Test sensation in the dermatomes of the arm
Ulnar nerve neuropathy.
specifically there may be sensory loss over
the T1 region, along the medial aspect of the (b) What investigations may help to confirm
arm around the elbow joint. the diagnosis?
There may be a cervical rib or prominent
Finish your examination here transverse process on the chest X-Ray or
thoracic outlet views
QUESTIONS Doppler examination may be useful in
quantifying the postural changes and
(a) What is the differential diagnosis of post-stenotic dilatation
thoracic outlet obstruction? Arteriograms of the subclavian artery may
show a marked kink in the artery or even the
This is often a difficult diagnosis to make
vein, and sometimes there is a localized
because the clinical signs are the result of a mix
aneurysm at the site of the narrowing.
of arterial, venous and neurological
complications of the obstruction.
Case 127 Circulation and lymphatic systems 221

(c) What is the pathogenesis of thoracic outlet


Mikito Takayasu (18601938) was a Japanese
obstruction? surgeon, describing an obliterative arteritis affecting
Congenital: the subclavian and carotid arteries of young Asian
Usually due to a cervical rib (arising from the women.
seventh cervical vertebra) and the subclavian Henry Khunrath Pancoast (18751939). Professor of
artery is compressed between the rib and Roentgenology, University of Pennsylvania,
either the scalenus anterior muscle or the Philadelphia, USA.
clavicle
Acquired:
The obstruction may also follow a fractured FURTHER READING
clavicle, hypertrophy of the scalene muscles,
or occasionally a pathological enlargement of Parziale JR, Akelman E, Weiss AP, et al:
the first rib. Thoracic outlet syndrome, Am J Orthop
29(5):353360, 2000.

CASE 127 THROMBOANGIITIS OBLITERANS


(BUERGERS DISEASE) *

INSTRUCTION QUESTIONS
Look at this mans legs and ask him some
questions. (a) What is the pathogenesis of
thromboangiitis obliterans?
It is a collagen vascular disease, caused by
APPROACH infiltrate of plasma cells into the arterial wall
Expose the patient and examine the legs as for This leads to luminal thrombosis and affects
any peripheral arterial case (see Case 111). small and medium-sized arteries of the lower
limb
Eventually, collagen is deposited and forms a
VITAL POINTS thick fibrous coat
Look Heavy smoking is very strongly associated
with this condition.
Nicotine staining of the fingers
(b) What specific investigations would you
The patient may complain of chronic
perform?
paronychia and early ulcers that heal poorly
Collagen antibodies are present in 45% of
There may be a history of intermittent
patients
claudication
There is an association with HLA-B5
Note the presence of distal gangrene and
other appearances of chronic ischaemia in Angiography has typical appearances of
the feet, and of erythema nodosum normal proximal vessels with distal occlusion
and corkscrew collaterals.
Patients often have multiple amputations.

Feel
Leo Buerger (18791943). North American urologist.
When examining the pulses, the typical pattern
is that the femoral and popliteal pulses are
present and the foot pulses are absent. FURTHER READING
Olin JW: Thromboangiitis obliterans (Buergers
Finish your examination here disease), N Engl J Med 343(12):864869, 2000.
222 Circulation and lymphatic systems Case 128

CASE 128 SUPERIOR VENA CAVA OBSTRUCTION *

INSTRUCTION ADVANCED QUESTIONS


Look at this ladys neck and tell me what the
(a) How can the extent of the obstruction be
problem is.
determined?
An intravenous injection of contrast into the
VITAL POINTS veins in the arm can illustrate the degree of
Look obstruction
A CT scan of the thorax may demonstrate
Note the tortuous, visible, dilated veins the cause of the obstruction and the length
overlying the chest wall and neck these of SVC affected.
veins would not be expected to be
compressible Johann Friedrich Horner (18311886). Professor of
The face may be plethoric and swollen Ophthalmology in Zurich who described Horners
Comment if the patient is dyspnoeic at rest. syndrome ipsilateral ptosis, miosis, hypohidrosis and
enophthalmos due to damage to the cervical
sympathetic chain.
Finish your examination here
William Harvey (15781657). Physician at St
Bartholomews Hospital, London and President of the
Royal College of Physicians. He gave the first account
Completion of the circulation of blood in his book De Motu Cordis
in 1628, although he first spoke of its existence in
Tell the examiner you would examine the patient 1616.
further to find a cause for the obstruction,
including looking for peripheral stigmata of lung
carcinoma (e.g. nicotine stains, digital clubbing
and Horners syndrome), lymphadenopathy and FURTHER READING
examining the chest.
Markman M: Diagnosis and management of
superior vena cava syndrome, Cleve Clin J Med
QUESTIONS 66(1):5961, 1999.

(a) What are the causes of superior vena cava


Notes
(SVC) obstruction?
Causes can be divided into pathology within There are two other conditions where dilated
and outside the SVC. Within the SVC veins can be observed across the trunk:
obstruction tends to be as a consequence of Inferior vena cava obstruction, where the
thrombosis within intravenous jugular or dilated veins occur across the lower
subclavian lines (CVP lines), especially when abdomen; the commonest cause is intra-
hyperosmolar solutions are infused for feeding. abdominal malignancy
Outside the superior vena cava (compression Caput medusa, dilated veins around a
from pathologies in adjacent structures): portosystemic anastomosis in the umbilical
Carcinoma of the lung veins.
Lymphoma The three causes of dilated abdominal wall
Carcinoma of the thyroid veins can be distinguished by the direction of
flow within the dilated veins. This is detected by
Aortic aneurysm
placing two fingers on the vein, sliding one
Mediastinal goitre
finger along the vein to empty it and then
Mediastinal fibrosis releasing one finger, watching to see which
Constrictive pericarditis. direction the empty segment fills (Harveys test).
Case 129 Circulation and lymphatic systems 223

In relation to the umbilicus: In caput medusa the direction of flow is


In SVC obstruction the direction of flow away from the umbilicus (both below and
above the umbilicus is downwards above).
In IVC obstruction the direction of flow below
the umbilicus is upwards

CASE 129 CAROTID ARTERY ANEURYSM AND


DILATED COMMON CAROTID ARTERY *

INSTRUCTION Completion
Examine this patients neck.
Say that you would like to:
Look for neurological associations (ipsilateral
APPROACH Horners syndrome and focal neurological
signs caused by embolization of the
Expose the patient and proceed as for the neck
aneurysm)
exam (see Case 6).
Examine for other cardiovascular
associations (measuring the blood pressure,
VITAL POINTS examining the peripheral pulses and heart).

Inspect
QUESTIONS
A pulsatile swelling can be noted in the line
(a) How would the patient be investigated?
of the carotid artery at the base of the neck
It is normally unilateral. Other risk factors and cardiovascular disease
elsewhere would be excluded and the neck
imaged with a duplex scan or occasionally an
Palpate intravenous digital subtraction angiogram.

The aneurysm is firm and expansile. (b) What is the cause of these aneurysms?
True aneurysms are uncommon and are
Auscultate generally caused by atherosclerosis, and
occasionally by dissection, trauma, previous
A bruit may be heard. carotid surgery or infection
When a true aneurysm has been excluded,
the patient can be reassured and discharged
Finish your examination here
Dilated, tortuous common carotid arteries
are much more common the artery is
kinked or coiled and there is a prominent
carotid bifurcation.
224 Circulation and lymphatic systems Case 131

CASE 130 LYMPHANGIOMA *

INSTRUCTION QUESTIONS
Examine this gentlemans neck.
(a) What is the origin of lymphangiomas?
Some 50% are present at birth and they are
APPROACH thought to represent a congenital abnormality
Begin to examine the neck as described in during the evolution of embryonic lymph nodes
Case 6. These are usually found in childhood into the adult type.
and rarely present in younger adults; they are
extremely rare in older adults. (b) How are they classified?
Lymphangiomas can be:
Cystic (cystic hygroma, as in this case for
VITAL POINTS further information, see Case 37)
Look Solid or diffuse may involve any part of the
body, usually present at birth; local
There is a swelling above the clavicle in the overgrowth of tissues and bone may occur
posterior triangle of the neck. which can render surgical correction
extremely difficult
Cutaneous (lymphangioma circumscriptum)
Feel present as groups of multiple small
transparent blisters lying close to each other.
The swelling feels soft and smooth They are usually not present at birth but
More solid areas may be palpable within the develop later. They tend to be cosmetically
mass more disfiguring and also ooze fluid or bleed
Characteristically brilliantly transilluminable frequently; early surgical treatment is
(because it is full of lymph) therefore warranted. An ellipse of skin and
The skin overlying the lump is normal. underlying subcutaneous tissue should be
excised.

Finish your examination here


FURTHER READING
Orvidas LJ, Kasperbauer JL: Pediatric
lymphangiomas of the head and neck, Ann Otol
Rhinol Laryngol 109(4):411421, 2000.

CASE 131 ARTERIOVENOUS FISTULA *

INSTRUCTION VITAL POINTS


Examine this gentlemans right wrist and tell Inspect
me the diagnosis.
There is a swelling over the distal forearm,
just proximal to the wrist joint
APPROACH
Describe this swelling as for any other lump
Expose the patients hands and place them (see Case 1)
palm upward on a white pillow if available The arteriovenous (AV) fistula may have been
Check that both hands and forearms are surgically created, i.e. a CiminoBrescia
exposed to compare one side with the other. fistula for haemodialysis in patients with
chronic renal failure (in which case there
Case 131 Circulation and lymphatic systems 225

should be a precise scar over the skin) or it should be avoided by the ligation of the
could be traumatic, or occasionally distal vein segment
congenital High-output cardiac failure secondary to
The lump may be pulsatile. massive run-off through the fistula
Pseudoaneurysm formation.
Palpate (c) How would you determine clinically the
degree of shunt caused by a large fistula?
Check that the patient does not have any
pain and then palpate the mass The BranhamNicoladoni sign indicates the
degree of shunting and cardiac impairment
There is also a thrill palpable.
resulting from a large AV fistula
The carotid pulse is palpated and then a
Auscultate tourniquet placed around the proximal
affected limb and inflated above systolic
The lump has a machinery murmur in systole. pressure
The pulse during the period when the
Finish your examination here tourniquet is inflated is compared with the
pulse beforehand
Normally an AV fistula causes a
hyperdynamic circulation sinus tachycardia
Completion may be present
When the fistula is cut off from the
Say that you would like to:
circulation, this is corrected and so the pulse
Examine the rest of the patient to try to will slow during the test
determine why the fistula had been formed in This indicates the presence of a left-to-right
the first place. shunt.

ADVANCED QUESTIONS
M. J. Brescia. Contemporary renal physician, VA
(a) How is a Cimino-Brescia arteriovenous Hospital, New York.
fistula fashioned? J. E. Cimino. Contemporary renal physician, VA
Hospital, New York.
The procedure can be performed under a
regional (brachial plexus), local or general H. H. Branham. Nineteenth-century North American
anaesthesia surgeon.
A longitudinal incision 34cm in length is
made over the distal third of the forearm
midway between the radial artery and the Other types of arteriovenous fistula
cephalic vein these are all rare
The cephalic vein is mobilized and tributaries
divided
(a) Congenital:
The radial artery is also identified and
dissected and a longitudinal venotomy and Most commonly occur in the head, neck and
parallel longitudinal arteriotomy performed limbs
Fine non-absorbable sutures are used to join They can lead to AV aneurysms
the two In the head, they most commonly involve the
The distal cephalic vein is ligated altogether. superficial temporal artery
If the overlying mucous membrane or skin
(b) What are the specific complications of a ulcerates, the fistula may haemorrhage
CiminoBrescia fistula? Small asymptomatic fistulae may be treated
Thrombosis during or just after expectantly or occasionally with therapeutic
haemodialysis, which may be due to relative embolization
hypotension and damage to the intima of the Surgical options include occlusion of the
vein feeding vessel and excision of the fistula and
Venous hypertension in the hand causes surrounding aneurysm if present, or
swelling and ischaemia of the fingertips. This radiological embolization.
226 Circulation and lymphatic systems Case 132

(b) Multiple arteriovenous fistulae (Parkes (c) Traumatic:


Weber syndrome) May follow a simultaneous partial laceration
These are almost always in the limbs and through a vein and artery lying in apposition
present with an overall increase in the size of Occur several days after the injury
the affected limb More common after open injuries
The limb has the appearances of extensive The patient may notice a thrill or buzzing
varicose veins The other situation where this may occur is
These are very often complicated by severe following cannulation of vessels by
lipodermatosclerosis and ulceration radiologists or cardiologists
Bruits and thrills may be present These fistulae normally need to be explored,
The BranhamNicoladoni sign is normally the vessels separated and the defect closed.
positive (see above)
Usually it is impossible to excise or embolize
each individual fistula unless they are all
derived from a single peripheral artery.

CASE 132 COARCTATION OF THE AORTA *

INSTRUCTION Examine for radio-femoral delay and examine


the precordium for an ejection systolic
Examine this ladys back and describe the murmur heard over the left sternal edge.
abnormalities you see.

QUESTIONS
APPROACH
(a) What is the pathophysiology of
Expose the patient to the waist, and ask her to
stand or sit forward on the side of the bed so
coarctation?
that you can examine the back adequately (see The aorta is narrowed below the origin of the
Case 99). left subclavian artery and therefore blood
flow to the abdomen and legs is reduced
The prominent vessels over the back are
VITAL POINTS large collaterals that have developed to
Look bypass the obstruction and supply the legs
The collaterals form between branches of the
Note the large, prominent, tortuous blood subclavian artery, especially the internal
vessels running over the left scapula mammary and scapular vessels, which feed
Palpate the vessels to demonstrate that they the intercostals from the third rib down.
are arteries
(b) What investigations would be helpful in
Listen to the vessels and confirm the
confirming the diagnosis?
presence of a systolic murmur.
Notching on the underside of the ribs may
be seen on a chest X-ray (CXR) this sign is
Finish your examination here caused by erosion by the intercostal
collateral vessels
On the CXR the aorta may be abnormal it
Completion contains two bulges the three sign
A barium swallow shows the opposite the
Say that you would like to: reverse three sign in the oesophagus
Compare the pulses in the arms and legs An echocardiogram shows the site of the
the upper limb pulses are much stronger coarctation and may demonstrate concurrent
than the leg pulses. The patient is usually aortic stenosis.
hypertensive
Case 133 Circulation and lymphatic systems 227

Surgical:
ADVANCED QUESTIONS
End-to-end anastomosis, patching and the
(a) What associations of coarctation are you use of the left subclavian artery as a flap are
aware of? all surgical options.

Coarctation may be associated with:


Bicuspid aortic valves FURTHER READING
Aortic stenosis McCrindle BW: Coarctation of the aorta, Curr
Aneurysms in the circle of Willis. Opin Cardiol 14(5):448452, 1999.

(b) What are the treatment options?


Non-surgical:
Investigation and treatment of concurrent
abnormalities (present in 50%)
Management of hypertension

CASE 133 ATRIAL FIBRILLATION *


Rate and rhythm (Table 32) should be
INSTRUCTION
ascertained from the radial pulse
Take this ladys pulse and comment on it. Character and volume are determined from
the carotid pulse
It is often easier to use the thumb to palpate
VITAL POINTS
the carotid pulse, but be careful to avoid
Ask for permission, and then take the giving the sensation of strangling the patient
patients right radial pulse and then a central by using the right thumb to take the pulse in
(carotid) pulse the left side of the neck.

Table 32 Assessment of rate, rhythm, character and volume of pulse


Rate Expressed in beats/min
Count for 15s and multiply beats by 4
Rhythm Regular or irregular
If irregular can be regularly or irregularly irregular
Regularly irregular:
Extrasystoles
Sinus arrhythmias (faster in inspiration)
Pulsus paradoxus (weaker in inspiration)
Pulses alternans (alternating weak and strong beats)
Irregularly irregular:
Atrial fibrillation (AF)
Character Rate of increase and decrease of the pressure within the wave of the pulse
Collapsing pulse (waterhammer pulse) steep rise then rapid fall is
characteristic of aortic regurgitation
Anacrotic pulse a slow rise and a slow fall in aortic stenosis, as the
normal dicrotic notch is lost
Volume The expansion of the artery with each beat is palpated in the carotid artery
High cardiac output leads to a strong pulse
A patient with severe blood loss and shock has a thready pulse
228 Circulation and lymphatic systems Case 133

Finish your examination here (b) What are the complications of


atrial fibrillation?
The major risk is of embolic stroke (4%
Completion per year), which results from thrombus
accumulating in an inefficiently contracting
Say that you would like to: left atrium
Emboli can also lodge in the mesenteric
Continue to examine the rest of the
vessels, causing intestinal ischaemia
cardiovascular system and look especially
for complications of atrial fibrillation (see Patients are also at risk from acute limb
below). ischaemia if emboli lodge in the arteries of
the leg.

QUESTIONS (c) What are the surgical problems associated


with atrial fibrillation?
(a) What are the causes of atrial fibrillation? Anaesthesia is more complicated because of
Cardiac disease: the increased risk of stroke
Hypertension In addition, patients with AF may be
anticoagulated and if on warfarin, this
Myocardial infarction, ischaemia
medication needs to be discontinued prior to
Mitral valve disease elective surgery
Cardiomyopathy Patients with controlled AF may
Endocarditis decompensate following the stress of
Respiratory disease: surgery and in the most severe cases, this
Pneumonia can lead to hypotension
Lung cancer Finally, the underlying cause for the AF, such
Sarcoidosis as ischaemic heart disease, is still present
and may contribute further to anaesthetic
Other:
risk.
Hyperthyroidism
Idiopathic (lone AF) where it is not due to
any of the causes listed above. FURTHER READING
Falk RH: Atrial fibrillation, N Engl J Med
344(14):10671078, 2001.
5
SECTION

COMMUNICATION SKILLS
WITH THOMAS CROMPTON

134 Introduction to communication skills *** 230


135 Information gathering back pain *** 231
136 Information gathering trauma call *** 232
137 Information gathering vascular referral *** 232
138 Information giving obtaining informed consent *** 233
139 Information giving breast carcinoma *** 234
140 Information giving testicular tumour *** 235
141 Information giving loss of function *** 236
142 Information giving the angry patient *** 237
230 Communication skills Case 134

CASE 134 INTRODUCTION TO COMMUNICATION


SKILLS ***

Specific stations may require the candidate to:


INTRODUCTION
1. Communicate with a patient or relative in
To communicate effectively you must: typical clinical situations. For example:
Listen to, ask for and respect patients views Explaining a diagnosis
about their health and respond to concerns Explaining investigations
and preferences
Conveying bad news
Share with patients, in a way they can
Obtaining informed consent
understand, the information they want or
need to know about their condition, its likely Explaining the uncertainties of diagnosis,
progression, and the treatment options outcome or prognosis.
available to them, including risks and 2. Take a focused medical history in a variety
uncertainties of clinical situations, for example outpatients
Answer questions and keep patients or Accident and Emergency (A&E)
informed about the progress of their care 3. Demonstrate an ability to convey information
Make sure that patients are informed about to colleagues and other healthcare
how information is shared with other professionals in an appropriate manner, to a
professionals involved in their care. satisfactory standard and using a variety of
methods. These might include:
Although communication skills are examined at
all levels in surgery, we have included a section Verbal communication such as case
here on what is expected of the candidate in presentations to colleagues
the MRCS examination. Written communications such as referral
or patient transfer letters
Investigation request forms
FORMAT OF MRCS EXAM Telephone communication.
Communication skills will be formally assessed
at three of the 16 OSCE stations in the MRCS
part B examination. Two manned stations
PREPARATION
examine skills that include talking with Read the case through carefully and make sure
colleagues, relatives and carers, making a you know how long you have to prepare. Paper
telephone call to a consultant and the handover and pens will be provided to make notes when
of a patient. One unmanned station tests written required.
communication skills, such as preparing a Some general points on the cases:
discharge letter from a patients records.
1. Do you know the patient/relative, e.g. is it
Usually actors replace patients and are someone you see every day on the ward
generally briefed thoroughly and carefully round or a new patient in clinic?
they will know the scenario much better than 2. Classify the problems into medical and
you will! psychosocial
One examiner may be a trained lay examiner 3. Deal with each separately
who is paired with a surgical examiner
4. What is the patients frame of mind likely to
Of the six domains being assessed in the be (e.g. angry/anxious/bereaved) this is
MRCS, the communication skills stations will best assessed by asking yourself the
award marks in the following three domains: question how would I feel if I was the
Communication 8 patient?
Decision-making, problem-solving and 5. Plan your conversation.
judgement 4
Organization and planning 4.
Clinical knowledge will not be specifically THE CONSULTATION
examined but beware, as obvious errors or Points are awarded for the approach to the
giving the wrong clinical information will result actor or patient and you should either introduce
in a fail. yourself formally, if you have never met the
Case 135 Communication skills 231

patient before, or remind the patient who you Finally, the actor usually has an input into
are if you have met frequently. the marks you are awarded for the case so
Diagrams often help patient understanding make sure you establish a good rapport with
them.
Avoid any medical jargon (like neck of femur
fracture)

CASE 135 INFORMATION GATHERING


BACK PAIN ***

SCENARIO Complete the medical history focusing on


surgically relevant information such as past
You are an orthopaedic core trainee on-call surgical history, medical conditions affecting
at a district general hospital. Your registrar fitness for surgery and medications that
has asked you to see a referral from a general may affect or contraindicate surgery (e.g.
practitioner a 43-year-old male mechanic with anticoagulants).
a long history of lower back pain and a recent
At the end, the examiners will most likely ask
exacerbation. On this occasion, he felt as if he
you for a plan. You suspect cauda equina
pulled a muscle in his back when lifting a heavy
syndrome. You would carry out a full
car part at work and he now has lower back
neurological examination and act on the
pain radiating down both legs. He has not
outcome. If your examination findings could not
passed more than a dribble of urine, despite
rule out a cauda equina, the patient requires
feeling the need to go. Take a history from him
radiological investigation. MRI scanning is the
and formulate a plan.
gold standard following standard radiography
and discussion with your local neurosurgical
KEY POINTS centre urgently is indicated. Dont forget to
involve your senior colleagues as early as
Medical possible.

Take a full history as you normally would in any


medical consultation beginning with age and
Psychological
occupation. You need to confirm your suspicion
of cauda equina syndrome (CES) by taking a You need to keep the patient informed. The
complete pain history, enquiring about urinary diagnosis you suspect will probably be a shock
symptoms, bowels symptoms and altered to the patient. There is an element of the giving
sensation in the perineum (e.g. you can ask a difficult diagnosis scenario that may also
if toilet paper feels different on wiping). come up in this consultation. The patient may
well ask what you think the diagnosis is and to
Although the history is not typical, you must tell them the possible outcomes. You may also
screen for the more rare causes of CES or other need to enquire as to the support, e.g. family
sinister causes of back pain. This will show your members that the patient has access to, and to
knowledge to the examiner. Enquire about involve any allied health professionals, e.g.
weight loss, lethargy, fevers, night sweats, senior nursing staff, as early as possible.
appetite, etc. to screen for spinal tumours,
metastases or spinal infections leading to
compression of the cauda equina.
232 Communication skills Case 137

CASE 136 INFORMATION GATHERING


TRAUMA CALL ***

SCENARIO nurses. Comment that he is now in a safe area


with a team of doctors and nurses looking after
You are an orthopaedic core trainee in a large him.
district general hospital. It is 7 p.m. and a
Give her the facts without any emotional
14-year-old boy has been brought in by the
comments. He has a head injury and is awaiting
ambulance service having been hit by a car.
a CT scan of his head, neck, chest, abdomen
He was a pedestrian and had bulls-eyed the
and pelvis. Explain this is needed to rule out
windscreen. He was combative at the scene
other life-threatening injuries and to define the
and so was intubated and ventilated. He has an
head injury further. He has an open tibial
obvious head injury and an open tibial fracture.
fracture that will require an operation tonight to
The primary and secondary surveys have been
prevent infection and for stabilization.
completed and no further injuries have been
found. He is awaiting a CT scan of his head, For his treatment you need to take a focused
cervical spine, chest and pelvis. The registrar trauma history and not a full medical history
is running the trauma call with the only nurse from birth to the present day. The AMPLE
available and asks you to speak to the mother history described as part of the ATLS protocol
who witnessed the event. The mother is is a useful guide.
obviously distressed and is sitting in the A Allergies
relatives room attached to A&E. M Medications
Speak to the mother giving her any information P Past medical history
you can about her son while obtaining the L Last meal
important information necessary for the boys E Events/Environment related to injury.
care. Once you have the information, you will
Once you have the information, you need to tell
present the key points back to your registrar.
the mother that you will be back to update her
after the scan results are available. You will then
KEY POINTS be able to give her much more information
about his injuries.
As you have been told in the scenario, the
Check her understanding and ask if she has any
mother is going to be distressed having
questions before presenting your information to
witnessed the accident. You will have to
the examiner/registrar. It may be worth asking if
comfort and reassure her as much as possible
she wants someone to stay with her, e.g. a
while getting the information you need.
nurse, so that she is not left alone in the room.
Find out what she already knows about her
sons injuries from the ambulance crew or

CASE 137 INFORMATION GATHERING


VASCULAR REFERRAL ***

SCENARIO referral letter to the local vascular consultant. In


the first station you will take the history from the
You are a general surgical core trainee in a busy patient, using notepaper as necessary, and then
clinic. The consultant has just seen a patient in the second station you will write the referral
with bilateral leg pain, referred by a GP. With letter to the vascular surgeon. You change jobs
further questioning it has become obvious that in a week and will be working for the vascular
the pain is vascular in nature. The consultant surgeon to whom you are referring.
has asked you to take a history and then write a
Case 138 Communication skills 233

PART 1 HISTORY TAKING Finally your details and contact information


should be included, and remember to copy
Key points the letter to the GP and the patient.

Name, age, sex and occupation General Surgical Consultant


General Surgical clinic
Initial symptoms, time of first presentation
St Elsewhere Hospital
and subsequent progression or regression of
London E1
symptoms are important
25th December 2012
The exact site of pain, walking distance at
which it develops, maximum walking Vascular Consultant
distance and time for pain to resolve on Another Hospital
resting need to be noted London
Direct questioning must exclude rest pain in E2
the limbs and other cardiovascular disease
The effect of the symptoms on the patients Dear Mr Vascular
work, life and hobbies need to be addressed RE: Mr SMITH GP: Dr GP
Take a full medical history to try and glean Acacia Avenue Another Road
the aetiology of the peripheral vascular London Anytown
disease including smoking, ischaemic heart K1 Z1
disease, diabetes, or any family history of NHS no. 987 654
vascular disease. 3421
Tel: 01234 567891 Tel: 01234
987654
PART 2 REFERRAL LETTER
This is a formal letter, which must include the This 64-year-old retired teacher presented to Mr
name and address of referring hospital, Abdomens clinic this morning having been
preferably on headed paper, with details of referred from his GP with bilateral leg pain
the referring consultant clinic, the name and etc. etc.
address of the receiving consultant and the date Yours sincerely
of referral.
Other details include the patients contact Mr Newly Qualified Surgeon
details including address and telephone, and cc. GP
the GP name, address and telephone cc. Patient
A concise letter detailing all relevant
information from your history should follow
the above
Finish with a polite request to review the
patient in clinic

CASE 138 INFORMATION GIVING OBTAINING


INFORMED CONSENT ***

SCENARIO You have examined Mrs Smith and taken a full


history. Examination is otherwise unremarkable
You are a registrar in general surgery and the but she states that her voice has changed in
consultant has asked you to consent Mrs Smith, recent weeks.
a 40-year-old singer, for a total thyroidectomy,
Obtain informed consent from this patient for a
as you have assisted with the operation on
total thyroidectomy. A generic consent form is
several occasions recently. The patient is aware
provided and you have a 5-minute preparation
of the diagnosis of thyroid cancer and the need
station to fill in the form including all the
for an operation.
patients details.
234 Communication skills Case 139

KEY POINTS COMPLICATIONS OF


Remember you have not met this patient before THYROIDECTOMY
so you must review their understanding of the
diagnosis first and explain the indications for
General
operation.
Early
Explain the procedure at an appropriate level
Bleeding/haematoma
for this patient, including where the incision
will be. Use diagrams if necessary to ensure Infection
understanding Scar
Mention the specific complications and the Anaesthetic risk
more general risks of surgery and ask about Late
previous anaesthetic problems. Any Hypertrophic scarring.
complication of greater than 1% must be
mentioned and dont forget to get Mrs Smith
to sign a consent form after you have
Specific
answered all her questions
Respiratory obstruction: oedema, bleeding or
If you are unsure of any points, you may
recurrent laryngeal nerve palsy
need to defer to your consultant say that
you will speak to him and get back to her. External laryngeal nerve damage: voice
Remember that your consultant is ultimately change preoperative cord inspection is
responsible for the care of this patient. essential
Hypocalcaemia due to deliberate or
inadvertent removal of all parathyroid tissue
TOP TIP Hypothyroidism and need for medication.
Complications
Complications from any surgical procedure can be
divided into early (days or weeks) and late (months)
Complications are also divided into general to any
operative procedure and specific to the operation in
question. This gives a framework on which to base
your discussion of complications from any procedure.

CASE 139 INFORMATION GIVING BREAST


CARCINOMA ***

SCENARIO KEY POINTS


You are a general surgery registrar in a breast Introduce yourself formally, as you have not met
clinic. A 52-year-old housewife is attending the patient before.
following a routine mammography showing
calcification consistent with ductal carcinoma Medical
in situ (DCIS) in the outer half of the right
breast. You have taken a history and performed Explain the mammography and clinical
both a general examination and clinical examination findings to the patient in appropriate
examination of the breasts, which are non-medical language. Explain carcinoma in situ
unremarkable. Explain the results to the patient as opposed to breast carcinoma/cancer (atypical
and inform her of the plan. cells in ducts but no invasion outside ducts as
Case 140 Communication skills 235

yet) it is essentially a pre-malignant condition Psychosocial


in which a proportion, if left untreated, will go on
to develop invasive cancer. Allow time for active listening to the patients
Explore her understanding of the diagnosis concerns, especially the shock of the suspected
and explain in simple terms what will happen diagnosis and the degree of uncertainty at this
next stage.
Knowledge of triple assessment is necessary There will be worries about treatment, as
Make sure the patient understands the need patients have preconceived ideas about
for biopsy and explain that the options for chemotherapy and/or radiotherapy. She may
surgery will depend on the biopsy results. know of other patients who have gone
She will probably require wide local excision through similar experiences
and radiotherapy but this will depend on the Give her the opportunity to return with her
histology family to discuss treatment and prognosis
If asked about prognosis, you must not give and offer her support and continuity of care.
specific information until you have the Let your consultant know, should she return
necessary results. You need to wait for when you are not in clinic
biopsy results and also the surgical histology It is often best to get other allied health
results with clearance margins and lymph professionals such as senior nurses involved
node involvement before an accurate in the patients support network as early as
prognosis can be given. possible. Give her any information leaflets
available and hand over to the Macmillan
nurse, if available, in clinic for further
discussion about the diagnosis.

CASE 140 INFORMATION GIVING TESTICULAR


TUMOUR ***

SCENARIO Medical
You are a urology core trainee in clinic. You
Explain that the ultrasound results and clinical
have just received a phone call from the
examination suggest that cancer is the
radiologist about a patient you sent for an
diagnosis. Give the news early in the
urgent testicular ultrasound scan. He is 30 years
consultation and use the term cancer not
old and newly married with no children. He
lesion or lump, as this leads to confusion.
presented with a painless swelling of the testicle
Explore the patients understanding of the
after much persuasion from his wife. The
diagnosis and explain in simple terms what will
radiologist confirms the results are consistent
happen next.
with a tumour as you suspected from your
clinical assessment. The scan has not been Make sure the patient understands the need for
discussed with the patient who is coming early surgery and also that surgery is partly
straight to clinic. He is anxious and suspecting diagnostic. He will need to have urgent blood
bad news. You are due to assist in theatre in 10 tests and a staging CT scan to assess
minutes time. prognosis. Sperm banking should also be
discussed at this stage.

KEY POINTS Psychosocial


No need for a formal introduction as you
sent him for the scan Ensure you will not be interrupted for your
Remember to phone the surgeon you are consultation by turning your pager off and
due to assist to avoid interruptions during informing the clinic nurse.
your consultation when theatres try to Allow time for active listening to the patients
contact you if your consultation is prolonged. concerns and consider the shock of the
236 Communication skills Case 141

suspected diagnosis but also a degree of You must offer the chance to return with his
uncertainty at this stage. He will have worries wife to discuss treatment and prognosis.
about treatment. Depending on surgical Offer support, continuity of care and any
results, he may require chemotherapy or less information leaflets you have available
likely radiotherapy If you have to rush away at the end, ensure
It is difficult to give any useful prognosis at that another health professional, e.g. clinic
this stage, as it will depend on the staging nurse, stays with him to explore any further
scan and histology following surgery issues.

CASE 141 INFORMATION GIVING LOSS


OF FUNCTION ***

SCENARIO irreversible loss of function. He cannot have a


femoropopliteal bypass operation as there are
You are a vascular surgery registrar. A week no distal vessels.
ago, the team admitted a 55-year-old policeman
Use a diagram if necessary to explain what
with an ischaemic right limb. He smokes 15
has happened to his leg
cigarettes per day and has insulin dependent
diabetes. An angiogram revealed an occluded You must be able to explain how to perform
right superficial femoral artery, with reasonable a below-knee amputation (BKA) and the
collaterals, but no distal runoff and no vessels reasons why this is a much better an option
seen at the ankle. An ECG, echocardiogram and for him than an above knee amputation
carotid duplex scan were all within normal You must be able to list the complications of
limits. He underwent an embolectomy 7 days BKA if he asks
previously, at which you assisted. Unfortunately, You must also be prepared to discuss ways
his leg is no better, there is now gangrene in which his life afterwards can be
affecting his toes, the foot is insensate and he normalized.
is unable to move his right ankle.
You have to explain that he needs a below-knee Psychosocial
amputation.
He is initially going to need a wheelchair at
home, so an occupational therapy
KEY POINTS assessment is required
You have met the patient before and He lives with his wife, so does have
therefore you should establish early on what someone at home with him and you should
he already understands about his leg and ask if he would like you to get his wife in to
what he has had done already. This case is discuss all of this with both of them together
about loss of function Empathy is key to the success of your
Find out what sort of work he does as a approach with this scenario and you should
policeman, as he may have to modify or try to understand the psychological
change his job consequences to this active man of losing
He may already expect the worst, as he has his leg
not been able to move his leg since the The use of a prosthetic limb will be crucial to
operation. his rehabilitation and this should be
mentioned. He will be able to drive a car and
Medical walk eventually, although he will initially need
assistance
You must be clear in your own mind about the Remember to enlist the assistance of allied
necessity for amputation here. His foot is health professionals such as prosthetists and
unviable and he has a fixed ankle, indicating senior nursing staff.
Case 142 Communication skills 237

CASE 142 INFORMATION GIVING THE ANGRY


PATIENT ***

SCENARIO Apologize for the cancellation and explain


the reasons of clinical priority. Explain how
You are an orthopaedic registrar in a large the operations are prioritized, as this may
trauma centre. At the end of a 12-hour on-call calm the situation. All patients will get their
shift, you go to the bedside of a 28-year-old operation as soon as space is available on
barrister with a closed ankle fracture whose the list available and it will not change her
operation has been cancelled for the third day long-term outcome from the operation
running. The patient was admitted on Saturday Avoid blaming anyone for what has
morning and it is now Monday of a Bank happened as this undermines
Holiday weekend. professionalism
She was seen by one of your colleagues this Offer analgesia early, as being comfortable
morning, who she states, promised the may change her perception of the situation
operation would definitely be today. Since then Give her a chance to vent her anger before
there have been two major trauma calls both trying to interject. You could apologize on
requiring emergency surgery. The nurses have behalf of your colleague but explain he had
been busy so she has not had analgesia for 6h good intentions trying to allay her anxiety
and she has spent much of the day off the ward and could not predict the arrival of major
with her leg not elevated to use her mobile traumas
phone and smoke. She has missed crucial
Try to move the discussion on to focus on
meetings with clients and wants to make a
her treatment and offer advice regarding
complaint.
elevation of her leg to reduce swelling and
You have to explain the reasons for cancellation prevent wound problems postoperatively.
(more urgent trauma cases and only limited Although obviously sensible advice, it is
capacity at the weekend), while advising her probably best to avoid advice against
about elevation of the leg. smoking at this time, as it is likely to inflame
the situation with no immediate benefit.

KEY POINTS
Psychological
Reading the case, it is clear that the
communication scenario is to deal with the
Be understanding by trying to put yourself
angry patient. This is difficult and must be
into this ladys shoes
planned carefully.
Expect that she will be angry, even
unreasonable, but that your reaction would
Medical be similar
Enlist the help of allied health professionals
If possible, find somewhere private to talk with such as senior nursing staff as she may need
a nursing colleague present and ensure you are someone to talk to following your departure
not interrupted. from the ward.
Begin by introducing yourself she has met
a lot of different doctors and it is important
she knows your role as the on-call surgeon
This page intentionally left blank
Index 239

INDEX

Notes
vs. indicates a comparison or differential diagnosis

Abdomen, 63120 Anaplastic thyroid cancer, 21


neurofibroma, 44 Anastomosis
Abdominal aortic aneurysms, 203205 direct, lymphoedema, 216
infections, 205 end-to-end, coarctation of the aorta, 227
repair, 204 Aneurysms
screening, 205 abdominal aortic see Abdominal aortic aneurysms
Abdominal examination, 6870 false, 218219, 219f
auscultation, 70, 70f false vs. true, 218
diabetic foot, 198 fusiform, 219f
exposure, 69 saccular, 219f
inspection, 68b true, 218
mouth, 116 Angiography
palpation, 6970 chemodectoma, 59
patient exposure, 68b diabetic foot, 200
percussion, 70 thromboangiitis obliterans (Buergers disease), 221
peripheral signs, 6869 Angiolipomas, 6
varicose veins, 190 Angry patient, 237
Abduction, shoulder, 160161, 161f Angular stomatitis, 116
Abductor pollicis brevis muscles, 140f Ankle brachial pressure index, 198
Abductor pollicis longus muscles, 158f diabetic foot, 200
ABO compatibility, kidney transplants, 92 Ankle swelling, varicose veins, 188
Acanthosis, seborrhoeic keratosis, 47 Antalgic gait, 166t
Acanthosis nigricans, 116 hip osteoarthritis, 124
Achalasia, 9899 Anterior cervical chain of lymph nodes, 12f
Achondroplasia, 185186 Anterior cruciate ligament (ACL)
Acne inversa, 55 anatomy, 156
Acral lentiginous melanoma, 30 injuries, 156
Addisons disease, 116 rupture therapy, 157
Addison, Thomas, 117 tests, 155
Adductor pollicis muscles, 146f Anterior drawer, knee osteoarthritis, 134, 134f
Adenocarcinomas, cervical lymphadenopathy, 14 Anterior interosseous nerve, 140f
Adenoid-cystic carcinoma, salivary glands, 43 Anterior interosseous syndrome, 141
Adenomas, solitary thyroid nodule, 20 Anterior tibial artery, 195f
Adults, umbilical/paraumbilical hernia, 8081 Anterior triangle, neck examination, 9, 9f
Age, surgical jaundice, 71 Antibodies, Sjgrens syndrome, 42
Albert, Jean Louis, 27 Antithyroid drugs, Graves disease, 24
Albright, Fuller, 110 Aorta, 195f
Alpha feto-protein, gynaecomastia, 109 abdominal aneurysms see Abdominal aortic
Alveolitis, fibrosing, 48 aneurysms
Amoebiasis, pyoderma granulosum vs., 60 coarctation of, 226227
Amputations, 200201 stenosis, 227
below-knee, 200 Aphthous ulcers, 116
complications, 201 Apparent leg length, hip osteoarthritis, 124125,
Dupuytrens contracture, 138 124f125f
indications, 201 Areola, 104
Raynauds phenomenon, 213 Arterial bruit, hepatomegaly, 77
Anaesthesia Arterial disease, ischaemic ulcers, 209
atrial fibrillation, 228 Arteries
breast surgery, 107 legs, 195f
Anakinra, rheumatoid hands treatment, 143 radiotherapy side effects, 53
Analgesia spermatic cord, 66
frozen shoulder, 164 thyroid gland, 19, 20f
hip osteoarthritis, 129 Arteriograms, thoracic outlet obstruction, 220
ischaemic ulcers, 209 Arteriovenous fistulae, 224226
knee osteoarthritis treatment, 136 CiminoBrescia, 224225
Analgesic ladder, ischaemic ulcers, 209, 209t congenital, 225226
240 Index

neuropathic ulcers, 215 Baumgartner, W, 78


right iliac fossa mass, 90 Becks triad, renal cell carcinoma, 101
traumatic, 226 BEDD mnemonic, lumps and ulcers, 5b
Arthritis Bed rest, pressure sores, 35
degenerative, ulnar nerve palsy, 147 Behet, Hulusi, 60, 117
hallux valgus, 149 Behets disease
see also Osteoarthritis; Rheumatoid arthritis pyoderma granulosum vs., 60
Arthrodesis ulceration, 116
Dupuytrens contracture, 138 Bell, Charles, 39, 181
hip osteoarthritis, 129 Bells palsy, 38
knee osteoarthritis treatment, 136 Below-knee amputation, 200
Arthroplasty Beta-blockers, Graves disease, 24
hand osteoarthritis, 145 Biceps, long head, 160
hip osteoarthritis, 129 Bicuspid aortic valves, coarctation of the aorta, 227
Arthroscopic debridement Bilateral ptosis, 37
knee osteoarthritis treatment, 136 Bilateral subtotal thyroidectomy, 25
rotator cuff, 164 Bilirubin, surgical jaundice, 72, 73f
Arthroscopic rotator cuff repair, 164 Blepharoplasty, 37
Ascites, 9395 Block dissection of neck, cervical lymphadenopathy,
fluid thrill, 94, 94f 14
frank dullness, 94 Blood count, full see Full blood count
shifting dullness, 94, 94f Blood cultures, enterocutaneous fistula, 115
shunts, 95 Blood film, splenectomy, 83
Ascitic taps, 95, 95t Blood tests
Aspirin, ischaemic ulcers, 209 atherosclerosis, 206
Asymmetry cervical lymphadenopathy, 13
face, salivary gland swellings, 40 enterocutaneous fistula, 115
winging of scapula, 181 hepatomegaly, 77
Atherosclerosis, carotid artery disease, 206 hip osteoarthritis, 129
Atrial fibrillation, 227228 inflammatory bowel disease, 84
Atrial myxoma, digital clubbing, 48 rheumatoid hands, 143
Auscultation Bone, rotator cuff impingement, 163
abdominal examination, 70, 70f Bone lesions, shoulder pain, 163
inflammatory bowel disease, 84 Bonneys blue dye, branchial cyst, 50
pleural effusion, 97 Bouchard, C J, 145
varicose veins, 190 Bouchards nodes
Autonomic neuropathy, neuropathic ulcers, 215 hand osteoarthritis, 144
Axillary vein thrombosis, thoracic outlet obstruction vs., surgery, 145
220 Boutonniere deformity, 142
mallet finger, 153
Back pain, information gathering, 231 Bowen, J T, 29
BADCaT acronym, Raynauds phenomenon, 213b Brachial plexus
Bakers cyst, 168 branches, 177, 177t
popliteal fossa swelling vs., 168, 168f nerve organization, 176f
Baker, William Morrant, 168 Brachial plexus lesions, 175178
Balloting, enlarged kidneys, 100f examination, 175
Bancroft, Joseph, 217 inspection, 175176
BannayanZonana syndrome, 6 management, 178
Barium enema, enterocutaneous fistula, 115 power testing, 176
Barium swallow pre- vs. post-ganglionic injuries, 177
coarctation of the aorta, 226 prognosis, 178
dysphagia, 99 reflexes, 177
enterocutaneous fistula, 115 tone, 176
pharyngeal pouch, 57 Brachioradialis muscle, 158f
Barretts oesophagus, 98 Branchial cyst, 10f, 4950
Barr, Yvonne, 14 Branham, H H, 225
Basal cell carcinoma (BCC), 3234 BranhamNicholson sign, 225
cicatricial, 33 BranhamNicoladoni sign, 226
cystic, 33 Breast(s), lymph nodes, 104f
dermatofibroma vs., 54 Breast cancer
nodular, 33 information giving, 234235
pigmented see Pigmented basal cell carcinoma male, 109110
sclerosing, 33 Breast examination, 103104
superficial, 33 cervical lymphadenopathy, 13
Index 241

Breast implants, 108t Carpal tunnel decompression, 141


Breast lumps, 105106 complications, 141
classification, 105 Carpal tunnel syndrome, 139142
Breast reconstruction, 107108 causes, 140141
types, 108 investigations, 141
Breast surgery motor assessment, 139
drains, 107 test light touch, 139
patient preparation, 107 Carpometacarpal joint (CMCJ), hand osteoarthritis, 144
Brescia, M J, 225 Casts, 170171
Breslows thickness, 30, 31f, 31t, 32 incomplete, 170
Broad-based gait, 166t newer materials, 170171
Bronchial carcinoma, digital clubbing, 48 tight, 170
Brown-Kelly, A, 99 Cavernous haemangioma, 61
Bruising, inguinal hernia repair, 67 Cervical disc protrusions, thoracic outlet obstruction
Bruits, varicose veins, 190 vs., 220
Buccinator muscle, facial nerve palsy, 38 Cervical lymphadenopathy, 1214
Buerger, Leo, 198, 221 Cervical lymph nodes, 1011, 12f
Buergers disease see Thromboangiitis obliterans thyroid examination, 16f
(Buergers disease) Cervical spondylosis, thoracic outlet obstruction vs., 220
Buffalo hump, lymphoedema, 215 Cervical sympathectomy
Bulge test, knee osteoarthritis, 132, 132f hyperhidrosis, 217
Bunion, hallux valgus, 148 Raynauds phenomenon, 213
Bunionectomy, 149 side effects, 217
Bunnell, W W, 14 Charcot, Jean-Martin, 152, 180
Burgess long posterior flap, below-knee amputation, CharcotMarieTooth syndrome, 152
200 Charcots joints, 180
Bursae diabetic foot, 199
popliteal fossa swelling vs., 167 Charles procedure, lymphoedema, 216
rotator cuff impingement, 163 Chemodectoma, 5859
Chemonucleolysis, chymopapain, 174
Caf-au-lait spots, neurofibromatosis, 44 Chemosis, thyroid examination, 18
Calcification, sebaceous cysts, 7 Chest drains, pleural effusion, 98
Campbell de Morgans spots, 61 Cheviers tap sign, 190
Cancer see Malignancies CHIASMA acronym
Candidiasis, abdominal disease, 116 hepatomegaly, 77b
Capillary refill, 196, 196f splenomegaly, 82
Caput medusa, superior vena cava obstruction, 222 Children, umbilical/paraumbilical hernia, 80
Carbuncle, 59 Cholesterol test, atherosclerosis, 206
Carcinogens, basal cell carcinoma, 33 Chondroma, 179
Carcinomas Choriocarcinoma, testicular tumours, 113
neuropathic ulcers, 214 Chronic cutaneous ulceration, squamous cell
oesophagus, 9899 carcinoma, 29
see also specific types Chymopapain chemonucleolysis, lumbar disc
Cardiac disease herniation, 174
atrial fibrillation, 228 Cicatricial basal cell carcinoma, 33
digital clubbing, 48 Cicatrix, umbilical/paraumbilical hernia, 80
Pagets disease of bone, 184 CiminoBrescia arteriovenous fistula, 224225
radiotherapy side effects, 53 Cimino, J E, 225
rheumatoid arthritis, 143 Circular frames, external fixators, 182
Cardiac failure, CiminoBrescia arteriovenous fistula, Circulation system, 187228
225 Cirrhosis, liver, 48
Cardiopulmonary bypass (CPB), 111112, 112f Clarkes test, 131
complications, 112 Clarks levels of invasion, malignant melanoma, 30,
Carotid angiography, carotid artery disease, 206 31f, 31t, 32
Carotid artery aneurysm, 223 Claudication, intermittent see Intermittent claudication
Carotid artery disease (CAD), 205207 Claw hand
atherosclerosis, 206 Klumpkes palsy, 176
carotid bruit, 205206 ulnar nerve lesions, 145
Carotid bruit, carotid artery disease, 205206 Claw toes, 150f, 151152
Carotid chemodectoma, 10f Clotting
Carotid duplex scan, carotid artery disease, 206 hepatomegaly, 77
Carotid endarterectomy, 206207, 206f obstructive jaundice, 72
patient preparation, 207 CMCJ (carpometacarpal joint), hand osteoarthritis, 144
Carotid pulse, CiminoBrescia arteriovenous fistula, 225 Coarctation of the aorta, 226227
242 Index

Cock, Edward, 7 Cushing, Harvey Williams, 117


Cocks peculiar tumour, 7 Cutaneous lymphangioma, 224
Collateral ligaments Cutaneous ulceration, squamous cell carcinoma, 29
knee osteoarthritis, 135, 135f CVP lines, superior vena cava obstruction, 222
tests, 155 Cystic basal cell carcinoma, 33
Colostomy, 75t Cystic hygroma, 10f, 5758, 224
Colour changes, peripheral arterial system examination, Cystic lymphangioma, 224
195 Cysts
Common iliac artery, 195f dermoid, 10f, 5051
Communication skills, 229237 epidermal, 7
consultation, 230231 epididymal, 8889
information gathering see Information gathering simple, enlarged kidneys, 101
listening, 230 thyroglossal see Thyroglossal cysts
MRCS exam, 230
patient information, 230 Dalrymple, J, 18
preparation, 230 DEAFEST PAIL mnemonic, Dupuytrens contracture, 138
question answering, 230 Deafness, neurofibroma, 44
Complete casts, 170 Debridement
Compressibility, lumps and ulcers, 4 diabetic foot, 200
Compression bandaging, venous ulcer, 194 knee rheumatoid arthritis, 137
Compression stockings pressure sore therapy, 35
lymphoedema, 216 Debulking, lymphoedema, 216
varicose vein treatment, 192 Decision making, MRCS exam, 230
Computed tomography (CT) Deep inferior epigastric perforator (DIEP) flap, breast
carotid artery disease, 206 reconstruction, 108
chemodectoma, 59 Deep inguinal ring, 64f
cystic hygroma, 58 Deep vein thrombosis (DVT)
enterocutaneous fistula, 115 amputation, 201
hepatomegaly, 77 hip replacement, 130
obstructive jaundice, 72 varicose veins, 192
right iliac fossa mass, 92 Deep venous insufficiency, venous ulcer, 193194
Congenital arteriovenous fistula, 225226 Deep venous occlusion, post-phlebitic limb, 210
Congenital hydrocele, 88 Deformity, orthopaedic history taking, 123
Consent, informed, 233234 Degenerative arthritis, ulnar nerve palsy, 147
Consultation, communication skills, 230231 Dejerine, Joseph Jules, 178, 181
Corticosteroid injections, rheumatoid hands treatment, DejerineKlumpke paralysis, 37
143 De la Peyronie, Francois Gigot, 138
Cough Deltoid muscle, power testing, 162, 162f
femoral hernia, 118 De Morgan, Campbell, 61
inguinal hernia, 65 Denver shunt, ascites, 95
Courvoisier, Ludwig Georg, 96 De Quervain, F, 18
Courvoisiers law, 96 Dercum, Francis X, 6
CPB see Cardiopulmonary bypass (CPB) Dermatofibromas, 54
C-reactive protein malignant melanoma vs., 30
hepatomegaly, 77 Dermatomes, lower limb, 174f
inflammatory bowel disease, 84 Dermatosis papulosa nigra, 46
Critical ischaemia, 203 Dermofasciectomy, Dupuytrens contracture, 138
peripheral arterial system, 202t Dermoid cysts, 10f, 5051
Crohn, Burrell Bernard, 86 Diabetes mellitus
Crohns disease Charcots joints vs., 180
pyoderma granulosum, 60 gangrene, 212
radiography, 84 neuropathic ulcers, 214
surgery, 8586 Diabetic foot, 199200
Cross-fluctuation test, knee osteoarthritis, 133, 133f Diabetic neuropathy, 201
Cruciate ligaments Diamond sign, digital clubbing, 48
anatomy, 156157 Diffuse joint osteoarthritis, hallux rigidus vs., 169
injuries, 156 Diffuse lymphangioma, 224
knee osteoarthritis, 134135 Diffuse thyroid enlargement, 16, 2425
tests, 155 neck examination, 2425
see also Anterior cruciate ligament (ACL); Posterior see also Graves disease; Thyroiditis
cruciate ligament (PCL) Digital clubbing, 4849, 48f
Cruveilhier, Jean, 78 abdominal examination, 68, 68f
Cruveilhiers sign, 189 hepatomegaly, 75
Cryotherapy, solar keratosis precursor, 47 right iliac fossa mass, 90
Index 243

Digital rectal examination (DRE), lumbar disc herniation, Endoscopy


173 dysphagia, 99
Dilated common carotid artery, 223 inflammatory bowel disease, 84
Direct anastomosis, lymphoedema, 216 rigid, pharyngeal pouch, 57
Discectomy Endovascular repair, abdominal aortic aneurysm repair,
endoscopic, lumbar disc herniation, 175 204205
percutaneous, lumbar disc herniation, 175 End-to-end anastomosis, coarctation of the aorta,
Disease-modifying anti-rheumatic drugs (DMARDS), 227
143 Enterocutaneous fistula, 114115
Distal interphalangeal joints (DIPJ) SNAPP acronym, 115b
hammer toes, 149 ENT examination, cervical lymphadenopathy, 13
hand osteoarthritis, 144 Environmental agents, squamous cell carcinoma, 28
Dohlmans procedure, pharyngeal pouch, 57 Epicondylectomy, medial, 147
Doigts Hippocratique, 48 Epidermal cysts, 7
Doppler ultrasound Epididymal cysts, 8889
ankle brachial pressure index, 198 Epidural analgesia, lumbar disc herniation, 173
thoracic outlet obstruction, 220 Epigastric hernia, 117118
varicose veins, 190192 Epigastric mass, 9596
Dorsal interossei, ulnar nerve lesions, 145 Epstein, M A, 14, 78
Dorsalis pedis (DPA) pulse, 197, 198f Erbs palsy, Waiters tip, 175176
Doxorubicin, anaplastic thyroid cancer, 21 Erb, Wilhelm, 178
Draining lymph nodes, malignant melanoma, 30 Etanercept, rheumatoid hands treatment, 143
Dressler, William, 98 European Carotid Surgery Trial (ECST), 206207
Drumstick appearance, digital clubbing, 48 Excision
Duchenne, Guillame Benjamine Armand, 178 basal cell carcinoma, 33
Duplex ultrasound squamous cell carcinoma, 29
carotid artery aneurysm, 223 Exophthalmos, 25
carotid artery disease, 206 thyroid examination, 18
chemodectoma, 59 Extended radical mastectomy, 106
post-phlebitic limb, 211 Extension
Dupuytren, Guillaume, 138 hip movements, 127
Dupuytrens contracture, 137139 knee osteoarthritis, 133, 133f
hepatomegaly, 76 lumbar disc herniation, 173
Durans test, 140 Extensor carpi radialis brevis muscle, 158f
Dysphagia, 9899 Extensor carpi radialis longus muscle, 158f
achalasia, 99 Extensor carpi ulnaris muscle, 158f
barium swallow, 99 Extensor digiti minimi muscle, 158f
causes, 99t Extensor digitorum muscle, 158f
endoscopy, 99 Extensor indicis muscle, 158f
Dysplastic naevus syndrome, 30 Extensor pollicis brevis muscle, 158f
Extensor pollicis longus muscle, 158f
Ear, nose and throat (ENT) examination, cervical External fixators, 182
lymphadenopathy, 13 External rotation, shoulder, 161f
Echocardiogram Exudates, pleural effusion, 66f, 97
carotid artery disease, 206 Eyelid(s), facial nerve palsy, 38
coarctation of the aorta, 226 Eyelid lag, 17, 17f, 25
Ectopic testes, right iliac fossa mass, 91f Eyelid retraction (Dalrymples sign), 17, 17f
Effusions, knee osteoarthritis, 132133 Eyes
Elbow flexion test, ulnar nerve lesions, 146 abdominal examination, 69
Electrocardiogram, atherosclerosis, 206 facial nerve palsy, 38
Electrolytes inflammatory bowel disease, 84
enterocutaneous fistula, 115 radiotherapy side effects, 53
inflammatory bowel disease, 84 surgical jaundice, 71b
Emboli, gangrene, 212 thyroid examination, 1718, 17f18f
Embolic stroke, atrial fibrillation, 228
Embryological origin, thyroglossal cysts, 52 Face
Embryonal carcinomas, testicular tumours, 113 asymmetry, salivary gland swellings, 40
Endarterectomy, carotid see Carotid brachial plexus lesions, 177
endarterectomy Facial nerve
Endocarditis, infective, 48 branches, 3839, 39f
Endoscopic discectomy, lumbar disc herniation, palsy, 3839
175 Fallot, E L A, 49
Endoscopic retrograde cholangiopancreatography False aneurysms, 218219, 219f
(ERCP), 72 Family history, malignant melanoma, 30
244 Index

Fascia defects, varicose veins, 189 Forward flexion


Fasciotomy lumbar disc herniation, 173
Dupuytrens contracture, 138 shoulder, 160
partial, Dupuytrens contracture, 138 4Ds, amputation indications, 201
Feel Fournier, Jean Alfred, 212
knee osteoarthritis, 131133 Fourniers gangrene, 212
lumbar disc herniation, 173 Fourniers sign, 212
shoulder, 122, 160, 160f Fourniers tibia, 212
Feet Fracture reduction, 171172
knee ligaments/cartilages, 155 Frank dullness, ascites, 94
loss of function, 122123 Freckles, malignant melanoma vs., 30
Felty, Augustus Roi, 83 Free superficial inferior epigastric artery (SIEA) flap,
Femoral hernia, 64f, 118120, 119f breast reconstruction, 108
inguinal hernia vs., 120t Frey, L, 42
L-SHAPE acronym, 120 Freys syndrome, hyperhidrosis, 218
NAVY acronym, 119f Froment, Jules, 147
Femoral pulses, 197, 197f Froments sign, 146
popliteal aneurysm, 207 Frozen shoulder, 164
Femoral stretch test (FST), lumbar disc herniation, Full blood count
173 enterocutaneous fistula, 115
Femoral vein, 189f hepatomegaly, 77
Fibrosing alveolitis, digital clubbing, 48 inflammatory bowel disease, 84
Fine-needle aspiration cytology (FNAC) obstructive jaundice, 72
cervical lymphadenopathy, 13 Functional assessment, rheumatoid hands, 143
malignant melanoma, 32 Fundoscopy, Pagets disease of bone, 184
parotid gland swelling, 41 Furuncles, 59
Fine tremor, thyroid examination, 17 Fusiform aneurysms, 219f
Fingers
clubbing see Digital clubbing Gait, 165
rheumatism, 142 high stepping, 166t
First lumbrical muscle, 140f hip osteoarthritis, 124
First rib, anatomy, 220f knee ligaments/cartilages, 155
Fistula knee osteoarthritis, 130131
arteriovenous see Arteriovenous fistulae lumbar disc herniation, 173
enterocutaneous see Enterocutaneous fistula Parkinsonian, 166t
traumatic arteriovenous, 226 short leg, 166t
Fistulogram, enterocutaneous fistula, 115 spastic, 166t
Fixators, external, 182 types, 166t
Fixed flexion deformity Galeazzi, Ricardo, 130
amputation, 201 Galeazzis test, 125126
hip osteoarthritis, 126 Ganglions, 78
Flexion Gangrene, 195, 211212
hip movements, 127 venous, 211
knee osteoarthritis, 133, 134f Gardner, Elton J, 7
Flexion compression test, carpal tunnel syndrome, Gardners syndrome, 7
140 Garrod, A E, 138
Flexor carpi muscle, 140f Gastrointestinal disease
Flexor carpi ulnaris muscle, 146f abdominal examination, 69
Flexor digitorum profundus muscle, 140f, 146f, 153b digital clubbing, 48
ulnar nerve lesions, 145 Gaucher, Philippe Charles Ernest, 83
Flexor digitorum superficialis muscle, 140f Gilbert, Nicholas Augustin, 73
trigger finger, 153b Girdlestone, Gathorne Robert, 150
Flexor pollicis brevis muscle, 146f Glucose test, atherosclerosis, 206
Flexor pollicis longus muscle, 140f Goitres
Flexor tenotomy, mallet toes, 151 definition, 1819
Fluctuation, lumps and ulcers, 4 multinodular see Multinodular goitre
Fluid thrill WHO classification, 1819
ascites, 94, 94f Goitrogens, diffuse thyroid enlargement, 24
lumps and ulcers, 4 Gonads, radiotherapy side effects, 53
Focus, external fixators, 182 Gorlin, R J, 34
Follicular thyroid cancer, 20 Gorlins syndrome, 3334
Follow-up, fracture reduction, 172 Graves disease, 24
Foot pulses, 197 digital clubbing, 49
Footwear, hallux valgus, 148149 thyroid examination, 16
Index 245

Graves, R J, 25 Hernia
Gridiron scar, 102f epigastric, 117118
Gynaecomastia, 109110 femoral see Femoral hernia
hepatomegaly, 76 incisional, 7879
inguinal see Inguinal hernia
Haemangioma, cavernous, 61 lumbar disc see Lumbar disc herniation
Haemangiomas, thrombosed vs. malignant melanoma, paraumbilical see Umbilical/paraumbilical hernia
30 umbilical see Umbilical/paraumbilical hernia
Haemarthrosis Herpes simplex virus infection, abdominal disease, 116
primary spontaneous, 156 Hibernomas, 6
secondary, 156 Hidradenitis suppurativa, 55
Haematology High approach, femoral hernia surgery, 120
atherosclerosis, 206 High stepping gait, 166t
splenomegaly, 82 Hip, loss of function, 122
Haematoma Hip osteoarthritis, 123130
amputation, 201 fixed flexion deformity, 126
inguinal hernia repair, 67 investigations, 129
Haemorrhage, thyroidectomy, 25 movements, 127, 127f128f, 127t
Haemothorax, cervical sympathectomy side effect, palpation, 126
217 patient lying down, 124126
Hallux rigidus, 169170 standing examination, 123
Hallux valgus, 147149 Thomas test, 126, 126f127f
bunion, 148 Trendelenburg test, 123124, 124f
Halstead mastectomy, 106 walking examination, 124
Halstead, William Stewart, 107 Hippocrates, 49
Hammer toes, 149150, 150f Hip replacement
Hand(s) complications, 129
abdominal aortic aneurysms, 203204 deep vein thrombosis, 130
abdominal examination, 6869 Histology
achondroplasia, 185 basal cell carcinoma, 34
osteoarthritis, 144145 cervical lymphadenopathy, 13
rheumatoid see Rheumatoid arthritis, hands neurofibroma, 44
thoracic outlet obstruction, 219 History taking
thyroid examination, 17 orthopaedic see Orthopaedic history taking
Harvesting, skin grafts, 36 thyroid gland, 2526
Harveys test, superior vena cava obstruction, 222 HIV infection, neuropathic ulcers, 214
Harvey, William, 222 HLA-B5, thromboangiitis obliterans (Buergers disease),
Hashimoto, Hakura, 18 221
Hearing HLA compatibility, kidney transplants, 92
facial nerve palsy, 38 Hoarseness, thyroidectomy, 25
Pagets disease of bone, 183 Hodgkin, Thomas, 14
Heberdens nodes Holding, fracture reduction, 172
hand osteoarthritis, 144 Homans, John, 217
surgery, 145 Homans procedure, 216
Heberden, William, 145 HOPOA (hypertrophic pulmonary osteoarthropathy), 48
Hedgehog signalling pathway, basal cell carcinoma, Horner, Johann Friedrich, 178, 222
34 Horners syndrome
Heel, ulcers, 195 cervical sympathectomy side effect, 217
Heel strike, gait, 165 Pancoasts tumour of the lung, 37
Height, achondroplasia, 185 ptosis, 37
Helicobacter pylori infection, epigastric hernia, 118 Human herpes virus 8 (HHV-8), 56
Heparin, deep vein thrombosis, 130 Hunterian (mid-thigh) perforating vein, 189f
Hepatomegaly, 7578 Hunter, John, 32, 208
arterial bruit, 77 Hunters ligation, 208
CHIASMA acronym, 77b Hunt, J Ramsay, 39
investigations, 77 Hutchinson, Jonathan, 32
liver examination, 76 Hybrid frames, 182
portal hypertension, 77 Hydrocele, 8788
splenomegaly vs., 82 congenital, 88
SPRUE acronym, 76b infantile, 88
venous hum, 77 secondary, 88
Hereditary osteochondromatosis, multiple, 167 spermatic cord, 8788
Hereditary telangiectasia (RenduOslerWeber testicular tumours, 113
disease), 116 vaginal, 8788
246 Index

Hygroma, cystic, 10f, 5758, 224 Information giving


Hypercalcaemia, Pagets disease of bone, 184 angry patient, 237
Hyperhidrosis, 217218 breast cancer, 234235
Hyperhidrosis erythematosus traumatica, 217 communication skills, 230
Hyperkeratosis loss of function, 236
seborrhoeic keratosis, 46 obtaining informed consent, 233234
solar keratosis, 47 testicular tumours, 235236
Hyperthyroidism, 26t Informed consent, obtaining, 233234
thyroidectomy, 25 Infraclavicular lymph node, 12f
Hypertrophic pulmonary osteoarthropathy (HOPOA), 48 Infraspinatus, power testing, 162, 163f
Hypertrophic scars, 2627, 27t28t Ingrowing toenail, 154155
Hypocalcaemia, thyroidectomy complications, 234 Inguinal hernia, 6467
Hypochondroplasia, 185 cough, 65
Hypothyroidism, 26t direct, 66
eyebrows, 17 femoral hernia vs., 120t
neuropathic ulcers, 214 indirect, 66
thyroidectomy complications, 234 palpation, 6566
scar, 102f
IBD see Inflammatory bowel disease (IBD) surface anatomy, 64f
Ileosigmoidoscopy, inflammatory bowel disease, 84 Inguinal ligament
Ileostomy, 75t anatomy, 65f66f
Ileostomy with subtotal colectomy, inflammatory bowel midpoint, 64b65b, 64f
disease, 85 Inguinal lymphadenopathy, scrotum examination, 87
Illizarov, Gabriel Abramovitch, 182 Inspection
Immobility, pressure sores, 35 brachial plexus lesions, 175176
Immunizations, splenectomy, 83 peripheral arterial system examination, 195196
Immunosuppression, squamous cell carcinoma, 28 varicose veins, 188
Immunotherapy, malignant melanoma, 32 Instability, shoulder pain, 163
Implants, breast reconstruction, 108t Intermedullary nails, 183
Incisional hernia, 7879 Intermittent claudication
Incompetence, varicose veins, 189190, 190f differential diagnosis, 203
Incomplete casts, 170 peripheral arterial system, 202t
Incompletely descended testes, right iliac fossa mass, Internal iliac artery, 195f
91f Internal rotation, shoulder, 161, 161f
Infantile hydrocele, 88 Intravenous digital subtraction angiogram, 223
Infections Iodine deficiency, diffuse thyroid enlargement, 24
abdominal aortic aneurysms, 205 Ischaemia, critical see Critical ischaemia
inguinal hernia repair, 67 Ischaemic orchitis, inguinal hernia repair, 67
lymphoedema, 216 Ischaemic ulcers, 208209, 210t
mouth, 116 diabetic foot, 199
sebaceous cysts, 7 Ivory osteoma, 178179
splenomegaly, 82
see also specific infections Jaboulays operation, hydrocele, 88
Infective endocarditis, digital clubbing, 48 Jaundice, obstructive, 72
Inflammation Jaundice, surgical, 7173
cervical lymphadenopathy, 14 causes, 72t
enterocutaneous fistula, 114 classification, 72
Inflammatory bowel disease (IBD), 8386 Jeghers, Harald Jos, 117
auscultation, 84 Joffroys sign, thyroid examination, 19
digital clubbing, 48 Joint arthrodesis, hand osteoarthritis, 145
hepatobiliary complications, 85 Joint disorders, shoulder pain, 163
investigations, 84 Jugular venous pressure, Pagets disease of bone, 183
palpation, 84 Jugular venous pulse, lymphoedema, 216
percussion, 84 Jugulodigastric lymph node, 12f
peripheral signs, 8384
surgery, 85 Kaposi, Moricz, 56
Truelove classification, 8485 Kaposis sarcoma, 5556
see also Crohns disease; Ulcerative colitis Kaposis sarcoma herpes virus (KSHV), 56
Inflammatory markers, enterocutaneous fistula, 115 Keller, William, 149
Infliximab, 143 Keloid scars, 2627, 27t28t
Information gathering Keratoacanthoma, 43
back pain, 231 basal cell carcinoma vs., 33
trauma, 232 Keratoconjunctivitis, Sjgrens syndrome, 42
vascular referral, 232233 Keratosis, seborrhoeic, 4647
Index 247

Kidneys, enlarged, 100102 Leser, E, 47


balloting, 100f Lesions, superficial, 161
differential diagnosis, 100101 Leukonychia, hepatomegaly, 76
simple cysts, 101 Leydig cell tumours, testicular tumours, 113
SPRUE acronym, 110b Lichen planus, 116
Kidney transplants, 9293 Lichtenstein mesh repair, inguinal hernia, 67
matching, 92 Lifestyle
rejection, 9293 hip osteoarthritis, 129
vascular supply, 93, 93f ischaemic ulcers, 209
Killians dehiscence, pharyngeal pouch, 57 knee osteoarthritis treatment, 136
Kingsley Robinson skew flap, 200 lumbar disc herniation, 172
Kirschner, Martin, 153 Ligaments
Kistners operation, 211 collateral see Collateral ligaments
Klinefelter, Harry Fitch Jr, 110 cruciate see Cruciate ligaments
Klippel, M, 192 inguinal see Inguinal ligament
KlippelTrenaunay syndrome, 61 patella, knee osteoarthritis, 131132, 131f
KlippelTrenaunayWeber syndrome, 192 posterior cruciate see Posterior cruciate ligament (PCL)
Klumpkes palsy, claw hand, 176 Light touch testing, brachial plexus lesions, 177
Knee, locked, 157 Lip disorders, 116
Knee(s) Lipoma, 56
achondroplasia, 185 popliteal fossa swelling vs., 167
cartilaginous lesions, 155157 Liposarcomas, classification, 6
see also specific cartilages Listening
ligamentous lesions, 155157 communication skills, 230
see also specific ligaments peripheral arterial system examination, 198
locked, 157 LIST mnemonic, cervical lymphadenopathy, 13
loss of function, 122 Liver cirrhosis, digital clubbing, 48
rheumatoid arthritis, 137 Liver examination, 76
Knee arthroplasty (knee replacement) Liver flaps
complications, 136137 abdominal examination, 69, 69f
knee osteoarthritis treatment, 136 hepatomegaly, 76
knee rheumatoid arthritis, 137 Liver function tests
Knee osteoarthritis, 130137 enterocutaneous fistula, 115
feel, 131133 epigastric hernia, 118
gait, 130131 hepatomegaly, 77
measurements, 131 inflammatory bowel disease, 84
movement, 133 obstructive jaundice, 72
standing examination, 130 LOAF mnemonic, carpal tunnel syndrome, 139
treatment, 136 Lobectomy, 110111
Kochers scar, 102f Local microvesicular ischaemia, Dupuytrens
Koilonychia, abdominal examination, 69 contracture, 138
Locked knee, 157
Lachman test, knee osteoarthritis, 134135 Lockwood approach, femoral hernia surgery, 120
Landouzy, L T J, 181 Long bones, Pagets disease of bone, 184
Lanz scar, 102f Long saphenous vein, 189f
Large congenital naevi, malignant melanoma, 30 Lord, Peter, 88
Laryngeal nerve damage, thyroidectomy complications, Lords plication, 88
234 LOSS mnemonic
Lateral collateral ligament (LCL), 155 hip osteoarthritis, 129
Lateral flexion, lumbar disc herniation, 173 Loss of function
Lateral joint line, knee osteoarthritis, 131132, 132f information giving, 236
Lateral meniscus, 156 orthopaedic history taking, 122123
Lateral thoracotomy scar, 110 Lotheissen repair, 120
Latissimus dorsi (LD) flap, breast reconstruction, 108 Lovibonds sign, 48
Ledderhose disease, 138 Low approach, femoral hernia surgery, 120
Ledderhose, G, 138 Lower brachial plexus injury, Horners syndrome, 37
Legs Lower limb
arteries, 195f dermatomes, 174f
length measurement, 165 loss of function, 122
superficial venous system, 189f see also Feet; Legs
LEGS acronym, varicose veins, 188b L-SHAPE acronym, 67b
Lentigo, malignant melanoma vs., 30 Lumbar disc herniation, 172175
Lentigo maligna melanoma, 30 digital rectal examination, 173
Leprosy, neuropathic ulcers, 214 examination, 173175
248 Index

feel, 173 Malabsorption, digital clubbing, 48


femoral stretch test, 173 Males, breast cancer, 109110
gait, 173 Malignancies
lifestyle effects, 172 enterocutaneous fistula, 114
movement, 173 lipoma, 6
neurological examination, 173 lymphoedema, 216
neurological symptoms, 172 secondary, radiotherapy side effects, 53
observation, 173 thyroid gland, 2022
sciatic stretch test, 173 Malignant melanoma, 2932
sphincter disturbance, 172 dermatofibroma vs., 54
straight leg raising, 173 draining lymph nodes, 30
treatment, 174175 nodular, 2930
Lumbar spine, loss of function, 122 staging, 3031
Lumbar sympathectomy, ischaemic ulcers, 209 superficial spreading, 2930
Lumps Mallet finger, 152153
examination, 35 Mallet toes, 150151
history taking, 3 Marie, Pierre, 152
Lung(s), radiotherapy side effects, 53 Marjolin, Rene, 194
Lung resection Marjolin ulcer, 194
indications, 110 Massive splenomegaly, 82
mortality, 111 Mastectomy
types, 110111 appearance after see Post-mastectomy breast
Luteinizing hormone (LH), gynaecomastia, 109 extended radical, 106
Lymphadenopathy Halstead, 106
cervical, 1214 indications for, 106
inguinal, 87 modified radical, 106
right iliac fossa mass, 90 radical see Radical mastectomy
Lymphangioma, 224 simple, 106
cutaneous, 224 types, 106107
cystic, 224 Mayo, Charles Horace, 81
diffuse, 224 Mayos vest-over-pants operation, 81
solid, 224 McEvedy approach, femoral hernia surgery, 120
Lymphatic system, 187228 McMurrays test, 135
Lymph node(s) modified, 135136, 135f
anterior cervical chain, 12f Medial collateral ligament (MCL), tests, 155
breast, 104f Medial epicondylectomy, ulnar nerve palsy, 147
cervical see Cervical lymph nodes Medial joint line, knee osteoarthritis, 131132, 131f
excision biopsy, 14 Medial meniscus, 156
infraclavicular, 12f Median nerve, 139b
malignant melanoma, 30 compression, 141
occipital, 12f see also Carpal tunnel decompression
postauricular, 12f muscles supplied, 140f
posterior triangular, 12f Median sternotomy, 111112
preauricular, 12f indications, 111
submandibular, 12f scar, 111
submental, 12f subtotal, 111
testes, 87 Medical history, 230
Lymphoedema, 215217 orthopaedic history taking, 123
primary vs. secondary, 216 Medullary thyroid cancer, 21
Lymphomas Melanoma, malignant see Malignant melanoma
cervical lymphadenopathy, 14 Menisci
testicular tumours, 114 anatomy, 156
thyroid cancer, 22 knee osteoarthritis, 135136
Lymphoreticular system, cervical lymphadenopathy, 13 lateral, 156
medial, 156
Mafucci, A, 179 tears, 156
Mafuccis syndrome, 180 tests, 155
Magnetic resonance cholangiopancreatography Mesothelioma, digital clubbing, 48
(MRCP), obstructive jaundice, 72 Metacarpophalangeal joints (MCPJ)
Magnetic resonance imaging (MRI) Dupuytrens contracture, 137
carotid artery disease, 206 volar subluxation, 142
chemodectoma, 59 Metatarsophalangeal joint (MTPJ), hallux rigidus, 169
cystic hygroma, 58 Midline scar, 102f103f
parotid gland swelling, 41 Midlingual point, 64b65b, 64f
Index 249

Mid-thigh (Hunterian) perforating vein, 189f lumps, differential diagnosis, 10t


Mikulicz syndrome, 42 palpation, thoracic outlet obstruction, 219220
Milroy, E F, 217 Neck decision circle, neck examination, 11f
Mobius sign, thyroid examination, 19 Neck examination, 812
Modified McMurrays test, knee osteoarthritis, anterior triangle, 9, 9f
135136, 135f neck decision circle, 11f
Modified radical mastectomy, 106 posterior triangle, 9, 9f
Mohs staged chemosurgery tongue protrusion, 9
basal cell carcinoma, 33 up and down technique, 11t
squamous cell carcinoma, 29 Nerve(s)
Moles, malignant melanoma vs., 30 injury, carpal tunnel decompression, 141
Mortality lesions, shoulder pain, 163
abdominal aortic aneurysm repair, 204 spermatic cord, 66
lung resection, 111 see also specific nerves
Motor assessment Nerve conduction studies, carpal tunnel syndrome, 141
carpal tunnel syndrome, 139 Neurofibroma, 4445
radial nerve lesions, 158159 Neurofibromatosis
rheumatoid hands, 143 caf-au-lait spots, 44
Motor neuropathy, neuropathic ulcers, 214215 definition, 44
Mouth Neurofibromatosis type I (von Recklinghausens
abdominal examination, 69 syndrome), 44
facial nerve palsy, 38 Neurofibromatosis type II (MISME syndrome), 44
infections, 116 Neurological disorders
inflammatory bowel disease, 84 claw toes, 152
pigmentation, 116 lumbar disc herniation, 172
ulceration, 116 Pagets disease of bone, 184
Movement rheumatoid arthritis, 143
knee ligaments/cartilages, 155 Neurological examination
knee osteoarthritis, 133 diabetic foot, 198
lumbar disc herniation, 173 lumbar disc herniation, 173
shoulder, 160162 Neuromas, popliteal fossa swelling vs., 167
MPS acronym, ulcerative colitis surgery, 85 Neuropathic ulcers, 210t, 214215
MRCS exam, communication skills, 230 diabetic foot, 199
MRI see Magnetic resonance imaging (MRI) Night, rest pain, 203
Mucoepidermoid tumours, salivary glands, 43 Nipples, examination, 104, 104b
Multinodular goitre, 2224 Nodular basal cell carcinoma, 33
Graves disease vs., 23, 23t Nodular malignant melanoma, 2930
neck examination, 22 Non-steroidal anti-inflammatory drugs (NSAIDs),
Multiple arteriovenous fistula (ParkesWeber rheumatoid hands treatment, 143
syndrome), 192, 226 North American Symptomatic Carotid Endarterectomy
Multiple endocrine neoplasia type I (Werner syndrome), Trial (NASCT), 206207
22
Multiple endocrine neoplasia type IIA (Sipple Observation
syndrome), 22 lumbar disc herniation, 173
Multiple endocrine neoplasia type IIb, 22 shoulder, 159160
Multiple hereditary osteochondromatosis, 167 Obstructive jaundice, 72
Muscle wasting/atrophy, brachial plexus lesions, 176 Obtaining informed consent, 233234
Muscular dystrophy, winging of scapula vs., 181 Occipital lymph node, 12f
Myocutaneous flaps, breast reconstruction, 108, Occipitofrontalis muscle, facial nerve palsy, 38
108t Occupational therapy
hip osteoarthritis, 129
Naevi knee osteoarthritis treatment, 136
large congenital, 30 Odynophagia, 99
malignant melanoma, 2930 Oesophagus, carcinomas, 9899
Naevus araneus, 61 Oestrogens, gynaecomastia, 109
Naevus vinosus, 61 Ollier, L, 179
Nail avulsion, ingrowing toenail, 154 Olliers disease, 179
Nail-bed vessel vasodilatation, digital clubbing, 49 Onycholysis (Plummers nails) see Plummers nails
NASCT (North American Symptomatic Carotid (onycholysis)
Endarterectomy Trial), 206207 Open hemilaminotomy and discectomy, lumbar disc
Nasolabial fold, facial nerve palsy, 38 herniation, 175
Neck Ophthalmic signs, rheumatoid arthritis, 143
abdominal examination, 69 Ophthalmoplegia, thyroid examination, 1718, 17f
block dissection, cervical lymphadenopathy, 14 Oral candidiasis, abdominal disease, 116
250 Index

Orbicularis oculi muscle, 38 Parkinsonian gait, 166t


Orbicularis oris muscle, 38 Parotidectomy
Orthopaedic history taking, 122123 complications, 42
deformity, 123 superficial, parotid gland swelling, 41
loss of function, 122123 Parotid gland, 10f
medical history, 123 anatomy, 40f
pain, 122 duct opening, 40
stiffness, 123 Parotid gland swelling, 40
swelling, 123 differential diagnosis, 41
OslerRenduWeber syndrome, 61 malignancies, 42
Osler, William, 61, 117 Parotid lymph node, 12f
Osteoarthritis Partial fasciotomy, Dupuytrens contracture, 138
diffuse joint, 169 Patella ligament, knee osteoarthritis, 131132, 131f
hands, 144145 Patellar hollow test, knee osteoarthritis, 132, 132f
hip see Hip osteoarthritis Patellar tap, knee osteoarthritis, 133
knee see Knee osteoarthritis Patella tendon bearing casts, 170
Pagets disease of bone, 183 Patellectomy, knee osteoarthritis treatment, 136
shoulder, 161 Paterson, D R, 99
Osteochondroma, 166167 Patey mastectomy, 106
Osteochondromatosis, multiple hereditary, 167 Patient anger, 237
Osteoma, ivory, 178179 Patient exposure, abdominal examination, 68b
Osteomyelitis, amputation, 201 Patient information, communication skills, 230
Osteotomy Patient lying down, hip osteoarthritis, 124126
hip osteoarthritis, 129 Patient preparation, stoma, 74
realignment, knee osteoarthritis treatment, 136 Paul, J R, 14
supracondylar, knee rheumatoid arthritis, 137 Pelvis, Pagets disease of bone, 184
Percussion
Paget, James, 184 abdominal examination, 70
PagetScroetter syndrome, 184 inflammatory bowel disease, 84
Pagets disease of bone, 183184 pleural effusion, 97
Pagets disease of the nipple, 184 splenomegaly, 8182
Pagets disease of the skin, 184 Percutaneous discectomy, lumbar disc herniation, 175
Pagets sign, 184 Peripheral arterial system examination, 194198
Pain history taking, 202203
inguinal hernia repair, 67 inspection, 195196
orthopaedic history taking, 122 listening, 198
phantom limb, 201 palpation, 196197
referred, shoulder pain, 163 Peripheral occlusive arterial disease, diabetic foot, 198
Palm, rheumatism, 142 Peripheral pulses, 196
Palma operation, post-phlebitic limb, 211 Peritovenous shunts, ascites, 95
Palmar erythema Perthes test, 210
hepatomegaly, 76 PeutzJeghers syndrome, 116
thyroid examination, 17 Peutz, John Law Augustine, 117
Palmar interossei, ulnar nerve lesions, 145 Pfannenstiel, Herman Johannes, 103
Palmaris longus muscle, 140f Pfannenstiel scar, 102f
Palomo operation, variocele, 90 Phaeochromocytoma, 217
Palpation Phalen, George S, 141
abdominal examination, 6970 Phalens sign, 140
hip osteoarthritis, 126 Phantom limb pain, 201
inflammatory bowel disease, 84 Pharyngeal pouch, 5657, 57f
peripheral arterial system examination, 196197 Physiotherapy, 39
pleural effusion, 97 frozen shoulder, 164
rheumatoid hands, 142 hallux valgus, 149
splenomegaly, 8182, 81f hip osteoarthritis, 129
Pancoast, Henry Khunrath, 37, 221 knee osteoarthritis treatment, 136
Pancoasts tumour of the lung lumbar disc herniation, 173
Horners syndrome, 37 Piano-key sign, 142
thoracic outlet obstruction vs., 220 Pigmentation
Papillary thyroid cancer, 20 mouth, 116
Papilloma, 45 skin, 210
Parakeratosis, solar keratosis, 47 Pigmented basal cell carcinoma, 33
Paraumbilical hernia see Umbilical/paraumbilical hernia malignant melanoma vs., 30
ParkesWeber syndrome (multiple arteriovenous Pigmented seborrhoeic keratoses, malignant melanoma
fistula), 192, 226 vs., 30
Index 251

Pinless frames, external fixators, 182 Primary spontaneous haemarthrosis, cruciate ligament
PIPJ see Proximal interphalangeal joints (PIPJ) injuries, 156
Plaster of Paris, casts, 170171 Proctocolectomy and permanent ileostomy,
Platelet precursors, digital clubbing, 49 inflammatory bowel disease, 85
Pleural effusion, 9698 Proctolectomy, restorative, inflammatory bowel
auscultation, 97 disease, 85
chest drains, 98 Profunda femoris artery, 195f
classification, 97 Pronator quadratus muscle, 140f
exudates, 66f Pronator syndrome, 141
palpation, 97 Pronator teres muscle, 140f
percussion, 97 Prophylaxis, pressure sore therapy, 35
pleural taps, 98 Proptosis, thyroid examination, 18, 18f
transudate, 66f, 97 Prostaglandins, ischaemic ulcers, 209
Pleural taps, pleural effusion, 98 Proximal interphalangeal joints (PIPJ)
Plexiform neurofibroma, 44 Dupuytrens contracture, 137
Plummer, H S, 23, 99 hand osteoarthritis, 144
Plummers nails (onycholysis), 17 Pseudoaneurysms, CiminoBrescia arteriovenous
thyroid examination, 17 fistula, 225
PlummerVinson syndrome, 23 Pseudochondroplasia, 185
dysphagia, 98 Psychological preparation, breast surgery, 107
Pneumonectomy, 110 Ptosis, 3637
Pneumothorax, cervical sympathectomy side effect, unilateral, 37
217 Pulses
Polycystic kidney disease, adult vs. infantile, 101t carotid, CiminoBrescia arteriovenous fistula, 225
Popeye bulge, 163 characteristics, 227t
Popliteal artery, 195f diabetic foot, 199
Popliteal artery aneurysm, 207208 dorsalis pedis, 197, 198f
popliteal fossa swelling vs., 167 femoral see Femoral pulses
Popliteal cyst, 168 foot, 197
Popliteal fossa swellings, 167168 peripheral, 196
Popliteal pulse, 197, 197f popliteal see Popliteal pulse
popliteal aneurysm, 207 popliteal aneurysm, 207
Popliteal swellings, knee osteoarthritis, 132, 132f posterior tibial, 197, 198f
Popliteal vein, 189f thyroid examination, 17
Portal hypertension see also specific pulses
hepatomegaly, 77 Pulsion diverticulum, pharyngeal pouch, 57
splenomegaly, 82 Pyoderma granulosum, 60
Port-wine stain, 61 neuropathic ulcers, 215
Postauricular lymph node, 12f Pyogenic granuloma, 4546
Posterior arch vein, 189f
Posterior cruciate ligament (PCL) Quadriceps wasting, knee osteoarthritis, 131, 131f
anatomy, 157 Question answering, communication skills, 230
tests, 155
Posterior drawer, knee osteoarthritis, 134 Radial neck dissection, cervical lymphadenopathy, 14
Posterior interosseous nerve, 158f Radial nerve, 139b
Posterior sag, knee osteoarthritis, 134 lesions, 157159
Posterior tibial (PTA) pulse, 197, 198f muscles supplied, 158f
Posterior triangle, neck examination, 9, 9f Radical mastectomy, 106
Posterior triangular lymph node, 12f extended, 106
Post-lobectomy, 110111 modified, 106
Post-mastectomy breast, 106107 Radiography
Post-phlebitic limb, 210211 cervical lymphadenopathy, 13
Post-pneumonectomy, 110111 coarctation of the aorta, 226
Post-surgical jaundice see Jaundice, surgical Crohns disease, 84
Power testing cystic hygroma, 58
brachial plexus lesions, 176 enterocutaneous fistula, 115
shoulder, 162, 162f fracture reduction, 172
Preauricular lymph node, 12f hallux rigidus, 169
Pre-existing skin lesions, squamous cell carcinoma, 28 hallux valgus, 148
Preparation, communication skills, 230 hepatomegaly, 77
Pressure effects, neurofibroma, 44 hip osteoarthritis, 129
Pressure sores, 3435 inflammatory bowel disease, 84
classification, 34b knee osteoarthritis, 136
Primary Raynauds phenomenon, 213 mallet finger, 153
252 Index

obstructive jaundice, 72 investigations, 143


Pagets disease of bone, 184 motor assessment, 143
renal cell carcinoma, 101102 palpation, 142
rheumatoid hands, 144 radiography, 144
right iliac fossa mass, 9192 sensory assessment, 143
thoracic outlet obstruction, 220 treatment, 143
Radioiodine Rheumatoid vasculitis, pyoderma granulosum vs., 60
Graves disease, 24 Ribs, anatomy, 220f
multinodular goitre, 23 Riedel, B, 78
Radiotherapy Right iliac fossa mass, 9092
anaplastic thyroid cancer, 21 arteriovenous fistula, 90
basal cell carcinoma, 33 digital clubbing, 90
enterocutaneous fistula, 115 ectopic testes, 91f
marks, 5354 incompletely descended testes, 91f
mechanism of action, 53 lymphadenopathy, 90
side effects, 53 radiography, 9192
squamous cell carcinoma, 29 Virchows node, 90
Ramsay Hunt syndrome, 38 Right paramedian scar, 102f
Raynaud, Maurice, 213 Rigid endoscopy, pharyngeal pouch, 57
Raynauds phenomenon, 212213 Rooftop scar, 102f
primary, 213 Rotation, lumbar disc herniation, 173
secondary, 213 Rotator cuff, tendons, 162b
thoracic outlet obstruction vs., 220 Rotator cuff disorders, 163
Raynauds syndrome, gangrene, 212 Rotator cuff impingement, 163
Realignment osteotomy, knee osteoarthritis treatment, Rotator cuff tear
136 examination, 161
Reaming, controversies, 183 treatment, 164
Reconstruction ladder, 36
Recurrence, ganglions, 8 Saccular aneurysms, 219f
Reducibility, lumps and ulcers, 4 Salivary glands
Reduction, fractures, 172 adenoid-cystic carcinoma, 43
Referred pain, shoulder pain, 163 mucoepidermoid tumours, 43
Reflexes, brachial plexus lesions, 177 see also specific salivary glands
Reidels thyroiditis, 138 Salivary gland swellings, 4043
Reifenstein, E C, 110 facial asymmetry, 40
Rejection, kidney transplants, 9293 Salmonella typhi infection, 205
Renal cell carcinoma (RCC), 101 Salt/sweet solution testing, facial nerve palsy, 38
radiography, 101102 Saphena varix, popliteal fossa swelling vs., 167
Renal function, obstructive jaundice, 72 Sarcomatous transformation, neurofibroma, 44
Rendu, Henry Jules Louis Marie, 61, 117 Saturday night palsy, 159
RenduOslerWeber disease (hereditary telangiectasia), Scapula, winging of, 181
116 Scars
Respiratory disease/disorders brachial plexus lesions, 176
atrial fibrillation, 228 hypertrophic, 2627, 27t28t
digital clubbing, 48 inguinal hernia, 102f
rheumatoid arthritis, 143 keloid, 2627, 27t28t
Respiratory obstruction, thyroidectomy complications, knee osteoarthritis, 130
234 midline, 102f103f
Restorative proctolectomy, inflammatory bowel Pfannenstiel, 102f
disease, 85 right paramedian, 102f
Rest pain rooftop, 102f
at night, 203 subcostal, 102f
peripheral arterial system, 202, 202t surgical, 102103, 102f
Reticuloendothelial manifestations, rheumatoid arthritis, thoraco-abdominal, 102f
143 transverse, 102f
Retroperitoneal fibrosis, 138 umbilical, 80
Rheumatoid arthritis SCC see Squamous cell carcinoma (SCC)
knee, 137 Schamroth. L, 70
mallet finger, 153 Schamroths test, 68, 69f
neuropathic ulcers, 215 Schirmer, O W A, 42
pyoderma granulosum, 60 Schirmers test, 42
Rheumatoid arthritis, hands, 142144 Schwann cells, neurofibroma, 44
clinical stages, 144 Sciatica, referred pain vs, 175
functional assessment, 143 Sciatic stretch test (SST), 173
Index 253

Sclerosing basal cell carcinoma, 33 Skin flaps, 3536


Sclerotherapy, varicose vein treatment, 192 classification, 36
Scratch marks, hepatomegaly, 76 indications, 36
Screw-threaded half pins, external fixators, 182 Skin tags, 45
Scrotum examination, 8687 Skull
inguinal lymphadenopathy, 87 achondroplasia, 185
see also Testes Pagets disease of bone, 183184
Sebaceous cysts, 67 Slip sign, lipoma, 5
Sebaceous horn formation, sebaceous cysts, 7 Smith Surgical papyrus, 27
Seborrhoeic keratosis, 4647 SNAPP acronym, enterocutaneous fistula, 115b
Secondary haemarthrosis, cruciate ligament injuries, 156 Solar keratosis, 47
Secondary hydrocele, 88 squamous cell carcinoma, 29
Secondary malignancies, radiotherapy side effects, 53 Solid lymphangioma, 224
Secondary Raynauds phenomenon, 213 Solitary thyroid nodule, 16, 1922
Second lumbrical muscle, 140f management, 21f
Segments, external fixators, 182 Spastic gait, 166t
Seminoma, testicular tumours, 113t Speeds test, 163
Sensory assessment Spermatic cord, 66
brachial plexus lesions, 177 hydrocele, 8788
radial nerve lesions, 158 Sphincter disturbance, lumbar disc herniation, 172
rheumatoid hands, 143 Spider naevus, 61
Sensory loss, neuropathic ulcers, 214 hepatomegaly, 76
Sertoli cell tumours, 114 Spinal cord, radiotherapy side effects, 53
Sertoli, Enrico, 114 Spine, achondroplasia, 185
Shape, shoulder, 160 Splenectomy
Shifting dullness, ascites, 94, 94f blood film, 83
Short leg gait, 166t immunizations, 83
Short saphenous vein, 189f Splenomegaly, 8183
Shoulder, 159164 causes, 82
asymmetry, 181 CHIASMA acronym, 82
external rotation, 161f hepatomegaly vs., 82
feel, 122, 160, 160f inspection, 81
frozen, 164 massive, 82
internal rotation, 161, 161f palpation, 8182, 81f
movement, 160162 percussion, 8182
observation, 159160 peripheral signs, 81
osteoarthritis, 161 SPRUE acronym, 8182
power testing, 162, 162f Split skin graft, venous ulcer, 194
Shouldice repair, inguinal hernia, 67 Spontaneous haemarthrosis, primary, 156
Should The Children Ever Find Lumps Readily SPRUE acronym
mnemonic, 5 enlarged kidneys, 110b
Shunts, ascites, 95 hepatomegaly, 76b
Sickle cell disease, neuropathic ulcers, 215 splenomegaly, 8182
Simple cysts, enlarged kidneys, 101 Squamous cell carcinoma (SCC), 2829
Simple mastectomy, 106 basal cell carcinoma vs., 33
Sipple, John H, 22 cervical lymphadenopathy, 1314
Sipple syndrome (multiple endocrine neoplasia type neuropathic ulcers, 215
IIA), 22 solar keratosis precursor, 47
Sites, stoma, 74, 74f Squaring the pelvis, hip osteoarthritis, 125, 125f
SITS mnemonic, 162b Staging, malignant melanoma, 3031
Sjgren, H S C, 42 Stance, gait, 165
Sjgrens syndrome, 42 Standing examination
Skin hip osteoarthritis, 123
amputation necrosis, 201 knee ligaments/cartilages, 155
blistering, gangrene, 211 knee osteoarthritis, 130
cast complications, 170 Staphylococcal infections, 205
Dupuytrens contracture, 138 Stellwags sign, 19
grafts, 3536 Stenosis, aorta, 227
pigmentation, post-phlebitic limb, 210 Stensen, N, 42
radiotherapy side effects, 53 Stensens duct, 40
shoulder examination, 160 Sternotomy
squamous cell carcinoma, 28 median see Median sternotomy
varicose veins, 188 subtotal median, 111
vascular malformations, 61 Stiff knee gait, knee osteoarthritis, 130
254 Index

Stiffness, orthopaedic history taking, 123 femoral hernia, 120


Stinstrunks operation, thyroglossal cysts, 52 frozen shoulder, 164
Stoma, 7375 furuncles, 59
complications, 75 ganglions, 8
indications, 74 Graves disease, 2425
patient preparation, 74 hallux valgus, 149
rehabilitation, 75 hammer toes, 150
sites, 74, 74f hand osteoarthritis, 145
Stomatitis, angular, 116 hip osteoarthritis, 129
Stool tests, inflammatory bowel disease, 84 hyperhidrosis, 217
Straight leg raising (SLR), 173 inflammatory bowel disease, 85
Strawberry patch, 61 keratoacanthoma, 43
Strictures, dysphagia, 98 knee osteoarthritis treatment, 136
Stroke lipoma, 6
carotid artery disease, 206 lumbar disc herniation, 174
embolic, atrial fibrillation, 228 lymphoedema, 216
Stump neuroma, amputation, 201 malignant melanoma, 3132
Stump ulceration, amputation, 201 mallet toes, 151
SturgeWeber syndrome, 61 multinodular goitre, 23
Sturge, William Allen, 61 osteochondroma, 167
Subcostal scar, 102f Pagets disease of bone, 184
Sublingual glands, tumours, 43 pharyngeal pouch, 57
Submandibular gland, 10f popliteal aneurysm, 208
duct opening, 40 post-phlebitic limb, 211
tumours, 43 pressure sore therapy, 35
Submandibular lymph node, 12f pyoderma granulosum, 60
Submental gland, 10f pyogenic granuloma, 46
Submental lymph node, 12f Raynauds phenomenon, 213
Subscapularis, power testing, 162, 163f rheumatoid hands treatment, 143
Subtotal colectomy with ileostomy, inflammatory bowel rotator cuff tear, 164
disease, 85 sebaceous cysts, 7
Subtotal median sternotomy, 111 testicular tumours, 113
Sunlight ulnar nerve palsy, 147
basal cell carcinoma, 33 varicose vein treatment, 192
malignant melanoma, 30 venous ulcer, 194
Superficial basal cell carcinoma, 33 winging of scapula, 181
Superficial circumflex iliac vein, 189f Surgical jaundice see Jaundice, surgical
Superficial epigastric vein, 189f Surgical scars, 102103, 102f
Superficial external pudendal vein, 189f Swallowing, thyroglossal cysts, 52
Superficial femoral artery, 195f Swallow test, thyroid examination, 15
Superficial lesions, 161 Swan neck deformity, 142, 142f
Superficial parotidectomy, parotid gland swelling, Swing, gait, 165
41 Syme, James, 201
Superficial spreading malignant melanoma, 2930 Symmetry, shoulder, 160
Superficial veins, post-phlebitic limb, 210 Sympathectomy
Superior vena cava obstruction, 222223 cervical see Cervical sympathectomy
Supinator muscle, 158f lumbar, 209
Supracondylar osteotomy, knee rheumatoid arthritis, Symphis pubis palpation, inguinal hernia, 65, 66f
137 Synovectomy, knee rheumatoid arthritis, 137
Supraspinatus tendon Syphilis
power testing, 162, 162f abdominal aortic aneurysms, 205
shoulder examination, 160 neuropathic ulcers, 215
Surgery pyoderma granulosum vs., 60
achondroplasia, 185 Syringomyelia, hyperhidrosis, 218
anterior cruciate ligament rupture, 157
carpal tunnel syndrome, 141 Takayasu, Mikito, 221
cervical lymphadenopathy, 14 Takayasus arteritis, thoracic outlet obstruction vs.,
chemodectoma, 59 220
claw toes, 152 Tattoos, hepatomegaly, 76
coarctation of the aorta, 227 Telangiectasis, 61
Crohns disease, 8586 Temperature
cystic hygroma, 58 knee osteoarthritis, 131133
Dupuytrens contracture, 138 peripheral arterial system examination, 196, 196f
enterocutaneous fistula, 115 Tenderness, knee osteoarthritis, 131
Index 255

Tendons Thyroid enlargement


Dupuytrens contracture, 138 classification, 16b
rotator cuff impingement, 163 diffuse enlargement see Diffuse thyroid enlargement
shoulder examination, 160 solitary nodule see Solitary thyroid nodule
see also specific tendons see also Thyroid cancer
Tensioned fine wires, external fixators, 182 Thyroid examination, 1419
Teratoma, testicular tumours, 113t from behind, 1516, 15f
Teres minor, power testing, 162, 163f cervical lymph nodes, 16f
Terry, R, 78 eyes, 1718, 17f18f
Terrys lines, hepatomegaly, 76 from front, 1516
Terry, T L, 184 Graves disease, 16
Tertiary syphilis, pyoderma granulosum vs., 60 hands, 17
Testes swallow test, 15
ectopic, 91f systemic examination, 18
examination, 86 thyroid status, 1619
incompletely descended, 91f tongue protrusion, 15
lymph drainage, 87 tracheal deviation, 16, 16f
tumours, 112114 voice hoarseness, 16
Test light touch, carpal tunnel syndrome, 139 Thyroid eye disease, 25
Testosterone, gynaecomastia, 109 NO SPECS classification, 25, 25t
Third lumbrical muscle, 146f Thyroid function tests, gynaecomastia, 109
Third nerve palsy, ptosis, 37 Thyroiditis, 24
Thomas, High Owen, 130 Thyroid nodules, 10f
Thomas test, 126, 126f127f solitary see Solitary thyroid nodule
Thoracic outlet obstruction, 219221 Thyroid status, thyroid examination, 1619
Thoraco-abdominal scar, 102f Thyroxine replacement
3 Ms, ulcerative colitis surgery, 85 multinodular goitre, 23
3S inspection, varicose veins, 188b post-thyroidectomy, 21
Thromboangiitis obliterans (Buergers disease), 221 Tibial tuberosity, knee osteoarthritis, 131132, 131f
gangrene, 212 Tight casts, 170
thoracic outlet obstruction vs., 220 Tinel, Jules, 141
Thromboembolic deterrent (TED) stockings, deep vein Tinels sign, carpal tunnel syndrome, 140
thrombosis, 130 TIPS (transjugular intrahepatic portosystemic stent
Thrombosed haemangiomas, malignant melanoma vs., shunt), ascites, 95
30 Toenails, ingrowing, 154155
Thrombosis Toe off, gait, 165
CiminoBrescia arteriovenous fistula, 225 Tone, brachial plexus lesions, 176
gangrene, 212 Tongue protrusion
Thumb, rheumatism, 142 neck examination, 9
Thyroglossal cysts, 10f, 5152 thyroglossal cysts, 51
swallowing, 52 thyroid examination, 15
tongue protrusion, 51 Tooth, Howard Henry, 152
Thyroid Total thyroidectomy, Graves disease, 25
acropathy, 17 Tourniquet test, varicose veins, 189, 190b, 191f192f
arterial supply, 19, 20f Tracheal deviation, thyroid examination, 16, 16f
cysts, 22 Traction, fracture reduction, 172
diffuse enlargement see Diffuse thyroid Trahere transplantation, post-phlebitic limb, 211
enlargement Transfixing pins, external fixators, 182
history taking, 2526 Transient ischaemic attacks (TIAs), carotid artery
lumps, tongue protrusion, 9 disease, 206
malignancies, 2022 Transilluminability, lumps and ulcers, 4
multinodular goitre see Multinodular goitre Transinguinal repair, femoral hernia surgery, 120
radiotherapy side effects, 53 Transjugular intrahepatic portosystemic stent shunt
structures around, 16 (TIPS), ascites, 95
Thyroid cancer Transplants, kidneys see Kidney transplants
medullary, 21 Transudate, pleural effusion, 66f, 97
papillary, 20 Transverse rectus abdominis myocutaneous (TRAM)
Thyroidectomy flaps, breast reconstruction, 108
complications, 25, 234 Transverse scar, 102f
hyperthyroidism, 25 Trapezius palsy, winging of scapula, 181
informed consent, 233234 Trauma
multinodular goitre, 23 brachial plexus lesions, 175
thyroid cancer, 20 diabetic foot, 199
total, Graves disease, 25 enterocutaneous fistula, 115
256 Index

information gathering, 232 Umbilical/paraumbilical hernia, 7981, 80f


mallet finger, 153 adults, 8081
ulnar nerve palsy, 147 children, 80
Traumatic arteriovenous fistula, 226 Mayos vest-over-pants operation, 81
Trlat, W, 47 umbilical scar, 80
Tremors, fine, 17 Umbilical scar, 80
Trenaunay, P, 192 Uni-axial frames, external fixators, 182
Trendelenburg, Friedrich, 130, 192 Unilateral ptosis, 37
Trendelenburg gait, 124, 166t Up-and-down technique
Trendelenburg test cervical lymphadenopathy, 1213
gait, 165 neck examination, 11t
hip osteoarthritis, 123124, 124f Upper limb, loss of function, 122
varicose veins, 189, 190f191f Urinalysis, atherosclerosis, 206
Triceps muscle, 158f Urinary retention, inguinal hernia repair, 67
Trichilemmal cysts (TCs), 7
Trigger finger, 153154 Vaginal hydrocele, 8788
Triple assessment Varicography, post-phlebitic limb, 211
breast lumps, 105 Varicose veins, 188193
solitary thyroid nodule, 1920 inspection, 188
Troisier, C E, 70 palpation, 188190
Truelove classification, 8485 3S inspection, 188b
Trunk, abdominal examination, 69 treatment, 192
Tuberculosis Varicosities, varicose veins, 188
cervical lymphadenopathy, 14 Variocele, 8990
neuropathic ulcers, 214215 Vascular (Buergers) angle, 195196, 196f
Vascular manifestations, rheumatoid arthritis, 143
UK Small Aneurysm Trial, 204 Vascular referral, information gathering, 232233
Ulcerative colitis Vasculitis, neuropathic ulcers, 214
pyoderma granulosum, 60 Venography, post-phlebitic limb, 211
surgery, 85 Venous gangrene, 211
Ulcers/ulceration Venous guttering, peripheral arterial system
amputation stump, 201 examination, 196
aphthous, 116 Venous hum, hepatomegaly, 77
Behets disease, 116 Venous hypertension, CiminoBrescia arteriovenous
cutaneous, squamous cell carcinoma, 29 fistula, 225
examination, 35 Venous leg ulcer, 210t
heel, 195 Venous ulcer, 193194
history taking, 3 Vertebrae, Pagets disease of bone, 184
ischaemic see Ischaemic ulcers Vessel wall calcification, diabetic foot, 200
mouth, 116 Vinson, P P, 99
neuropathic see Neuropathic ulcers Virchow, R L K, 70
sebaceous cysts, 7 Virchows node, right iliac fossa mass, 90
venous, 193194 Viscera, radiotherapy side effects, 53
venous leg, 210t Viscosupplementation, knee osteoarthritis treatment, 136
Ulnar nerve, 139b Vitamin B12 deficiency, neuropathic ulcers, 214
anterior transposition, 147 Vitiligo, thyroid examination, 17
decompression, 147 Voice hoarseness, thyroid examination, 16
muscles supplied, 146f Von Leydis, Franz, 114
neuropathy vs. thoracic outlet obstruction, 220 Von Mickulicz-Radecki, J, 42
Ulnar nerve lesions, 145147 Von Recklinghausen, Friedrich Daniel, 45
inspection, 145147
motor assessment, 145 Waiters tip, Erbs palsy, 175176
sensory assessment, 145 Walking see Gait
Ulnar nerve palsy Warfarin necrosis, pyoderma granulosum vs., 60
cause, 147 Warren bypass, post-phlebitic limb, 211
Dupuytrens contracture, 138 Wartenbergs syndrome, 159
treatment, 147 Warthin, A S, 42
Ultrasound WBC acronym, Raynauds phenomenon, 213b
Doppler see Doppler ultrasound Weber, Frederick Parkes, 61, 117, 192
Duplex see Duplex ultrasound Wedge excision, ingrowing toenail, 154
hepatomegaly, 77 Weight-loss, surgical jaundice, 71
obstructive jaundice, 72 Wermer, P, 22
right iliac fossa mass, 9192 Werner syndrome (multiple endocrine neoplasia type I),
see also Duplex ultrasound 22
Index 257

Whartons duct, 40 Xeroderma pigmentosum


Wharton, T, 42 basal cell carcinoma, 33
Willis, Thomas, 207 malignant melanoma, 30
World Health Organization (WHO), goitre classification, Xerophthalmia, Schirmers test, 42
1819 Xerostomia, Sjgrens syndrome, 42
Wounds X-rays see Radiography
ganglions, 8
Yergasons test, 163
hypertrophic scars, 27
Yolk sac tumours, testicular tumours, 113
keloid scars, 27
pressure sores, 35 Zadeks procedure, ingrowing toenail, 154
Wrist, rheumatism, 142 Z-thumb appearance, 142
This page intentionally left blank
This page intentionally left blank
This page intentionally left blank

Você também pode gostar